GI Questions

Réussis tes devoirs et examens dès maintenant avec Quizwiz!

A patient calls the clinic to report a severe diarrhea lasting 4 days. What would the nurse anticipate that the patient will need to do? A. Collect a stool specimen. B. Prepare for colonoscopy. C. Schedule a barium enema. D. Have blood cultures drawn.

A Acute diarrhea is often caused by an infectious process, so stool specimens are obtained for culture and examined for parasites or white blood cells. There is no indication that the patient needs a colonoscopy, blood cultures, or a barium enema.

Which action will the nurse include in the plan of care for a 25-yr-old male patient with a new diagnosis of irritable bowel syndrome (IBS)? A. Encourage the patient to express concerns and ask questions about IBS. B. Suggest that the patient increase the intake of milk and other dairy products. C. Teach the patient to avoid using nonsteroidal antiinflammatory drugs (NSAIDs). D. Teach the patient about the use of alosetron (Lotronex) to reduce IBS symptoms.

A Because psychologic and emotional factors can affect the symptoms for IBS, encouraging the patient to discuss emotions and ask questions is an important intervention. Alosetron has serious side effects and is used only for female patients who have not responded to other therapies. Although yogurt may be beneficial, milk is avoided because lactose intolerance can contribute to symptoms in some patients. NSAIDs can be used by patients with IBS.

Which information obtained by the nurse interviewing a patient is most important to communicate to the health care provider? A. Blood in the stool B. History of constipation C. Appendectomy 3 years ago D. Smokes a pack/day of cigarettes

A Blood in the stool is a possible clinical manifestation of colorectal cancer and requires further assessment by the health care provider. The other patient information will also be communicated to the health care provider but does not indicate an urgent need for further testing or intervention.

The nurse receives the following information about a patient who is scheduled for a colonoscopy. Which information would the nurse communicate to the health care provider before preparing the patient for the procedure? A. The patient declined to drink the prescribed laxative solution. B. The patient has had an allergic reaction to shellfish and iodine. C. The patient has a permanent pacemaker to prevent bradycardia. D. The patient is worried about discomfort during the examination.

A If the patient has had inadequate bowel preparation, the colon cannot be visualized and the procedure would be rescheduled. Because contrast solution is not used during colonoscopy, the iodine allergy is not pertinent. A pacemaker is a contraindication to magnetic resonance imaging but not to colonoscopy. The nurse should instruct the patient about the sedation used during the examination to decrease the patient's anxiety about discomfort.

A patient has just returned to the nursing unit after an esophagogastroduodenoscopy (EGD). Which action by assistive personnel (AP) requires that the registered nurse (RN) intervene? A. Offering the patient a pitcher of water B. Positioning the patient on the right side C. Checking the vital signs every 30 minutes D. Swabbing the patient's mouth with a wet cloth

A Immediately after EGD, the patient will have a decreased gag reflex and is at risk for aspiration. Assessment for return of the gag reflex should be done by the RN. The other actions by the AP are appropriate.

The client has a hiatal hernia. How does the nurse describe this to the client? A. A part of the stomach has exited through the diaphragm B. A part of the abdominal contents has exited the inguinal canal C. A part of the abdominal contents has exited through the abdominal wall D. A part of the abdominal contents has exited the femoral canal

A In a hiatal hernia, a part of the stomach has pushed up through a hole in the diagpragm Ventral Hernia - part of the abdominal contents exited through the abdominal wall Femoral Hernia - part of the abdominal contents exited through the femoral canal Inguinal Hernia - part of the abdominal contents exited through the inguinal canal

Which laboratory test result will the nurse monitor to evaluate the effects of therapy for a patient who has acute pancreatitis? A. Lipase B. Calcium C.Bilirubin D. Potassium

A Lipase is elevated in acute pancreatitis. Although changes in the other values may occur, they would not be useful in evaluating whether the prescribed therapies have been effective.

After having frequent diarrhea and a weight loss of 10 lb (4.5 kg) over 2 months, a patient has a new diagnosis of Crohn's disease. What would the nurse plan to teach the patient? A. Medication use B. Fluid restriction C. Enteral nutrition D. Activity restrictions

A Medications are used to induce and maintain remission in patients with inflammatory bowel disease (IBD). Decreased activity level is indicated only if the patient has severe fatigue and weakness. Fluids are needed to prevent dehydration. There is no advantage to enteral feedings if the patient is able to eat.

Which action would the nurse in the emergency department anticipate for a young adult patient who has had several acute episodes of bloody diarrhea? A. Obtain a stool specimen for culture. B. Administer antidiarrheal medication. C. Provide teaching about antibiotic therapy. D. Teach the adverse effects of acetaminophen (Tylenol).

A Patients with bloody diarrhea should have a stool culture for Escherichia coli O157:H7. Antidiarrheal medications are usually avoided for possible infectious diarrhea to avoid prolonging the infection. Antibiotic therapy in the treatment of infectious diarrhea is controversial because it may precipitate kidney complications. Acetaminophen does not cause bloody diarrhea.

Which prescribed intervention for a patient with chronic short bowel syndrome would the nurse question? A. Senna 1 tablet daily B. Ferrous sulfate 325 mg daily C. Psyllium (Metamucil) 3 times daily D. Diphenoxylate with atropine (Lomotil) PRN loose stools

A Patients with short bowel syndrome have diarrhea because of decreased nutrient and fluid absorption and would not need stimulant laxatives. Iron supplements are used to prevent iron-deficiency anemia, bulk-forming laxatives help make stools less watery, and opioid antidiarrheal drugs are helpful in slowing intestinal transit time.

A patient with Crohn's disease who is taking infliximab (Remicade) calls the nurse in the outpatient clinic about new symptoms. Which symptom is most important to communicate to the health care provider? A. Fever B. Nausea C. Joint pain D. Headache

A Since infliximab suppresses the immune response, rapid treatment of infection is essential. Nausea, joint pain, and headache are common side effects of the medication, but they do not indicate any potentially life-threatening complications.

A patient is admitted with possible botulism poisoning after eating home-canned green beans. Which intervention ordered by the health care provider would the nurse question? A. Encourage oral fluids to 3 L/day. B. Document neurologic symptoms. C. Position patient lying on the side. D. Observe respiratory status closely.

A The patient should not have oral fluids because neuromuscular weakness increases risk for aspiration. Side-lying position and assessment of neurologic and respiratory status are expected interventions.

A patient is being scheduled for endoscopic retrograde cholangiopancreatography (ERCP) as soon as possible. Which prescribed action would the nurse take first? A. Place the patient on NPO status. B. Administer sedative medications. C. Ensure the consent form is signed. D. Teach the patient about the procedure.

A The patient will need to be NPO for 8 hours before the ERCP is done, so the nurse's initial action should be to place the patient on NPO status. The other actions can be done after the patient is NPO.

Which patient would the nurse assess first after receiving change-of-shift report? A. A 40-yr-old patient who has a distended abdomen and tachycardia B. A 60-yr-old patient whose ileostomy has drained 800 mL over 8 hours C. A 30-yr-old patient with ulcerative colitis who had six liquid stools in 4 hours D. A 50-yr-old patient with familial adenomatous polyposis who has occult blood in the stool

A The patient's abdominal distention and tachycardia suggest hypovolemic shock caused by problems such as peritonitis or intestinal obstruction, which will require rapid intervention. The other patients would be assessed as quickly as possible, but the data do not indicate any life-threatening complications associated with their diagnoses.

The nurse is caring for a client with a hiatal hernia. Which of the following is an appropriate treatment? Select all that apply. A. Teach the client to avoid anticholinergics B. Recommend a high fat diet C. Teach the client to avoid straining D. Administer antacids E. Recommend eating large meals

A, C, D Weight loss is one way to improve a hiatal hernia. Meals should be small, and the client should be taught not to lie down for one hour after meals. Large meals should be avoided because gastric fullness can worsen the hernia.

A 45-year-old client is preparing to undergo hernia surgery. What information would the nurse provide for this client during the pre-operative teaching session? Select all that apply A. Leave valuables at home when coming to the hospital B. Do not bathe or shower for three days before the surgery C. Remove all jewelry and body piercings D. Stop taking insulin on the morning of the surgery E. Avoid eating or drinking anything for eight hours before surgery

A, C, E Bathing is important for any person, but particularly for the client undergoing surgery, as bacteria should be kept to a minimum to decrease the risk of infection. The general guideline for a diabetic client who must have surgery is to keep glucose values within a normal range. There is risk of hyperglycemia due to surgery's effect on the body's glucose metabolism. There is risk of hypoglycemia due to NPO status and some anesthesia drugs. Therefore, the client must ensure that their glucose level is kept within normal range, and the hospital staff will monitor this level throughout the peri-operative and post-operative periods.

Which of the following veins should be avoided when initiating an IV? Select all that apply A. lower extremities of diabetic pt B. previously used veins and sclerotic veins C. veins in unaffected arm of mastectomy D. veins in arm of dialysis AV fistula

A,B, D

A nurse is caring for a client with Zollinger Ellison Syndrome. Which interventions should be included in the plan of care? Select all that apply. A. Assess for melena B. Genetic counseling C. Vitamin D supplements D. Pantoprazole daily

A,B,D Clients with Zollinger Ellison syndrome are at risk for bleeding so stools should be assessed for occult blood. People at higher risk include males ages 25 to 50 and people who have a parent with the disorder. Zollinger Ellison syndrome cannot be prevented. Pantoprazole is used to treat damage from gastroesophageal reflux (GERD), A condition in which backward flow of acid from the stomach causes heartburn and possible injury of the esophagus (the tube between the throat and stomach) in adults and children five years of age and older. It is also used to treat conditions where the stomach produced too much acid, such as Zollinger Ellison syndrome in adults.

The nurse is caring for a client with an intestinal ulcer who takes lansoprazole. The nurse knows to monitor the client for which of the following adverse reactions? Select all that apply. A. Nausea B. Diarrhea C. Oliguria D. Headache E. Anxiety

A,B,D Lansoprazole is a proton pump inhibitor that carries the risk of adverse reactions. These include abdominal pain, headache, nausea, and diarrhea. Anxiety and Oliguria are not associated with PPI administration

After an unimmunized person is exposed to hepatitis B through a needle-stick injury, which actions will the nurse plan to take? (Select all that apply.) A. Administer hepatitis B vaccine. B. Test for antibodies to hepatitis B. C. c. Teach about a-interferon therapy. D. Give hepatitis B immune globulin. E. Explain options for oral antiviral therapy.

A,B,D The recommendations for hepatitis B exposure include both vaccination and immune globulin administration. In addition, baseline testing for hepatitis B antibodies will be needed. Interferon and oral antivirals are not used for hepatitis B prophylaxis.

The nurse is doing pre-op teaching with the client who is about to undergo the creation of a Kock pouch. The nurse interprets that the client has the best understanding of the nature of the surgery if the client makes which of the following statements? A. "I will need to drain the pouch regularly with a catheter." B. "I will need to wear a drainage bag for the rest of my life." C. "The drainage from this type of ostomy will be formed." D. "I will be able to pass stool from my rectum eventually."

A. "I will need to drain the pouch regularly with a catheter." A Kock pouch is a continent ileostomy. As the ileostomy begins to function, the client drains it every 3 to 4 hours and then decreases the draining to about 3 times a day or as needed when full. In this new operation, a pouch or reservoir is fashioned out of terminal ileum with a valve mechanism at its exit to the skin surface. This allows storage of the liquid bowel contents in an expandable container with no leakage of stool or gas and therefore no skin problems. Option B: The client does not need to wear a drainage bag but should wear an absorbent dressing to absorb mucus drainage from the stoma. There is no need for appliances or bags, no embarrassment from the involuntary noise and smell of flatus through the ileostomy. The stoma is created flush and within the bikini line. The patient catheterizes the pouch an average of three times a day. Option C: Ileostomy drainage is liquid. Ileostomy output will be liquid to pasty, depending on what the client eats, his medicines, and other factors. Because the output is constant, the client will need to empty the pouch 5 to 8 times a day. Option D: The client would be able to pass stool only from the rectum if an ileal-anal pouch or anastomosis were created. This type of operation is a two-stage procedure. Not only does this procedure solve many of the complications of a conventional ileostomy, but it helps decrease the amount of emotional trauma suffered by the young ileostomist, greatly improving the quality of life.

Michael, 42 y.o. The man is admitted to the med-surg floor with a diagnosis of acute pancreatitis. His BP is 136/76, pulse 96, Resps 22, and temp 10 His past history includes hyperlipidemia and alcohol abuse. The doctor prescribes an NG tube. Before inserting the tube, you explain the purpose to the patient. Which of the following is the most accurate explanation? A. "It empties the stomach of fluids and gas." B. "It prevents spasms at the sphincter of Oddi." C. "It prevents air from forming in the small intestine and large intestine." D. "It removes bile from the gallbladder."

A. "It empties the stomach of fluids and gas." An NG tube is inserted into the patient's stomach to drain fluid and gas. Nasogastric tubes are part of the standard of care in treating intestinal obstruction and can also be used to provide nutritional support. They are most common in surgical patients but are useful in any patient population where gastric decompression or nutritional support is necessary. Option B: An NGT does not prevent spasms at the sphincter of Oddi. The most common indication for placement of a nasogastric tube is to decompress the stomach in the setting of distal obstruction. Small bowel obstruction from adhesions or hernias, ileus, obstructing neoplasms, volvulus, intussusception, and many other causes may block the normal passage of bodily fluids such as salivary, gastric, hepatobiliary, and enteric secretions. Option C: Similarly, intractable nausea or emesis, whether caused by medications, intoxication, or other reasons, can be an indication for the placement of a nasogastric tube in order to prevent aspiration. Prophylactic placement of the NG tube in patients with abdominal surgery is not recommended. Patients who develop postoperative ileus tend to recover faster without the placement of an NG tube. Option D: Placement of an NGT does not remove bile from the gallbladder. NG tubes have been used for various reasons in patients with GI bleeding. In the past, NG lavage was thought to help control GI bleeding. However, recent studies have shown that this is not helpful. Another indication for placement of a nasogastric tube is in the setting of massive hematochezia.

When teaching a community group about measures to prevent colon cancer, which instruction should the nurse include? A. "Limit fat intake to 20% to 25% of your total daily calories." B. "Include 15 to 20 grams of fiber into your daily diet." C. "Get an annual rectal examination after age 35." D. "Undergo sigmoidoscopy annually after age 50."

A. "Limit fat intake to 20% to 25% of your total daily calories." To help prevent colon cancer, fats should account for no more than 20% to 25% of total daily calories and the diet should include 25 to 30 grams of fiber per day. Long-chain n-3 fatty acids have been suggested to play a protective role in colorectal cancer development in laboratory and animal studies, with the mechanism of action conjectured to be inhibition of the cyclooxygenase-2 (COX-2) enzyme and the production of arachidonic acid (n-6) derived eicosanoids. Option B: Many observational studies have reported an inverse association between dietary fiber and CRC risk, with a relative reduction of up to 40%, although a few large cohort studies reported small, statistically null associations. A large pooled analysis of thirteen prospective cohorts suggested that dietary fiber intake was inversely associated with CRC risk in age-adjusted analyses. Option C: A digital rectal examination isn't recommended as a stand-alone test for colorectal cancer. All new CRC cases should be universally screened for DNA mismatch repair/microsatellite status, and RAS/BRAF mutational testing when considering for prognostic and predictive of chemotherapy efficacy. In almost all patients, a diagnostic or screening colonoscopy is required for tissue biopsy pathological confirmation of colon carcinoma. Option D: For colorectal cancer screening, the American Cancer Society advises clients over age 50 to have a flexible sigmoidoscopy every 5 years, yearly fecal occult blood tests, yearly fecal occult blood tests PLUS a flexible sigmoidoscopy every 5 years, a double-contrast barium enema every 5 years, or a colonoscopy every 10 years.

A male client with extreme weakness, pallor, weak peripheral pulses, and disorientation is admitted to the emergency department. His wife reports that he has been "spitting up blood." A Mallory-Weiss tear is suspected, and the nurse begins taking the client's history from the client's wife. The by the nurse that demonstrates her understanding of Mallory-Weiss tearing is: A. "Tell me about your husband's alcohol usage." B. "Is your husband being treated for tuberculosis?" C. "Has your husband recently fallen or injured his chest?" D. "Describe spices and condiments your husband uses on food."

A. "Tell me about your husband's alcohol usage." A Mallory-Weiss tear is associated with massive bleeding after a tear occurs in the mucous membrane at the junction of the esophagus and stomach. There is a strong relationship between ethanol usage, resultant vomiting, and a Mallory-Weiss tear. Mallory-Weiss tears account for an estimated 1-15% of cases of upper gastrointestinal bleeding. Although the age range varies widely, affected individuals are generally in middle age (40s-50s), and men reportedly have a higher incidence than women by a ratio of 2-4:1. Option B: The bleeding is coming from the stomach, not from the lungs as would be true in some cases of tuberculosis. The presence of a hiatal hernia is a predisposing factor and is found in 35-100% of patients with Mallory-Weiss tears. During retching or vomiting, the transmural pressure gradient is greater within the hernia than the rest of the stomach, and it is the location most likely to sustain a tear Option C: A Mallory-Weiss tear doesn't occur from chest injuries or falls. Precipitating factors include retching, vomiting, straining, hiccupping, coughing, primal scream therapy, blunt abdominal trauma, and cardiopulmonary resuscitation. In a few cases, no apparent precipitating factor can be identified. One study reported that 25% of patients had no identifiable risk factor. Option D: A Mallory-Weiss tear isn't associated with eating spicy foods. Mallory-Weiss tears are usually associated with other mucosal lesions. In one study, 83% of patients had additional mucosal abnormalities potentially contributing to bleeding or actually causing retching and vomiting that would induce these tears.

The nurse provides medication instructions to a client with peptic ulcer disease. Which statement, if made by the client, indicates the best understanding of the medication therapy? A. "The cimetidine (Tagamet) will cause me to produce less stomach acid." B. "Sucralfate (Carafate) will change the fluid in my stomach." C. "Antacids will coat my stomach." D. "Omeprazole (Prilosec) will coat the ulcer and help it heal."

A. "The cimetidine (Tagamet) will cause me to produce less stomach acid." Cimetidine (Tagamet), a histamine H2 receptor antagonist, will decrease the secretion of gastric acid. The H2-receptor antagonist cimetidine competitively blocks histamine from stimulating the H2-receptors located on the gastric parietal cells (these cells are responsible for hydrochloric acid secretion and secretion of the intrinsic factor). The effect results in reducing the volume of gastric acid secretion from stimuli, including histamine, food, caffeine, and insulin. Option B: Sucralfate (Carafate) promotes healing by coating the ulcer. By forming a polyanion gel, it acts as a physical barrier between luminal contents and mucosa. It also increases the production of mucus by increasing prostaglandin production. Sucralfate prevents the breakdown of mucus by pepsin A, reducing ulcerogenesis. Option C: Antacids neutralize acid in the stomach. The antacids reduce the acid reaching the duodenum by neutralizing the acid present in the stomach. The salts' mechanism of neutralization of acid varies, and each salt has a different mechanism with the ultimate goal of acid neutralization. Option D: Omeprazole (Prilosec) inhibits gastric acid secretion. Omeprazole is a proton pump inhibitor. It inhibits the parietal cell H+ / K+ ATP pump, the final step of acid production. In turn, omeprazole suppresses gastric basal and stimulated acid secretion. The inhibitory effects of omeprazole occur rapidly within 1 hour of administration, with the maximum effect occurring in 2 hours.

A female client who has just been diagnosed with hepatitis A asks, "How could I have gotten this disease?" What is the nurse's best response? A. "You may have eaten contaminated restaurant food." B. "You could have gotten it by using I.V. drugs." C. "You must have received an infected blood transfusion." D. "You probably got it by engaging in unprotected sex."

A. "You may have eaten contaminated restaurant food." Hepatitis A virus typically is transmitted by the oral-fecal route — commonly by consuming food contaminated by infected food handlers. The hepatitis A virus (HAV) is a common infectious etiology of acute hepatitis worldwide. HAV is most commonly transmitted through the oral-fecal route via exposure to contaminated food, water, or close physical contact with an infectious person. The virus isn't transmitted by the I.V. route, blood transfusions, or unprotected sex. Option B: Hepatitis B can be transmitted by I.V. drug use. In the United States, estimates are about 2.2 million people have chronic hepatitis B virus infection. It is transmitted parenterally and sexually when individuals come in contact with mucous membranes or body fluids of infected individuals. Option C: Hepatitis B can be transmitted by blood transfusion. Transfusion of blood and blood products, injection drug use with shared needles, needlesticks, or wounds caused by other instruments in healthcare workers and hemodialysis are all examples of parenteral and percutaneous exposures, but parenteral mode remains the dominant mode of transmission both globally and in the United States. Option D: Hepatitis C can be transmitted by unprotected sex. Transmission can be parenteral, perinatal, and sexual, with the most common mode being the sharing of contaminated needles among IV drug users. Also, other high-risk groups include people who require frequent blood transfusions and organ transplantation of organs from infected donors.

Gail is scheduled for a cholecystectomy. After completion of preoperative teaching, Gail states," If I lie still and avoid turning after the operation, I'll avoid pain. Do you think this is a good idea?" What is the bestresponse? A. "You'll need to turn from side to side every 2 hours." B. "It's always a good idea to rest quietly after surgery." C. "The doctor will probably order you to lie flat for 24 hours." D. "Why don't you decide about activity after you return from the recovery room?"

A. "You'll need to turn from side to side every 2 hours." To prevent venous stasis and improve muscle tone, circulation, and respiratory function, encourage her to move after surgery. Insufflation of the abdominal cavity with carbon dioxide during laparoscopic surgery increases abdominal pressure, which may cause stasis of the blood flow in inferior vena cava and common iliac veins. Moreover, the reverse Trendelenburg position, in which laparoscopic cholecystectomy is performed, decreases venous return. Option B: The patient may need to walk around the same day of surgery, or the day after. The movement will help prevent blood clots. She may also be given exercises to do in bed. The patient should not get out of bed on her own until the healthcare provider says you can. She should talk to healthcare providers before she gets up the first time. They may need to help the patient stand up safely. Option C: The patient should take deep breaths and cough 10 times each hour. This will decrease the risk of a lung infection. She should take a deep breath and hold it for as long as she can. Let the air out and then cough strongly. Deep breaths help open the airway. The patient may be given an incentive spirometer to help take deep breaths. Option D: The patient may be taken to a recovery room, where she will stay until she is fully awake. Healthcare providers will watch closely for problems. She should not attempt to get out of bed until the healthcare provider says it is okay. When healthcare providers see that she is okay, she will be taken back to a hospital room.

You're teaching Anthony how to use his new colostomy. How much skin should remain exposed between the stoma and the ring of the appliance? A. 1/16" B. 1/4" C. 1/2" D. 1"

A. 1/16" Only a small amount of skin should be exposed and more than 1/16" of skin allows the excrement to irritate the skin. It is expected that the stoma will change its size (get smaller) for the first four to six weeks after surgery due to the swelling that occurs post-op. Option B: Cut the wafer slightly larger to accommodate the expansion. Doing this also helps to prevent the wafer from "strangulating" the stoma by putting too much pressure around it. Option C: Make sure that there's about a 1/16 - 1/8? (approx. 5 - 3mm) gap between the stoma and the edge of the hole. Remember, as wafers do swell, the patient may need to adjust this gap accordingly. If using a barrier ring or similar product, it is okay to cut the hole slightly larger Option D: The pouching system must be completely sealed to prevent leaking of the effluent and to protect the surrounding peristomal skin. The disposable pouching systems can be either a one-piece or a two-piece flexible system consisting of a plastic bag and a flange (skin barrier) that sit against the patient's skin. The flange may be flat or convex.{

When used with hyperacidic disorders of the stomach, antacids are given to elevate the gastric pH to: A. 2.0 B. 4.0 C. 6.0 D. >8.0

A. 2.0 Antacids are a group of drugs that have been on the market for many years. They were initially first-line defense against peptic ulcer disease; however, the discovery of proton pump inhibitors revolutionized the treatment of peptic ulcer disease. Currently, antacid use is restricted to the relief of mild intermittent gastroesophageal reflux disease (GERD) associated with heartburn. Option B: The formulation of aluminum hydrochloride and water results in the neutralization of the acid in the stomach. It is also known to inhibit pepsin activity. Aluminum hydroxide is complexed with a sulfated polysaccharide sucrose octasulfate to form sucralfate. This complex does not have a significant buffering action against the acid or has no effect on the pepsin secretion and does not alter the gastric acid production in any way. Option C: Calcium salts neutralize gastric acidity resulting in increased gastric and duodenal bulb pH; they additionally inhibit the proteolytic activity of pepsin if the pH is greater than 4 and increase lower esophageal sphincter tone. The calcium released from calcium carbonate is known to increase peristalsis in the esophagus, pushing the acid into the stomach and providing relief from symptoms of heartburn. Option D: The dose for antacids depends upon the age of the patient, the purpose of administration (neutralization of acid or off-label use), and the presence of other comorbidities like renal or hepatic impairment. As all the forms of these medications are available as over-the-counter medication, the dosing recommendation varies by product/and or manufacturer.

Nurse Ryan is assessing for correct placement of a nasogastric tube. The nurse aspirates the stomach contents and checks the contents for pH. The nurse verifies correct tube placement if which pH value is noted? A. 3.5 B. 7.0 C. 7.35 D. 7.5

A. 3.5 If the nasogastric tube is in the stomach, the pH of the contents will be acidic. Gastric aspirates have acidic pH values and should be 3.5 or lower. The pH test performed with reagent strips is sensitive to identify the correct placement of the gastric tube, so it can be used as an adjuvant technique in the evaluation of the gastric tube placement. In interpreting the results, pH ?5.5 points to correct placement, and values > 5.5 require radiological confirmation. Option B: 7.0 indicates a slightly acidic pH. There is evidence that the use of histamine H2 receptor antagonist drugs may increase the pH value and cause confusion in the evaluation of gastric tube placement. Option C: 7.35 indicates a neutral pH. Verifying the pH of the aspirated secretion using reagent strips is a quick bedside test. Currently, there is a consensus among experts that this is the safest method available and is recommended as the first choice when verifying gastric tube placement in adults and children. Option D: 7.5 indicates an alkaline pH. The use of pH reagent strips is a sensitive but non-specific test to verify the placement of the gastric tube in newborns in the sample studied. That is, pH values ???5.5 in the aspirated gastric tube secretion are sensitive indicators of the correct positioning of the tip of the tube.

Christina is receiving an enteral feeding that requires a concentration of 80 ml of supplement mixed with 20 ml of water. How much water do you mix with an 8 oz (240ml) can of feeding? A. 60 ml B. 70 ml C. 80 ml D. 90 ml

A. 60 ml Dosage problem. It's 80/20 = 240/X. X=60. There are many different formulas that are used for enteral nutrition, and each formula is specific to the medical condition. For example, there are formulas for relatively healthy people, and those with cancer, diabetes, and kidney or liver disease, among other medical conditions. Most formulas will provide between 0-2.0 calories per mL of formula. Option B: On average, enteral formulas deliver between 0-2.0 calories/mL of formula. To calculate how much formula is needed, divide the total calorie needs by the amount of calories per mL of formula. The total calories a person needs over a 24-hr period is calculated by multiplying BMR by the activity factor. The activity factor is a number used to represent how many calories a person burns through physical activity. Option C: Enteral nutrition involves supplying nutrients directly into the stomach or intestines (the word 'enteral' comes from the word 'entrails', which means 'intestines'). Enteral nutrition is often referred to as 'tube feeding' because a tube is used to supply a liquid formula directly into the GI tract. This liquid formula contains calories, fats, vitamins, minerals, and other nutrients needed for proper health and survival. Option D: Enteral formulas are composed of carbohydrates, fats, proteins, micronutrients, and free water. The majority of the formulation (60-85%) is free water and can be easily calculated.

Which of the following complications is thought to be the most common cause of appendicitis? A. A fecalith B. Bowel kinking C. Internal bowel occlusion D. Abdominal bowel swelling

A. A fecalith A fecalith is a fecal calculus, or stone, that occludes the lumen of the appendix and is the most common cause of appendicitis. The cause of appendicitis is usually an obstruction of the appendiceal lumen. This can be from an appendicolith (stone of the appendix), or some other mechanical etiologies. Appendiceal tumors such as carcinoid tumors, intestinal parasites, and hypertrophied lymphatic tissue are all known causes of appendiceal obstruction and appendicitis. Option B: Kinking of the appendix is one of the causes of appendicitis. Appendicitis is inflammation of the vermiform appendix. This is a hollow organ located at the tip of the cecum, usually in the right lower quadrant of the abdomen. Option C: External, not internal, occlusion of the bowel by adhesion is a cause of appendicitis. When the appendiceal lumen gets obstructed, bacteria will build up in the appendix and cause acute inflammation with perforation and abscess formation. Option D: Bowel wall swelling is one of the causes of appendicitis. The pathophysiology of appendicitis likely stems from obstruction of the appendiceal orifice. This results in inflammation, localized ischemia, perforation, and the development of a contained abscess or frank perforation with resultant peritonitis.

A client with which of the following conditions may be likely to develop rectal cancer? A. Adenomatous polyps B. Diverticulitis C. Hemorrhoids D. Peptic ulcer disease

A. Adenomatous polyps A client with adenomatous polyps has a higher risk of developing rectal cancer than others do. Familial adenomatous polyposis (FAP), a rare disorder that causes hundreds or even thousands of polyps to develop in the lining of the colon beginning during teenage years. If the polyps aren't treated, the risk of developing colorectal cancer is nearly 100 percent, usually before age 40. Option B: Clients with diverticulitis are more likely to develop colon cancer. The risk of colorectal cancer is increased in the short-term period after hospitalization related to diverticular disease. According to a recent systematic review and meta-analysis, the prevalence of colorectal cancer is 6% in patients with acute diverticulitis who underwent colonoscopy. Option C: Hemorrhoids don't increase the chance of any type of cancer. For many, cancer is the first thing that comes to mind when experiencing blood in their stool for the first time. While colorectal cancer can cause similar symptoms, hemorrhoids are far more common. As uncomfortable as hemorrhoids can be, they're easily treatable and don't cause cancer. Option D: Clients with peptic ulcer disease have a higher incidence of gastric cancer. Helicobacter pylori infection, now considered to be a cause of gastric cancer, is also strongly associated with gastric and duodenal ulcer disease. The discovery of these relations has brought the long-controversial connection between peptic ulcers and gastric cancer into focus.

Hepatic encephalopathy develops when the blood level of which substance increases? A. Ammonia B. Amylase C. Calcium D. Potassium

A. Ammonia Ammonia levels increase d/t improper shunting of blood, causing ammonia to enter systemic circulation, which carries it to the brain. Under normal conditions, ammonia is produced by bacteria in the gastrointestinal tract (e.g., breakdown product of amines, amino acids, purines, and urea) followed by metabolism and clearance by the liver. In the case of cirrhosis or advanced liver dysfunction, however, there is either a decrease in the number of functioning hepatocytes, portosystemic shunting, or both, resulting in decreased ammonia clearance and hyperammonemia. Option B: Amylase is indirectly affected in hepatic encephalopathy. Hepatic encephalopathy (HE) is a reversible syndrome observed in patients with advanced liver dysfunction. The syndrome is characterized by a spectrum of neuropsychiatric abnormalities resulting from the accumulation of neurotoxic substances in the bloodstream (and ultimately in the brain). Option C: Elevated blood ammonia levels, not calcium levels, are often seen in patients with hepatic encephalopathy. It is more useful, however, to assess the clinical improvement or deterioration of a patient undergoing treatment rather than monitor serial arterial blood ammonia measurements. Option D: Once ammonia crosses the blood-brain barrier, it has multiple neurotoxic effects. These include alterations in molecular transport (e.g., amino acids, electrolytes, water) in astrocytes and neurons, increased synthesis of glutamine from glutamate by astrocytes, inhibition of excitatory and inhibitory postsynaptic potential generation, impaired amino acid metabolism, and impaired energy utilization as a result of increased GABA activity.

Which of the following areas is the most common site of fistulas in clients with Crohn's disease? A. Anorectal B. Ileum C. Rectovaginal D. Transverse colon

A. Anorectal Fistulas occur in all these areas, but the anorectal area is most common because of the relative thinness of the intestinal wall in this area. The initial lesion starts out as an infiltrate around an intestinal crypt. This goes on to develop ulceration first in the superficial mucosa and involves deeper layers. As the inflammation progresses, non-caseating granulomas form involving all layers of the intestinal wall. Option B: Small bowel follow-through is often used to assess the involvement of the terminal ileum and can also detect fistulas. The classic string sign due to stricture formation or spasm is often seen. Option C: Granuloma formation is very common in Crohn's disease but their absence does not exclude the diagnosis. The ongoing inflammation and scarring lead to bowel obstruction and stricture formation. Option D: It can develop into the classic cobblestone mucosal appearances and skip lesions along the length of the intestine sparing areas with normal mucosa. As the flare of Crohn's settles, scarring replaces the inflamed areas of the intestines.

You're caring for Betty with liver cirrhosis. Which of the following assessment findings leads you to suspect hepatic encephalopathy in her? A. Asterixis B. Chvostek's sign C. Trousseau's sign D. Hepatojugular reflux

A. Asterixis Asterixis is an early neurologic sign of hepatic encephalopathy elicited by asking the patient to hold her arms stretched out. Asterixis is present if the hands rapidly extend and flex. Asterixis is a clinical sign that describes the inability to maintain sustained posture with subsequent brief, shock-like, involuntary movements. This motor disorder is myoclonus characterized by muscular inhibition (whereas muscle contractions produce positive myoclonus). Option B: In the late 1800s, Dr. Chvostek noticed that mechanical stimulation of the facial nerve (as with the examiner's fingertip, for example) could lead to twitching of the ipsilateral facial muscles. The long-accepted explanation is that this resulted from hypocalcemia, and this relationship became known as the Chvostek sign. Option C: Trousseau's sign for latent tetany is most commonly positive in the setting of hypocalcemia. The sign is observable as a carpopedal spasm induced by ischemia secondary to the inflation of a sphygmomanometer cuff, commonly on an individual's arm, to 20 mmHg over their systolic blood pressure for 3 minutes. Option D: Deep inspiration generates negative intrathoracic pressure, leading to an increased venous return to the right atrium and, subsequently, the right ventricle. This phenomenon, in the healthy adult, enhances blood flow in the right heart chambers and causes decreased jugular venous pressure (JVP).

Which of the following nursing interventions should the nurse perform for a female client receiving enteral feedings through a gastrostomy tube? A. Change the tube feeding solutions and tubing at least every 24 hours. B. Maintain the head of the bed at a 15-degree elevation continuously. C. Check the gastrostomy tube for position every 2 days. D. Maintain the client on bed rest during the feedings.

A. Change the tube feeding solutions and tubing at least every 24 hours. Tube feeding solutions and tubing should be changed every 24 hours, or more frequently if the feeding requires it. Doing so prevents contamination and bacterial growth. The feeding bag should be changed every 24 hours. Food (formula) should not be left in the bag for more than 4 hours. So, only put 4 hours (or less) worth of food in the feeding bag at a time. Option B: The head of the bed should be elevated 30 to 45 degrees continuously to prevent aspiration. Lying prone/supine during feeding increases the risk of aspiration and therefore where clinically possible the child should be placed in an upright position. If unable to sit up for a bolus feed or if receiving continuous feeding, the head of the bed should be elevated 30-45 degrees during feeding and for at least 30 minutes after the feed to reduce the risk of aspiration. Option C: Checking for gastrostomy tube placement is performed before initiating the feedings and every 4 hours during continuous feedings. Correct placement of the tube should be confirmed prior to administration of an enteral feed by checking the insertion site at the abdominal wall and observing the client for abdominal pain or discomfort. If the nurse is unsure regarding the position of the gastrostomy or jejunostomy tube contact the medical team immediately. Option D: Clients may ambulate during feedings. Tube feeding doesn't need to keep the client from most physical activities. He can run or walk, but the client should talk to a doctor about yoga or other exercises that work the abdomen muscles. Even swimming is fine if the incision site has healed and the water is clean.

The nurse instructs the ileostomy client to do which of the following as a part of essential care of the stoma? A. Cleanse the peristomal skin meticulously. B. Take in high-fiber foods such as nuts. C. Massage the area below the stoma. D. Limit fluid intake to prevent diarrhea.

A. Cleanse the peristomal skin meticulously. The peristomal skin must receive meticulous cleansing because the ileostomy drainage has more enzymes and is more caustic to the skin than colostomy drainage. Dry the skin completely before putting on the skin barrier or pouch. Watch for sensitivities and allergies to the adhesive, skin barrier, paste, tape, or pouch material. They can develop after weeks, months, or even years of using a product because the client can become sensitized over time. Option B: Foods such as nuts and those with seeds will pass through the ileostomy. The client should be taught that these foods will remain undigested. Many factors, such as foods, normal bacteria in your intestine, illness, certain medicines, and vitamins can cause odor. Option C: The area below the ileostomy may be massaged if needed if the ileostomy becomes blocked by high fiber foods. There will be times when an ostomy does not have an output for short periods of time. This is normal. But, if the stoma is not active for 4 to 6 hours and the client has cramps, pain, and/or nausea, the intestine could be blocked (the medical word is obstructed). Option D: Fluid intake should be maintained to at least six to eight glasses of water per day to prevent dehydration. Drinks such as Gatorade®, PowerAde®, or Pedialyte® contain potassium and sodium. But any liquid containing water (soda, milk, juice, tea, etc.) helps to meet the daily need for fluid.

Histamine2-receptor antagonists: A. Compete with histamine for binding sites on the parietal cells. B. Irreversibly bind to H+/K+ATPase. C. Cause a decrease in stomach pH. D. Decrease signs and symptoms of allergies related to histamine release.

A. Compete with histamine for binding sites on the parietal cells Histamine receptor blocking agents decrease gastric acid by competing with histamine for binding sites on the parietal cells. H2 receptor blockers, or H2 receptor antagonists (H2RAs), are a class of gastric acid-suppressing agents frequently used in various gastric conditions. They are FDA-approved for short-term use in treating uncomplicated gastroesophageal reflux disease (GERD), gastric or duodenal ulcers, gastric hypersecretion, and mild to infrequent heartburn or indigestion. Option B: H2RAs decrease gastric acid secretion by reversibly binding to histamine H2 receptors located on gastric parietal cells, thereby inhibiting the binding and action of the endogenous ligand histamine. H2 blockers thus function as competitive antagonists. Option C: By blocking the histamine receptor and thus histamine stimulation of parietal cell acid secretion, H2RAs suppress both stimulated and basal gastric acid secretion induced by histamine. The onset of gastric relief provided by H2RAs is approximately 60 minutes with a duration of action that ranges from 4 to 10 hours, making them useful for the on-demand treatment of occasional symptoms. All H2RAs have similar efficacy in decreasing gastric acid secretion. Option D: Normally, after a meal, gastrin stimulates histamine release from enterochromaffin-like cells, which then binds to histamine H2 receptors on gastric parietal cells and leads to gastric acid release. This increase in gastric acid release occurs through the activation of adenylate cyclase, which raises intracellular cAMP levels.

Fistulas are most common with which of the following bowel disorders? A. Crohn's disease B. Diverticulitis C. Diverticulosis D. Ulcerative colitis

A. Crohn's disease The lesions of Crohn's disease are transmural; that is, they involve all thickness of the bowel. These lesions may perforate the bowel wall, forming fistulas with adjacent structures. Crohn's disease can progress from an initially mild to moderate inflammatory condition to severe penetrating (fistulization) and/or stricturing disease. The initial lesion starts out as an infiltrate around an intestinal crypt. This goes on to develop ulceration first in the superficial mucosa and involves deeper layers. Option B: Fistulas don't develop in diverticulitis. Diverticulitis is the result of microscopic and macroscopic perforations of the diverticular wall. Previously, practitioners thought that obstruction of colonic diverticulum with fecaliths led to increased pressure within the diverticulum and subsequent perforation. They now theorized that increased luminal pressure is due to food particles that lead to erosion of the diverticular wall. Option C: Diverticula occurs in weaker portions of the colonic wall where the vasa recta infiltrates the circular muscular layer. The vast majority of colonic diverticula are typically "false" diverticula, which are mucosa and submucosa herniating through a defect or weakness in the muscularis layer, covered externally only by serosa. Option D: The ulcers that occur in the submucosal and mucosal layers of the intestine in ulcerative colitis usually don't progress to fistula formation as in Crohn's disease. Colonoscopy or proctosigmoidoscopy might reveal loss of typical vascular pattern, granularity, friability, and ulceration which involve the distal rectum and proceed proximally in a symmetric, continuous, and circumferential pattern.

After a subtotal gastrectomy, the nurse should anticipate that nasogastric tube drainage will be what color for about 12 to 24 hours after surgery? A. Dark brown B. Bile green C. Bright red D. Cloudy white

A. Dark brown About 12 to 24 hours after a subtotal gastrectomy, gastric drainage is normally brown, which indicates digested blood. The aims of prophylactic drainage are to prevent repeated infection (for example by discharging remnant blood and preventing abscess formation), control possible leakage from the surgical seam (by drainage of the digestive closure, for example, a colonic anastomosis), and to provide a warning of potential complications. Option B: Bile green is not expected during the first 12 to 24 hours after subtotal gastrectomy. Bile-colored (greenish) drainage is characteristic when the tube is in the duodenum. Measure and record the amount of drainage. Dispose of measured drainage by flushing into the hopper or toilet. Option C: Drainage during the first 6 to 12 hours contains some bright red blood, but large amounts of blood or excessively bloody drainage should be reported to the physician promptly. In gastrointestinal drainage, blood varies in color-it may be dark red when fresh, dark brownish-red, or in brown particles ("coffee ground drainage") if it has been partially digested. Option D: Cloudy, pale-yellowish drainage is characteristic when the tube is in the stomach. However, this is not expected within 12 to 24 hours. Measure the contents and empty the drainage bottle at the hours ordered by the physician, when the drainage bottle is two-thirds full or when suction is discontinued.

You're preparing a patient with a malignant tumor for colorectal surgery and subsequent colostomy. The patient tells you he's anxious. What should your initial step be in working with this patient? A. Determine what the patient already knows about colostomies. B. Show the patient some pictures of colostomies. C. Arrange for someone who has a colostomy to visit the patient. D. Provide the patient with written material about colostomy care.

A. Determine what the patient already knows about colostomies. Initially, you should assess the patient's knowledge about colostomies and how it will affect his lifestyle. Review anatomy, physiology, and implications of surgical intervention. Discuss future expectations, including anticipated changes in the character of effluent. Provides knowledge base from which the patient can make informed choices, and offers an opportunity to clarify misconceptions regarding an individual situation. Option B: Include written, picture (photo, video, Internet) learning resources. This provides references for obtaining support, equipment, and additional information after discharge to support patient efforts for independence in self-care. Option C: Ascertain whether support and counseling were initiated when the possibility and/or necessity of ostomy was first discussed. This provides information about the patient's/SO's level of knowledge and anxiety about an individual situation. Option D: The patient may find it easier to accept or deal with an ostomy done to correct chronic or long-term disease than for traumatic injury, even if ostomy is only temporary. Also, patient who will be undergoing a second procedure (to convert ostomy to a continent or anal reservoir) may possibly encounter less severe self-image problems because body function eventually will be "more normal."

The nurse can expect a 60-year old patient with ischemic bowel to report a history of: A. Diabetes mellitus B. Asthma C. Addison's Disease D. Cancer of the bowel

A. Diabetes mellitus Ischemic bowel occurs in patients over 50 with a history of diabetes mellitus. Diabetes mellitus is the most common endocrine disorder affecting multiple organs including the gastrointestinal (GI) tract where manifestations and/or complications relate to disordered gut motility possibly as a result of autonomic neuropathy. Option B: Asthma is not related to an ischemic bowel. An increased prevalence of GI symptoms or complications has been documented in diabetic patients compared with nondiabetic control subjects including symptoms from both the upper and lower GI tract such as gastroparesis, anorexia, vomiting, early satiety, intestinal enteropathy, diarrhea, constipation, or fecal incontinence. Option C: Addison disease is an acquired primary adrenal insufficiency. A primary adrenal insufficiency is termed Addison disease when an autoimmune process causes the condition. It is a rare but potentially life-threatening emergency condition. It results from bilateral adrenal cortex destruction leading to decreased adrenocortical hormones, which may include cortisol, aldosterone, and androgens. Option D: Approximately 5% of patients with ischemic colitis have an obstructing lesion, usually in the distal colon. Half of these patients have colon cancer while the remainder has strictures caused by disorders such as diverticulitis, radiation, and previous surgery.

The nurse is reviewing the record of a female client with Crohn's disease. Which stool characteristics should the nurse expect to note documented in the client's record? A. Diarrhea B. Chronic constipation C. Constipation alternating with diarrhea D. Stools constantly oozing from the rectum

A. Diarrhea Crohn's disease is characterized by non-bloody diarrhea of usually not more than four to five stools daily. Over time, the diarrhea episodes increase in frequency, duration, and severity. In CD, the inflammation extends through the entire thickness of the bowel wall from the mucosa to the serosa. The disease runs a relapsing and remitting course. The other options are not associated with diarrhea. Option B: Patients with flare-ups of Crohn's disease typically present with abdominal pain (right lower quadrant), flatulence/bloating, diarrhea (can include mucus and blood), fever, weight loss, anemia. In severe cases, perianal abscess, perianal Crohn's disease, and cutaneous fistulas can be seen. Option C: When the small bowel is involved, it may present with diarrhea, malabsorption, weight loss, abdominal pain, and anorexia. Enterovesical fistulae may present with pneumaturia, recurrent urinary tract infections, and feculent vaginal discharge. Option D: Granuloma formation is very common in Crohn's disease but their absence does not exclude the diagnosis. The ongoing inflammation and scarring lead to bowel obstruction and stricture formation. Crohn's disease is also associated with enterovesical, enteroenteral, enterocutaneous and enterovaginal fistulas.

Side effects of loperamide (Imodium) include all of the following except? A. Diarrhea B. Epigastric pain C. Dry mouth D. Anorexia

A. Diarrhea Side effects associated with loperamide include CNS fatigue and dizziness, epigastric pain, abdominal cramps, nausea, dry mouth, vomiting, and anorexia. Diarrhea is an indication, not a side effect. Loperamide is a medication used in the treatment of diarrhea. It classifies as an anti-diarrheal agent. Option B: A number of the adverse events reported during the clinical investigations and post-marketing experience with loperamide are frequent symptoms of the underlying diarrheal syndrome (abdominal pain/discomfort, nausea, vomiting, dry mouth, tiredness, drowsiness, dizziness, constipation, and flatulence). Option C: Dry mouth has been reported as a side effect of prescription and over-the-counter (OTC) medications, like loperamide, to stop diarrhea. Dry mouth isn't always just thirst - other symptoms can include difficulty eating, speaking, swallowing, and bad breath, to name but a few. Option D: Bloating, loss of appetite, stomach pain, and skin rash are also rare side effects of loperamide. Anorexia is found among people who take Imodium, especially for people who are male, 60+ old, and have been taking the drug for < 1 month.

Stephen is a 62 y.o. patient that has had a liver biopsy. Which of the following groups of signs alert you to a possible pneumothorax? A. Dyspnea and reduced or absent breath sound over the right lung. B. Tachycardia, hypotension, and cool, clammy skin. C. Fever, rebound tenderness, and abdominal rigidity. D. Redness, warmth, and drainage at the biopsy site.

A. Dyspnea and reduced or absent breath sounds over the right lung Signs and symptoms of pneumothorax include dyspnea and decreased or absent breath sounds over the affected lung (right lung). A pneumothorax is defined as a collection of air outside the lung but within the pleural cavity. It occurs when air accumulates between the parietal and visceral pleura inside the chest. The air accumulation can apply pressure on the lung and make it collapse. Option B: The risk of fatal hemorrhage in patients without malignant disease is 0.04%, and the risk of nonfatal hemorrhage is 0.16%. In those with malignancy, the risk of nonfatal hemorrhage is 0.4% and 0.57% for nonfatal hemorrhage. Option C: This can occur with the inadvertent puncture of the gallbladder or in patients with obstructive jaundice and dilated bile ducts. It usually presents with abdominal pain, fever, leukocytosis. It can also be painless in some patients. Biliary scintigraphy demonstrates the leak. Treatment is usually with fluids and antibiotics. Very rarely, endoscopic procedures like ERCP or surgery may be required. Option D: This is usually clinically insignificant except in patients with obstructive jaundice like primary sclerosing cholangitis or in the post-transplant setting. Currently, there is no recommendation for treating with prophylactic antibiotics, and treatment can be offered on a case by case basis.

A client is suspected of having hepatitis. Which diagnostic test result will assist in confirming this diagnosis? A. Elevated hemoglobin level B. Elevated serum bilirubin level C. Elevated blood urea nitrogen level D. Decreased erythrocyte sedimentation rate

A. Elevated serum bilirubin level. Laboratory indicators of hepatitis include elevated liver enzyme levels, elevated serum bilirubin levels, elevated erythrocyte sedimentation rates, and leukopenia. Baseline evaluation in a patient suspected to have viral hepatitis can be started by checking a hepatic function panel. Patients who have a severe disease can have elevated total bilirubin levels. Typically, levels of alkaline phosphatase (ALP) remain in the reference range, but if it is elevated significantly, the clinician should consider biliary obstruction or liver abscess. Option A: A hemoglobin level is unrelated to this diagnosis. In advanced liver disease, prothrombin time (PT) and international normalized ratio (INR) may appear prolonged. Patients may also have leukopenia and thrombocytopenia. Patients who suffer from easy bruising, variceal bleed, or hemorrhoidal bleed due to advanced liver disease may have anemia with low hemoglobin and hematocrit levels. Option C: An elevated blood urea nitrogen level may indicate renal dysfunction. Blood urea nitrogen (BUN) and serum creatinine levels are also necessary for patients suspected to have advanced liver disease to look for renal impairment. Patients who present with altered mental status should have serum ammonia levels checked and are usually elevated in the presence of hepatic encephalopathy. Option D: Elevated erythrocyte sedimentation rate is a laboratory indicator of hepatitis. The increase in the ESR in type A hepatitis could be explained by changes in the serum protein levels in the course of acute viral hepatitis and/or by the different inflammatory activity of the underlying disease.

You're caring for Jane, a 57 y.o. patient with liver cirrhosis who developed ascites and requires paracentesis. Before her paracentesis, you instruct her to: A. Empty her bladder. B. Lie supine in bed. C. Remain NPO for 4 hours. D. Clean her bowels with an enema.

A. Empty her bladder. A full bladder can interfere with paracentesis and be punctured inadvertently. The preferred site for the procedure is in either the lower quadrant of the abdomen lateral to the rectus sheath. Placing the patient in the lateral decubitus position can aid in identifying fluid pockets in patients with lower fluid volumes. Ask the patient to empty his or her bladder before starting the procedure. Option B: Placing the patient in the lateral decubitus position can aid in identifying fluid pockets in patients with lower fluid volumes. Paracentesis is done in a lateral decubitus or supine position. The ascites fluid level is percussed, and a needle is inserted either in the midline or lateral lower quadrant (lateral to rectus abdominis muscle, 2 cm to 4 cm superomedial to anterior superior iliac spine). This positioning avoids puncture of the inferior epigastric arteries. Option C: NPO is not necessary for the procedure. There are few absolute contraindications for paracentesis. Coagulopathy and thrombocytopenia (both very common in cirrhotic patients) are themselves not absolute contraindications as the incidence of bleeding complications from the procedure has been shown to be very low. Option D: An enema is not necessary for the procedure. A bedside ultrasound should be used to identify an appropriate location for the procedure. Ultrasound can confirm the presence of fluid and identify an area with a sufficient amount of fluid for aspiration, thereby decreasing the incidence of both unsuccessful aspiration and complications.

The MD orders a paracentesis for pt with ascites. Before the procedure, RN should instruct pt to: A. empty the bladder B. eat foods low in fat C. remain NPO for 24 hrs D. assume the supine position

A. Empty the bladder

You're caring for a 28 y.o. woman with hepatitis B. She's concerned about the duration of her recovery. Which response isn't appropriate? A. Encourage her to not worry about the future. B. Encourage her to express her feelings about the illness. C. Discuss the effects of hepatitis B on future health problems. D. Provide avenues for financial counseling if she expresses the need.

A. Encourage her to not worry about the future. Telling her not to worry minimizes her feelings. Contract with the patient regarding time for listening. Encourage discussion of feelings/concerns. Establishing time enhances trusting relationships. Providing an opportunity to express feelings allows the patient to feel more in control of the situation. Verbalization can decrease anxiety and depression and facilitate positive coping behaviors. Option B: The patient may need to express feelings about being ill, length and cost of illness, possibility of infecting others, and (in severe illness) fear of death. She may have concerns regarding the stigma of the disease. The recovery period may be prolonged (up to 6 mo), potentiating family and/or situational stress and necessitating the need for planning, support, and follow-up. Option C: Avoid making moral judgments regarding lifestyle. The patient may already feel upset and angry and condemn self; judgments from others will further damage self-esteem. This can also start distrust issues with care workers. Option D: Assess the effect of illness on economic factors of the patient and SO. Financial problems may exist because of loss of the patient's role functioning in the family and prolonged recovery. Make appropriate referrals for help as needed: case manager, discharge planner, social services, and/or other community agencies.

Which diagnostic test would be used first to evaluate a client with upper GI bleeding? A. Endoscopy B. Upper GI series C. Hemoglobin (Hb) levels and hematocrit (HCT) D. Arteriography

A. Endoscopy Endoscopy permits direct evaluation of the upper GI tract and can detect 90% of bleeding lesions. Endoscopy is the insertion of a long, thin tube directly into the body to observe an internal organ or tissue in detail. It can also be used to carry out other tasks including imaging and minor surgery. Endoscopes are minimally invasive and can be inserted into the openings of the body such as the mouth or anus. An upper GI series, or barium study, usually isn't the diagnostic method of choice, especially in a client with acute active bleeding who's vomiting and unstable. Option B: An upper GI series is also less accurate than endoscopy. Although an upper GI series might confirm the presence of a lesion, it wouldn't necessarily reveal whether the lesion is bleeding. An upper gastrointestinal series (UGI) is a radiographic (X-ray) examination of the upper gastrointestinal (GI) tract. The esophagus, stomach, and duodenum (first part of the small intestine) are made visible on X-ray film by a liquid suspension. This liquid suspension may be barium or a water-soluble contrast. Option C: Hb levels and HCT, which indicate loss of blood volume, aren't always reliable indicators of GI bleeding because a decrease in these values may not be seen for several hours. Upper GI bleeding is sometimes fatal, and the European Society of Gastrointestinal Endoscopy recommends maintaining Hb levels between 7.0-9.0 g/dl using blood transfusion in order to reduce mortality. Option D: Arteriography is an invasive study associated with life-threatening complications and wouldn't be used for an initial evaluation. An arteriogram is an imaging test that uses x-rays and a special dye to see inside the arteries. It can be used to view arteries in the heart, brain, kidney, and other parts of the body.

The client being seen in a physician's office has just been scheduled for a barium swallow the next day. The nurse writes down which of the following instructions for the client to follow before the test? A. Fast for 8 hours before the test. B. Eat a regular supper and breakfast. C. Continue to take all oral medications as scheduled. D. Monitor own bowel movement pattern for constipation.

A. Fast for 8 hours before the test A barium swallow is an x-ray study that uses a substance called barium for contrast to highlight abnormalities in the GI tract. The client should fast for 8 to 12 hours before the test, depending on the physician's instructions. A barium swallow test (cine esophagram, swallowing study, esophagography, modified barium swallow study, videofluoroscopy swallow study) is a special type of imaging test that uses barium and X-rays to create images of the upper gastrointestinal (GI) tract. Option B: The client will need to stop eating and drinking for about 8 hours before the swallowing test. Generally, this means after midnight. A barium swallow test may be performed as an outpatient procedure or as part of the client's stay in a hospital. The way the test is done may vary depending on the client's condition and his healthcare provider's practices. Option C: Most oral medications also are withheld before the test. The client should tell the provider about all medicines he is taking. This includes prescriptions, over-the-counter medicines, and herbal supplements. He may need to stop taking these before the swallowing test. Option D: After the procedure, the nurse must monitor for constipation, which can occur as a result of the presence of barium in the GI tract. The client may have constipation or impacted stool after the swallowing test if all of the barium does not pass out of the body. The client should drink plenty of fluids in the days following the exam.

A female client being seen in a physician's office has just been scheduled for a barium swallow the next day. The nurse writes down which instruction for the client to follow before the test? A. Fast for 8 hours before the test B. Eat a regular supper and breakfast C. Continue to take all oral medications as scheduled D. Monitor own bowel movement pattern for constipation

A. Fast for 8 hours before the test. A barium swallow is an x-ray study that uses a substance called barium for contrast to highlight abnormalities in the gastrointestinal tract. The client should fast for 8 to 12 hours before the test, depending on physician instructions. The barium swallow study, also known as a barium esophagogram or esophagram, is a contrast-enhanced radiographic study commonly used to assess structural characteristics of the entire esophagus. Option B: Patients must be capable of swallowing relatively large amounts of contrast without assistance and be able to protect their airways. For visual studies focusing on the pharynx and esophagus, minimal preparation is required. However, patients should be able to tolerate swallowing liquids. Option C: Most oral medications also are withheld before the test. Oral barium contrast has relatively few adverse effects in standard practice. Most commonly, patients complain of nausea and vomiting within 30 minutes of ingestion. Hypersensitivity reactions have been reported but are uncommon. Most adverse effects are related to the extravasation of contrast into the mediastinum or from aspiration. Option D: After the procedure, the nurse must monitor for constipation, which can occur as a result of the presence of barium in the gastrointestinal tract. The barium esophagogram is noninvasive and readily performed, requiring only radiographic still-image capability and contrast medium. As such, it is a useful exam despite the current wide availability of CT imaging.

The hospital administrator had undergone percutaneous transhepatic cholangiography. Which assessment finding indicates complication after the operation? A. Fever and chills B. Hypertension C. Bradycardia D. Nausea and diarrhea

A. Fever and chills Septicemia is a common complication after a percutaneous transhepatic cholangiography. Evidence of fever and chills, possibly indicative of septicemia, is important. Although PTC may be performed to treat the obstruction that is the cause of sepsis, PTC itself may also cause sepsis. Antibiotics, IV fluids, oxygen, and vasopressors in the setting of an intensive care unit should be considered. Option B: Hypotension, not hypertension, is associated with septicemia. The Society of Interventional Radiology (SIR) has published complication rates for PTC and PBD. The rate of major complications is around 2% to 10%. Major complications include inducing sepsis, other severe infections (such as an abscess), bile leak/biloma, hemorrhage (subcapsular hematoma, pseudoaneurysm), pneumothorax, and death. Option C: Tachycardia, not bradycardia, is most likely to occur. Transgression of blood vessels during PTC is to be expected. Coagulation usually occurs successfully, and hemorrhage ceases entirely within 2 to 3 days. Bleeding through the catheter can occur if a catheter side hole is left in communication with a hepatic vessel or if a pseudoaneurysm develops. Option D: Nausea and diarrhea may occur but are not classic signs of sepsis. If electrolyte depletion occurs due to high-output external drainage, then the electrolytes should be replaced and considerations should be made on converting the catheter to internal drainage as soon as possible.

46. Which of the following aspects is the priority focus of nursing management for a client with peritonitis? A. Fluid and electrolyte balance B. Gastric irrigation C. Pain management D. Psychosocial issues

A. Fluid and electrolyte balance Peritonitis can advance to shock and circulatory failure, so fluid and electrolyte balance is the priority focus of nursing management. Maintain accurate I&O and correlate with daily weights. Include measured losses. Include measurements from gastric suction, drains, dressings, Hemovacs, diaphoresis, and abdominal girth for third spacing of fluid. Option B: Gastric irrigation may be needed periodically to ensure patency of the nasogastric tube. Nasogastric tubes used for suction or drainage will be irrigated at least every 2 hours and when needed (PRN) as per patient reports of abdominal discomfort, nausea or vomiting, leaking from the tube, gastric distention, or gastric distress. Option C: Although pain management is important for comfort, focusing on fluid and electrolyte imbalance will maintain hemodynamic stability. Maintain semi-Fowler's position as indicated; facilitates fluid or wound drainage by gravity, reducing diaphragmatic irritation and/or abdominal tension, and thereby reducing pain. Option D: Although psychosocial care will address concerns such as anxiety, focusing on fluid and electrolyte imbalance will maintain hemodynamic stability. Evaluate anxiety level, noting patient's verbal and nonverbal response. Encourage free expression of emotions. The apprehension may be escalated by severe pain, increasingly ill-feeling, the urgency of diagnostic procedures, and the possibility of surgery.

You're caring for a patient with a sigmoid colostomy. The stool from this colostomy is: A. Formed B. Semisolid C. Semiliquid D. Watery

A. Formed A colostomy in the sigmoid colon produces a solid, formed stool. This is the most common type. It is located in the bottom part of the large intestine. The sigmoid colon moves waste to the rectum. Sigmoid colostomies produce stool that is more solid and regular than other colostomies. Option B: The transverse colon crosses the top of the abdomen. Stool in this area is usually soft. This is because only a small portion of the colon has absorbed water from the indigestible material. This common type of colostomy has 3 versions. Option C: A double-barrel colostomy divides the colon into 2 ends that form separate stomas. Stool exits from 1 of the stomas. Mucus made by the colon exits from the other. This type of transverse colostomy is the least common. A loop colostomy creates a stoma through which stool exits. In this type, the colon stays connected to the rectum. As a result, people will sometimes pass stool or gas through the rectum. Option D: The ascending colon runs from the beginning of the large intestine to the right side of the abdomen. In this procedure, only part of the colon still works. As a result, little water is absorbed from the waste. This means the stool is usually liquid. This type of colostomy is rare. An ileostomy is more appropriate for this portion of the colon. You're advising a 21 y.o. with a colostomy who reports problems with flatus. What food should you recommend? A. Peas B. Cabbage C. Broccoli D. Yogurt D. Yogurt High-fiber foods stimulate peristalsis, and as a result, flatus. Yogurt reduces gas formation. Eating bland foods will help avoid uncomfortable symptoms such as diarrhea (loose or watery bowel movements), bloating, and gas. Bland foods are cooked, easy-to-digest foods that aren't spicy, heavy, or fried. Option A: Excessive gas, or flatulence, can be caused by foods, such as beans and cabbage, that have high amounts of fiber and carbohydrates which tend to ferment a lot during digestion and in turn have a greater tendency to cause bad-smelling flatulence. Option B: Well-cooked vegetables without skins or seeds, such as peeled potatoes, peeled zucchini with the seeds removed, and peeled tomatoes with the seeds removed should be included in the diet. Some vegetables, like cabbage, may cause gas or odor for some people. Option C: Include lettuce instead of broccoli in the diet. For the first few weeks after surgery, it's normal to have gas in the pouch and odor when the pouch is opened. Buttermilk, cranberry juice, parsley, and kefir help prevent gas, odor, or both.

Elmer is scheduled for a proctoscopy and has an I.V. The doctor wrote an order for 5mg of I.V. diazepam(Valium). Which order is correct regarding diazepam? A. Give diazepam in the I.V. port closest to the vein. B. Mix diazepam with 50 ml of dextrose 5% in water and give it over 15 minutes. C. Give diazepam rapidly I.V. to prevent the bloodstream from diluting the drug mixture. D. the order because I.V. administration of diazepam is contraindicated.

A. Give diazepam in the I.V. port closest to the vein. Diazepam is absorbed by the plastic I.V. tubing and should be given in the port closest to the vein. Diazepam is a benzodiazepine medication that is FDA approved for the management of anxiety disorders, short-term relief of anxiety symptoms, spasticity associated with upper motor neuron disorders, adjunct therapy for muscle spasms, preoperative anxiety relief, management of certain refractory epilepsy patients and adjunct in severe recurrent convulsive seizures, and an adjunct in status epilepticus. Option B: Diazepam is available in multiple formulations, including oral tablets, intramuscular injections (IM), intravenous injection (IV), or rectal gel. Of note, oral tablets have a more reliable absorption and controlled release when compared to IM. Option C: Diazepam is highly lipophilic. While there is a moderately quick onset of action, the drug quickly redistributes. Once in the body, diazepam is mostly broken down by the CYP2C19 and CYP3A4 enzymes to several active metabolites, mainly desmethyldiazepam. Other minor active metabolites include oxazepam and temazepam. The average half-lives of oral diazepam and desmethyldiazepam are about 46 hours and 100 hours, respectively. Option D: When administered intravenously, diazepam actS within 1 to 3 minutes, while oral dosing onset ranges between 15 to 60 minutes. Diazepam is long-lasting with a duration of action of more than 12 hours.

Nathaniel has severe pruritus due to having hepatitis B. What is the best intervention for his comfort? A. Give tepid baths. B. Avoid lotions and creams. C. Use hot water to increase vasodilation. D. Use cold water to decrease the itching.

A. Give tepid baths. For pruritus, care should include tepid sponge baths and use of emollient creams and lotions. Bathe or shower using lukewarm water and mild soap or nonsoap cleansers. Long bathing or showering in hot water causes drying of the skin and can aggravate itching through vasodilation. Option B: Lubrication with fragrance-free creams or ointments serves as a barrier to prevent further drying of the skin through evaporation. Moisturizing is the cornerstone of treatment. Over-the-counter moisturizing lotions include Eucerin, Lubriderm, and Nivea. Lotions are lighter and less emollient than creams. Option C: Encourage the patient to adopt skincare routines to decrease skin irritation. After bathing, allow the skin to air dry or gently pat the skin dry. Avoid rubbing or brisk drying. Rubbing the skin with a towel can irritate the skin and exacerbate the itch-scratch cycle. Option D: If more moisturizing is required than a lotion can provide, a cream is recommended. These include Keri cream, Cetaphil cream, Eucerin cream, and Neutrogena Norwegian formula. Ointments are the most emollient. Vaseline Pure Petroleum Jelly or Aquaphor Natural Healing Ointment may be beneficial.

Dr. Smith has determined that the client with hepatitis has contracted the infection from contaminated food. The nurse understands that this client is most likely experiencing what type of hepatitis? A. Hepatitis A B. Hepatitis B C. Hepatitis C D. Hepatitis D

A. Hepatitis A Hepatitis A is transmitted by the fecal-oral route via contaminated food or infected food handlers. The most common mode of transmission of hepatitis A is via the fecal-oral route from contact with food, water, or objects contaminated by fecal matter from an infected individual. It is more commonly encountered in developing countries where due to poverty and lack of sanitation, there is a higher chance of fecal-oral spread. Option B: Hepatitis B is transmitted parenterally and sexually when individuals come in contact with mucous membranes or body fluids of infected individuals. Transfusion of blood and blood products, injection drug use with shared needles, needlesticks, or wounds caused by other instruments in healthcare workers and hemodialysis are all examples of parenteral and percutaneous exposures, but parenteral mode remains the dominant mode of transmission both globally and in the United States. Option C: Transmission of Hepatitis C can be parenteral, perinatal, and sexual, with the most common mode being the sharing of contaminated needles among IV drug users. Also, other high-risk groups include people who require frequent blood transfusions and organ transplantation of organs from infected donors. Option D: Hepatitis D is an RNA virus and a single species in the Deltavirus genus. It contains the hepatitis D antigen and RNA strand and uses HBsAg as its envelope protein; therefore, those who get hepatitis D virus infection have coinfection with the hepatitis B virus as well. Hepatitis D virus has similar modes of transmission as the hepatitis B virus, but perinatal transmission is uncommon.

Nurse Oliver checks for residual before administering a bolus tube feeding to a client with a nasogastric tube and obtains a residual amount of 150 mL. What is the appropriate action for the nurse to take? A. Hold the feeding B. Reinstill the amount and continue with administering the feeding C. Elevate the client's head at least 45 degrees and administer the feeding D. Discard the residual amount and proceed with administering the feeding

A. Hold the feeding Unless specifically indicated, residual amounts more than 100 mL require holding the feeding. Gastric residual refers to the volume of fluid remaining in the stomach at a point in time during enteral nutrition feeding. Nurses withdraw this fluid via the feeding tube by pulling back on the plunger of a large (usually 60 mL) syringe at intervals typically ranging from four to eight hours. Option B: When interpreting GRV, clinicians must keep in mind that the stomach has a reservoir function and that the stomach fluid is a mixture of both the infused EN formula and normal gastric secretions. Option C: Patients at risk for delayed gastric emptying include those with gastroparesis, poorly controlled diabetes mellitus, gastric outlet obstruction, ileus, recent surgery, trauma, or sepsis, and those using a large amount of narcotic pain medication. Efforts to prevent aspiration of gastric contents are important in these patients. Option D: The feeding is not discarded unless its contents are abnormal in color or characteristics. In a review article, "Measurement of Gastric Residual Volume: State of the Science," published in 2000 in MEDSURG Nursing, Edwards and Metheny reported that the literature contained a variety of recommendations for what is considered a high GRV, ranging from 100 to 500 mL.

A patient with chronic alcohol abuse is admitted with liver failure. You closely monitor the patient's blood pressure because of which change that is associated with liver failure? A. Hypoalbuminemia B. Increased capillary permeability C. Abnormal peripheral vasodilation D. Excess renin release from the kidneys

A. Hypoalbuminemia Blood pressure decreases as the body is unable to maintain normal oncotic pressure with liver failure, so patients with liver failure require close blood pressure monitoring. Increased capillary permeability, abnormal peripheral vasodilation, and excess renin released from the kidneys aren't direct ramifications of liver failure. Option B: Once ascites is present, most therapeutic modalities are directed on maintaining negative sodium balance, including salt restriction, bed rest, and diuretics. Paracentesis and albumin infusion is applied to tense ascites. Transjugular intrahepatic portosystemic shunt is considered for refractory ascites. With worsening of liver disease, fluid retention is associated with other complications; such as spontaneous bacterial peritonitis. Option C: Hepatorenal syndrome is a state of functional renal failure in the setting of low cardiac output and impaired renal perfusion. Its management is based on drugs that restore normal renal blood flow through peripheral arterial and splanchnic vasoconstriction, renal vasodilation, and/or plasma volume expansion. However, the definitive treatment is liver transplantation. Option D: The most acceptable theory for ascites formation is peripheral arterial vasodilation leading to underfilling of circulatory volume. This triggers the baroreceptor-mediated activation of the renin-angiotensin-aldosterone system, sympathetic nervous system, and nonosmotic release of vasopressin to restore circulatory integrity. The result is an avid sodium and water retention, identified as a pre ascitic state. This condition will evolve in overt fluid retention and ascites, as the liver disease progresses.

Which stoma would you expect a malodorous, enzyme-rich, caustic liquid output that is yellow, green, or brown? A. Ileostomy B. Ascending colostomy C. Transverse colostomy D. Descending colostomy

A. Ileostomy The output from an Ileostomy is described. The consistency of the ileostomy output will be liquid to pasty, depending on one's diet, medications, and other factors. Because the output is constant, the pouch will need to be emptied 5-8 times a day. Average ileostomy output ranges from 800 - 1,200 milliliters (mL) or 3 - 5 cups per day. Right after surgery, output may be watery. During the first few weeks after surgery, the output should thicken to the consistency of applesauce. Option B: Ascending colostomy is made from the ascending part of the colon. The ascending colostomy is usually located in the low to the middle right side of the abdomen. The output is often liquid to semiliquid, and gas is common. Option C: Transverse colostomy is made from the transverse part of the colon. The transverse colostomy is usually located in the center of the abdomen above the navel. The output often is liquid to pasty, and gas is common. Option D: Descending colostomy is made from the descending part of the colon. The descending colostomy is typically located on the lower left-hand side of the abdomen. The output may be pasty to a formed consistency, and gas is common.

The nurse is teaching the client how to perform a colostomy irrigation. To enhance the effectiveness of the irrigation and fecal returns, what measure should the nurse instruct the client to do? A. Increase fluid intake. B. Reduce the amount of irrigation solution. C. Perform the irrigation in the evening. D. Place heat on the abdomen.

A. Increase fluid intake. To enhance the effectiveness of the irrigation and fecal returns, the client is instructed to increase fluid intake and prevent constipation. A colostomy may make the client more prone to constipation or diarrhea. It's important to get enough fiber in the diet and drink plenty of water to prevent these problems. Some people experience a small amount of stool leakage between irrigations. Option B: Fill the irrigating container with about 16 to 50 ounces (500 to 1500 mL) of lukewarm water. The water should not be cold or hot. Ask how much water will be needed to irrigate. Hang the irrigation container so that it is level with the shoulder. Option C: It is best to perform irrigation at the same time each day. The client may want to try irrigating at the time of day he typically had a bowel movement (before getting the colostomy). Irrigation may be easier after a meal or hot drink. Option D: Abdominal pain or nausea may occur during irrigation, and may mean that water flow is too fast or the water is too cold. In six to eight weeks, the bowels will typically adjust, and bowel movements will become regular.

The nurse is teaching a female client how to perform a colostomy irrigation. To enhance the effectiveness of the irrigation and fecal returns, what measure should the nurse instruct the client to do? A. Increase fluid intake B. Place heat on the abdomen C. Perform the irrigation in the evening D. Reduce the amount of irrigation solution

A. Increase fluid intake. To enhance the effectiveness of the irrigation and fecal returns, the client is instructed to increase fluid intake and to take other measures to prevent constipation. A colostomy may make the client more prone to constipation or diarrhea. It's important to get enough fiber in the diet and drink plenty of water to prevent these problems. Some people experience a small amount of stool leakage between irrigations. Option B: Abdominal pain or nausea may occur during irrigation, and may mean that water flow is too fast or the water is too cold. In six to eight weeks, the bowels will typically adjust, and bowel movements will become regular. Option C: It is best to perform irrigation at the same time each day. The client may want to try irrigating at the time of day he typically had a bowel movement (before getting the colostomy). Irrigation may be easier after a meal or hot drink. Option D: Fill the irrigating container with about 16 to 50 ounces (500 to 1500 mL) of lukewarm water. The water should not be cold or hot. Ask how much water will be needed to irrigate. Hang the irrigation container so that it is level with the shoulder.

A male client is recovering from an ileostomy that was performed to treat inflammatory bowel disease. During discharge teaching, the nurse should stress the importance of: A. Increasing fluid intake to prevent dehydration. B. Wearing an appliance pouch only at bedtime. C. Consuming a low-protein, high-fiber diet. D. Taking only enteric-coated medications.

A. Increasing fluid intake to prevent dehydration. Because stool forms in the large intestine, an ileostomy typically drain liquid waste. To avoid fluid loss through ileostomy drainage, the nurse should instruct the client to increase fluid intake. Monitor I&O. Note number, character, and amount of stools; estimate insensible fluid losses (diaphoresis). Measure urine specific gravity; observe for oliguria. Provides information about overall fluid balance, renal function, and bowel disease control, as well as guidelines for fluid replacement. Option B: The nurse should teach the client to wear a collection appliance at all times because ileostomy drainage is incontinent. Resume or advance diet as indicated (clear liquids progressing to bland, low residue; then high-protein, high-calorie, caffeine-free, non-spicy, and low-fiber as indicated). Option C: The nurse should teach the client to avoid high-fiber foods because they may irritate the intestines. Avoid or limit foods that might cause or exacerbate abdominal cramping, flatulence (milk products, foods high in fiber or fat, alcohol, caffeinated beverages, chocolate, peppermint, tomatoes, orange juice). Option D: The nurse should teach the client to avoid enteric-coated medications because the body can't absorb them after an ileostomy. Allows the intestinal tract to readjust to the digestive process. Protein is necessary for tissue healing integrity. Low bulk decreases peristaltic response to meals.

A patient has become very depressed postoperatively after receiving a colostomy for GI cancer. He does not participate in his colostomy care or looks at the stoma. An appropriate nursing diagnosis for this situation is: A. Ineffective Individual Coping B. Knowledge Deficit C. Impaired Adjustment D. Anxiety

A. Ineffective Individual Coping The patient is dealing with a disturbance in self-concept and difficulty coping with the newly established stoma. Encourage the patient/SO to verbalize feelings regarding the ostomy. Acknowledge normality of feelings of anger, depression and grief over a loss. Discuss daily "ups and downs" that can occur. Option B: Provide opportunities for patient/SO to view and touch stoma, using the moment to point out positive signs of healing, normal appearance, and so forth. Remind the patient that it will take time to adjust, both physically and emotionally. Option C: Although integration of stoma into body image can take months or even years, looking at the stoma and hearing comments (made in a normal, matter-of-fact manner) can help the patient with this acceptance. Touching stoma reassures patient/SO that it is not fragile and that slight movements of stoma actually reflect normal peristalsis. Option D: Maintain a positive approach during care activities, avoiding expressions of disdain or revulsion. Do not take angry expressions of the patient and SO personally. A person who is living with an ostomy can be a good support system/role model. Helps reinforce teaching (shared experiences) and facilitates acceptance of change as the patient realizes "life does go on" and can be relatively normal.

The client has orders for a nasogastric (NG) tube insertion. During the procedure, instructions that will assist in the insertion would be: A. Instruct the client to tilt his head back for insertion in the nostril, then flex his neck for the final insertion. B. After insertion into the nostril, instruct the client to extend his neck. C. Introduce the tube with the client's head tilted back, then instruct him to keep his head upright for final insertion. D. Instruct the client to hold his chin down, then back for insertion of the tube.

A. Instruct the client to tilt his head back for insertion in the nostril, then flex his neck for the final insertion. NG insertion technique is to have the client first tilt his head back for insertion into the nostril, then to flex his neck forward and swallow. A common error when placing the tube is to direct the tube in an upward direction as it enters the nares; this will cause the tube to push against the top of the sinus cavity and cause increased discomfort. The tip should instead be directed parallel to the floor, directly toward the back of the patient's throat. Option B: Extension of the neck will impede NG tube insertion. The patient can be given a cup of water with a straw in it to sip from to help ease the passage of the tube. The tube should be advanced with firm, constant pressure while the patient is sipping. If there is a great deal of difficulty in passing the tube, a helpful maneuver is to withdraw the tube and attempt again after a short break in the contralateral nares as the tube may have become coiled in the oropharynx or nasal sinus. Option C: In intubated patients, the use of reverse Sellick's maneuver (pulling the thyroid cartilage up rather than pushing it down during intubation) and freezing the NG tube may help facilitate placement of the tube. Once the tube has been inserted an appropriate length, typically around 55 cm as previously noted, it should be secured to the patient's nose with tape. Option D: Nasogastric tubes are, as one might surmise from their name, tubes that are inserted through the nares to pass through the posterior oropharynx, down the esophagus, and into the stomach. The most common complications related to the placement of nasogastric tubes are discomfort, sinusitis, or epistaxis, all of which typically resolve spontaneously with the removal of the nasogastric tube.

While a female client is being prepared for discharge, the nasogastric (NG) feeding tube becomes clogged. To remedy this problem and teach the client's family how to deal with it at home, what should the nurse do? A. Irrigate the tube with warm water. B. Advance the tube into the intestine. C. Apply intermittent suction to the tube. D. Withdraw the obstruction with a 30-ml syringe.

A. Irrigate the tube with warm water. The American Society for Parenteral and Enteral Nutrition (ASPEN) recommends warm water as the best initial choice for trying to unclog a feeding tube. First, attach a 30- or 60-mL piston syringe to the feeding tube and pull back the plunger to help dislodge the clog. Next, fill the flush syringe with warm water, reattach it to the tube, and attempt a flush. Option B: Advancing the NG tube is inappropriate because the tube is designed to stay in the stomach and isn't long enough to reach the intestines. If there is still continued resistance, gently move the syringe plunger back and forth to help loosen the clog. Then, clamp the tube to allow the warm water to penetrate the clog for up to 20 minutes. Option C: Applying intermittent suction or using a syringe for aspiration is unlikely to dislodge the material clogging the tube but may create excess pressure.If the patient's feeding tube becomes clogged, attempt to unclog it before replacing it, which is both costly and uncomfortable for the patient. The belief that carbonated beverages or cranberry juice will unclog a feeding tube is a persistent nursing myth. In fact, these beverages have an acidic pH that can worsen the occlusion by causing proteins in the EN formula to precipitate within the tube. Option D: Intermittent suction may even collapse the tube. Consistently flushing feeding tubes with water as scheduled during EN therapy and medication administration is the best way to minimize the risk of occlusions.

The nurse is caring for a client following a Billroth II procedure. On review of the post-operative orders, which of the following, if prescribed, would the nurse and verify? A. Irrigating the nasogastric tube. B. Coughing and deep breathing exercises. C. Leg exercises D. Early ambulation

A. Irrigating the nasogastric tube. In a Billroth II procedure, the proximal remnant of the stomach is anastomosed to the proximal jejunum. Patency of the NG tube is critical for preventing the retention of gastric secretions. The nurse should never irrigate or reposition the gastric tube after gastric surgery unless specifically ordered by the physician. In this situation, the nurse would clarify the order. Option B: The client may do coughing and deep breathing exercises. Discuss and identify stressful situations and how to avoid them. Investigate job-related issues. This can alter gastric motility, interfering with optimal digestion. The patient may require vocational counseling if a change in employment is indicated. Option C: Leg exercises are allowed to prevent thrombosis. Discuss the importance of eating small, frequent meals slowly and in a relaxed atmosphere; resting after meals; avoiding extremely hot or cold food; restricting high-fiber foods, caffeine, milk products, and alcohol, excess sugars and salt; and taking fluids between meals, rather than with food. Option D: Early ambulation is recommended after the procedure. Also, discuss the reasons for and importance of cessation of smoking. Smoking stimulates gastric acid production and may cause vasoconstriction, compromising mucous membranes and increasing the risk of gastric irritation and ulceration.

A clinical manifestation of acute pancreatitis is epigastric pain. Your nursing intervention to facilitate relief of pain would place the patient in a: A. Knee-chest position B. Semi-Fowler's position C. Recumbent position D. Low-Fowler's position

A. Knee-chest position Flexion of the trunk lessens the pain and decreases restlessness. Promote position of comfort on one side with knees flexed, sitting up, and leaning forward. Reduces abdominal pressure and tension, providing some measure of comfort and pain relief. Other positions do not decrease the pain. Option B: The Semi-Fowler's position is a position in which a patient, typically in a hospital or nursing home is positioned on their back with the head and trunk raised to between 15 and 45 degrees, although 30 degrees is the most frequently used bed angle. Option C: The word "lateral" means "to the side," while "recumbent" means "lying down." In the right or left lateral recumbent position, the individual is lying on their right or left side. This position makes it easier to access a patient's side. Option D: Supine position often increases pain. The Semi-Fowler's position is often used for purposes similar to those of the regular Fowler's position, including feeding and lung expansion, cardiac or respiratory conditions, and for patients with a nasogastric tube.

You observe changes in mentation, irritability, restlessness, and decreased concentration in a patient with cancer of the liver. Hepatic encephalopathy is suspected and the patient is ordered neomycin enemas. Which of the following information in the patient's history would be a contraindication of this order? A. Left nephrectomy B. Glaucoma in both eyes C. Myocardial infarction D. Peripheral neuropathy

A. Left nephrectomy Neomycin prevents the release of ammonia from the intestinal bacteria flora and from the breakdown of red blood cells. Common side-effects of this drug are nephrotoxicity and ototoxicity. Patients with renal disease or renal impairment should not take this drug. A baseline serum BUN/creatinine should be obtained with subsequent periodical follow-up blood tests during chronic therapy to monitor for effects on renal function. Prompt discontinuation of the drug should occur with any signs of renal or otologic damage. Option B: Glaucoma is not affected by neomycin. Neomycin belongs to a group of antibiotics known as aminoglycosides. Like others in the aminoglycoside family, neomycin works by inhibiting bacterial protein synthesis leading to its bactericidal effect. This group of medications is particularly effective in killing gram-negative organisms allowing for good coverage of enteric organisms. Option C: MI is not affected by neomycin. Neomycin's mechanism of action is very similar to most aminoglycosides as it binds to the 30s ribosomal subunit interfering with bacterial protein synthesis. This action thereby disrupts the process of translation by bacteria leading to the medication's bactericidal effects. Option D: Peripheral neuropathy is a chronic complication of diabetes mellitus. Neomycin's bactericidal effects decrease ammonia-producing bacteria residing in the GI tract, thereby decreasing the burden of ammonia on the patient. Neomycin is often reserved for patients that cannot tolerate rifaximin.

Which of the following types of diets is implicated in the development of diverticulosis? A. Low-fiber diet B. High-fiber diet C. High-protein diet D. Low-carbohydrate diet

A. Low-fiber diet Low-fiber diets have been implicated in the development of diverticula because these diets decrease the bulk in the stool and predispose the person to the development of constipation. A low-fiber diet is linked to causing diverticulosis and also linked to the onset of diverticulitis. The goal will be to cut back on the low fiber foods and increase the high fiber ones. Option B: A high-fiber diet is recommended to help prevent diverticulosis. High fiber foods are the ones with whole wheat, oat, bran, whole-grain cereal, vegetables, fruit, and legumes. The dietary treatment for diverticulosis is also one of the primary ways to prevent it. Option C: A high-protein diet has no effect on the development of diverticulosis. Fiber softens and adds bulk to stools, helping them pass more easily through the colon. It also reduces pressure in the digestive tract. Option D: A low-carbohydrate diet has no effect on the development of diverticulosis. Many studies show that eating fiber-rich foods can help control diverticular symptoms. Women younger than 51 should aim for 25 grams of fiber daily. Men younger than 51 should aim for 38 grams of fiber daily. Women 51 and older should get 21 grams daily. Men 51 and older should get 30 grams daily.

Which of the following diets is most commonly associated with colon cancer? A. Low-fiber, high fat B. Low-fat, high-fiber C. Low-protein, high-carbohydrate D. Low carbohydrate, high protein

A. Low-fiber, high fat A low-fiber, high-fat diet reduces motility and increases the chance of constipation. The metabolic end products of this type of diet are carcinogenic. A high-fat, low-fiber diet is implicated in the development of colorectal cancer. Specifically, people who ingest a diet high in saturated animal fats and highly saturated vegetable oils (eg, corn, safflower) have a higher incidence of colorectal cancer. Option B: A low-fat, high-fiber diet is recommended to prevent colon cancer. The ingestion of a high-fiber diet may be protective against colorectal cancer. Fiber causes the formation of a soft, bulky stool that dilutes carcinogens; it also decreases colonic transit time, allowing less time for harmful substances to contact the mucosa. The decreased incidence of colorectal cancer in Africans is attributed to their high-fiber, low-animal-fat diet. Option C: Saturated fats from dairy products do not have the same carcinogenic effect, nor do oils containing oleic acid (eg, olive, coconut, fish oils). Omega-3 monounsaturated fatty acids and omega-6 monounsaturated fatty acids also appear to be less carcinogenic than unsaturated or polyunsaturated fats. In fact, epidemiologic data suggest that high fish consumption may provide a protective effect against the development of colorectal cancer. Long-term diets high in red meat or processed meats appear to increase the risk of distal colon and rectal cancers. Option D: Increased dietary intake of calcium appears to have a protective effect on colorectal mucosa by binding with bile acids and fatty acids. The resulting calcium salts may have antiproliferative effects, decreasing crypt cell production in the mucosa. A double-blind placebo-controlled study showed a statistically significant reduction in the incidence of metachronous colorectal adenomas.

The client with Crohn's disease has a nursing diagnosis of acute pain. The nurse would teach the client to avoid which of the following in managing this problem? A. Lying supine with the legs straight. B. Massaging the abdomen. C. Using antispasmodic medication. D. Using relaxation techniques.

A. Lying supine with the legs straight. Lying with the legs flexed is recommended. Lying with the legs extended is not useful because it increases the muscle tension in the abdomen, which could aggravate the inflamed intestinal tissues as the abdominal muscles are stretched. Encourage the patient to assume a position of comfort (knees flexed) to reduce abdominal tension and promote a sense of control. Option B: Applying local cold or heat to the abdomen or massaging the abdomen can help alleviate the pain in Crohn's disease. Provide comfort measures (back rub, reposition) and diversional activities. Promotes relaxation, refocuses attention, and may enhance coping abilities. Option C: The pain associated with Crohn's disease is alleviated by the use of analgesics and antispasmodics. Implement prescribed dietary modifications (commence with liquids and increase to solid foods as tolerated). Complete bowel rest can reduce pain, cramping. Option D: The pain associated with Crohn's disease is reduced by having the client practice relaxation techniques. Provide sitz baths as appropriate. Enhances cleanliness and comfort in the presence of perianal irritation or fissures.

The student nurse is teaching the family of a patient with liver failure. You instruct them to limit which foods in the patient's diet? A. Meats and beans B. Butter and gravies C. Potatoes and pasta D. Cakes and pastries

A. Meats and beans Meats and beans are high-protein foods. In liver failure, the liver is unable to metabolize protein adequately, causing protein by-products to build up in the body rather than be excreted. Have about 2 to 5 grams of protein per kilogram of body weight. This means that a 154-pound (70-kilogram) man should eat 84 to 105 grams of protein per day. Look for non-meat protein sources such as beans, tofu, and dairy products when you can. Option B: Eat a moderate intake of fat, as prescribed by the provider. The increased carbohydrates and fat help prevent protein breakdown in the liver. Eat fruits and vegetables and lean protein such as legumes, poultry, and fish. Avoid uncooked shellfish. Option C: Eat large amounts of carbohydrate foods. Carbohydrates should be the major source of calories in this diet. Increase the intake of carbohydrates to be in proportion with the amount of protein the client eats. Option D: Liver disease can affect the absorption of food and the production of proteins and vitamins. Therefore, diet may influence the weight, appetite, and the amounts of vitamins in the body. Do not limit protein too much, because it can result in a lack of certain amino acids.

The nurse would monitor a patient using sodium bicarbonate to treat gastric hyperacidity for signs and symptoms of:A. Metabolic alkalosis A. Metabolic alkalosis B. Metabolic acidosis C. Hyperkalemia D. Hypercalcemia

A. Metabolic alkalosis Solutions containing sodium bicarbonate (a base) can cause metabolic alkalosis. Serum K and serum calcium would decrease with alkalosis, not increase. Due to rapid alkalotic effects, sodium bicarbonate is contraindicated in those with signs/symptoms or laboratory values indicating underlying metabolic or respiratory alkalosis due to the potential for exacerbation of symptoms. Option B: Sodium bicarbonate is indicated for acute metabolic acidosis. If pH is less than 7.1 or pH less than 7.1 to 7.2 in patients with severe acute kidney injury (oliguria or 2-fold or larger increase in serum creatinine level). It is also indicated for chronic metabolic acidosis. 50-100 mEq oral tablet can be initiated and titrated according to the ongoing evaluation of acid-base balance. Option C: When patients with severe hyperkalemia (serum potassium level of more than 6 mEq/L or more than 5.5 mEq/L with arrhythmia or EKG changes) have metabolic acidosis, sodium bicarbonate should be administered. The dose needed is empirical and is unpredictable. Initially, 150 mEq of sodium bicarbonate can be given in 1 liter of 5% dextrose over 4 hours. More can be given if acidosis does not correct with this regimen. Option D: Sodium bicarbonate administration in a rapid infusion or as large boluses can result in acute metabolic alkalosis resulting in reduced serum ionized calcium. This acute shift in ionized calcium can result in tetany. This severe alkalosis is potentially treatable with ammonium chloride. Hypocalcemia may be addressed with calcium gluconate. An addition of 0.9 % NS infusion and potassium supplementation may also be indicated.

Which of the following symptoms indicated diverticulosis? A. No symptoms exist. B. Change in bowel habits. C. Anorexia with low-grade fever. D. Episodic, dull, or steady midabdominal pain.

A. No symptoms exist. Diverticulosis is an asymptomatic condition. The other choices are signs and symptoms of diverticulitis. The majority of individuals with diverticulosis are asymptomatic. Diverticular disease occurs when there is symptomatic diverticulosis (e.g., diverticular bleeding); diverticulitis (e.g., acute or chronic inflammation that may or may not is complicated by abscess formation, fistula formation, bowel obstruction, or perforation); or associated segmental colitis (e.g., inflammation in segments of the mucosal segments of the colon in between diverticula). Option B: Change in bowel habits, either diarrhea (35%) or constipation (50%), can be associated with abdominal pain. Patients may also experience nausea and vomiting, possibly secondary to bowel obstruction. Option C: Fever is not uncommon in patients with abscesses and perforation. Dysuria, frequency, and urgency can occur in patients when the inflamed portion of the bowel comes into direct contact with the bladder wall, which is called sympathetic cystitis. Option D: Clinical manifestation of acute diverticulitis varies depending on the severity of the disease. Patients with uncomplicated diverticulitis typically present with left lower quadrant abdominal pain, reflecting that propensity of left-sided disease in Western nations.

A male client undergoes total gastrectomy. Several hours after surgery, the nurse notes that the client's nasogastric (NG) tube has stopped draining. How should the nurse respond? A. Notify the physician. B. Reposition the tube. C. Irrigate the tube. D. Increase the suction level.

A. Notify the physician An NG tube that fails to drain during the postoperative period should be reported to the physician immediately. It may be clogged, which could increase pressure on the suture site because fluid isn't draining adequately. Nasogastric decompression has been routinely used in most abdominal operations to prevent the consequences of postoperative ileus. Option B: Repositioning an NG tube in a client who has undergone gastric surgery can disrupt the anastomosis. Routine use of nasogastric tubes after abdominal operations is intended to hasten the return of bowel function, prevent pulmonary complications, diminish the risk of anastomotic leakage, increase patient comfort and shorten hospital stay. Option C: Irrigating an NG tube in a client who has undergone gastric surgery can disrupt the anastomosis. Most surgeons traditionally continue to use nasogastric decompression, believing that its use facilitates a better surgical field and reduces complications such as nausea, vomiting, aspiration, and anastomotic leakage caused by postoperative ileus. Option D: Increasing the level of suction may cause trauma to GI mucosa or the suture line. The reason to perform such activity may be either therapeutic, as in patients with distention and vomiting from bowel obstruction, diagnostic, as in the case of gastrointestinal bleeding or peptic ulcer disease, or prophylactic, as in patients having major abdominal surgery.

Risk factors for the development of Hiatal hernias are those that lead to increased abdominal pressure. Which of the following complications can cause increased abdominal pressure? A. Obesity B. Volvulus C. Constipation D. Intestinal obstruction

A. Obesity Obesity may cause increased abdominal pressure that pushes the lower portion of the stomach into the thorax. In a hiatal hernia, the stomach pushes through that opening and into the chest and compromises the lower esophageal sphincter (LES). This laxity of the LES can allow gastric content and acid to back up into the esophagus and is the leading cause of gastroesophageal reflux disease (GERD). Option B: Volvulus occurs when a loop of intestine twists around itself and the mesentery that supports it, causing bowel obstruction. Symptoms include abdominal distension, pain, vomiting, constipation, and bloody stools. The onset of symptoms may be insidious or sudden. The mesentery becomes so tightly twisted that blood supply becomes cut off, resulting in an ischemic bowel. Option C: The cause of constipation is multifactorial. The problem may arise in the colon or rectum or it may be due to an external cause. In most people, slow colonic motility that occurs after years of laxative abuse is the problem. In a few patients, the cause may be related to an outlet obstruction like rectal prolapse or a rectocele. Option D: A bowel obstruction can either be a mechanical or functional obstruction of the small or large intestines. The obstruction occurs when the lumen of the bowel becomes either partially or completely blocked. Obstruction frequently causes abdominal pain, nausea, vomiting, constipation-to-obstipation, and distention. This, in turn, prevents the normal movement of digested products.]

A patient with severe cirrhosis of the liver develops hepatorenal syndrome. Which of the following nursing assessment data would support this? A. Oliguria and azotemia B. Metabolic alkalosis C. Decreased urinary concentration D. Weight gain of less than 1 lb per week

A. Oliguria and azotemia Hepatorenal syndrome is a functional disorder resulting from a redistribution of renal blood flow. Oliguria and azotemia occur abruptly as a result of this complication. Confusion due to hepatic encephalopathy is likely the last and most severe stage of liver disease as a result of the liver failing to break down toxic metabolites. Most importantly these patients notice they urinate less frequently in smaller and smaller volumes as they become oliguric. Option B: Excess organic acids are not being excreted by the damaged kidneys, resulting in an elevated concentration of hydrogen ions; decreased pH occurs, causing metabolic acidosis. Cirrhosis and portal hypertension can trigger the neurohormonal cascade which leads to the development of HRS. This, in turn, causes the production and release of vasodilators and cytokines like nitric oxide and prostaglandins which cause splanchnic and systemic vasodilation. Option C: Concentration of the urine is increased with decreased renal function. The systemic drop in circulating pressure triggers the carotid and aortic arch baroreceptors to activate three separate compensatory mechanisms. These include the renin-angiotensin-aldosterone system, vasopressin release, and activation of the sympathetic nervous system (SNS). Option D: With renal insufficiency, significant weight gain is expected due to fluid retention. The progression of cirrhosis causes a fall in cardiac output and a fall in systemic vascular resistance in a cycle that induces further renal vasoconstriction. This leads to further renal hypoperfusion, worsened by renal vasoconstriction with the endpoint of renal failure.

You're developing the plan of care for a patient experiencing dumping syndrome after a Billroth II procedure. Which dietary instructions do you include? A. Omit fluids with meals. B. Increase carbohydrate intake. C. Decrease protein intake. D. Decrease fat intake.

A. Omit fluids with meals. Gastric emptying time can be delayed by omitting fluids from your patient's meal. A diet low in carbs and high in fat & protein is recommended to treat dumping syndrome. Most cases of dumping syndrome are successfully treated with dietary adjustments. The suggested division of meals recommended is at least six times per day. Liquids should be withheld until 30 minutes after the meal. Option B: In addition, simple sugars and milk products should be avoided. Protein and fat calories should be increased to compensate for the decreased carbohydrate intake. Fiber-rich food is encouraged and has shown to promote a longer transit time in the bowel. Option C: Increase protein intake. The early symptoms are caused when concentrated sugar passes too rapidly from the stomach into the intestine. The body dilutes this sugar mixture by bringing fluid from body tissues into the intestine, giving a sense of fullness, cramping, and occasionally, diarrhea. The loss of water from tissues can produce a temporary drop in blood pressure, resulting in weakness and faintness. Option D: Increase fat intake. The later symptoms are caused by the rapid absorption of sugar into the bloodstream, which raises the amount of blood sugar. A high level of blood sugar signals the body to produce more insulin. The excess insulin, in turn, drives blood sugar levels down. Low blood sugar (hypoglycemia) produces weakness, hunger, and rapid heart rate that may occur about two to three hours after eating. This diet is really a regular diet with frequent small meals and reduced simple sugars.

Glenda has cholelithiasis (gallstones). You expect her to complain of: A. Pain in the right upper quadrant, radiating to the shoulder. B. Pain in the right lower quadrant, with rebound tenderness. C. Pain in the left upper quadrant, with shortness of breath. D. Pain in the left lower quadrant, with mild cramping.

A. Pain in the right upper quadrant, radiating to the shoulder. The gallbladder is located in the RUQ and a frequent sign of gallstones is pain radiating to the shoulder. Patients with gallstone disease typically present with symptoms of biliary colic (intermittent episodes of constant, sharp, right upper quadrant (RUQ) abdominal pain often associated with nausea and vomiting), normal physical examination findings, and normal laboratory test results. Option B: Clinical symptoms and signs suggestive of appendicitis include a history of central abdominal pain migrating to the right lower quadrant (RLQ), anorexia, fever, and nausea/vomiting. On examination, RLQ tenderness, along with "classical" signs of peritoneal irritation (e.g., rebound tenderness, guarding, rigidity, referred pain), may be present. Option C: LUQ pain can originate from the chest, abdomen, diaphragm/peritoneum, or from general 'medical' causes. Note that intra-abdominal organs may not localize pain accurately and diaphragmatic pain can be referred to the shoulder tip. Option D: Crampy pain may be due to gas, indigestion, inflammation, or infection, or it may result from menstrual cramps, endometriosis, or pelvic inflammatory disease in women. Severe pain that comes in waves may be caused by kidney stones. Trauma to the body wall, hernias, and shingles can also cause left lower quadrant pain.

The nurse is doing an admission assessment on a client with a history of duodenal ulcer. To determine whether the problem is currently active, the nurse would assess the client for which of the following most frequent symptom(s) of duodenal ulcer? A. Pain that is relieved by food intake. B. Pain that radiates down the right arm. C. N/V D. Weight loss

A. Pain that is relieved by food intake. The most frequent symptom of a duodenal ulcer is pain that is relieved by food intake. These clients generally describe the pain as burning, heavy, sharp, or "hungry" pain that often localizes in the midepigastric area. Duodenal ulcers occur when there is a disruption to the surface of the mucosa of the duodenum. These ulcers are part of peptic ulcer disease, which involves the stomach and the first part of the duodenum. Option B: The degree of disease progression before the initial diagnosis can affect the symptoms with which a patient may present. The location of the disease can also be differentiated based on symptoms. The pain associated with duodenal ulcers improves after meals, while the pain associated with gastric ulcers generally intensifies after meals. Option C: The typical presentation of a patient with gastric ulcers is epigastric pain that is worse with eating. It often correlates with mild nausea and early satiety. They often describe this pain as a sharp or burning type of pain that typically doesn't radiate. The most common finding on the physical exam is epigastric tenderness. Option D: The client with a duodenal ulcer usually does not experience weight loss. These symptoms are usually more typical in the client with a gastric ulcer. Patients may present with upper GI bleeding. The clinician should ask if they are having any black tarry stools, hematemesis, coffee-ground emesis, or bright red blood per rectum. It is important to remember that up to 15% of patients who present with bright red rectal bleeding have a brisk upper GI bleed.

Which assessment finding indicates that lactulose is effective in decreasing the ammonia level in the client with hepatic encephalopathy? A. Passage of two or three soft stools daily B. Evidence of watery diarrhea C. Daily deterioration in the client's handwriting D. Appearance of frothy, foul-smelling stools

A. Passage of two or three soft stools daily. Lactulose reduces serum ammonia levels by inducing catharsis, subsequently decreasing colonic pH and inhibiting fecal flora from producing ammonia from urea. Ammonia is removed with the stool. Two or three soft stools daily indicate the effectiveness of the drug. Lactulose, also known as 1,4 beta galactoside-fructose, is a non-absorbable synthetic disaccharide made up of galactose and fructose. The human small intestinal mucosa does not have the enzymes to split lactulose, and hence lactulose reaches the large bowel unchanged. Lactulose is metabolized in the colon by colonic bacteria to monosaccharides, and then to volatile fatty acids, hydrogen, and methane. Option B: Watery diarrhea indicates overdose. Since its intended use is to soften the stool quantity and increase the stool amount, its most significant side effect remains diarrhea. The diarrhea is dose-dependent and decreases in severity with a reduction in the dose of lactulose. Option C: Daily deterioration in the client's handwriting indicates an increase in the ammonia level and worsening of hepatic encephalopathy. From a pharmacokinetic standpoint, lactulose has negligible systemic absorption. However, like most laxatives, it has a propensity to bring about large changes in the body's fluid and electrolyte status. This activity would require periodic electrolyte monitoring, especially in the elderly and critically ill population. Option D: Frothy, foul-smelling stools indicate steatorrhea, caused by impaired fat digestion. Because lactulose has insignificant absorption by the gut and undergoes rapid excretion by the kidneys, its effects remain localized to the gut microenvironment.

A client has just had surgery for colon cancer. Which of the following disorders might the client develop? A. Peritonitis B. Diverticulosis C. Partial bowel obstruction D. Complete bowel obstruction

A. Peritonitis Bowel spillage could occur during surgery, resulting in peritonitis. Intestinal perforations occur most commonly in CRC and in diverticular disease. They are seen less often in other diseases of the colon (ulcerative colitis, Lesniowski-Crohn disease), abdominal trauma therein iatrogenic (complications after surgery, after endoscopic examination, or after radiation therapy), colonic ischemia, and necrosis. Option B: Diverticulosis doesn't result from surgery or colon cancer. Diverticulosis is a clinical condition in which multiple sac-like protrusions (diverticula) develop along the gastrointestinal tract. Though diverticula may form at weak points in the walls of either the small or large intestines, the majority occur in the large intestine (most commonly the sigmoid colon). Option C: Partial bowel obstruction may occur before bowel resection. Acute colonic obstruction produces a dilated bowel with a large amount of fecal loading that is proximal to the blockage and is associated with bacterial overgrowth and impairment of blood flow. Option D: Complete bowel obstruction may occur before bowel resection. Colorectal cancer is the single most common cause of large intestinal obstruction. Approximately 2% to 5% of colorectal cancer patients have an obstruction. Cancer arising in the rectum or left colon is more likely to obstruct than cancer arising in the proximal colon.

Nurse Juvy is caring for a client with cirrhosis of the liver. To minimize the effects of the disorder, the nurse teaches the client about foods that are high in thiamine. The nurse determines that the client has the best understanding of the dietary measures to follow if the client states an intention to increase the intake of: A. Pork B. Milk C. Chicken D. Broccoli

A. Pork The client with cirrhosis needs to consume foods high in thiamine. Thiamine is present in a variety of foods of plant and animal origin. Pork products are especially rich in this vitamin. Other good food sources include nuts, whole grain cereals, and legumes. Thiamine helps turn carbohydrates into energy. It is required for the metabolism of glucose, amino acids, and lipids. Option B: Milk contains vitamins A, D, and B2. Milk and dairy foods provide the right amount of bone-building nutrients, specifically calcium, vitamin D, protein, phosphorus, magnesium, potassium, vitamin B12, and zinc. Option C: Poultry contains niacin. Meat represents an excellent source of the majority of hydrophilic vitamins, and it is the ideal dietary source of vitamin B12. The amounts of B-group vitamins (e.g. niacin, vitamin B6, and pantothenic acid) in poultry are very similar to those of other meats and do not significantly diminish during cooking. Option D: Broccoli contains vitamins C, E, and K, and folic acid. Broccoli is a rich source of multiple vitamins, minerals, and fiber. Different cooking methods may affect the vegetable's nutrient composition, but broccoli is a healthy addition to the diet whether cooked or raw.

The nurse must be alert for complications with Sengstaken-Blakemore intubation including: A. Pulmonary obstruction B. Pericardiectomy syndrome C. Pulmonary embolization D. Cor pulmonale

A. Pulmonary obstruction. Rupture or deflation of the balloon could result in upper airway obstruction. Esophageal rupture is a well-known but rarely reported fatal complication of the management of bleeding esophageal varices with the Sengstaken-Blakemore (SB) tube. The most common complications of esophageal balloon therapy for varices include aspiration, esophageal perforation, and pressure necrosis of the mucosa. The other choices are not related to the tube. Option B: A pericardiectomy is a procedure done on the sac around the heart. A surgeon cuts away this sac or a large part of this sac. This allows the heart to move freely. Pericardiectomy is most often needed in people with chronic constrictive pericarditis. It's not usually an option for people who have had a single instance of pericarditis. With chronic constrictive pericarditis, the pericardium has become stiff and thick. This is due to repeated scarring. This scarring constricts the heart's movement. Option C: A pulmonary embolism is a blood clot that occurs in the lungs. It can damage part of the lung due to restricted blood flow, decrease oxygen levels in the blood, and affect other organs as well. Large or multiple blood clots can be fatal. The blockage can be life-threatening. Option D: Cor pulmonale is a condition that most commonly arises out of complications from high blood pressure in the pulmonary arteries (pulmonary hypertension). It's also known as right-sided heart failure because it occurs within the right ventricle of the heart.

A 40-year-old male client has been hospitalized with peptic ulcer disease. He is being treated with a histamine receptor antagonist (cimetidine), antacids, and diet. The nurse doing discharge planning will teach him that the action of cimetidine is to: A. Reduce gastric acid output. B. Protect the ulcer surface. C. Inhibit the production of hydrochloric acid (HCl). D. Inhibit vagus nerve stimulation.

A. Reduce gastric acid output. These drugs inhibit the action of histamine on the H2 receptors of parietal cells, thus reducing gastric acid output. The H2-receptor antagonist cimetidine competitively blocks histamine from stimulating the H2-receptors located on the gastric parietal cells (these cells are responsible for hydrochloric acid secretion and secretion of the intrinsic factor). The effect results in reducing the volume of gastric acid secretion from stimuli, including histamine, food, caffeine, and insulin. Option B: Sucralfate exhibits its action by forming a protective layer, increasing bicarbonate production, exhibiting anti-peptic effects, promoting tissue growth, regeneration, and repair. Sucralfate is a medication used to treat duodenal ulcers, epithelial wounds, chemotherapy-induced mucositis, radiation proctitis, ulcers in Behcet disease, and burn wounds. Option C: Ultimately, PPIs function to decrease acid secretion in the stomach. The proximal small bowel absorbs these drugs, and once in circulation, affects the parietal cells of the stomach. The parietal cells contain the H+/K+ ATPase enzyme, the proton pump, that PPIs block. This enzyme serves as the final step of acid secretion into the stomach. Option D: Atropine is an antimuscarinic that works through competitive inhibition of postganglionic acetylcholine receptors and direct vagolytic action, which leads to parasympathetic inhibition of the acetylcholine receptors in smooth muscle. The end effect of increased parasympathetic inhibition allows for preexisting sympathetic stimulation to predominate, creating increased cardiac output and other associated antimuscarinic side effects as described herein.

Radiation therapy is used to treat colon cancer before surgery for which of the following reasons? A. Reducing the size of the tumor. B. Eliminating the malignant cells. C. Curing cancer. D. Helping the bowel heal after surgery.

A. Reducing the size of the tumor Radiation therapy is used to treat colon cancer before surgery to reduce the size of the tumor, making it easier to be resected. Radiation therapy is a treatment using high-energy rays (such as x-rays) or particles to destroy cancer cells. For some colon and rectal cancers, treating with chemotherapy at the same time can make radiation therapy work better. Using these 2 treatments together is called chemoradiation. Option B: Radiation therapy can't eliminate the malignant cells (though it helps define tumor margins). Radiation therapy, often with chemotherapy, is frequently used in the adjuvant or neoadjuvant setting for the treatment of rectal cancers, whereas chemotherapy alone is more common for the adjuvant and neoadjuvant treatment of colon cancers. Option C: Radiation therapy isn't curative. The progress that has been made in the treatment of colon cancer has resulted from improved development of radiation treatments and surgical techniques and participation in clinical trials. Option D: Radiation therapy could slow postoperative healing. It's not common to use radiation therapy to treat colon cancer, but it may be used after surgery if cancer has attached to an internal organ or the lining of the belly (abdomen). If this happens, the surgeon can't be sure that all of cancer has been removed. Radiation therapy may be used to try to kill any cancer cells that may have been left behind.

Your patient's ABG reveals an acidic pH, an acidic CO2, and a normal bicarbonate level. Which of the following indicates this acid-base disturbance? A. Respiratory acidosis B. Respiratory alkalosis C. Metabolic acidosis D. Metabolic alkalosis

A. Respiratory acidosis A pH of 7.35 indicates acidosis, as does an acidic CO2 and bicarbonate. The primary disturbance of elevated arterial PCO2 is the decreased ratio of arterial bicarbonate to arterial PCO2, which leads to a lowering of the pH. In the presence of alveolar hypoventilation, 2 features commonly are seen are respiratory acidosis and hypercapnia. To compensate for the disturbance in the balance between carbon dioxide and bicarbonate (HCO3-), the kidneys begin to excrete more acid in the forms of hydrogen and ammonium and reabsorb more base in the form of bicarbonate. See also: 8-Step Guide to ABG Analysis: Tic-Tac-Toe Method Option B: Respiratory alkalosis is 1 of the 4 basic classifications of blood pH imbalances. Normal human physiological pH is 7.35 to 7.45. A decrease in pH below this range is acidosis, an increase above this range is alkalosis. Respiratory alkalosis is by definition a disease state where the body's pH is elevated to greater than 7.45 secondary to some respiratory or pulmonary process. Option C: Determining the type of metabolic acidosis can help clinicians narrow down the cause of the disturbance. Acidemia refers to a pH less than the normal range of 7.35 to 7.45. In addition, metabolic acidosis requires a bicarbonate value less than 24 mEq/L. Further classification of metabolic acidosis is based on the presence or absence of an anion gap, or concentration of unmeasured serum anions. Option D: HCO3 functions as an alkalotic substance. CO2 functions as an acidic substance. Therefore, increases in HCO3 or decreases in CO2 will make blood more alkalotic. The opposite is also true where decreases in HCO3 or an increase in CO2 will make blood more acidic. CO2 levels are physiologically regulated by the pulmonary system through respiration, whereas the HCO3 levels are regulated through the renal system with reabsorption rates. Therefore, metabolic alkalosis is an increase in serum HCO3.

Sharon has cirrhosis of the liver and develops ascites. What intervention is necessary to decrease the excessive accumulation of serous fluid in her peritoneal cavity? A. Restrict fluids. B. Encourage ambulation. C. Increase sodium in the diet. D. Give antacids as prescribed.

A. Restrict fluids Restricting fluids decrease the amount of body fluid and the accumulation of fluid in the peritoneal space. Restrict sodium and fluids as indicated. Sodium may be restricted to minimize fluid retention in extravascular spaces. Fluid restriction may be necessary to correct dilutional hyponatremia. Option B: Encourage bed rest when ascites is present; may promote recumbency-induced diuresis. Measure abdominal girth. Reflects accumulation of fluid (ascites) resulting from loss of plasma proteins/fluid into peritoneal space. Excessive fluid accumulation can reduce circulating volume, creating a deficit (signs of dehydration). Option C: Monitor serum albumin and electrolytes (particularly potassium and sodium). Decreased serum albumin affects plasma colloid osmotic pressure, resulting in edema formation. Reduced renal blood flow accompanied by elevated ADH and aldosterone levels and the use of diuretics (to reduce total body water) may cause various electrolyte shifts/imbalances. Option D: Administer salt-free albumin/plasma expanders as indicated. Albumin may be used to increase the colloid osmotic pressure in the vascular compartment (pulling fluid into vascular space), thereby increasing effective circulating volume and decreasing the formation of ascites.

A client has a percutaneous endoscopic gastrostomy tube inserted for tube feedings. Before starting a continuous feeding, the nurse should place the client in which position? A. Semi-Fowlers B. Supine C. Reverse Trendelenburg D. High Fowler's

A. Semi-Fowlers To prevent aspiration of stomach contents, the nurse should place the client in semi-Fowler's position. Keep the head of the bed elevated when feeding and for at least a half-hour afterward. Maintaining a sitting position after meals may help decrease aspiration pneumonia in the elderly. Option B: Position patients with a decreased level of consciousness on their side. This positioning (rescue positioning) decreases the risk for aspiration by promoting the drainage of secretions out of the mouth instead of down the pharynx, where they could be aspirated. Option C: In the reverse Trendelenburg group, patients are placed in a supine position, which is inclined at a 30-degree angle, so that the head is higher than the pelvis and the leg is lower than the hip. Option D: High Fowler's position isn't necessary and may not be tolerated as well as semi-Fowler's. During enteral feedings, position the patient with head of bed elevated 30 to 40 degrees; maintain for 30 to 45 minutes after feeding. Keeping the patient's head elevated helps keep food in the stomach and decreases the incidence of aspiration.

Which of the following laboratory values would be the most important to monitor for a patient with pancreatic cancer? A. Serum glucose B. Radioimmunoassay (RIA) C. Creatine phosphokinase (CPK) D. Carcinoembryonic antigen (CEA)

A. Serum glucose In pancreatitis, hypersecretion of the insulin from a tumor may affect the islets of Langerhans, resulting in hyperinsulinemia, a complication of pancreatic cancer. Pancreatitis damages the cells that produce insulin and glucagon, which are the hormones that control the amount of glucose in the blood. This can lead to an increase in blood glucose levels. Option B: RIA should also be monitored to measure the effects of therapy, but hypoglycemia may be life-threatening. Determination of serum pancreatic enzymes remains the gold standard for the diagnosis of acute pancreatitis. More expensive and cumbersome methods such as RIA or ELISA for pancreatic elastase are useful only in special clinical circumstances. Option C: Creatine phosphokinase is an enzyme that reflects normal tissue catabolism. Elevated serum levels indicate trauma to cells with high CPK content. CPK and CPK-isoenzymes are used to detect myocardial infarction. Serum CPK is elevated in most alcoholics including patients with delirium tremens (6, 9) or acute pancreatitis, a finding which possibly reflects a certain degree of myopathy known to occur in patients with acute alcohol intoxication as well as with chronic alcoholism Option D: Carcinoembryonic antigen (CEA) is one of the most widely used tumor markers and is increased in 30%-60% of patients with pancreatic cancer. Although carbohydrate antigen 19-9 (CA19-9) is the most important serum biomarker in pancreatic cancer, the diagnostic and prognostic value of CEA is gradually being recognized.

What information is correct about stomach cancer? A. Stomach pain is often a late symptom. B. Surgery is often a successful treatment. C. Chemotherapy and radiation are often successful treatments. D. The patient can survive for an extended time with TPN.

A. Stomach pain is often a late symptom. Stomach pain is often a late sign of stomach cancer; outcomes are particularly poor when cancer reaches that point. In the United States, most patients have symptoms of an advanced stage at the time of presentation. The most common presenting symptoms for gastric cancers are non-specific weight loss, persistent abdominal pain, dysphagia, hematemesis, anorexia, nausea, early satiety, and dyspepsia. Option B: Surgery has minimal positive effects. Patients with localized, resectable gastric cancer have the best chance of long-term survival with surgery alone. The main goal of surgery is complete resection with adequate margins (more than 4 cm), and only 50% of patients will obtain R0. Option C: Chemotherapy and radiation have minimal positive effects. Neoadjuvant chemotherapy has been shown to downstage primary tumors and regional lymph nodes to attempt higher long-term curative resections. Neoadjuvant therapy should be offered to patients at high risk of developing distant metastases (bulky T3/T4, perigastric nodes, linitis plastica, or positive peritoneal cytology), sparing unnecessary surgery in case an emerging metastasis appears. Option D: TPN may enhance the growth of cancer. Total parenteral nutrition is known to be effective in cases of malnutrition in patients who do not have cancer. However, TPN has not been shown to positively affect the nutritional status in patients with cancer. This is due in part to the metabolic changes associated with cancer.

The nurse is monitoring a female client for the early signs and symptoms of dumping syndrome. Which of the following indicate this occurrence? A. Sweating and pallor B. Bradycardia and indigestion C. Double vision and chest pain D. Abdominal cramping and pain

A. Sweating and pallor Early manifestations of dumping syndrome occur 5 to 30 minutes after eating. Symptoms include vertigo, tachycardia, syncope, sweating, pallor, palpitations, and the desire to lie down. In early dumping, the symptoms usually occur within 10 to 30 minutes after a meal. The rapid transit of hyperosmolar chyme from the stomach into the duodenum causes fluid to shift from the vasculature to the intestinal lumen, leading to increased volume in the small bowel. Option B: Bradycardia and indigestion are not signs of dumping syndrome. There may be GI or vasomotor symptoms. GI symptoms include nausea, vomiting, diarrhea, or belching. Vasomotor symptoms include shock, syncope, near-syncope, palpitations, dizziness, desire to lie down, or diaphoresis. Option C: Double vision and chest pain are not signs of dumping syndrome. GI hormones such as enteroglucagon, pancreatic polypeptide, peptide YY, vasoactive intestinal polypeptide, glucagon-like peptide, and neurotensin have been evident with higher values after meals. Hormonal imbalances may cause delayed motility, decreased gastric and intestinal secretions, which delay the digestion and transit of food that arrives at the small bowel. Option D: Late dumping, also known as postprandial hyperinsulinemic hypoglycemia, usually occurs 1 to 3 hours after a high-carbohydrate meal. There is an association with hypoglycemia, but the exact mechanism is unknown. It is proposed that the rapid absorption of carbohydrates exaggerates the glucose-mediated insulin response.

For a client in hepatic coma, which outcome would be the most appropriate? A. The client is oriented to time, place, and person. B. The client exhibits no ecchymotic areas. C. The client increases oral intake to 2,000 calories/day. D. The client exhibits increased serum albumin level.

A. The client is oriented to time, place, and person. Hepatic coma is the most advanced stage of hepatic encephalopathy. As hepatic coma resolves, improvement in the client's level of consciousness occurs. The client should be able to express orientation to time, place, and person. Throughout the intermediate stages, patients tend to experience worsening levels of confusion, lethargy, and personality changes. Option B: Ecchymotic areas are related to decreased synthesis of clotting factors. In order to make a diagnosis of hepatic encephalopathy, there must be confirmed the presence of liver disease (e.g., abnormal liver function tests, ultrasound or liver biopsy demonstrating liver disease) or a portosystemic shunt, and exclusion of other potential etiologies (e.g., intracranial lesions, masses, hemorrhage or stroke; seizure activity; post-seizure encephalopathy; intracranial infections; or toxic encephalopathy from other causes). Option C: Although oral intake may be related to the level of consciousness, it is more closely related to anorexia. Triggers of hepatic encephalopathy include renal failure, gastrointestinal bleeding (e.g., esophageal varices), constipation, infection, medication non-compliance, excessive dietary protein intake, dehydration (e.g., fluid restriction, diuretics, diarrhea, vomiting, excessive paracentesis), electrolyte imbalance, consumption of alcohol, or consumption of certain sedatives, analgesics or diuretics all in the setting of chronic liver disease. Option D: The serum albumin level reflects hepatic synthetic ability, not level of consciousness. Elevated blood ammonia levels are often seen in patients with hepatic encephalopathy. It is more useful, however, to assess the clinical improvement or deterioration of a patient undergoing treatment rather than monitor serial arterial blood ammonia measurements.

You are caring for Rona, a 35-year-old female in a hepatic coma. Which evaluation criteria would be the most appropriate? A. The patient demonstrates an increase in the level of consciousness. B. The patient exhibits improved skin integrity. C. The patient experiences no evident signs of bleeding. D. The patient verbalizes decreased episodes of pain.

A. The patient demonstrates an increase in the level of consciousness. Increased level of consciousness indicates resolving of a comatose state. Ongoing assessment of behavior and mental status is important because of the fluctuating nature of impending hepatic coma. Other options are important evaluations but do not evaluate a patient in a hepatic coma who is responding to external stimuli. Option B: Inspect pressure points and skin surfaces closely and routinely. Gently massage bony prominences or areas of continued stress. Use of emollient lotions and limiting the use of soap for bathing may help. Keep linens dry and free of wrinkles. Option C: Closely assess for signs and symptoms of GI bleeding: check all secretions for frank or occult blood. Observe color and consistency of stools, NG drainage, or vomitus. The esophagus and rectum are the most usual sources of bleeding because of their mucosal fragility and alterations in hemostasis associated with cirrhosis. Option D: Use small needles for injections. Apply pressure to small bleeding and venipuncture sites for longer than usual. Minimizes damage to tissues, reducing risk of bleeding and hematoma. Encourage the use of soft toothbrush, electric razor, avoiding straining for stool, vigorous nose blowing, and so forth.

A patient has an acute upper GI hemorrhage. Your interventions include: A. Treating hypovolemia. B. Treating hypervolemia. C. Controlling the bleeding source. D. Treating shock and diagnosing the bleeding source.

A. Treating hypovolemia. A patient with an acute upper GI hemorrhage must be treated for hypovolemic and hemorrhagic shock. Monitor patient's vital signs, especially BP and HR. Look for signs of orthostatic hypotension. Hypotension and tachycardia are initial compensatory mechanisms usually noted with bleeding. Orthostasis (a drip of 20 mm Hg in systolic BP or 10 mm Hg in diastolic BP when changing from supine to sitting position) indicates reduced circulating fluids. Option B: Educate the patient and family members about signs of bleeding that need to be reported to a health care provider. Early evaluation and treatment of bleeding by a health care provider reduce the risk for complications from blood loss. Option C: Controlling the bleeding may require surgery or intensive medical treatment. Tell the family members to be active in decision-making about the treatment of the patient at risk for bleeding. Active participation encourages a fuller understanding of the rationale and compliance with the treatment. Option D: Inform the patient to check the color and consistency of stools. Bright red blood in the stools is an indicator of lower gastrointestinal bleeding. A stool that has a dark greenish-black color and a tarry consistency is linked with upper gastrointestinal bleeding.

A nursing intervention for a patient with hepatitis B would include which of the following types of isolation. A. Universal precautions B. Blood transfusions C. Enteric isolation D. Strict isolation

A. Universal precautions Universal precautions are indicated for the patient with hepatitis B. Hepatitis B is contracted via blood and blood products, body secretions, and punctures from contaminated needles. Universal precautions apply to blood and to other body fluids containing visible blood. Occupational transmission of HIV and HBV to healthcare workers by blood is documented. Blood is the single most important source of HIV, HBV, and other bloodborne pathogens in the occupational setting. Option B: Currently, guidelines for transfusion of red blood cells (RBC), generally follow a restrictive threshold. While there is some variation in the number for the threshold, 7 g/dL is an agreed-upon value for asymptomatic healthy patients. Transfusion may also be indicated in patients with active or acute bleeding as well as in patients with symptoms related to anemia (for example, tachycardia, weakness, dyspnea on exertion) and hemoglobin less than 8 g/dL. Option C: Enteric precautions are taken to prevent infections that are transmitted primarily by direct or indirect contact with fecal material. They're indicated for patients with known or suspected infectious diarrhea or gastroenteritis. Clostridium difficile is the most common cause of hospital-acquired infectious diarrhea. Option D: Strict isolation includes suspected highly infectious and transmissible virulent and pathogenic microbes, highly resistant bacterial strains, and agents that are not accepted in any form of distribution in society or in the environment. Examples are completely resistant Mycobacterium tuberculosis, viral hemorrhagic fevers like Ebola and Lassa, pandemic severe influenza, and coronavirus like SARS, MERS, etc.

Nitrosocarcinogen production can be inhibited with the intake of: A. Vitamin C B. Vitamin E C. Carbohydrates D. Fiber

A. Vitamin C Vitamin C and refrigeration of foods inhibit nitroso carcinogen. Humans are exposed to a wide range of nitrogen-containing compounds and nitrosating agents, such as nitrite, nitrate, and nitrogen oxides (NOx), that can react in vivo to form potentially carcinogenic N-nitroso compounds (NOCs), as well as several carcinogenic C-nitro(so) or reactive diazo compounds. Option B: Ascorbic acid, alpha-tocopherol, phenolic compounds, and fruit, vegetable and plant extracts inhibit NOC formation by destroying nitrosating agents. Fresh fruits and vegetables (sources of nitrosation inhibitors) exert a protective effect against various epithelial cancers. Option C: Although vitamin C has been known to stimulate immune function, inhibit nitrosamine formation, and block the metabolic activation of carcinogens, its cancer-preventive effects may be associated mainly with its protective effects against oxidative stress. Option D: Vitamin C, not fiber, is considered to be one of the most prevalent antioxidative components of fruit and vegetables, and it could exert chemopreventive effects without apparent toxicity at doses higher than the current recommended dietary allowance of 60 mg/d. It has also been used as a dietary supplement intended to prevent oxidative stress-mediated chronic diseases such as cancer, cardiovascular disease, hypertension, stroke, and neurodegenerative disorder.

You have a patient with achalasia (incomplete muscle relaxation of the GI tract, especially sphincter muscles). Which medications do you anticipate to administer? A. isosorbide dinitrate (Isordil) B. digoxin (Lanoxin) C. captopril (Capoten) D. propranolol (Inderal)

A. isosorbide dinitrate (Isordil) Achalasia is characterized by incomplete relaxation of the LES, dilation of the lower esophagus, and a lack of esophageal peristalsis. Because nitrates relax the lower esophageal sphincter, expect to give Isordil orally or sublingually. Isosorbide is a nitrate that exerts its pharmacologic effect by releasing nitric oxide (NO), an endothelium-derived relaxing factor (EDRF).NO is endogenously produced in the endothelium to dilate the blood vessels. Option B: Digoxin is a medication used in the management and treatment of heart failure and certain arrhythmias and abortion. It is in the cardiac glycoside class of drugs. It is used for rate control in atrial fibrillation or atrial flutter when conventional therapies have not achieved goal heart rate. Digoxin should not be administered in cases of pre-excitation caused by accessory pathways as digoxin induces AV blockade and may trigger ventricular tachyarrhythmias. Option C: The benefits of captopril in hypertension and heart failure result primarily from suppressing the renin-angiotensin-aldosterone system (RAAS). As an angiotensin-converting enzyme (ACE) inhibitor, it inhibits ACE, which converts angiotensin I to angiotensin II. Angiotensin II binds to AT1 receptors on smooth muscles to produce vasoconstriction of precapillary arterioles and postcapillary venules, inhibits the reuptake of norepinephrine, and release of catecholamines from the adrenal medulla, which all increases blood pressure. Option D: Propranolol can be used to ameliorate the sympathetic response in angina, tachyarrhythmias, prevention of acute ischemic attacks, migraine prophylaxis, and restless leg syndrome. Propranolol can be used in almost all cases if the desired result is to slow contractility and decrease a patient's heart rate.

The nurse should anticipate which of these dietary measures for pt admitted with diagnosis of acute pancreatits A. maintain NPO status B. low-fat, high-fiber diet C. full liquid diet D. soft, bland diet

A. maintain NPO status

After receiving change of shift report, which pt will the RN plan to assess first? A. young adult pt with acute pancreatitis who is dyspneic and has RR of 34 breaths/min B. adult pt admitted with cholecystitis who is having severe RUQ pain C. middle age pt who has elevated temp after ERCP D. older adult receiving TPN after whipple with blood glucose of 235 mg/dL

A. young adult pt with acute pancreatitis who is dyspneic and has RR of 34 breaths/min

A patient who recently had a colon resection for cancer of the colon asks about the purpose of the carcinoembryonic antigen (CEA) test. Which explanation would the nurse provide? A. Identify any metastasis of the cancer. B. Monitor for tumor growth after surgery. C. Confirm the diagnosis of a specific type of cancer. D. Determine the need for postoperative chemotherapy.

B CEA is used to monitor for cancer recurrence after surgery. CEA levels do not help to determine whether there is metastasis of the cancer. Confirmation of the diagnosis is made based on the biopsy. Chemotherapy use is based on factors other than CEA.

Which item would the nurse offer to the patient restarting oral intake after being NPO due to nausea and vomiting? A. Glass of orange juice B. Dish of lemon gelatin C. Cup of coffee with cream D. Bowl of hot chicken broth

B Clear cool liquids are usually the first foods started after a patient has been nauseated. Acidic foods such as orange juice, very hot foods, and coffee are poorly tolerated when patients have been nauseated.

After a total proctocolectomy and permanent ileostomy, the patient tells the nurse, "I cannot manage all this. I don't want to look at the stoma." Which action would the nurse take? A. Reassure the patient that ileostomy care will become easier. B. Ask the patient about the concerns with stoma management. C. Postpone any teaching until the patient adjusts to the ileostomy. D. Develop a detailed written list of ostomy care tasks for the patient.

B Encouraging the patient to share concerns assists in helping the patient adjust to the body changes. Acknowledgment of the patient's feelings and concerns is important rather than offering false reassurance. Because the patient indicates that the feelings about the ostomy are the reason for the difficulty with the many changes, development of a detailed ostomy care plan will not improve the patient's ability to manage the ostomy. Although detailed ostomy teaching could be postponed, the nurse should begin to offer teaching about some aspects of living with an ostomy.

Which action would the nurse take to evaluate treatment effectiveness for a patient who has hepatic encephalopathy? A. Request that the patient stand on one foot. B. Ask the patient to extend both arms forward. C. Request that the patient walk with eyes closed. D. Ask the patient to perform the Valsalva maneuver.

B Extending the arms allows the nurse to check for asterixis, a classic sign of hepatic encephalopathy. The other tests might be done as part of the neurologic assessment but would not be diagnostic for hepatic encephalopathy.

Which information will the nurse include when teaching a patient with peptic ulcer disease about the effect of famotidine (Pepcid)? A. "Famotidine absorbs the excess gastric acid." B. "Famotidine decreases gastric acid secretion." C. "Famotidine constricts the blood vessels near the ulcer." D. "Famotidine covers the ulcer with a protective material."

B Famotidine is a histamine-2 (H2) receptor blocker that decreases the secretion of gastric acid. Famotidine does not constrict the blood vessels, absorb the gastric acid, or cover the ulcer.

A young adult patient is hospitalized with massive abdominal trauma from a motor vehicle crash. The patient asks about the purpose of receiving famotidine (Pepcid). Which information would the nurse explain about the action of the medication? A. "It decreases nausea and vomiting." B. "It inhibits development of stress ulcers." C. "It lowers the risk for H. pylori infection." D. "It prevents aspiration of gastric contents."

B Famotidine is administered to prevent the development of physiologic stress ulcers, which are associated with a major physiologic insult such as massive trauma. Famotidine does not decrease nausea or vomiting, prevent aspiration, or prevent H. pylori infection.

A critically ill patient with sepsis is frequently incontinent of watery stools. Which action by the nurse will prevent complications associated with ongoing incontinence? A. Apply incontinence briefs. B. Use a fecal management system. C. Insert a rectal tube with a drainage bag. D. Assist the patient to a commode frequently.

B Fecal management systems are designed to contain loose stools and can be in place for as long as 4 weeks without causing damage to the rectum or anal sphincters. Although incontinence briefs may be helpful, unless they are changed frequently, they are likely to increase the risk for skin breakdown. Rectal tubes are avoided because of possible damage to the anal sphincter and ulceration of the rectal mucosa. A critically ill patient will not be able to tolerate getting up frequently to use the commode or bathroom.

A patient is admitted to the outpatient testing area for an ultrasound of the gallbladder. Which information obtained by the nurse indicates that the ultrasound may need to be rescheduled? A. The patient has a gastrostomy tube. B. The patient ate a bagel 4 hours ago. C. The patient took a laxative the day before. D. The patient had a high-fat meal the previous evening.

B Food intake can cause the gallbladder to contract and result in a suboptimal study. The patient would be NPO for 8 to 12 hours before the test. A high-fat meal the previous evening, laxative use, or a gastrostomy tube will not affect the results of the study.

A patient who has gastroesophageal reflux disease (GERD) is experiencing increasing discomfort. Which patient statement to the nurse indicates a need for additional teaching about GERD? A. "I quit smoking years ago, but I chew gum." B. "I eat small meals and have a bedtime snack." C. "I take antacids between meals and at bedtime each night." D. "I sleep with the head of the bed elevated on 4-inch blocks."

B GERD is exacerbated by eating late at night, and the nurse would plan to teach the patient to avoid eating within 3 hours of bedtime. Smoking cessation, taking antacids, and elevating the head of the bed are appropriate actions to control symptoms of GERD.

Which information will the nurse include in teaching a patient who had a proctocolectomy and ileostomy for ulcerative colitis? A. Restrict fluid intake to prevent constant liquid drainage from the stoma. B. Use care when eating high-fiber foods to avoid obstruction of the ileum. C. Irrigate the ileostomy daily to avoid having to wear a drainage appliance. D. Change the pouch every day to prevent leakage of contents onto the skin.

B High-fiber foods are introduced gradually and should be well chewed to avoid obstruction of the ileostomy. Patients with ileostomies do not have a colon for the absorption of water; they need to take in increased amounts of fluid. The pouch should be drained frequently but is changed every 5 to 7 days. The drainage from an ileostomy is liquid and continuous, so control by irrigation is not possible.

A patient who underwent a gastroduodenostomy (Billroth I) 12 hours ago reports increasing abdominal pain. The patient has no bowel sounds and 200 mL of bright red nasogastric (NG) drainage in the past hour. Which nursing action is the highest priority? A. Monitor drainage. B. Contact the surgeon. C. Irrigate the NG tube. D. Give prescribed morphine.

B Increased pain and 200 mL of bright red NG drainage 12 hours after surgery indicate possible postoperative hemorrhage, and immediate actions such as blood transfusion or return to surgery are needed (or both). Because the NG is draining, there is no indication that irrigation is needed. Continuing to monitor the NG drainage is needed but not an adequate response to the findings. The patient may need morphine, but this is not the highest priority action.

Which information will the nurse plan to teach a patient who has lactose intolerance? A. Ice cream is relatively low in lactose. B. Live-culture yogurt is usually tolerated. C. Heating milk will break down the lactose. D. Nonfat milk is tolerated better than whole milk.

B Lactose-intolerant persons can usually eat yogurt without experiencing discomfort. Ice cream, nonfat milk, and milk that have been heated are all high in lactose.

After change-of-shift report, which patient would the nurse assess first? A. A 40-yr-old male patient with celiac disease who has frequent frothy diarrhea B. A 30-yr-old female patient with a femoral hernia who has abdominal pain and vomiting C. A 30-yr-old male patient with ulcerative colitis who has severe perianal skin breakdown D. A 40-yr-old female patient with a colostomy bag that is pulling away from the adhesive wafer

B Pain and vomiting with a femoral hernia suggest strangulation, which will require emergency surgery. All the other patients require assessment or care but have less urgent problems.

A 19-yr-old patient has familial adenomatous polyposis (FAP). Which action will the nurse in the gastrointestinal clinic include in the plan of care? A. Obtain blood samples for DNA analysis. B. Schedule the patient for yearly colonoscopy. C. Provide preoperative teaching about total colectomy. D. Discuss lifestyle modifications to decrease cancer risk.

B Patients with FAP should have annual colonoscopy starting at age 16 years and usually have total colectomy by age 25 years to avoid developing colorectal cancer. DNA analysis is used to make the diagnosis but is not needed now for this patient. Lifestyle modifications will not decrease cancer risk for this patient.

A 26-yr-old patient who was admitted with viral hepatitis has severe anorexia and fatigue, and is homeless. Which goal has the highest priority in the plan of care? A. Increase activity level. B. Maintain adequate nutrition. C. Establish a stable place of residence. D. Identify source of hepatitis exposure.

B The highest priority outcome is to maintain nutrition because adequate nutrition is needed for hepatocyte regeneration. Finding a home for the patient and identifying the source of the infection would be appropriate activities, but they do not have as high a priority as ensuring adequate nutrition. Although the patient's activity level will be gradually increased, rest is indicated during the acute phase of hepatitis.

Which is the correct technique for the nurse to palpate the liver during a head-to-toe physical assessment? A. Place one hand on top of the other and use the upper fingers to apply pressure and the bottom fingers to feel for the liver edge. B. Place one hand on the patient's back and press upward and inward with the other hand below the patient's right costal margin. C. Press slowly and firmly over the right costal margin with one hand and withdraw the fingers quickly after the liver edge is felt. D. Place one hand under the patient's lower ribs and press the left lower rib cage forward, palpating below the costal margin with the other hand.

B The liver is normally not palpable below the costal margin. The nurse needs to push inward below the right costal margin while lifting the patient's back slightly with the left hand. The other methods will not allow palpation of the liver.

A patient is being treated for bleeding esophageal varices with balloon tamponade. Which nursing action will be included in the plan of care? A. Instruct the patient to cough every hour. B. Monitor the patient for shortness of breath. C. Verify the position of the balloon every 4 hours. D. Deflate the gastric balloon if the patient reports nausea.

B The most common complication of balloon tamponade is aspiration pneumonia. In addition, if the gastric balloon ruptures, the esophageal balloon may slip upward and occlude the airway. Coughing increases the pressure on the varices and increases the risk for bleeding. Balloon position is verified after insertion and does not require further verification. Balloons may be deflated briefly every 8 to 12 hours to avoid tissue necrosis, but if only the gastric balloon is deflated, the esophageal balloon may occlude the airway. Balloons are not deflated for nausea.

A 74-yr-old male patient tells the nurse that growing old causes constipation, so he has been using a suppository to prevent constipation every morning. Which action would the nurse take first? A. Encourage the patient to increase oral fluid intake. B. Question the patient about risk factors for constipation. C. Suggest that the patient increase intake of high-fiber foods. D. Teach the patient that a daily bowel movement is unnecessary.

B The nurse's initial action should be further assessment of the patient for risk factors for constipation and for his usual bowel pattern. The other actions may be appropriate but will be based on the assessment.

The nurse and a licensed practical/vocational nurse (LPN/VN) are working together to care for a patient who had an esophagectomy 2 days ago. Which action by the LPN/VN requires that the nurse intervene? A. The LPN/VN uses soft swabs to provide oral care. B. The LPN/VN positions the head of the bed in the flat position. C. The LPN/VN includes the enteral feeding volume when calculating intake. D. The LPN/VN encourages the patient to use pain medications before coughing.

B The patient's bed would be in Fowler's position to prevent reflux and aspiration of gastric contents. The other actions by the LPN/LVN are appropriate.

The nurse is administering IV fluid boluses and nasogastric irrigation to a patient with acute gastrointestinal (GI) bleeding. Which assessment finding is most important for the nurse to communicate to the health care provider? A. The bowel sounds are hyperactive in all four quadrants. B. The patient's lungs have crackles audible to the midchest. C. The nasogastric (NG) suction is returning coffee-ground material. D. The patient's blood pressure (BP) has increased to 142/84 mm Hg.

B The patient's lung sounds indicate that pulmonary edema may be developing because of the rapid infusion of IV fluid and that the fluid infusion rate would be slowed. The return of coffee-ground material in an NG tube is expected for a patient with upper GI bleeding. The BP is slightly elevated but would not be an indication to contact the health care provider immediately. Hyperactive bowel sounds are common when a patient has GI bleeding.

A patient admitted with a peptic ulcer has a nasogastric (NG) tube in place. When the patient develops sudden, severe upper abdominal pain, diaphoresis, and a firm abdomen, which action would the nurse take? A. Irrigate the NG tube. B. Check the vital signs. C. Give the ordered antacid. D. Elevate the foot of the bed.

B The patient's symptoms suggest acute perforation, and the nurse should assess for signs of hypovolemic shock. Irrigation of the NG tube, administration of antacids, or both would be contraindicated because any material in the stomach will increase the spillage into the peritoneal cavity. Elevating the foot of the bed may increase abdominal pressure and discomfort, as well as making it more difficult for the patient to breathe.

The nurse evaluates that administration of hepatitis B vaccine to a healthy patient was effective when the patient's later blood specimen reveals the presence of A. HBsAg. B. anti-HBs. C. anti-HBc IgG. D. anti-HBc IgM.

B The presence of surface antibody to hepatitis B (anti-HBs) is a marker of a positive response to the vaccine or previous illness with hepatitis B. The other laboratory values indicate current infection with hepatitis B.

Which area of the abdomen shown in the accompanying figure will the nurse palpate to assess for splenomegaly? A. 1 B. 2 C. 3 D. 4

B The spleen is usually not palpable, but when palpated, it is located in left upper quadrant of abdomen.

The nurse is assessing an alert and independent older adult patient for malnutrition risk. Which is the most useful initial question? A. "How do you get to the store to buy your food?" B. "Can you tell me the food that you ate yesterday?" C. "Do you have any difficulty in preparing or eating food?" D. "Are you taking any medications that alter your taste for food?"

B This question is the most open-ended and will provide the best overall information about the patient's daily intake and risk for poor nutrition. The other questions may be asked, depending on the patient's response to the first question.

Which scheduling would the nurse teach a patient with chronic pancreatitis to use for the prescribed pancrelipase (Viokase)? A. Bedtime B. Mealtime C. When needed for pain D. When feeling nauseated

B Pancreatic enzymes are used to help with digestion of nutrients and would be taken with every meal.

A client diagnosed with chronic cirrhosis who has ascites and pitting peripheral edema also has hepatic encephalopathy. Which of the following nursing interventions are appropriate to prevent skin breakdown? Select all that apply. A. Range of motion every 4 hours B. Turn and reposition every 2 hours C. Abdominal and foot massages every 2 hours D. Alternating air pressure mattress E. Sit in chair for 30 minutes each shift

B & D Edematous tissue must receive meticulous care to prevent tissue breakdown. An air pressure mattress, careful repositioning can prevent skin breakdown. Inspect pressure points and skin surfaces closely and routinely. Gently massage bony prominences or areas of continued stress. Use of emollient lotions and limiting use of soap for bathing may help. Option A: Range of motion exercises preserve joint function but do not prevent skin breakdown. Encourage and assist the patient with reposition on a regular schedule. Assist with active and passive ROM exercises as appropriate. Option B: Repositioning reduces pressure on edematous tissues to improve circulation. Exercises enhance circulation and improve and/or maintain joint mobility. Edematous tissues are more prone to breakdown and to the formation of decubitus. Ascites may stretch the skin to the point of tearing in severe cirrhosis. Option C: Abdominal or foot massage will not prevent skin breakdown but must be cleaned carefully to prevent breaks in skin integrity. Keep linens dry and free of wrinkles. Moisture aggravates pruritus and increases the risk of skin breakdown. Option D: Use an alternating pressure mattress, egg-crate mattress, waterbed, sheepskins, as indicated. Reduces dermal pressure, increases circulation, and diminishes the risk of tissue ischemia. Option E: The feet should be kept at the level of the heart or higher so Fowler's position should not be employed. Recommend elevating lower extremities. Enhances venous return and reduces edema formation in extremities.

While caring for a client with peptic ulcer disease, the client reports that he has been nauseated most of the day and is now feeling lightheaded and dizzy. Based upon these findings, which nursing actions would be most appropriate for the nurse to take? Select all that apply. A. Administering an antacid hourly until nausea subsides. B. Monitoring the client's vital signs. C. Notifying the physician of the client's symptoms. D. Initiating oxygen therapy. E. Reassessing the client in an hour.

B and C. The symptoms of nausea and dizziness in a client with peptic ulcer disease may be indicative of hemorrhage and should not be ignored. The appropriate nursing actions at this time are for the nurse to monitor the client's vital signs and notify the physician of the client's symptoms. Option A: To administer an antacid hourly or to wait one hour to reassess the client would be inappropriate; prompt intervention is essential in a client who is potentially experiencing a gastrointestinal hemorrhage. Option B: Instruct the client on what signs and symptoms to report to the health care provider. Recognizing the signs and symptoms can help ensure the early initiation of treatment. Explain the pathophysiology of disease and how it relates to the functioning of the body. Option C: The nurse would notify the physician of assessment findings. Assess for body weight changes. Weight loss is an indication of inadequate nutritional intake. Gastric ulcers are more likely to be associated with vomiting, loss of appetite, and weight loss than duodenal ulcers. Option D: Then the nurse would initiate oxygen therapy if ordered by the physician. Monitor laboratory values for serum albumin. This test indicates the degree of protein depletion (2.5 g/dL indicates severe depletion; 3.8 to 4.5 g/dL is normal). Option E: Obtain a nutritional history. Clients may often overestimate the amount of food eaten. The client may not eat sufficient calories or essential nutrients as a way to reduce pain episodes with peptic ulcer disease. Because of this, clients are at high risk for malnutrition.

The nurse is giving a client cimetidine. Which of the following are adverse reactions for which the nurse should monitor? Select all that apply. A. Tinnitus B. Diarrhea C. Dizziness D. Mental confusion E. Alopecia

B, C, D Cimetidine is a histamine H2 agonist. This drug is used to treat ulcers, prevent GI bleeding, and heartburn. It has the potential to cause diarrhea, mental confusion, nausea, and dizziness, especially in the elderly

The nurse is caring for a client who has had a gastroscopy. Which of the following symptoms may indicate that the client is developing a complication related to the procedure? Select all that apply. A. The client complains of a sore throat. B. The client has a temperature of 100*F. C. The client appears drowsy following the procedure. D. The client complains of epigastric pain. E. The client experiences hematemesis.

B, D, and E. Following a gastroscopy, the nurse should monitor the client for complications, which include perforation and the potential for aspiration. An elevated temperature, complaints of epigastric pain, or the vomiting of blood (hematemesis) are all indications of a possible perforation and should be reported promptly. Option A: A sore throat is a common occurrence following a gastroscopy. Before the procedure, the throat will be numbed with a local anesthetic spray. The local anesthetic spray is then given and a small plastic mouth guard placed in the mouth, to hold it open and protect the teeth. Option B: During a gastroscopy, there's a very small risk of the endoscope tearing the lining of the esophagus, stomach, or the first section of the small intestine (duodenum). This is known as perforation. Option C: Clients are usually sedated to decrease anxiety and the nurse would anticipate that the client will be drowsy following the procedure. Possible complications that can occur include a reaction to the sedative, which can cause problems with breathing, heart rate, and blood pressure. Option D: Signs of perforation can include neck, chest, or stomach pain, a high temperature of 38C or above, breathing difficulties, and neck, chest, or stomach pain. If the perforation isn't severe, it can usually be left to heal by itself. Option E: Sometimes, during a gastroscopy, the endoscope can accidentally damage a blood vessel, causing it to bleed. However, significant bleeding is very rare. Signs of bleeding can include vomiting and passing black or "tar-like" poo.

When planning care for a client with ulcerative colitis who is experiencing symptoms, which client care activities can the nurse appropriately delegate to an unlicensed assistant? Select all that apply. A. Assessing the client's bowel sounds. B. Providing skincare following bowel movements. C. Evaluating the client's response to antidiarrheal medications. D. Maintaining intake and output records. E. Obtaining the client's weight.

B, D, and E. The nurse can delegate the following basic care activities to the unlicensed assistant: providing skincare following bowel movements, maintaining intake and output records, and obtaining the client's weight. Assessing the client's bowel sounds and evaluating the client's response to medication are registered nurse activities that cannot be delegated. Option A: Assessing a patient's abdomen can provide critical information about his internal organs. Always follow this sequence: inspection, auscultation, percussion, and palpation. Changing the order of these assessment techniques could alter the frequency of bowel sounds and make your findings less accurate. Option C: The evaluation of the effectiveness of pharmacotherapy includes measurable improvement in clinical signs and symptoms and/or laboratory values. The evaluation of the safety of pharmacotherapy includes evidence of adverse drug reactions and/or toxicity.

Which information will the nurse include when teaching a patient how to avoid chronic constipation? (Select all that apply.) A. Stimulant and saline laxatives can be used regularly. B. Bulk-forming laxatives are an excellent source of fiber. C. Walking or cycling frequently will help bowel motility. D. A good time for a bowel movement may be after breakfast. E. Some over-the-counter (OTC) medications cause constipation.

B,C,D,E Stimulant and saline laxatives should be used infrequently. Use of bulk-forming laxatives, regular early morning timing of defecation, regular exercise, and avoiding many OTC medications will help the patient avoid constipation.

To prevent gastroesophageal reflux in a male client with a hiatal hernia, the nurse should provide which of the following discharge instructions? A. "Lie down after meals to promote digestion." B. "Avoid coffee and alcoholic beverages." C. "Take antacids with meals." D. "Limit fluid intake with meals."

B. "Avoid coffee and alcoholic beverages." To prevent reflux of stomach acid into the esophagus, the nurse should advise the client to avoid foods and beverages that increase stomach acids, such as coffee and alcohol. Instruct the patient regarding avoidance of alcohol, smoking, and caffeinated beverages. These increase acid production and may cause esophageal spasms. Option A: The nurse also should teach the client to avoid lying down after meals, which can aggravate reflux. Instruct to remain in an upright position at least 2 hours after meals; avoiding eating 3 hours before bedtime. This helps control reflux and causes less irritation from reflux action into the esophagus. Option C: The nurse also should teach the client to take antacids after eating. Instruct the patient in medications, effects, side effects, and to report to the physician if symptoms persist despite medication treatment. Promotes knowledge, facilitates compliance with treatment, and allows for prompt identification of potential need for changes in medication regimen to prevent complications. Option D: The client need not limit fluid intake with meals as long as the fluids aren't gastric irritants. Instruct the patient to avoid highly seasoned food, acidic juices, alcoholic drinks, bedtime snacks, and foods high in fat. These can reduce the lower esophageal sphincter pressure.

For Jayvin who is taking antacids, which instruction would be included in the teaching plan? A. "Take the antacids with 8 oz of water." B. "Avoid taking other medications within 2 hours of this one." C. "Continue taking antacids even when pain subsides." D. "Weigh yourself daily when taking this medication."

B. "Avoid taking other medications within 2 hours of this one." Antacids neutralize gastric acid and decrease the absorption of other medications. The client should be instructed to avoid taking other medications within 2 hours of the antacid. The antacids act by neutralizing the acid in the stomach and by inhibiting pepsin, which is a proteolytic enzyme. Each of these cationic salts has a characteristic pharmacological property that determines its clinical use. Option A: Water, which dilutes the antacid, should not be taken with an antacid. The dose for antacids depends upon the age of the patient, the purpose of administration (neutralization of acid or off-label use), and the presence of other comorbidities like renal or hepatic impairment. Option C: A histamine receptor antagonist should be taken even when the pain subsides. Calcium salts neutralize gastric acidity resulting in increased gastric and duodenal bulb pH; they additionally inhibit the proteolytic activity of pepsin if the pH is greater than 4 and increase lower esophageal sphincter tone. Option D: Daily weights are indicated if the client is taking a diuretic, not an antacid. The average therapeutic dose of antacid is 10 to 15 mL (1 tablespoon or one package content) of liquid or 1 to 2 tablets 3 to 4 times a day. Periodic monitoring of calcium and phosphorus plasma concentrations is a suggested practice in patients on chronic therapy.

George has a T tube in place after gallbladder surgery. Before discharge, what information or instructions should be given regarding the T tube drainage? A. "If there is any drainage, notify the surgeon immediately." B. "The drainage will decrease daily until the bile duct heals." C. "First, the drainage is dark green; then it becomes dark yellow." D. "If the drainage stops, milk the tube toward the puncture wound."

B. "The drainage will decrease daily until the bile duct heals." As healing occurs from the bile duct, bile drains from the tube; the amount of bile should decrease. Teach the patient to expect dark green drainage and to notify the doctor if drainage stops. Monitor intake and output (I&O) carefully, measure liquid stool. Weigh regularly. This provides direct indicators of fluid balance. Greatest fluid losses occur with an ileostomy, but they generally do not exceed 500-800 mL/day. Option A: Monitor vital signs, noting postural hypotension, tachycardia. Evaluate skin turgor, capillary refill, and mucous membranes. Reflects hydration status and/or the possible need for increased fluid replacement. Option C: Recommend foods used to manage constipation (bran, celery, raw fruits), and discuss the importance of increased fluid intake. Proper management can prevent or minimize problems of constipation. Option D: Irrigations may be done on a daily basis if appropriate, although there are differing views on this practice. Many believe cleaning the bowel on a regular basis is helpful. Others believe that this interferes with normal functioning.

The nurse is developing a weight loss plan for a 21-year-old obese patient. Which statement by the nurse is most likely to help the patient in losing weight on the planned 800-calorie diet? A. "You will decrease your risk for future health problems such as diabetes by losing weight now." B. "You are likely to start to notice changes in how you feel with just a few weeks of diet and exercise." C. "Most of the weight that you lose during the first weeks of dieting is water weight rather than fat." D. "It will be necessary to change lifestyle habits permanently to maintain weight loss."

B. "You are likely to start to notice changes in how you feel with just a few weeks of diet and exercise."

Develop a teaching care plan for Angie who is about to undergo a liver biopsy. Which of the following points do you include? A. "You'll need to lie on your stomach during the test." B. "You'll need to lie on your right side after the test." C. "During the biopsy, you'll be asked to exhale deeply and hold it." D. "The biopsy is performed under general anesthesia."

B. "You'll need to lie on your right side after the test." After a liver biopsy, the patient is placed on the right side to compress the liver and to reduce the risk of bleeding or bile leakage. The risk of fatal hemorrhage in patients without malignant disease is 0.04%, and the risk of nonfatal hemorrhage is 0.16%. In those with malignancy, the risk of nonfatal hemorrhage is 0.4% and 0.57% for nonfatal hemorrhage. Option A: The patient is usually kept in the right decubitus position. The duration of observation varies across centers ranging from 1 hour to 6 hours. The American Association for the Study of Liver Diseases guidelines recommends observation for 2 to 4 hours. The vital signs are monitored every 15 minutes for the first hour, every 30 minutes for the next hour, and hourly till discharge. Option C: The patients are made to lie in a comfortable supine position. The right hand is placed under the head in a neutral position. By percussion, the area of maximum dullness is identified over the right hemithorax. This is typically between the 6 and 9 intercostal spaces between the anterior and the midclavicular line. Option D: The skin is prepped and draped in a sterile fashion. The overlying skin is anesthetized using 1% lidocaine. The peritoneum is also anesthetized by inserting the needle along the upper border of the rib avoiding vascular structures.

A male client is recovering from a small bowel resection. To relieve pain, the physician prescribes meperidine (Demerol), 75 mg I.M. every 4 hours. How soon after administration should meperidine onset of action occur? A. 5 to 10 minutes B. 15 to 30 minutes C. 30 to 60 minutes D. 2 to 4 hours

B. 15 to 30 minutes Meperidine onset of action is 15 to 30 minutes. It peaks between 30 and 60 minutes and has a duration of action of 2 to 4 hours. Meperidine, also known as pethidine, is in the class of phenylpiperidine as a hydrochloride salt synthetic form of the opioid, which is a white crystalline with a melting point of 186°C. This medication has calcium sulfate, dibasic calcium phosphate, starch, stearic acid, and talc as inactive ingredients. Clinicians use meperidine for the treatment of moderate to severe pain. Option A: Oral form is not recommended for acute pain control. Using Meperidine for pain control should be considered if no other options are available, in which case the duration of medication use should be limited to less than 48 hours, and the total dosage administered should not exceed 600 mg in 24 hours. Option C: Meperidine has the same mechanism of action as morphine, which is acting as an agonist to the mu-opioid receptor. The anti-shivering effect may involve the stimulation of k-opioid receptors. Option D: Meperidine also has some local anesthetic effects because of its interactions with sodium ion channels. Also, meperidine has stimulant effects by inhibition of the dopamine transporter (DAT) and norepinephrine transporter (NET).

A male client is recovering from a small bowel resection. To relieve pain, the physician prescribes meperidine (Demerol), 75 mg I.M. every 4 hours. How soon after administration should meperidine onset of action occur? A. 5 to 10 minutes B. 15 to 30 minutes C. 30 to 60 minutes D. 2 to 4 hours

B. 15 to 30 minutes Meperidine's onset of action is 15 to 30 minutes. It peaks between 30 and 60 minutes and has a duration of action of 2 to 4 hours. Meperidine is in the class of phenylpiperidine as a hydrochloride salt synthetic form of the opioid. Meperidine is used for the treatment of moderate to severe pain. It has intramuscular, subcutaneous, intravenous injection, syrup, and tablet forms. Option A: Injection should be into large muscle mass, and it is preferable to subcutaneous injection. Using Meperidine for pain control should be considered if no other options are available, in which case the duration of medication use should be limited to less than 48 hours, and the total dosage administered should not exceed 600 mg in 24 hours. Option C: For intravenous (IV) injection, inject the dose of 10 mg/ml slowly. The injection should be a consideration only when an opiate antagonist and the administration of oxygen and respiratory monitoring facilities are available. In the 20th century, it was the drug of choice amongst the opioids in the management of acute pain by most physicians and the management of some patients with chronic pain. Meperidine is also being used as an adjunct to preoperative medications to reduce shivering. Option D: Meperidine has the same mechanism of action as morphine, which is acting as an agonist to the mu-opioid receptor. The anti-shivering effect may involve the stimulation of k-opioid receptors. The nurse is caring for a male client with cirrhosis. Which assessment findings indicate that the client has deficient vitamin K absorption caused by this hepatic disease? A. Dyspnea and fatigue B. Ascites and orthopnea C. Purpura and petechiae D. Gynecomastia and testicular atrophy C. Purpura and petechiae A hepatic disorder, such as cirrhosis, may disrupt the liver's normal use of vitamin K to produce prothrombin (a clotting factor). Consequently, the nurse should monitor the client for signs of bleeding, including purpura and petechiae. Petechiae and purpura result from a wide variety of underlying disorders and may occur at any age. Petechiae are small (1-3 mm), red, non-blanching macular lesions caused by intradermal capillary bleeding. Purpura are larger, typically raised lesions resulting from bleeding within the skin Option A: Dyspnea and fatigue suggest anemia. When patients with severe anemia develop a high output state, cardiac failure can ensue causing them to have shortness of breath from cardiac causes as well. Diseases in other organs, such as the kidneys and the liver, may cause dyspnea by a combination of the interactions discussed. Option B: Ascites and orthopnea are unrelated to vitamin K absorption. Patients with malignant ascites can have symptoms related to malignancy, which may include weight loss. On the other hand, patients with ascites due to heart failure may report dyspnea, orthopnea, and peripheral edema, and those with chylous ascites report diarrhea, steatorrhea, malnutrition, edema, nausea, enlarged lymph nodes, early satiety, fevers, and night sweats. Option D: Gynecomastia and testicular atrophy result from decreased estrogen metabolism by the diseased liver. Gynecomastia is most commonly caused by an imbalance between the hormones estrogen and testosterone. Estrogen controls female traits, including breast growth. Testosterone controls male traits, such as muscle mass and body hair.

Which of the following symptoms is a client with colon cancer most likely to exhibit? A. A change in appetite. B. A change in bowel habits. C. An increase in body weight. D. An increase in body temperature.

B. A change in bowel habits. The most common complaint of the client with colon cancer is a change in bowel habits. Tumor location on the clinical presentation can be separated on left-sided with more changes in bowel habits and hematochezia, and right-sided with obscured anemia impacting on late-stage at diagnosis. Option A: The client may have anorexia, secondary abdominal distention. The provider should perform a thorough physical examination for signs of ascites, hepatomegaly, and lymphadenopathy. Comprehensive family history is of great relevance to identify familial clusters and inherent patterns that would alter surveillance and therapy on the high-risk patient. Option C: The client may have weight loss. Late presentation with metastatic disease at diagnosis will depend on the symptoms at the organ affected by the route of spread; to the liver via the portal system, to lungs via the inferior vena cava, to supraclavicular adenopathy via lymphatic or to neighbor structures by contiguous invasion. Option D: Fever isn't associated with colon cancer. Diagnostic colonoscopy's triggers are blood per rectum (37%), abdominal pain (34%), and anemia (23%). The most common indications of emergency surgery are obstruction (57%), peritonitis (25%), and perforation (18%).

Which of the following definitions best describes diverticulosis? A. An inflamed outpouching of the intestine. B. A noninflamed outpouching of the intestine. C. The partial impairment of the forward flow of intestinal contents. D. An abnormal protrusion of an organ through the structure that usually holds it.

B. A noninflamed outpouching of the intestine. Diverticulosis involves a noninflamed outpouching of the intestine. Diverticulosis is a clinical condition in which multiple sac-like protrusions (diverticula) develop along the gastrointestinal tract. Though diverticula may form at weak points in the walls of either the small or large intestines, the majority occur in the large intestine (most commonly the sigmoid colon). Option B: Diverticulitis involves an inflamed outpouching. Acute diverticulitis is inflammation due to micro-perforation of a diverticulum. The diverticulum is a sac-like protrusion of the colon wall. Diverticulitis can present in about 10% to 25% of patients with diverticulosis. Diverticulitis can be simple or uncomplicated and complicated. Option C: The partial impairment of forward flow of the intestine is an obstruction. Obstruction causes dilation of the bowel proximal to the transition point and collapses distally. A result of partial or complete blockage of digested products during obstruction is emesis. Frequent emesis can lead to fluid deficits and electrolyte abnormalities. Option D: Abnormal protrusion of an organ is a hernia. A hernia occurs when an organ pushes through an opening in the muscle or tissue that holds it in place. For example, the intestines may break through a weakened area in the abdominal wall. Many hernias occur in the abdomen between the chest and hips, but they can also appear in the upper thigh and groin areas.

Which of the following symptoms would a client in the early stages of peritonitis exhibit? A. Abdominal distention B. Abdominal pain and rigidity C. Hyperactive bowel sounds D. Right upper quadrant pain

B. Abdominal pain and rigidity Abdominal pain-causing rigidity of the abdominal muscles is characteristic of peritonitis. During the physical exam, pertinent findings include fever and abdominal tenderness to palpation which usually is diffuse with wall rigidity in more septic presentations. Option A: Abdominal distention may occur as a late sign but not early on. It is important to conduct a thorough exam as certain thoracic or pelvic pathologies can mimic peritoneal irritation (empyema causing diaphragmatic irritation and cystitis/pyelonephritis causing peritoneum adjacent pain). Option C: Bowel sounds may be normal or decreased but not increased. Important to note, approximately 30% of individuals with SBP will be asymptomatic on presentation. Patients will present with a variable amount of clinical manifestation of the underlying disease process, ranging from insidious mild limited disease to an acute fulminant systemic process. Option D: Right upper quadrant pain is characteristic of cholecystitis or hepatitis. Concerning signs present in a high percentage of individuals with diagnosed peritonitis include vague constitutional symptoms such as fever chills, abdominal pain +/- discomfort, diarrhea, and ileus.

Which of the following tests is most commonly used to diagnose cholecystitis? A. Abdominal CT scan B. Abdominal ultrasound C. Barium swallow D. Endoscopy

B. Abdominal ultrasound An abdominal ultrasound can show if the gallbladder is enlarged, if gallstones are present, if the gallbladder wall is thickened, or if distention of the gallbladder lumen is present. A gallbladder ultrasound is the best test to evaluate gallbladder disease initially. A thickened gallbladder wall and gallstones are common findings with this condition. Option A: An abdominal CT scan can be used to diagnose cholecystitis, but it usually isn't necessary. Often a CT scan is ordered in the emergency department as the first test in the workup. Findings of cholecystitis and gallstones can often be seen in this imaging. Option C: A barium swallow looks at the stomach and the duodenum. The barium swallow study, also known as a barium esophagogram or esophagram, is a contrast-enhanced radiographic study commonly used to assess structural characteristics of the entire esophagus. It may be used for the diagnosis of a wide range of pathologies including esophageal motility disorders, strictures, and perforations. Option D: Endoscopy looks at the esophagus, stomach, and duodenum. Endoscopy is the insertion of a long, thin tube directly into the body to observe an internal organ or tissue in detail. It can also be used to carry out other tasks including imaging and minor surgery. Endoscopes are minimally invasive and can be inserted into the openings of the body such as the mouth or anus.

You have to teach ostomy self-care to a patient with a colostomy. You tell the patient to measure and cut the wafer: A. To the exact size of the stoma. B. About 1/16" larger than the stoma. C. About 1/8" larger than the stoma.

B. About 1/16" larger than the stoma. A proper fit protects the skin but doesn't impair circulation. A 1/16" should be cut. After the stoma has been measured, the patient will need to cut an opening that matches the measurement on the back of the barrier. If a one-piece pouch is being used, be sure to pull the pouch away to prevent cutting into the plastic. The opening should match the size of the stoma leaving no exposed skin. Option A: Not all wafers will act the same when they are worn. While all wafers tend to swell a bit as they come in contact with fluids or output, the degree in which they swell can differ from brand to brand. Because of this, the patient may need to cut the wafer slightly larger to accommodate the expansion. Doing this also helps to prevent the wafer from "strangulating" the stoma by putting too much pressure around it. Option C: Another thing about wafers is that they come in all kinds of sizes and materials. If the patient uses a wafer that doesn't give enough space, he may end up having difficulties cutting it, or worse, develop leaks. Option D: The patient should make sure that there's about a 1/16 - 1/8? (approx. 5 - 3mm) gap between the stoma and the edge of the hole. Remember, as wafers do swell, he may need to adjust this gap accordingly.

Which of the following will the nurse include in the care plan for a client hospitalized with viral hepatitis? A. Increase fluid intake to 3000 ml per day B. Adequate bed rest C. Bland diet D. Administer antibiotics as ordered

B. Adequate bed rest. Treatment of hepatitis consists of bed rest during the acute phase to reduce metabolic demands on the liver, thus increasing blood supply and cell regeneration. Institute bed red or chair rest during the toxic state. Provide a quiet environment; limit visitors as needed. Promotes rest and relaxation. Available energy is used for healing. Activity and an upright position are believed to decrease hepatic blood flow, which prevents optimal circulation to the liver cells. Option A: Monitor I&O, compare with periodic weight. Note enteric losses: vomiting and diarrhea. Diarrhea may be due to transient flu-like response to viral infection or may represent a more serious problem of obstructed portal blood flow with vascular congestion in the GI tract, or it may be the intended result of medication use (neomycin, lactulose) to decrease serum ammonia levels in the presence of hepatic encephalopathy. Option C: Encourage intake of fruit juices, carbonated beverages, and hard candy throughout the day. Monitor dietary intake and caloric count. Suggest several small feedings and offer the "largest" meal at breakfast. Large meals are difficult to manage when a patient is anorexic. Anorexia may also worsen during the day, making intake of food difficult later in the day. Option D: Establish isolation techniques for enteric and respiratory infections according to infection guidelines and policy. Encourage or model effective handwashing. Prevents transmission of viral disease to others. Thorough hand washing is effective in preventing virus transmission.

Which of the following factors is believed to cause ulcerative colitis? A. Acidic diet B. Altered immunity C. Chronic constipation D. Emotional stress

B. Altered immunity Several theories exist regarding the cause of ulcerative colitis. One suggests altered immunity as the cause based on the extraintestinal characteristics of the disease, such as peripheral arthritis and cholangitis. Although there is little evidence to support this, it has been postulated that alterations in the composition of the gut microbiota and defects in mucosal immunity could lead to ulcerative colitis. Autoimmunity may also play an important role in the etiology of ulcerative colitis. Option A: Diet has no effect on the development of ulcerative colitis. The specific cause of inflammatory bowel disease is not known. There seems to be a primary genetic component since the most important independent risk factor is a family history of the disease (8% to 14% of patients). Option C: Chronic constipation does not affect the development of ulcerative colitis. A first-degree relative of a patient with ulcerative colitis has a four times higher risk of developing the disease. Additionally, ulcerative colitis has a higher incidence in Jewish populations than other ethnicities. Option D: Emotional stress can exacerbate the attacks but isn't believed to be the primary cause. Some evidence suggests that smoking may be protective but so far no one has been able to confirm a direct relationship between the two.

A male client has just been diagnosed with hepatitis A. On assessment, the nurse expects to note: A. Severe abdominal pain radiating to the shoulder. B. Anorexia, nausea, and vomiting. C. Eructation and constipation. D. Abdominal ascites.

B. Anorexia, nausea, and vomiting. Hallmark signs and symptoms of hepatitis A include anorexia, nausea, vomiting, fatigue, and weakness. Acute hepatitis usually presents as a self-limited illness; development of fulminant hepatitis is rare. Typical symptoms of acute infection include nausea, vomiting, abdominal pain, fatigue, malaise, poor appetite, and fever; management is with supportive care. Option A: Abdominal pain may occur but doesn't radiate to the shoulder. Extrahepatic manifestations rarely occur but may include pancreatitis, rash, acute kidney injury with interstitial nephritis or glomerulonephritis, pneumonitis, pericarditis, hemolysis, and acute cholecystitis. Option C: Eructation and constipation are common in gallbladder disease, not hepatitis A. Patients may develop dark urine and pale stools within a week, followed by jaundice, icteric (yellow-tinted) sclera, and pruritus. Patients usually have elevated levels of serum alanine aminotransferase, aspartate aminotransferase, bilirubin, alkaline phosphatase, and lambda-glutamyl transpeptidase. Option D: Abdominal ascites is a sign of advanced hepatic disease, not an early sign of hepatitis A. Ascites is the pathologic accumulation of fluid within the peritoneal cavity. It is the most common complication of cirrhosis and occurs in about 50% of patients with decompensated cirrhosis in 10 years. The development of ascites denotes the transition from compensated to decompensated cirrhosis.

If a gastric acid perforates, which of the following actions should not be included in the immediate management of the client? A. Blood replacement B. Antacid administration C. Nasogastric tube suction D. Fluid and electrolyte replacement

B. Antacid administration Antacids aren't helpful in perforation. The client should be treated with antibiotics. Perforation of the stomach is a full-thickness injury of the wall of the organ. Since peritoneum completely covers the stomach, perforation of the wall creates a communication between the gastric lumen and the peritoneal cavity. If the perforation occurs acutely, there is no time for an inflammatory reaction to wall off the perforation, and the gastric content is free to enter the general peritoneal cavity, causing chemical peritonitis. Option A: The client should be treated with blood replacement. Broad-spectrum antibiotics have been shown to reduce the risk of wound infection. Metronidazole and either a cephalosporin or an aminoglycoside will suffice. Option C: NG tube suction should also be performed to prevent further spillage of stomach contents into the peritoneal cavity. A nasogastric tube should also be placed. Intravenous analgesia and PPIs should be given as necessary. A urinary catheter enables close monitoring of urine output. Surgical management is the mainstay of treatment for most stomach perforations. Option D: The client should be treated with fluid and electrolyte replacement. Initial management consists of aggressive resuscitation, oxygen therapy, intravenous fluids, and broad-spectrum antibiotics. Definitive surgical treatment should be done at the earliest possible time.

The nurse is monitoring a female client receiving paregoric to treat diarrhea for drug interactions. Which drugs can produce additive constipation when given with an opium preparation? A. Antiarrhythmic drugs B. Anticholinergic drugs C. Anticoagulant drugs D. Antihypertensive drugs

B. Anticholinergic drugs Paregoric has an additive effect of constipation when used with anticholinergic drugs. The opiate anhydrous morphine, which is contained in paregoric, can decrease motility more than loperamide or the combination of diphenoxylate and atropine can. Antiarrhythmics, anticoagulants, and antihypertensives aren't known to interact with paregoric. Option A: Of the Class III antiarrhythmics, amiodarone is involved in a significant number of interactions since it is a potent inhibitor of several cytochrome P450 enzymes. It can significantly impair the metabolism of digoxin, theophylline and warfarin. Dosages of digoxin and warfarin should empirically be decreased by one-half when amiodarone therapy is added. Option C: The anticoagulant effect of warfarin is inhibited by drugs like barbiturates, rifampin, azathioprine, and carbamazepine, which increase its clearance by inducing hepatic metabolism. Azathioprine also reduces the anticoagulant effect of warfarin, presumably through a potentiating effect on hepatic clearance. Option D: Nonsteroidal anti-inflammatory drugs (NSAIDs) can induce an increase in blood pressure (BP) and may potentially reduce the efficacy of several antihypertensive drugs. Probably the main mechanism of action is inhibition of prostaglandin (PG) synthesis since NSAIDs have a higher propensity to increase BP as the regulation of BP (and renal function) is more PG-dependent and to interact with drugs (diuretics, beta-blockers, and ACE inhibitors) that may act through the increase of PG formation.

Which of the following factors should be the main focus of nursing management for a client hospitalized for cholecystitis? A. Administration of antibiotics. B. Assessment for complications. C. Preparation for lithotripsy. D. Preparation for surgery.

B. Assessment for complications. The client with acute cholecystitis should first be monitored for perforation, fever, abscess, fistula, and sepsis. Review signs and symptoms requiring medical intervention: recurrent fever; persistent nausea and vomiting, or pain; jaundice of skin or eyes, itching; dark urine; clay-colored stools; blood in urine, stools, vomitus; or bleeding from mucous membranes. Option A: After an assessment, antibiotics will be administered to reduce the infection. Occlusion of the cystic duct or malfunction of the mechanics of gallbladder emptying is the pathophysiology of this disease. Cases of acute untreated cholecystitis could lead to perforation of the gallbladder, sepsis, and death. Option C: Lithotripsy is used only for a small percentage of clients. The preferred recommended treatment is the removal of the gallbladder. In the past, this was done through an open laparotomy incision. Now laparoscopic cholecystectomy is the procedure of choice. This procedure has low mortality and morbidity, a quick recovery time (usually one week), and good results. Option D: Surgery is usually done after the acute infection has subsided. The most appropriate management of cholecystitis is laparoscopic cholecystectomy. There are low morbidity and mortality rates with quick recovery. This can also be done with an open technique in cases where the patient is not a good laparoscopic candidate.

The nurse is preparing a discharge teaching plan for the male client who had umbilical hernia repair. What should the nurse include in the plan? A. Irrigating the drain B. Avoiding coughing C. Maintaining bed rest D. Restricting pain medication

B. Avoiding coughing. Coughing is avoided following umbilical hernia repair to prevent disruption of tissue integrity, which can occur because of the location of this surgical procedure. Splint the stomach by placing a pillow over the abdomen with firm pressure before coughing or movement to help reduce the pain. Option A: A drain is not used in this surgical procedure, although the client may be instructed in simple dressing changes. Do not soak in a bathtub until the stitches or staples are removed. A small amount of drainage from the incision is normal. Option C: Bed rest is not required following this surgical procedure. The client may slowly increase his activity. He should get up and walk every hour or so to prevent blood clot formation. After recovery, the client may return to work within 2 or 3 days. There should be no lifting anything above 10 lbs, climbing, or any strenuous activities for 4 to 6 weeks. Option D: The client should take analgesics as needed and as prescribed to control pain. Most non-opioid analgesics are classified as non-steroidal anti-inflammatory drugs (NSAIDs). They are used to treat mild pain and inflammation or combined with narcotics. Narcotics or opioids are used for severe pain.

Prophylaxis for hepatitis A includes: A. preventing constipation B. screening of blood donors C. avoiding shellfish in diet D. limiting hepatotoxic drug therapy

B. Avoiding shellfish in diet

Which of the following tests should be administered to a client suspected of having diverticulosis? A. Abdominal ultrasound B. Barium enema C. Barium swallow D. Gastroscopy

B. Barium enema A barium enema will cause diverticula to fill with barium and be easily seen on x-ray. A barium enema is a radiographic (X-ray) examination of the lower gastrointestinal (GI) tract. The large intestine, including the rectum, is made visible on X-ray film by filling the colon with a liquid suspension called barium sulfate (barium). Barium highlights certain areas in the body to create a clearer picture. Option A: The abdominal US can tell more about structures, such as the gallbladder, liver, and spleen, than the intestine. An abdominal ultrasound is a noninvasive procedure used to assess the organs and structures within the abdomen. This includes the liver, gallbladder, pancreas, bile ducts, spleen, and abdominal aorta. Ultrasound technology allows quick visualization of the abdominal organs and structures from outside the body. Option C: A barium swallow can view upper GI structures. A barium swallow also called an esophagram, is an imaging test that checks for problems in the upper GI tract. The upper GI tract includes the mouth, back of the throat, esophagus, stomach, and first part of the small intestine. The test uses a special type of x-ray called fluoroscopy. Option D: A gastroscopy is a procedure where a thin, flexible tube called an endoscope is used to look inside the esophagus (gullet), stomach, and the first part of the small intestine (duodenum). It's also sometimes referred to as an upper gastrointestinal endoscopy. The endoscope has a light and a camera at one end. The camera sends images of the inside of the esophagus, stomach, and duodenum to a monitor.

A client has a serious complication of chronic acid entering the esophagus over many years. The client's esophageal lining has pathologically changed to resemble tissue in the intestine. What is this called? A. GERD B. Barrett's esophagus C. Heartburn D. Esophagitis

B. Barrett's esophagus Barrett's esophagus is a serious condition that can lead to esophageal cancer due to acid chronically entering the esophagus over many years, which changes the lining to resemble intestinal lining from irritation GERD - Gastroesophageal reflux disease can lead to Barrett's esophagus, but it is not the definition of Barrett's esophagus Esophagitis - Esophagitis is an infection leading to inflammation of the esophagus rather than from acid Heartburn - heartburn is caused when stomach acids back up into the throat, but is not necessarily chronic nor does it change the lining of the esophagus

Regina is a 46 y.o. woman with ulcerative colitis. You expect her stools to look like: A. Watery and frothy B. Bloody and mucous C. Firm and well-formed D. Alternating constipation and diarrhea

B. Bloody and mucous Stools from ulcerative colitis are often bloody and contain mucus. Ulcerative colitis is an idiopathic inflammatory condition of the colon which results in diffuse friability and superficial erosions on the colonic wall and associated bleeding. It is the most common form of inflammatory bowel disease worldwide. Option A: A bacterial, parasitic, or viral infection can invade the gastrointestinal tract and create gas bubbles, making stool appear foamy. A common source of infection is the Giardia parasite. A person may be infected after consuming contaminated water or food. They may also come into contact with contaminated water when swimming, for example. Option C: A healthy bowel movement should be painless and require minimal strain. Soft to firm in texture: Stool that is passed in one single piece or a few smaller pieces is typically considered to be a sign of a healthy bowel. The long, sausage-like shape of the stool is due to the shape of the intestines. Option D: Ulcerative colitis's main symptom is bloody diarrhea, with or without mucus. Associated symptoms also include urgency or tenesmus, abdominal pain, malaise, weight loss, and fever, depending on the extent and severity of the disease. The onset of the disease is typically gradual, and patients will likely experience periods of spontaneous remission and subsequent relapses.

The nurse is caring for a male client postoperatively following the creation of a colostomy. Which nursing diagnosis should the nurse include in the plan of care? A. Sexual dysfunction B. Body image, disturbed C. Fear related to poor prognosis D. Nutrition: more than body requirements, imbalanced

B. Body image, disturbed Body image, disturbed relates to loss of bowel control, the presence of a stoma, the release of fecal material onto the abdomen, the passage of flatus, odor, and the need for an appliance (external pouch). Encourage the patient/SO to verbalize feelings regarding the ostomy. Acknowledge normality of feelings of anger, depression, and grief over a loss. Discuss daily "ups and downs" that can occur. Option A: Review with the patient and/or SO sexual functioning in relation to their own situation. Understanding if nerve damage has altered normal sexual functioning helps the patient/SO to understand the need for exploring alternative methods of satisfaction. Option C: Provide opportunities for patient/SO to view and touch stoma, using the moment to point out positive signs of healing, normal appearance, and so forth. Remind the patient that it will take time to adjust, both physically and emotionally. Option D: Nutrition: less than body requirements, imbalanced is the more likely nursing diagnosis. Recommend patient increase use of yogurt, buttermilk, and acidophilus preparations. Identify odor-causing foods (e.g., cabbage, fish, beans) and temporarily restrict them from the diet. Gradually reintroduce one food at a time.

In developing a teaching plan for the client with a hiatal hernia, the nurse's assessment of which work-related factors would be most useful? A. Number and length of breaks. B. Body mechanics used in lifting. C. Temperature in the work area. D. Cleaning solvents used.

B. Body mechanics used in lifting.

Your patient, Christopher, has a diagnosis of ulcerative colitis and has severe abdominal pain aggravated by movement, rebound tenderness, fever, nausea, and decreased urine output. This may indicate which complication? A. Fistula B. Bowel perforation C. Bowel obstruction D. Abscess

B. Bowel perforation An inflammatory condition that affects the surface of the colon, ulcerative colitis causes friability and erosions with bleeding. Patients with ulcerative colitis are at increased risk for bowel perforation, toxic megacolon, hemorrhage, cancer, and other anorectal and systemic complications. Colonic perforations are usually a complication of a toxic megacolon. However, perforation can also present in severe ulcerative colitis even in the absence of toxic megacolon. Most perforations occur in the left colon, commonly in the sigmoid colon. Option A: Fistulas can occur anywhere in the bowel. The longer the client has Crohn's, the more likely he is to develop a fistula. In ulcerative colitis the inflammation doesn't spread through the full thickness of the bowel wall, so fistulas are less likely to form. The symptoms the client experiences depend on where the fistula is. Option C: Crohn's disease affects the entire thickness of the bowel wall. This makes strictures more common in people who have ulcerative colitis, which typically affects only the inner lining of the bowel. Bowel obstructions with strictures may either be temporary or permanent. Option D: The typical histological (microscopic) lesion of ulcerative colitis is the crypt abscess, in which the epithelium of the crypt breaks down and the lumen fills with polymorphonuclear cells. The lamina propria is infiltrated with leukocytes.

When teaching a client about pancreatic function, the nurse understands that pancreatic lipase performs which function? A. Transports fatty acids into the brush border B. Breaks down fat into fatty acids and glycerol C. Triggers cholecystokinin to contract the gallbladder D. Breaks down protein into dipeptides and amino acids

B. Breaks down fat into fatty acids and glycerol. Lipase hydrolyses or breaks down fat into fatty acids and glycerol. Lipase is an enzyme that breaks down triglycerides into free fatty acids and glycerol. Lipases are present in pancreatic secretions and are responsible for fat digestion. There are many different types of lipases; for example, hepatic lipases are in the liver, hormone-sensitive lipases are in adipocytes, lipoprotein lipase is in the vascular endothelial surface, and pancreatic lipase in the small intestine. Option A: Lipase is not involved with the transport of fatty acids into the brush border. Lipases are enzymes that play a crucial role in lipid transport. Hepatic lipase plays a crucial role in the formation and delivery of low-density lipoprotein(LDL). LDL is formed by the modification of intermediate density lipoprotein in the peripheral tissue and liver by hepatic lipase. These LDL particles are taken up, or endocytosed, via receptor-mediated endocytosis by target cell tissue. LDL serves to ultimately transport cholesterol from the liver to peripheral tissue. Option C: Fat itself triggers cholecystokinin release. Hormone-sensitive lipase is found within fat tissue and is responsible for degrading the triglycerides that are stored within adipocytes. Fat necrosis occurs enzymatically and non-enzymatically. In acute pancreatitis, saponification of peripancreatic fat occurs. During traumatic events, such as physical injury in breast tissue, non-enzymatic fat necrosis takes place. Option D: Protein breakdown into dipeptides and amino acids is the function of trypsin, not lipase. Lipoprotein lipase is found on the vascular endothelial surface and is responsible for degrading triglycerides that circulating from chylomicrons and VLDLs. Pancreatic lipase is found within the small intestine and is responsible for degrading dietary triglycerides.

A client presents to the emergency department with chest pain. The EKG and troponin labs are all within normal limits. Upon reviewing the client's medication list, the nurse notes and medication for acid reflux. Which of the following medications is indicated for this condition? A. Captopril B. Cimetidine C. Cephalexin D. Clopidogrel

B. Cimetidine Cimetidine Is an anti-ulcer H2 antagonist that treats GERD, ulcers and is used to prevent GI bleeding. When a client with chest pain has negative troponins and a normal EKG, acid reflux is sometimes found to be the cause of the chest pain. Captopril Is an ACE inhibitor that treats hypertension and CHF Cephalexin is an anti-infective medication used for skin infections, pneumonia, UTI's and otitis media Clopidogrel is an anti-platelet agent used to manage cerebral vascular accidents, myocardial infarctions, and peripheral vascular disease

Stephanie, a 28 y.o. accident victim, requires TPN. The rationale for TPN is to provide: A. Necessary fluids and electrolytes to the body. B. Complete nutrition by the I.V. route. C. Tube feedings for nutritional supplementation. D. Dietary supplementation with liquid protein given between meals.

B. Complete nutrition by the I.V. route. TPN is given I.V. to provide all the nutrients your patient needs. Parenteral nutrition is the intravenous administration of nutrition outside of the gastrointestinal tract. Total parenteral nutrition (TPN) is when the IV administered nutrition is the only source of nutrition the patient is receiving. Total parenteral nutrition is indicated when there is an inadequate gastrointestinal function and contraindications to enteral nutrition. Option A: Enteral diet intake is preferred over parenteral as it is inexpensive and associated with fewer complications such as infection and blood clots but requires a functional GI system. TPN is a mixture of separate components which contain lipid emulsions, dextrose, amino acids, vitamins, electrolytes, minerals, and trace elements. TPN composition should be adjusted to fulfill individual patients' needs. The main three macronutrients are lipids emulsions, proteins, and dextrose. Option C: TPN isn't tube feeding. Total parenteral nutrition administration is through a central venous catheter. A central venous catheter is an access device that terminates in the superior vena cava or the right atrium and is used to administer nutrition, medication, chemotherapy, etc. Establishing this access could be through a peripherally inserted central catheter (PICC), central venous catheter, or an implanted port. Option D: TPN is not a liquid dietary supplement. A 3-in-1 solution and intravenous lipid emulsions) mixed with electrolytes, trace elements, vitamins, and water. Parenteral solution with only dextrose and amino acids with a separate intravenous lipid emulsions infusion, the 2-in-1 solution has also been previously used. Research has shown TNA to be the standard of care for adult TPN.

The nurse would monitor for which of the following adverse reactions to aluminum-containing antacids such as aluminum hydroxide (Amphojel)? A. Diarrhea B. Constipation C. GI upset D. Fluid retention

B. Constipation Aluminum- and calcium-containing antacids cause constipation. The primary side effects of aluminum hydroxide include hypomagnesemia, hypophosphatemia, constipation, and anemia. Additionally, due to its ability to stimulate the immune system, there have been observed cases of persistent granulomas at the injection site of vaccines containing aluminum hydroxide. Option A: Magnesium-containing antacids cause diarrhea. Antacids that contain magnesium have a laxative effect that may cause diarrhea, and in patients with renal failure, they may cause increased magnesium levels in the blood, because of the reduced ability of the kidneys to eliminate magnesium from the body in the urine. Option C: Antacids work by counteracting (neutralizing) the acid in the stomach. They do this because the chemicals in antacids are bases (alkalis) which are the opposite of acids. A reaction between an acid and base is called neutralization. This neutralization makes the stomach contents less corrosive. Option D: Sodium-containing antacids cause sodium and fluid retention. High-dose antacid intake may lead to fluid retention in the body depending on the sodium content of the different antacid preparations. Sodium bicarbonate ingestion provokes metabolic alkalosis and alkalemia, the "nonsystemic calcium, and magnesium-containing antacids" cause these changes too, but to a lower degree.

A client with gastric cancer can expect to have surgery for resection. Which of the following should be the nursing management priority for the preoperative client with gastric cancer? A. Discharge planning B. Correction of nutritional deficits C. Prevention of DVT D. Instruction regarding radiation treatment

B. Correction of nutritional deficits Client's with gastric cancer commonly have nutritional deficits and may be cachectic. For patients undergoing surgery, the preoperative nutritional condition directly affects postoperative prognosis, overall survival, and disease-specific survival. The goal of nutritional therapy is to improve the nutritional status, metabolism, adherence to antitumor therapies, quality of life, and course of the disease. Option A: Discharge planning before surgery is important, but correcting the nutrition deficit is a higher priority. Provide accurate, consistent information regarding diagnosis and prognosis. Avoid arguing about the patient's perceptions of the situation. Option C: Prevention of DVT also isn't a high priority to surgery, though it assumes greater importance after surgery. The link between thromboembolism and cancer has been recognized for over 100 years. Venous thromboembolism (VTE) is associated with considerable morbidity in patients with cancer, with emerging research also indicating a detrimental effect on survival. Option D: At present, radiation therapy hasn't been proven effective for gastric cancer, and teaching about it preoperatively wouldn't be appropriate. People with stomach cancer usually receive external-beam radiation therapy, which is radiation given from a machine outside the body. Radiation therapy may be used before surgery to shrink the size of the tumor or after surgery to destroy any remaining cancer cells.

For Rico who has chronic pancreatitis, which nursing intervention would be most helpful? A. Allowing liberalized fluid intake B. Counseling to stop alcohol consumption C. Encouraging daily exercise D. Modifying dietary protein

B. Counseling to stop alcohol consumption. Chronic pancreatitis typically results from repeated episodes of acute pancreatitis. More than half of chronic pancreatitis cases are associated with alcoholism. Counseling to stop alcohol consumption would be the most helpful for the client. Explore the availability of treatment programs and rehabilitation of chemical dependency if indicated. Option A: Resume oral intake with clear liquids and advance diet slowly to provide a high-protein, high-carbohydrate diet, when indicated. Oral feedings given too early in the course of illness may exacerbate symptoms. Loss of pancreatic function and reduced insulin production may require the initiation of a diabetic diet. Option C: Daily exercise would be helpful but not the most beneficial intervention. Review the importance of initially continuing a bland, low-fat diet with frequent small feedings and restricted caffeine, with a gradual resumption of a normal diet within individual tolerance. Option D: Dietary protein modification is not necessary for chronic pancreatitis. Maintain NPO status and gastric suctioning in the acute phase. Prevents stimulation and release of pancreatic enzymes (secretin), released when chyme and HCl enter the duodenum.

After abdominal surgery, your patient has a severe coughing episode that causes wound evisceration. In addition to calling the doctor, which intervention is most appropriate? A. Irrigate the wound & organs with Betadine. B. Cover the wound with a saline-soaked sterile dressing. C. Apply a dry sterile dressing & binder. D. Push the organs back & cover with moist sterile dressings.

B. Cover the wound with a saline-soaked sterile dressing. Cover the organs with a sterile, non-adherent dressing moistened with normal saline. Do this to prevent infection and to keep the organs from drying out. Dehiscence of abdominal surgical wounds is a medical emergency and requires immediate action to reduce further complications. Option A: To decrease intra-abdominal pressure and stress on the wound, place the patient supine in the low Fowler's position with his knees slightly bent and cover the wound with a saline-moistened, sterile gauze dressing. Option C: The physician may order an abdominal binder to help prevent evisceration. To prevent dehiscence, teach patients to splint the surgical site when coughing, vomiting, or sneezing. An abdominal binder for those at risk for dehiscence may be helpful, but evidence supporting its use is still needed. Option D: Do not push back the organs because it may cause injuries. Although dehiscence occurs in less than 3% of abdominal surgeries, it's associated with a mortality of 14% to 50%, with evisceration increasing the risk of death. Conditions that increase intra-abdominal pressure (obesity) or may disrupt skin health (poor nutrition, steroid use, diabetes) place patients at increased risk of dehiscence.

Findings during an endoscopic exam include a cobblestone appearance of the colon in your patient. The findings are characteristic of which disorder? A. Ulcer B. Crohn's disease C. Chronic gastritis D. Ulcerative colitis

B. Crohn's disease Crohn's disease penetrates the mucosa of the colon through all layers and destroys the colon in patches, which creates a cobblestone appearance. As the inflammation progresses, non-caseating granulomas form involving all layers of the intestinal wall. It can develop into the classic cobblestone mucosal appearances and skip lesions along the length of the intestine sparing areas with normal mucosa. Option A: In a gastric ulcer, on histopathology, one will see an ulcer base with clear margins that penetrate the muscularis propria and into the submucosa. Inflammatory debris on the epithelial surface is often present. In the submucosa, one will see fibrosis and thickened blood vessels. Option C: H. pylori infection's first appearance of gastritis tends to be antral. The inflammation, composing mainly of mononuclear inflammatory cells and plasma cells are superficial and mostly in the upper layers of the mucosa of the corpus (body of the stomach). The chronic inflammation of gastric mucosa is associated with neutrophilic inflammation; the effects are dependent on the cytotoxicity of the H. pylori strain. Option D: Histologically, the mucosal layer of the colon in a patient with ulcerative colitis includes infiltrates of varying density and composition, depending on the stage of the disease. These infiltrates primarily consist of lymphocytes, plasma cells, and granulocytes, with the latter being more prominent during acute flares of the disease.

Digoxin preparations and absorbents should not be given simultaneously. As a nurse, you are aware that if these agents are given simultaneously, which of the following will occur? A. Increased absorption of digoxin. B. Decreased absorption of digoxin. C. Increased absorption of the absorbent. D. Decreased absorption of the absorbent.

B. Decreased absorption of digoxin. Digoxin comes from the foxgloves plant known as Digitalis purpurea. It is a cardiotonic glycoside and belongs to the digitalis class. The chemical formula of digoxin is C41 H64 O14. Cardiac glycosides, including digitalis and digoxin, have long-standing use in clinical practice. It is used for rate control in atrial fibrillation or atrial flutter when conventional therapies have not achieved the goal heart rate. Option A: The absorption of digoxin is not increased. Digoxin should not be administered in cases of pre-excitation caused by accessory pathways as digoxin induces AV blockade and may trigger ventricular tachyarrhythmias. It is ineffective in states of high sympathetic activity. Beta-blockers are preferable in such cases. Option C: There is no increase in the absorption of the absorbent. The kidneys excrete digoxin in direct proportion to the glomerular filtration rate; the liver metabolizes 16 %. Digoxin has a half-life that varies from 36 to 48 hours, which may increase in cases of renal failure. Option D: The absorption of the absorbent is not decreased. Digoxin has an oral bioavailability of approximately 75%, although intake efficacy might diminish when taking digoxin with high fiber foods. Some patients possess gut flora that metabolizes digoxin to dihydrodigoxin, thereby decreasing the drug's absorption.

When evaluating a male client for complications of acute pancreatitis, the nurse would observe for: A. Increased intracranial pressure B. Decreased urine output C. Bradycardia D. Hypertension

B. Decreased urine output Acute pancreatitis can cause decreased urine output, which results from the renal failure that sometimes accompanies this condition. AKI develops late in the course of acute pancreatitis, usually after failure of other organs. Remarkably, the kidney was the first organ to fail in only 8.9% of patients with AKI, and only a minority of patients develop isolated AKI Option A: Intracranial pressure neither increases nor decreases in a client with pancreatitis. The causes of increased intracranial pressure (ICP) can be divided based on the intracerebral components causing elevated pressures. Generalized swelling of the brain or cerebral edema from a variety of causes such as trauma, ischemia, hyperammonemia, uremic encephalopathy, and hyponatremia. Option C: Tachycardia, not bradycardia, usually is associated with pulmonary or hypovolemic complications of pancreatitis. Tachycardia and mild hypotension may result from hypovolemia from sequestration of fluid in the pancreatic bed. About 60% of patients develop low-grade pyrexia from peripancreatic inflammation without evident infection. Option D: Hypotension can be caused by a hypovolemic complication, but hypertension usually isn't related to acute pancreatitis. Release into the systemic circulation of activated enzymes and proteases may cause endothelial damage leading to extravasation of fluids from the vascular space, hypovolemia, hypotension, increased abdominal pressure, intense kidney vasoconstriction, hypercoagulability, and fibrin deposition in the glomeruli.

A client with peptic ulcer disease tells the nurse that he has black stools, which he has not reported to his physician. Based on this information, which nursing diagnosis would be appropriate for this client? A. Ineffective coping related to fear of diagnosis of chronic illness. B. Deficient knowledge related to unfamiliarity with significant signs and symptoms. C. Constipation related to decreased gastric motility. D. Imbalanced nutrition: Less than body requirements due to gastric bleeding.

B. Deficient knowledge related to unfamiliarity with significant signs and symptoms. Black, tarry stools are an important warning sign of bleeding in peptic ulcer disease. Digested blood in the stomach causes it to be black. The odor of the stool is very stinky. Clients with peptic ulcer disease should be instructed to report the incidence of black stools promptly to their physician. Assess the client's knowledge and misconceptions regarding peptic ulcer disease, lifestyle behaviors, and the treatment regimen. Option A: Provide emotional support to the client. Providing emotional support will give a client a calming and relaxing mood that will lower anxiety and stress related to the condition. Assist the client in developing anxiety-reducing measures such as biofeedback, positive imagery, and behavior modification. Option C: Teach about the importance of eating a balanced diet with meals at regular intervals. Specific dietary restrictions are no longer part of the treatment for PUD. During the symptomatic phase of an ulcer, the client may find benefit from eating small meals at more frequent intervals. Option D: Assist the client with identifying foods that cause gastric irritation. Clients need to learn what foods they can tolerate without gastric pain. Soft, bland, non-acidic foods cause less gastric irritation. The client is more likely to increase food intake if the foods are not associated with pain. Foods that may contribute to mucosal irritation include spicy foods, pepper, aNd raw fruits and vegetables.

The nurse is reviewing the record of a client with Crohn's disease. Which of the following stool characteristics would the nurse expect to note documented on the client's record? A. Chronic constipation B. Diarrhea C. Constipation alternating with diarrhea. D. Stool constantly oozes from the rectum.

B. Diarrhea Crohn's disease is characterized by nonbloody diarrhea of usually not more than four to five stools daily. Over time, the diarrhea episodes increase in frequency, duration, and severity. In CD, the inflammation extends through the entire thickness of the bowel wall from the mucosa to the serosa. The disease runs a relapsing and remitting course. The other options are not associated with diarrhea. Option A: Patients with flare-ups of Crohn's disease typically present with abdominal pain (right lower quadrant), flatulence/bloating, diarrhea (can include mucus and blood), fever, weight loss, anemia. In severe cases, perianal abscess, perianal Crohn's disease, and cutaneous fistulas can be seen. Option C: When the small bowel is involved, it may present with diarrhea, malabsorption, weight loss, abdominal pain, and anorexia. Enterovesical fistulae may present with pneumaturia, recurrent urinary tract infections, and feculent vaginal discharge. Option D: Granuloma formation is very common in Crohn's disease but their absence does not exclude the diagnosis. The ongoing inflammation and scarring lead to bowel obstruction and stricture formation. Crohn's disease is also associated with enterovesical, enteroenteral, enterocutaneous, and enterovaginal fistulas.

A client with a peptic ulcer reports epigastric pain that frequently awakens her at night, a feeling of fullness in the abdomen, and a feeling of anxiety about her health. Based on this information, which nursing diagnosis would be most appropriate? A. Imbalanced Nutrition: Less than Body Requirements related to anorexia. B. Disturbed Sleep Pattern related to epigastric pain. C. Ineffective Coping related to exacerbation of duodenal ulcer. D. Activity Intolerance related to abdominal pain.

B. Disturbed Sleep Pattern related to epigastric pain. Based on the data provided, the most appropriate nursing diagnosis would be Disturbed Sleep pattern. A client with a duodenal ulcer commonly awakens at night with pain. Clients with a gastric ulcer typically demonstrate pain 1 to 2 hours after eating. The client with duodenal ulcers demonstrates pain 2 to 4 hours after eating or in the middle of the night. With both gastric and duodenal ulcers, the pain is located in the upper abdomen and is intermittent. The client may report relief after eating or taking an antacid. Option A: Clients need to learn what foods they can tolerate without gastric pain. Soft, bland, non-acidic foods cause less gastric irritation. The client is more likely to increase food intake if the foods are not associated with pain. Foods that may contribute to mucosal irritation include spicy foods, pepper, aNd raw fruits and vegetables. Option C: The client's feelings of anxiety do not necessarily indicate that she is coping ineffectively. Provide emotional support to the client. Providing emotional support will give a client a calming and relaxing mood that will lower anxiety and stress related to the condition. Option D: Encourage the use of nonpharmacological pain relief measures. Non Pharmacological relaxation techniques will decrease the production of gastric acid, which in turn will reduce pain.

Which of the following instructions should the nurse include in the teaching plan for a client who is experiencing gastroesophageal reflux disease (GERD)? A. Limit caffeine intake to two cups of coffee per day. B. Do not lie down for 2 hours after eating. C. Follow a low-protein diet. D. Take medications with milk to decrease irritation.

B. Do not lie down for 2 hours after eating.

The nurse is assessing a client 24 hours following a cholecystectomy. The nurse notes that the T-tube has drained 750ml of green-brown drainage. Which nursing intervention is most appropriate? A. Notify the physician. B. Document the findings. C. Irrigate the T-tube. D. Clamp the T-tube.

B. Document the findings. Following cholecystectomy, drainage from the T-tube is initially bloody and then turns green-brown. Fresh post-op (1-2 days): drainage starts out with some blood and then progresses to a greenish/yellow/brown liquid drainage. The drainage is measured as output. The amount of expected drainage will range from 500 to 1000 ml per day. The nurse would document the output. Option A: Notifying the physician is unnecessary. The fluid may appear bloody for the first day or 2. The color will eventually be golden yellow or greenish, depending on exactly where the catheter is inside the body. There will be bile (yellow-green fluid) flowing into the bag. Option C: There is no need to irrigate the T-tube. The client will need to flush the catheter with normal saline twice a day. If the doctor instructed to flush with less than 10 mL, squirt the extra saline out before connecting the syringe. Push the plunger of the syringe to push 1/3 of the normal saline into the catheter, and then pause. Push in another 1/3 of the normal saline, and pause again. Push in the rest of the normal saline into the catheter. Option D: The doctor may order the t-tube to be clamped at times so bile can drain to the duodenum so fats can be digested during meal times. Assess how well the patient tolerated the t-tube being clamped. If a patient develops abdominal pain, nausea, vomiting, etc. unclamp it and notify the physician.

Which of the following complications of gastric resection should the nurse teach the client to watch for? A. Constipation B. Dumping syndrome C. Gastric spasm D. Intestinal spasms

B. Dumping syndrome Dumping syndrome is a problem that occurs postprandially after gastric resection because ingested food rapidly enters the jejunum without proper mixing and without the normal duodenal digestive processing. Dumping syndrome is treated primarily by diet modification, medical treatment with somatostatin analogs, or surgical intervention for refractory cases. Option A: Diarrhea, not constipation, may also be a symptom. The most common post-gastrectomy complications following gastric resection include nutritional deficiencies, dumping syndrome, small gastric remnant, post-vagotomy diarrhea, delayed gastric emptying, afferent or efferent loop syndrome, roux stasis, and bile reflux gastritis. Option C: Gastric spasms don't occur, but antispasmodics may be given to slow gastric emptying. Internal hernias are a known cause of acute abdominal pain in patients with gastric resection and Roux-en-Y reconstruction. Three types of trans-mesenteric hernias commonly occur in these patients. Option D: Intestinal spasms don't occur, but antispasmodics may be given to slow gastric emptying. Afferent and Efferent Loop Syndromes are well-established complications of gastric resection. Afferent loop syndrome is an uncommon obstruction that may result from the internal hernia, marginal ulceration, adhesions, recurrent cancer, or intussusception in patients with Billroth II gastrectomy.

You're caring for Lewis, a 67 y.o. patient with liver cirrhosis who developed ascites and requires paracentesis. Relief of which symptom indicates that the paracentesis was effective? A. Pruritus B. Dyspnea C. Jaundice D. Peripheral Neuropathy

B. Dyspnea Ascites put pressure on the diaphragm. Paracentesis is done to remove fluid and reduce pressure on the diaphragm. The goal is to improve the patient's breathing. The others are signs of cirrhosis that aren't relieved by paracentesis. Dyspnea from tense ascites might only be relieved with large-volume paracentesis. Care should also be taken with this procedure because patients with cirrhosis who have unrecognized cardiomyopathy can develop pulmonary edema following large-volume paracentesis. Option A: These medical treatments range from bile salts, rifampicin, and opioid receptor antagonists to antihistamines. Additionally, nonpharmacological management such as skin moisturizers, avoidance of skin irritants, and avoiding hot environments can also prove to be very beneficial in reducing pruritus. Option C: The underlying disorder and any problems it causes are treated as needed. If jaundice is due to acute viral hepatitis, it may disappear gradually, without treatment, as the condition of the liver improves. However, hepatitis may become chronic, even if jaundice disappears. Jaundice itself requires no treatment in adults Option D: Treatment of peripheral neuropathy has two goals: controlling the underlying disease process and treating troublesome symptoms. The former is usually achieved by eliminating offending agents, such as toxins or medications; correcting a nutritional deficiency; or treating the underlying disease.

You're preparing a teaching plan for a 27 y.o. named Jeff who underwent surgery to close a temporary ileostomy. Which nutritional guidelines do you include in this plan? A. There is no need to change eating habits. B. Eat six small meals a day. C. Eat the largest meal in the evening. D. Restrict fluid intake.

B. Eat six small meals a day. To avoid overloading the small intestine, encourage the patient to eat six small, regularly spaced meals. An ileostomy closure surgery is done to reverse the ileostomy so the client can have bowel movements as he did before the surgery. Ileostomy closure surgery is usually done through the stoma. Option A: The patient should eat 5 to 6 small meals throughout the day. He should remember to eat slowly and chew his food well. It's important the patient tries to maintain his weight. After surgery, foods may affect the patient differently. Certain foods may make him have bowel movements right after he eats them. Option C: Certain foods may cause diarrhea (loose or watery bowel movements). The patient may need to change his diet after surgery. During the first few months after the surgery, the patient will need to test foods and see how he reacts to them. It may be helpful to keep a food diary. This will help keep track of which foods cause discomfort. Option D: The patient should drink 8 to 10 (8-ounce) glasses of liquids every day. The amount of alcohol the patient drinks can affect him during and after the surgery. If the patient stops drinking alcohol suddenly, it can cause seizures, delirium, and death.

The nurse is instructing the male client who has an inguinal hernia repair how to reduce postoperative swelling following the procedure. What should the nurse tell the client? A. Limit oral fluid B. Elevate the scrotum C. Apply heat to the abdomen D. Remain in a low-fiber diet

B. Elevate the scrotum. Following inguinal hernia repair, the client should be instructed to elevate the scrotum and apply ice packs while in bed to decrease pain and swelling. The nurse also should instruct the client to apply a scrotal support when out of bed. In the beginning phases of healing, the body produces extra fluid that helps with the healing process. This fluid brings nutrients and cells that can help tissues repair themselves, and appears as swelling at the site where there was an injury to the tissues (similar to swelling after a sprained ankle). When this swelling sits in one area for a few days, it often turns hard and can feel like a firm lump. Option A: Limiting oral fluids can cause constipation. Sometimes the swelling is soft and does not seem to go away after surgery. Again, this is most likely due to the body's normal fluid pooling in the space the hernia used to be. If this swelling persists longer than three to six months after surgery, it should be re-evaluated by the surgical team to determine if hernia has recurred. Option C: Heat will increase swelling. While most swelling after hernia surgery is normal, it can rarely be an indicator of a complication, including infection and hernia recurrence. Swelling after hernia surgery is rarely a sign of infection. Since infection from hernia surgery occurs less than 1% of the time, swelling is normal the other 99% of the time. Option D: A low-fiber diet can cause constipation. Pain or swelling that continues to worsen after hernia surgery or pain and swelling accompanied by red discoloration of the skin are signs of infection and require attention by the surgical team.

What laboratory finding is the primary diagnostic indicator for pancreatitis? A. Elevated blood urea nitrogen (BUN) B. Elevated serum lipase C. Elevated aspartate aminotransferase (AST) D. Increased lactate dehydrogenase (LD)

B. Elevated serum lipase Elevation of serum lipase is the most reliable indicator of pancreatitis because this enzyme is produced solely by the pancreas. Serum lipase typically increases 3-6 hours after the onset of acute pancreatitis and usually peaks at 24 hours. Unlike amylase, there is significant reabsorption of lipase in the renal tubules so the serum concentrations remain elevated for 8-14 days. Option A: A client's BUN is typically elevated in relation to renal dysfunction. A BUN test is done to see how well the kidneys are working. If the kidneys are not able to remove urea from the blood normally, the BUN level rises. Heart failure, dehydration, or a diet high in protein can also make the BUN level higher. Liver disease or damage can lower the BUN level. Option C: A client's AST is typically elevated in relation to liver dysfunction. The elevated AST-to-ALT ratio in alcoholic liver disease results in part from the depletion of vitamin B6 (pyridoxine) in chronic alcoholics. ALT and AST both use pyridoxine as a coenzyme, but the synthesis of ALT is more strongly inhibited by pyridoxine deficiency than is the synthesis of AST. Option D: A client's LD is typically elevated in relation to damaged cardiac muscle. Usually, LDH isoenzyme levels increase 24-72 hours following myocardial infarction and reach a peak concentration in 3-4 days. Glycogen phosphorylase BB is released into circulation 2-4 h after onset of cardiac ischemia and returns to baseline levels 1-2 days after acute myocardial infarction, making it an early marker.

After a right hemicolectomy for treatment of colon cancer, a 57-year old client is reluctant to turn while on bed rest. Which action by the nurse would be appropriate? A. Asking a coworker to help turn the client. B. Explaining to the client why turning is important. C. Allowing the client to turn when he's ready to do so. D. Telling the client that the physician's order states he must turn every 2 hours.

B. Explaining to the client why turning is important. The appropriate action is to explain the importance of turning to avoid postoperative complications. Postoperative patients must be monitored and assessed closely for any deterioration in condition and the relevant postoperative care plan or pathway must be implemented. Option A: Asking a coworker to help turn the client would infringe on his rights. Postoperative patients are at risk of clinical deterioration, and it is vital that this is minimized. Knowledge and understanding of the key areas of risk and local policies will help reduce potential problems (National Patient Safety Agency, 2007; National Institute for Health and Clinical Excellence, 2007). Option C: Allowing him to turn when he's ready would increase his risk for postoperative complications. The National Early Warning Score (NEWS) was developed by a working party to provide a national standard for assessing, monitoring, and tracking acutely and critically ill patients (not for use with children under 16 years or in pregnancy); the intention was that trusts would use it to replace their locally adapted early warning systems (Royal College of Physicians, 2012). Option D: Telling him he must turn because of the physician's orders would put him on the defensive and exclude him from participating in care decisions. The NCEPOD (2011) report found that patients whose condition was deteriorating were not always identified and referred for a higher level of care. Patients should be made as comfortable as possible before postoperative checks are performed.

The client has just had surgery to create an ileostomy. The nurse assesses the client in the immediate post-op period for which of the following most frequent complications of this type of surgery? A. Intestinal obstruction B. Fluid and electrolyte imbalance C. Malabsorption of fat D. Folate deficiency

B. Fluid and electrolyte imbalance A major complication that occurs most frequently following an ileostomy is fluid and electrolyte imbalance. The client requires constant monitoring of intake and output to prevent this from happening. Losses require replacement by intravenous infusion until the client can tolerate a diet orally. Monitor I&O carefully, measure liquid stool. Weigh regularly. This provides direct indicators of fluid balance. Greatest fluid losses occur with an ileostomy, but they generally do not exceed 500-800 mL/day. Option A: Intestinal obstruction is a less frequent complication. Small bowel obstruction is a recognized complication of ileostomy. Small bowel obstruction may occur because of too small trephine in the anterior abdominal wall, a twisted loop, small bowel adhesions in the area around the ileostomy, intraperitoneal adhesions, or an acute parastomal hernia. Option C: Fat malabsorption is a complication that could occur later in the postoperative period. Suggest the patient with ileostomy limit prunes, dates, stewed apricots, strawberries, grapes, bananas, cabbage family, beans, and avoid foods high in cellulose, e.g., peanuts. These products increase ileal effluent. Digestion of cellulose requires colon bacteria that are no longer present. Option D: Folate deficiency is a complication that could occur later in the postoperative period. Vitamin B12 plays an important role in keeping the brain and nervous system healthy. This decrease is thought to occur because the part of the intestine removed during the procedure is responsible for absorbing some vitamin B12 from the food you eat.

Ralph has a history of alcohol abuse and has acute pancreatitis. Which lab value is most likely to be elevated? A. Calcium B. Glucose C. Magnesium D. Potassium

B. Glucose Glucose level increases and diabetes mellitus may result d/t the pancreatic damage to the islets of Langerhans. Acute pancreatitis is associated with damage to both the endocrine and exocrine pancreas. Glucose intolerance seen with this disease appears to be the result of hyperglucagonemia and relative hypoinsulinemia. Option A: Initial evaluation of suspected acute pancreatitis involves laboratory abnormalities suggesting biliary cholestasis, hypercalcemia or severe hyperlipidemia will help in determining the etiology of pancreatitis. An abdominal ultrasound is recommended in all the patients to assess for choledocholithiasis and bile duct dilatation. Option C: The diagnosis of acute pancreatitis has been defined by the Revised Atlanta Classification and requires at least 2 of 3 criteria be met: 1) a lipase or amylase level that is three times the upper limit of normal 2) abdominal pain that is consistent with pancreatitis 3) abdominal imaging consistent with acute pancreatitis. Option D: A thorough history regarding alcohol use and medications should be gathered, keeping in mind that over five years of heavy alcohol use is often needed to induce alcohol-related pancreatitis. Smoking history is also important as a risk factor for acute pancreatitis.

A client with irritable bowel syndrome is being prepared for discharge. Which of the following meal plans should the nurse give the client? A. Low fiber, low-fat B. High fiber, low-fat C. Low fiber, high-fat D. High-fiber, high-fat

B. High-fiber, low-fat The client with irritable bowel syndrome needs to be on a diet that contains at least 25 grams of fiber per day. Fatty foods are to be avoided because they may precipitate symptoms. Fiber adds bulk to stools, which helps aid in movement. The average adult should eat 20 to 35 grams of fiber per day. While this seems simple enough, the National Institute of Diabetes and Digestive and Kidney Diseases estimates that most people only eat 5 to 14 grams per day. Option A: Fiber-rich foods, such as fruits, vegetables, and whole grains, are nutritious and help prevent constipation. However, if you experience bloating from increased fiber intake, try focusing solely on soluble fiber found in fruits and vegetables instead of grains. Option C: High-fat foods are generally low in fiber, which can be problematic for IBS-related constipation. According to the Cleveland Clinic, fatty foods are particularly bad for people with mixed IBS, which is characterized by a combination of constipation and diarrhea. Embarking on a low-fat diet is good for your heart and may improve uncomfortable bowel symptoms. Option D: Chronic consumption of high-fat foods is a known contributor to a variety of health issues, such as obesity. However, it can be especially hard on those with IBS by worsening symptoms. Instead of eating fried foods and animal fats, focus on lean meats, fruits, vegetables, grains, and low-fat dairy products.

Spironolactone (Aldactone) is prescribed for a client with chronic cirrhosis and ascites. The nurse should monitor the client for which of the following medication-related side effects? A. Jaundice B. Hyperkalemia C. Tachycardia D. Constipation

B. Hyperkalemia This is a potassium-sparing diuretic so clients should be monitored closely for hyperkalemia. Diarrhea, dizziness, and headaches are other more common side effects. Spironolactone is a medication used in the management and treatment of hypertension and heart failure with some indications aside from cardiovascular disease. It is in the mineralocorticoid receptor antagonist class of drugs. Option A: Jaundice is not an expected side effect of spironolactone (Aldactone). The most common non-electrolyte and electrolyte adverse effects of spironolactone are breast complaints and hyperkalemia, respectively. Men specifically may experience gynecomastia, loss of libido, and general feminization; the drug is a category C pregnancy drug because animal studies showed that the feminization of male fetuses is a concern. Option C: Tachycardia is not an expected side effect of spironolactone (Aldactone). Women can experience menstrual irregularities. One study mentions the following additional adverse effects in order from more to less common: dehydration, hyponatremia, gastrointestinal problems (nausea, vomiting, diarrhea or anorexia), neurological abnormalities (headache, drowsiness, asterixis, confusion, or coma), and skin rashes. Option D: Constipation is not an expected side effect of spironolactone (Aldactone). Hyperkalemia can be due to spironolactone alone or a synergistic side effect from multiple medications such as beta-blockers, angiotensin-converting enzyme inhibitors, and angiotensin receptor blockers that clinicians often prescribe to patients for indications such as hypertension or heart failure.

Matt is a 49 y.o. with a hiatal hernia that you are about to counsel. Health care counseling for Matt should include which of the following instructions? A. Restrict intake of high-carbohydrate foods. B. Increase fluid intake with meals. C. Increase fat intake. D. Eat three regular meals a day.

B. Increase fluid intake with meals. Increasing fluids help empty the stomach. A hiatal hernia is a condition in which the upper part of the stomach or other internal organ bulges through the hiatus of the diaphragm. When there is laxity in this hiatus, gastric content can back up into the esophagus and is the leading cause of gastroesophageal reflux disease (GERD). Option A: A high-carb diet isn't restricted. Non- or low-acidic foods will reduce the likelihood and severity of hiatal hernia symptoms. The best food choices for people with hiatal hernias are non-acidic, minimally processed, and contain dietary fiber. Option C: Fat intake shouldn't be increased. Fermented or cultured foods that are rich in probiotics (acid-neutralizing stomach bacteria) may also help reduce hiatal hernia symptoms. Option D: Diet plays a significant role in the development, severity, and length of hiatal hernia symptoms. But aside from causing inflammation and irritation, researchers are not sure how and why certain foods cause a hiatal hernia to develop.

Patients with esophageal varices would reveal the following assessment: A. Increased blood pressure B. Increased heart rate C. Decreased respiratory rate D. Increased urinary output

B. Increased heart rate Tachycardia is an early sign of compensation for patients with esophageal varices. Since the portal venous system has no valves, resistance at any level between the splanchnic vessels and the right side of the heart results in retrograde flow and elevated pressure. The collaterals slowly enlarge and connect the systemic circulation to the portal venous system. Option A: Esophageal varices are a direct result of high blood pressure in the portal vein. This condition is called portal hypertension. It causes blood to build up in nearby blood vessels, including those in your esophagus. Veins begin to dilate and swell as a result of increased blood flow. Option C: The respiratory rate is not decreased in esophageal varices. Esophageal varices are the major complication of portal hypertension. It is detected in about 50% of cirrhosis patients, and approximately 5-15% of cirrhosis patients show newly formed varices or worsening of varices each year. Option D: Effective resuscitation, accurate diagnosis, and early treatment are key to reducing mortality in variceal bleeding. The aims are not only to stop bleeding as soon as possible but also to prevent early re-bleeding. Early rebleeding, as with peptic ulcer disease, is significantly associated with worsening mortality.

The correct sequence for abdominal assessment is: A. Inspection, percussion, palpation, auscultation. B. Inspection, auscultation, percussion, palpation. C. Inspection, palpation, auscultation, percussion. D. Inspection, percussion, auscultation, palpation.

B. Inspection, auscultation, percussion, palpation. Auscultation is done before percussion and palpation to avoid stimulating peristaltic movements and distorting auscultatory sounds. The diaphragm of the stethoscope should be placed on the right side of the umbilicus to listen to the bowel sounds, and their rate should be calculated after listening for at least two minutes. Normal bowel sounds are low-pitched and gurgling, and the rate is normally 2-5/min. Option A: Begin with the general inspection of the patient and then proceed to the abdominal area. This should be performed at the foot end of the bed. The general inspection can give multiple clues regarding the diagnosis of the patient, for example, yellowish discoloration of the skin (jaundice) indicates a possible hepatic abnormality. Option C: There are three stages of palpation that include superficial or light palpation, deep palpation, and organ palpation, and should be performed in the same order. Maneuvers specific to certain diseases are also a part of abdominal palpation. The examiner should begin with superficial or light palpation from the area furthest from the point of maximal pain and move systematically through the nine regions of the abdomen. Option D: A proper technique of percussion is necessary to gain maximum information regarding abdominal pathology. While percussing, it is important to appreciate tympany over air-filled structures such as the stomach and dullness to percussion which may be present due to an underlying mass or organomegaly (for example, hepatomegaly or splenomegaly).

Before bowel surgery, Lee is to administer enemas until clear. During administration, he complains of intestinal cramps. What do you do next? A. Discontinue the procedure. B. Lower the height of the enema container. C. Complete the procedure as quickly as possible. D. Continue administration of the enema as ordered without making any adjustments.

B. Lower the height of the enema container. Lowering the height decreases the amount of flow, allowing him to tolerate more fluid. Position the patient on the left side, lying with the knees drawn to the abdomen. This eases the passage and flow of fluid into the rectum. Gravity and the anatomical structure of the sigmoid colon also suggest that this will aid enema distribution and retention. Option A: A person can expect that they may feel some discomfort due to an enema. Adverse effects, such as stomach cramping and anal irritation, should go away on their own. However, if any adverse effect is severe or persistent, see a doctor. Option C: Enemas may also be used before medical examinations of the colon. The doctor may order an enema prior to an X-ray of the colon to detect polyps so that they can get a clearer picture. This procedure may also be done prior to a colonoscopy. Option D: The risks for enema-related complications are greater if the patient administers the tubes multiple times a day. The best course of action is to use the enema once a day, and around the same time every day, as directed by a doctor. This not only reduces side effects but will also help to train the body to release waste regularly.

A male client with a recent history of rectal bleeding is being prepared for a colonoscopy. How should the nurse position the client for this test initially? A. Lying on the right side with legs straight B. Lying on the left side with knees bent C. Prone with the torso elevated D. Bent over with hands touching the floor

B. Lying on the left side with knees bent For a colonoscopy, the nurse initially should position the client on the left side with knees bent. Preparation for a colonoscopy is the biggest complaint that most patients have about receiving the procedure, and is a primary reason for non-compliance to screening colonoscopies. The technician or nurse is there to assist with preserving stability and preventing the patient from rolling forward or backward. Also, they are there to help provide counter pressure to the abdomen to assist the endoscopist in navigating corners and turns. Option A: Placing the client on the right side with legs straight wouldn't allow proper visualization of the large intestine. Performing a colonoscopy requires practice and is a skill that is difficult to master. While watching an experienced clinician perform a colonoscopy may appear simple, the technique is something that requires time, patience, and a lot of practice. Navigating through a cylindrical tube that can flex, dilate, contract, and move is not an easy task. Option B: Placing the client prone with the torso elevated wouldn't allow proper visualization of the large intestine. The patient should be positioned in the left lateral decubitus position. Although, some clinicians may prefer the patient on their back or right side if circumstances require. The legs being flexed toward the chest help to relax the puborectalis and pubococcygeus muscles. This allows for easier entry and traversing past the angle at the sacral prominence. Option D: Placing the client bent over with hands touching the floor wouldn't allow proper visualization of the large intestine. On the left-sided position, the patient's legs should be flexed, and pillows should be placed around their back, head, and between their knees to help prevent injury to the bony prominence and to help maintain position.

Which goal of the client's care should take priority during the first days of hospitalization for an exacerbation of ulcerative colitis? A. Promoting self-care and independence. B. Managing diarrhea. C. Maintaining adequate nutrition. D. Promoting rest and comfort.

B. Managing diarrhea Diarrhea is the primary symptom in an exacerbation of ulcerative colitis, and decreasing the frequency of stools is the first goal of treatment. Observe and record stool frequency, characteristics, amount, and precipitating factors. The other goals are ongoing and will be best achieved by halting the exacerbation. The client may receive antidiarrheal medications, antispasmodic agents, bulk hydrophilic agents, or anti-inflammatory drugs. Option A: Include patient and SO in team conferences to develop an individualized program. Promotes continuity of care and enables the patient and SO to feel a part of the plan, imparting a sense of control and increasing cooperation with the therapeutic regimen. Option C: Identify and restrict foods and fluids that precipitate diarrhea (vegetables and fruits, whole-grain cereals, condiments, carbonated drinks, milk products). Avoiding intestinal irritants promotes intestinal rest and reduces intestinal workload. Option D: Encourage use of stress management skills, (relaxation techniques, visualization, guided imagery, deep-breathing exercises). Refocuses attention, promotes relaxation, and enhances coping abilities.

Claire, a 33 y.o. is on your floor with a possible bowel obstruction. Which intervention is a priority for her? A. Obtain daily weights. B. Measure abdominal girth. C. Keep strict intake and output. D. Encourage her to increase fluids.

B. Measure abdominal girth. Measuring abdominal girth provides quantitative information about increases or decreases in the amount of distention. Abdominal girths should be measured daily. Use the same measuring tape each time. Place the patient in the same position each time. Ensure that the tape measure is placed in the same position each time. This can be done by drawing small tick marks on the patient's abdomen to indicate the position of the tape. Measure the patient at the same time each day. Option A: Weigh daily; provides information about dietary needs and effectiveness of therapy. Avoid or limit foods that might cause or exacerbate abdominal cramping, flatulence (milk products, foods high in fiber or fat, alcohol, caffeinated beverages, chocolate, peppermint, tomatoes, orange juice). Option C: Monitor I&O closely. Fluid and electrolyte losses must be replaced. Record intake and changes in symptomatology. Useful in identifying specific deficiencies and determining GI response to foods. Monitor I&O. Note number, character, and amount of stools; estimate insensible fluid losses (diaphoresis). Measure urine specific gravity; observe for oliguria. Option D: Administer parenteral fluids, blood transfusions as indicated. Maintenance of bowel rest requires alternative fluid replacement to correct losses and anemia. Fluids containing sodium may be restricted in presence of regional enteritis.

A male client with pancreatitis complains of pain. The nurse expects the physician to prescribe meperidine (Demerol) instead of morphine to relieve pain because: A. Meperidine provides a better, more prolonged analgesic effect. B. Morphine may cause spasms of Oddi's sphincter. C. Meperidine is less addictive than morphine. D. Morphine may cause hepatic dysfunction.

B. Morphine may cause spasms of Oddi's sphincter. For a client with pancreatitis, the physician will probably avoid prescribing morphine because this drug may trigger spasms of the sphincter of Oddi (a sphincter at the end of the pancreatic duct), causing irritation of the pancreas. Morphine showed an excitatory effect on the sphincter of Oddi, and might be a cause of Oddi's sphincter dysfunction(SOD). SO may function as a peristaltic pump to actively expel fluid from the sphincter segment into the duodenum. Option A: Meperidine has a somewhat shorter duration of action than morphine. Morphine is the standard opium-based analgesic with well-known analgesic effects and side effects. Meperidine (pethidine) is another opioid, but the use of meperidine is not very popular in countries such as the USA and Canada, however, it is still used in Iran for pain management in the ED. Option C: The two drugs are equally addictive. Meperidine appeared to be safer with a lower risk of addiction when compared to other opioids and because of the anticholinergic effects associated with less biliary spasm or renal colic. Option D: Morphine isn't associated with hepatic dysfunction. Respiratory depression is among the more serious adverse reactions with opiate use that is especially important to monitor in the postoperative patient population. Other reported side effects include lightheadedness, sedation, and dizziness.

The nurse is reviewing the physician's orders written for a male client admitted to the hospital with acute pancreatitis. Which physician order should the nurse question if noted on the client's chart? A. NPO status B. Nasogastric tube inserted C. Morphine sulfate for pain D. An anticholinergic medication

B. Morphine sulfate for pain Meperidine (Demerol) rather than morphine sulfate is the medication of choice to treat pain because morphine sulfate can cause spasms in the sphincter of Oddi. Histological data show that treatment with morphine after induction of acute pancreatitis exacerbates the disease with increased pancreatic neutrophilic infiltration and necrosis in all three models of acute pancreatitis. Morphine also exacerbated acute pancreatitis-induced gut permeabilization and bacteremia. Option A: Historically, patients with acute pancreatitis would be kept without food by mouth (nil per os or NPO) until their physician team deemed them ready to eat again, usually based on blood tests or the reported level of pain. Option B: NGT is used if vomiting is a problem. The tube can be used for a few weeks. It can be used to remove fluid and air and give the pancreas more time to heal. It can also be used to put liquid food into the stomach as the client heals. Option D: Anticholinergics are used to a variable extent in the treatment of many gastrointestinal conditions, including acute or relapsing chronic pancreatitis, acid-peptic disorders of the upper gastrointestinal tract, chronic inflammations of the intestines, and so-called functional gastrointestinal disorders caused by excessive or abnormal motility.

A nurse is monitoring a client admitted to the hospital with a diagnosis of appendicitis. The client is scheduled for surgery in 2 hours. The client begins to complain of increased abdominal pain and begins to vomit. On assessment, the nurse notes that the abdomen is distended and the bowel sounds are diminished. Which of the following is the most appropriate nursing intervention? A. Administer dilaudid. B. Notify the physician. C. Call and ask the operating room team to perform the surgery as soon as possible. D. Reposition the client and apply a heating pad in a warm setting to the client's abdomen.

B. Notify the physician Based on the signs and symptoms presented in the , the nurse should suspect peritonitis and should notify the physician. If left untreated, appendicitis can lead to abscess formation with the development of an enterocutaneous fistula. Diffuse peritonitis and sepsis can also develop, which may progress to significant morbidity and possible death. Option A: Administering pain medication is not an appropriate intervention. While in the emergency department, the patient must be kept NPO and hydrated intravenously with crystalloid. Antibiotics should be administered intravenously as per the surgeon. The responsibility for the consent falls on the surgeon. Option C: Scheduling surgical time is not within the scope of nursing practice, although the physician probably would perform the surgery earlier than the pre-scheduled time. The gold-standard treatment for acute appendicitis is to perform an appendectomy. Laparoscopic appendectomy is preferred over the open approach. Most uncomplicated appendectomies are performed laparoscopically. Option D: Heat should never be applied to the abdomen of a client with suspected appendicitis. Complications of appendicitis and appendectomy include surgical site infections, intra-abdominal abscess formation (3% to 4% in open appendectomy and 9% to 24% in laparoscopic appendectomy), prolonged ileus, enterocutaneous fistula, and small bowel obstruction.

Care for the postoperative client after gastric resection should focus on which of the following problems? A. Body image B. Nutritional needs C. Skincare D. Spiritual needs

B. Nutritional needs After gastric resection, a client may require total parenteral nutrition or jejunostomy tube feedings to maintain adequate nutritional status. This provides rest for the GI tract during the acute postoperative phase until the return of normal function. The physician or surgeon may need to reposition the tube endoscopically to prevent injury to the operative area. Option A: Suggest the patient weigh himself on a regular basis. Change in dietary pattern, early satiety, and efforts to avoid dumping syndrome may limit intake, causing weight loss. Option C: Identify signs and symptoms requiring medical evaluation such as persistent nausea and vomiting or abdominal fullness; weight loss; diarrhea; foul-smelling fatty or tarry stools; bloody or coffee-ground vomitus or presence of bile, fever. Instruct the patient to report changes in pain characteristics. Option D: Review medication purpose, dosage, and schedule, and possible side effects. Understanding rationale and therapeutic needs can reduce the risk of complications ( anticholinergics or pectin powder may be given to reduce the incidence of dumping syndrome; antacids and histamine antagonists reduce gastric irritation).

You're performing an abdominal assessment on Brent who is 52 y.o. In which order do you proceed? A. Observation, percussion, palpation, auscultation B. Observation, auscultation, percussion, palpation C. Percussion, palpation, auscultation, observation D. Palpation, percussion, observation, auscultation

B. Observation, auscultation, percussion, palpation Begin with the general inspection of the patient and then proceed to the abdominal area. This should be performed at the foot end of the bed. The general inspection can give multiple clues regarding the diagnosis of the patient, for example, yellowish discoloration of the skin (jaundice) indicates a possible hepatic abnormality. Option A: A proper technique of percussion is necessary to gain maximum information regarding abdominal pathology. While percussing, it is important to appreciate tympany over air-filled structures such as the stomach and dullness to percussion which may be present due to an underlying mass or organomegaly (for example, hepatomegaly or splenomegaly). Option C: Auscultation is done before palpation to avoid stimulating peristaltic movements and distorting auscultatory sounds. The diaphragm of the stethoscope should be placed on the right side of the umbilicus to listen to the bowel sounds, and their rate should be calculated after listening for at least two minutes. Normal bowel sounds are low-pitched and gurgling, and the rate is normally 2-5/min. Option D: There are three stages of palpation that include superficial or light palpation, deep palpation, and organ palpation and should be performed in the same order. Maneuvers specific to certain diseases are also a part of abdominal palpation. The examiner should begin with superficial or light palpation from the area furthest from the point of maximal pain and move systematically through the nine regions of the abdomen.

Your patient is complaining of abdominal pain during assessment. What is your priority? A. Auscultate to determine changes in bowel sounds. B. Observe the contour of the abdomen. C. Palpate the abdomen for a mass. D. Percuss the abdomen to determine if fluid is present.

B. Observe the contour of the abdomen. The first step in assessing the abdomen is to observe its shape and contour, then auscultate, palpate, and then percuss. It is important to begin with the general examination of the abdomen with the patient in a completely supine position. The presence of any of the following signs may indicate specific disorders. Distension of the abdomen could be present due to small bowel obstruction, masses, tumors, cancer, hepatomegaly, splenomegaly, constipation, abdominal aortic aneurysm, and pregnancy. Option A: The last step of the abdominal examination is auscultation with a stethoscope. The diaphragm of the stethoscope should be placed on the right side of the umbilicus to listen to the bowel sounds, and their rate should be calculated after listening for at least two minutes. Normal bowel sounds are low-pitched and gurgling, and the rate is normally 2-5/min. Option C: The examiner should begin with superficial or light palpation from the area furthest from the point of maximal pain and move systematically through the nine regions of the abdomen. It is important to press slowly as pressing too fast may trap a gas pocket within the intestinal lumen and distend the wall resulting in false-positive tenderness. Option D: A proper technique of percussion is necessary to gain maximum information regarding abdominal pathology. While percussing, it is important to appreciate tympany over air-filled structures such as the stomach and dullness to percussion which may be present due to an underlying mass or organomegaly (for example, hepatomegaly or splenomegaly).

When preparing a male client, age 51, for surgery to treat appendicitis, the nurse formulates a nursing diagnosis of Risk for infection related to inflammation, perforation, and surgery. What is the rationale for choosing this nursing diagnosis? A. Obstruction of the appendix may increase venous drainage and cause the appendix to rupture. B. Obstruction of the appendix reduces arterial flow, leading to ischemia, inflammation, and rupture of the appendix. C. The appendix may develop gangrene and rupture, especially in a middle-aged client. D. Infection of the appendix diminishes necrotic arterial blood flow and increases venous drainage.

B. Obstruction of the appendix reduces arterial flow, leading to ischemia, inflammation, and rupture of the appendix. A client with appendicitis is at risk for infection related to inflammation, perforation, and surgery because obstruction of the appendix causes mucus fluid to build up, increasing pressure in the appendix and compressing venous outflow drainage. The pressure continues to rise with venous obstruction; arterial blood flow then decreases, leading to ischemia from lack of perfusion. Option A: Inflammation and bacterial growth follow, and swelling continues to raise pressure within the appendix, resulting in gangrene and rupture. The pathophysiology of appendicitis likely stems from obstruction of the appendiceal orifice. This results in inflammation, localized ischemia, perforation, and the development of a contained abscess or frank perforation with resultant peritonitis. Option C: Geriatric, not middle-aged, clients are especially susceptible to appendix rupture. Appendicitis occurs most often between the ages of 5 and 45 with a mean age of 28. The incidence is approximately 233/100,000 people. Males have a slightly higher predisposition of developing acute appendicitis compared to females, with a lifetime incidence of 8.6% for men and 6.7 % for women. Option D: When an obstruction is the cause of appendicitis, it leads to an increase in intraluminal and intramural pressure, resulting in small vessel occlusion and lymphatic stasis. Once obstructed, the appendix fills with mucus and becomes distended, and as lymphatic and vascular compromise advances, the wall of the appendix becomes ischemic and necrotic.

During the first few days of recovery from ostomy surgery for ulcerative colitis, which of the following aspects should be the first priority of client care? A. Body image B. Ostomy care C. Sexual concerns D. Skin care

B. Ostomy care Although all of these are concerns the nurse should address, being able to safely manage the ostomy is crucial for the client before discharge. Patients may have comorbidities that affect their ability to manage their ostomy care. Conditions such as arthritis, vision changes, Parkinson's disease, or post-stroke complications may hinder a patient's coordination and function to manage the ostomy. Option A: Help the patient identify and initiate positive coping behaviors used in the past. Successful behaviors can be fostered in dealing with current problems and stress, enhancing a patient's sense of self-control. Option C: Provide opportunity for the patient to discuss how illness has affected relationships, including sexual concerns. Stressors of illness affect all areas of life, and the patient may have difficulty coping with feelings of fatigue and pain in relation to relationship and sexual needs. Option D: Observe excessively dry skin and mucous membranes, decreased skin turgor, slowed capillary refill; Indicates excessive fluid loss or resultant dehydration.

Which condition is most likely to have a nursing diagnosis of fluid volume deficit? A. Appendicitis B. Pancreatitis C. Cholecystitis D. Gastric ulcer

B. Pancreatitis Hypovolemic shock from fluid shifts is a major factor in acute pancreatitis. The other conditions are less likely to exhibit fluid volume deficit. Early fluid therapy is the cornerstone of treatment and is universally recommended; however, there is a lack of consensus regarding the type, rate, amount, and endpoints of fluid replacement. The basic goal of fluid depletion should be to prevent or minimize the systemic response to inflammatory markers. Option A: A client with appendicitis may be at risk for deficient fluid volume. The peritoneum reacts to irritation and infection by producing large amounts of intestinal fluid, possibly reducing the circulating blood volume, resulting in dehydration and relative electrolyte imbalances. Option C: A client with cholecystitis may be at risk for deficient fluid volume. Prolonged vomiting, gastric aspiration, and restricted oral intake can lead to deficits in sodium, potassium, and chloride. Maintain accurate record of I&O, noting output less than intake, increased urine specific gravity. Option D: A client with a gastric ulcer may be at risk for deficient fluid volume. The erosion of an ulcer through the gastric or duodenal mucosal layer may cause GI bleeding. The client may develop anemia. If bleeding is brisk, changes in vital signs and physical symptoms of hypovolemia may develop rapidly.

A female client complains of gnawing epigastric pain for a few hours after meals. At times, when the pain is severe, vomiting occurs. Specific tests are indicated to rule out: A. Cancer of the stomach B. Peptic ulcer disease C. Chronic gastritis D. Pylorospasm

B. Peptic ulcer disease Peptic ulcer disease is characteristically gnawing epigastric pain that may radiate to the back. Vomiting usually reflects pyloric spasm from muscular spasm or obstruction. Peptic ulcer disease is characterized by discontinuation in the inner lining of the gastrointestinal (GI) tract because of gastric acid secretion or pepsin. It extends into the muscularis propria layer of the gastric epithelium. It usually occurs in the stomach and proximal duodenum. Option A: Cancer would not evidence pain or vomiting unless the pylorus was obstructed. In the United States, most patients have symptoms of an advanced stage at the time of presentation. The most common presenting symptoms for gastric cancers are non-specific weight loss, persistent abdominal pain, dysphagia, hematemesis, anorexia, nausea, early satiety, and dyspepsia. Option C: The current classification of gastritis is based on time course (acute versus chronic), histological features, anatomic distribution, and underlying pathological mechanisms. Acute gastritis will evolve to chronic, if not treated. There are no typical clinical manifestations of gastritis. Sudden onset of epigastric pain, nausea, and vomiting have been described to accompany acute gastritis. Option D: There has been much uncertainty about the concept of "pylorospasm". For many years radiologists considered pylorospasm to be due to spasm of the pyloric ring, where the ring was equated with the pyloric sphincter. It was thought that spasm of the ring (or "sphincter") closed the pyloric aperture, thereby delaying gastric emptying and causing retention.

A male client has undergone a colon resection. While turning him, wound dehiscence with evisceration occurs. The nurse's first response is to: A. Call the physician. B. Place saline-soaked sterile dressings on the wound. C. Take blood pressure and pulse. D. Pull the dehiscence closed.

B. Place saline-soaked sterile dressings on the wound. The nurse should first place saline-soaked sterile dressings on the open wound to prevent tissue drying and possible infection. Ask the client to bend the knees to reduce abdominal tension. Note the color of the tissue before it is covered. Then, cover the moistened dressings with a sterile drape. Option A: After placing saline-soaked sterile dressings, the nurse should call the physician. The bed should be lowered until it is flat or not higher than 20 degrees. Dehiscence occurs when an incisional wound separates after surgery; evisceration occurs when an internal bodily organ protrudes through the incision. Dehiscence and evisceration can be a life-threatening emergency. Option C: Take the client's vital signs and SPO2 while notifying the physician. Check every 15 minutes and assess for signs of shock. The nurse may also start a patent IV line with 0.9% sodium chloride solution for surgery. Option D: The dehiscence needs to be surgically closed, so the nurse should never try to close it. Be prepared to insert a nasogastric tube if ordered, to decompress the stomach. Keep the dressings wet and check the wound frequently, using a new pair of sterile gloves each time.

Which of the following nursing diagnoses is appropriate for a patient receiving famotidine (Pepcid)? A. Increased risk for infection due to immunosuppression. B. Potential risk for bleeding related to thrombocytopenia. C. Alteration in urinary elimination related to retention. D. Alteration in tissue perfusion related to hypertension.

B. Potential risk for bleeding related to thrombocytopenia. A serious side effect of famotidine is thrombocytopenia, which is manifested by a decrease in platelet count and an increased risk of bleeding. The over-the-counter tablets should not be used if the patient has trouble and/or pain when swallowing food, vomiting with blood, or bloody or black stools. Option A: Due to central nervous system (CNS) adverse effects, longer dosing intervals or reduced doses may be used instead to adjust for the resulting longer elimination half-life of famotidine. An increased risk of developing community-acquired pneumonia and acute gastroenteritis have been linked to the use of famotidine as well as other gastric acid inhibitors in the pediatric population. Option C: As famotidine is excreted mainly by the kidney, the risk of toxic reactions may be greater in patients with impaired renal function. Dose adjustment in patients who have moderate to severe renal impairment is necessary. Per famotidine's package insert, oral doses outside of FDA-approved doses of up to 640 mg per day have been given to adult patients with pathological hypersecretory states with no serious adverse outcomes. Option D: Famotidine is substantially excreted by the kidney; thus, it may be useful for healthcare professionals to monitor renal function especially in elderly patients. A patient's complete blood count (CBC), gastric pH, and occult blood in patients with gastrointestinal (GI) bleeding should be monitored.

After pancreatic surgery, pts are at risk for developing repiratory tract infections because of A. length of time required for surgery B. proximity of incision to diaphragm C. lowered resistance caused by bile in blood D. transfer of bacteria from pancreas to blood

B. Proximity of incision to diaphragm

Impaired bile metabolism and excretion are directly related to which symptom of cirrhosis? A. prolonged PTT B. pruritus C. ascites D. esophageal varices

B. Pruritus

The RN would assess the pt experiencing an acute episode of cholecystitis for pain that is located in the A. RUQ and radiates to left shoulder B. RUQ and radiates to right shoulder C. RLQ and radiates to umbilicus D. RLQ and radiates to back

B. RUQ and radiates to right shoulder

Which of the following symptoms is associated with ulcerative colitis? A. Dumping syndrome B. Rectal bleeding C. Soft stools D. Fistulas

B. Rectal bleeding In ulcerative colitis, rectal bleeding is the predominant symptom. Ulcerative colitis is an idiopathic inflammatory condition of the colon which results in diffuse friability and superficial erosions on the colonic wall associated with bleeding. It is the most common form of inflammatory bowel disease worldwide. Option A: Dumping syndrome occurs after gastric surgeries. Dumping syndrome is likely caused by the rapid movement of chyme. In patients without gastric surgery, digestion is initiated in the stomach, and the transition to the duodenum occurs progressively. The severity of dumping syndrome is correlated to the extent of gastric surgery. Surgical etiologies include gastrojejunostomy, antrectomy, pylorectomy, pyloroplasty, esophagectomy, vagotomy, Roux-en-Y bypass, and Nissen fundoplication. Option C: Soft stools are more commonly associated with Crohn's disease, in which malabsorption is more of a problem. Steatorrhea from malabsorption can occur with Crohn's disease. The inflammatory process caused by Crohn's disease, especially if it involves large tracts of intestine, alters the absorption mechanisms of different substances due to the lack of reabsorption of the bile salts, which normally stimulate the reabsorption of food fats. Option D: Fistulas are associated with Crohn's disease. Fistulas occur in all these areas, but the anorectal area is most common because of the relative thinness of the intestinal wall in this area. The initial lesion starts out as an infiltrate around an intestinal crypt. This goes on to develop ulceration first in the superficial mucosa and involves deeper layers. As the inflammation progresses, non-caseating granulomas form involving all layers of the intestinal wall.

You're assessing the stoma of a patient with a healthy, well-healed colostomy. You expect the stoma to appear: A. Pale, pink and moist B. Red and moist C. Dark or purple-colored D. Dry and black

B. Red and moist Good circulation causes tissues to be moist and red, so a healthy, well-healed stoma appears red and moist. A stoma should be pink to red in color, preferably raised above skin level, and moist. Stomas that are flat or convex can still be healthy but they can present challenges in terms of ostomy management and directing waste into the pouch. Option A: Major color changes in a stoma, with the stoma becoming pale or dark, are a sign that the tissue is not receiving the blood supply that it should. This type of change should be immediately reported to the surgeon, whether the surgery was recent or in the past. Option C: Necrosis occurs if the blood supply to the stoma is restricted. Initially, the stoma will become a darker red/purple and may even turn black, which is an indication that the blood supply is impaired. It may also feel cold and hard to touch. Option D: Stoma necrosis usually occurs within the first 5 postoperative days. The stoma appears discolored rather than red, moist, and shiny. Discoloration may be cyanotic, black, dark red, dusky bluish-purple, or brown. The stoma mucosa may be hard and dry or flaccid.

Which of the following best describes the method of action of medications, such as ranitidine (Zantac), which are used in the treatment of peptic ulcer disease? A. Neutralize acid. B. Reduce acid secretions. C. Stimulate gastrin release. D. Protect the mucosal barrier.

B. Reduce acid secretions. Ranitidine is a histamine-2 receptor antagonist that reduces acid secretion by inhibiting gastrin secretion. Ranitidine is a competitive inhibitor of histamine H2-receptors. The reversible inhibition of H2-receptors in gastric parietal cells results in a reduction in both gastric acid volume and concentration. Ranitidine's acid-lowering effect is more pronounced for basal and nocturnal acid secretion than it is for food-stimulated acid secretion. Option A: The antacids reduce the acid reaching the duodenum by neutralizing the acid present in the stomach. The salts' mechanism of neutralization of acid varies, and each salt has a different mechanism with the ultimate goal of acid neutralization. The formulation of aluminum hydrochloride and water results in the neutralization of the acid in the stomach. It is also known to inhibit pepsin activity. Option C: Gastrin release is stimulated by the stretching of the stomach walls during a meal, the presence of certain foods (particularly proteins) within the stomach cavity, and an increase in the pH levels of the stomach (i.e. the stomach becoming less acidic). Option D: Sucralfate, a polymer of sucrose with aluminum hydroxide, forms a protective coating on the mucosal lining, particularly in ulcerated areas. In the presence of acid, it becomes a gel that adheres to epithelial cells and ulcer craters. Sucralfate is only minimally absorbed and can cause constipation.

What is the primary nursing diagnosis for a 4th to 10th-day postoperative liver transplant patient? A. Excess Fluid Volume B. Risk for Rejection C. Impaired Skin Integrity D. Decreased Cardiac Output

B. Risk for Rejection Risk for rejection is always a possibility, especially during the 4th to 10th day postoperatively. LT patients are at risk for several complications. The primary care NP should be aware of these complications and needs to know when referral back to a transplant center or hepatologist is appropriate. The most serious issues are problems with the vasculature of the liver, biliary issues, rejection, and infection. Lab abnormalities—specifically elevation in alkaline phosphatase, alanine aminotransferase (ALT), and serum bilirubin levels—are usually the first indication of a problem in one or more of these areas. Option A: Post-transplant acute kidney injury (AKI) has been reported to occur in 9-78% of cases with 10% progressing to end-stage renal failure. Early identification of potential AKI is crucial to improving patient outcomes as evidence shows that even small increases in serum creatinine are associated with a decline in overall mortality. Option C: Patients are also at risk for specific opportunistic infections in the early postoperative period. Herpes simplex virus (HSV) reactivation disease is the most common opportunistic viral infection and can quickly progress to disseminated multi-organ infection and failure. Option D: In the first month postoperatively patients are most likely to develop infections related to the surgical procedure and hospitalization, such as bacterial and fungal wound infections, urinary tract infections, bloodstream infections, pneumonia, and Clostridium difficile colitis.

To inhibit pancreatic secretions, which pharmacologic agent would you anticipate administering to a patient with acute pancreatitis? A. Nitroglycerin B. Somatostatin C. Pancrelipase D. Pepcid

B. Somatostatin Somatostatin, a treatment for acute pancreatitis, inhibits the release of pancreatic enzymes. Somatostatin produces predominantly neuroendocrine inhibitory effects across multiple systems. It is known to inhibit GI, endocrine, exocrine, pancreatic, and pituitary secretions, as well as modify neurotransmission and memory formation in the CNS. Option A: Nitroglycerin is a vasodilator and does not affect pancreatic secretions. Nitroglycerin is a vasodilatory drug used primarily to provide relief from anginal chest pain. It is currently FDA approved for the acute relief of an attack or acute prophylaxis of angina pectoris secondary to coronary artery disease. Option C: Pancrelipase is an enzyme that aids in the digestion and absorption of fats and proteins. Pancrelipase refers to a class of medications designed to treat malabsorption and abdominal pain secondary to exocrine pancreatic insufficiency. These agents serve as exogenous versions of digestive hormones and enzymes required for normal digestion and are ingested with meals to improve digestion, absorption, and abdominal pain frequently seen in chronic pancreatitis and exocrine pancreatic insufficiency. Option D: Pepcid is an H2 blocker and is used to decrease gastric motility. H2 receptor blockers, or H2 receptor antagonists (H2RAs), are a class of gastric acid-suppressing agents frequently used in various gastric conditions. They are FDA-approved for short-term use in treating uncomplicated gastroesophageal reflux disease (GERD), gastric or duodenal ulcers, gastric hypersecretion, and mild to infrequent heartburn or indigestion.

An intubated patient is receiving continuous enteral feedings through a Salem pump tube at a rate of 60ml/hr. Gastric residuals have been 30-40ml when monitored Q4H. You check the gastric residual and aspirate 220ml. What is your first response to this finding? A. Notify the doctor immediately. B. Stop the feeding, and clamp the NG tube. C. Discard the 220ml and clamp the NG tube. D. Give a prescribed GI stimulant such as metoclopramide (Reglan).

B. Stop the feeding, and clamp the NG tube. A gastric residual greater than 2 hours worth of feeding or 100-150ml is considered too high. The feeding should be stopped; the NG tube clamped, and then allowed time for the stomach to empty before additional feeding is added. Gastric residual volume is the amount of liquid drained from a stomach following administration of enteral feed; this liquid consists mainly of infused nutritional formula or water, and secreted GI juice. Gastric residual volume is measured either by aspiration using a syringe, or by gravity drainage to a reservoir. Option A: It is unnecessary to notify the physician immediately. Monitoring GRV involves obtaining frequent GRV measurements and employing appropriate interventions in patients with large GRVs. Gastric residual volume monitoring (monitoring of residual volume of the enteral nutrients including digestive juices) is an essential component of EN patient care and aids in preventing complications due to EN. Option C: Administering additional enteral nutrients in patients with increased GRVs may cause aspiration and an increase in intra?abdominal pressure, which increases the risk of respiratory and circulatory failure, and intestinal necrosis. For this reason, it is particularly important to monitor GRV in the early stages of administration of enteral nutrition, especially in critically ill patients. Option D: Frequency of GRV measurement (e.g. every six hours) and the intervention strategy for large GRVs (e.g. if GRV is above 500 mL, hold feeding for two hours and re?check GRV) is usually decided as per institution?specific protocols and needs of the inpatient population.

The nurse is performing colostomy irrigation on a male client. During the irrigation, the client begins to complain of abdominal cramps. What is the appropriate nursing action? A. Notify the physician B. Stop the irrigation temporarily C. Increase the height of the irrigation D. Medicate for pain and resume the irrigation

B. Stop the irrigation temporarily. If cramping occurs during colostomy irrigation, the irrigation flow is stopped temporarily and the client is allowed to rest. Cramping may occur from an infusion that is too rapid or is causing too much pressure. Have the colostomy patient sit on or near the toilet for about 15 to 20 minutes so the initial colostomy returns can drain into the toilet. (If the patient is on bed rest, allow the colostomy to drain into the bedpan.) Option A: The physician does not need to be notified. Unless contraindicated or otherwise ordered by the physician, it is best to establish a routine of daily irrigation in accordance with the patient's former bowel habits. Option C: Increasing the height of the irrigation will cause further discomfort. Hold the enema can approximately 12 inches above the bed and allow the solution to flow in slowly to avoid painful cramps usually caused by too rapid flow. Option D: Medicating the client for pain is not the appropriate action in this situation. If cramping occurs, slow down the flow rate and ask the patient to deep breathe until cramps subside. Cramping during irrigation may indicate that the flow is too fast or the water is too cold.

The client with a colostomy has an order for irrigation of the colostomy. The nurse used which solution for irrigation?A. Distilled water A. Distilled water B. Tap water C. Sterile water D. Lactated Ringer's

B. Tap water Warm tap water or saline solution is used to irrigate a colostomy. If tap water is not suitable for drinking, then bottled water should be used. The prescribed irrigating solution is usually 500-1000cc warm (100º-105ºF) tap water. Fill the irrigation bag with the prescribed solution and hang it on the IV pole or hook. Option A: Distilled water is not used for irrigation. When performed safely and correctly, CI is an excellent method to regulate the evacuation of stool from the ostomy in a selected population of patients with an end colostomy and has rarely been associated with complications. It may also help to improve quality of life Option C: Sterile water is not used for irrigation. Using water that is too warm or too cool, or instilling water too quickly, may cause abdominal cramps or a vasovagal response.6 Therefore, it is important to provide careful written and oral instructions to the patient. In addition, the first irrigation should be performed in the presence of a health care professional because a small number of patients may experience dizziness as a result of a fall in blood pressure or pulse rate. Option D: Lactated Ringer's are not used for irrigation. Irrigations for colostomies involve the regular installation of moderate to large volumes of water to clear the colon of stool. Essentially, it is an enema into the stoma resulting in the clearing of stool from the colon. Some individuals who have colostomies choose to use irrigations as a means to regulate the function of their stoma.

A female client with dysphagia is being prepared for discharge. Which outcome indicates that the client is ready for discharge? A. The client doesn't exhibit rectal tenesmus. B. The client is free from esophagitis and achalasia. C. The client reports diminished duodenal inflammation. D. The client has normal gastric structures.

B. The client is free from esophagitis and achalasia. Dysphagia may be the reason why a client with esophagitis or achalasia seeks treatment. Dysphagia is common in patients with erosive esophagitis but is not a reliable clinical predictor of severe erosive esophagitis. Dysphagia resolved with PPI therapy in most cases, but persistent dysphagia may indicate failed healing. Dysphagia isn't associated with rectal tenesmus, duodenal inflammation, or abnormal gastric structures. Option A: Rectal tenesmus can happen for several reasons. The most common is colon inflammation, either from a noninfectious or infectious cause. Inflammatory bowel disease (IBD) is one cause of colon inflammation. IBD is an umbrella term for several long-term conditions involving chronic inflammation of the gut. Option C: The most common cause of duodenitis is infection by Helicobacter pylori (H. pylori) bacteria. Another common cause is the long-term use of NSAIDs (such as aspirin and ibuprofen). Celiac disease, an allergy to gluten, causes a particular type of inflammation in the duodenum along with other changes. Option D: Dysphagia means difficulty swallowing. For this diagnosis, it is critical that related symptoms be associated with the act of swallowing a liquid or solid bolus. When unassociated with swallowing, the sensation of fullness in the upper esophagus suggests globus hystericus, which is distinct from dysphagia.

A client has been diagnosed with adenocarcinoma of the stomach and is scheduled to undergo a subtotal gastrectomy (Billroth II procedure). During preoperative teaching, the nurse is reinforcing information about the procedure. Which of the following explanations is most accurate? A. The procedure will result in enlargement of the pyloric sphincter. B. The procedure will result in anastomosis of the gastric stump to the jejunum. C. The procedure will result in removal of the duodenum. D. The procedure will result in repositioning of the vagus nerve.

B. The procedure will result in anastomosis of the gastric stump to the jejunum. A Billroth II procedure bypasses the duodenum and connects the gastric stump directly to the jejunum. The pyloric sphincter is removed, along with some of the stomach fundus. If the stomach cannot be reconnected to the duodenum, a Billroth II is performed, in which an opening hole is made in the next section of the small intestine, the jejunum, and the stomach attached at that opening. Option A: A pyloromyotomy is an operation to loosen the tight muscle causing the blockage between the stomach and small intestine. During the operation, the surgeon cuts the tight muscle between the stomach and small intestine. This loosens the muscle so the stomach can empty and food will be able to pass easily into the small intestine. Option C: The Whipple procedure (pancreaticoduodenectomy) is an operation to remove the head of the pancreas, the first part of the small intestine (duodenum), the gallbladder, and the bile duct. The remaining organs are reattached to allow the client to digest food normally after surgery. Option D: Billroth II gastrojejunostomy is a procedure that has been performed for tumor or severe ulcer disease in the distal stomach. There are many variations on the procedure, but they generally involve resection of the diseased portion of the distal stomach and a side-to-side anastomosis of the residual stomach to jejunum through the transverse mesocolon. It can be performed with either an antecolic or a retrocolic anastomosis.

The nurse evaluates the client's stoma during the initial post-op period. Which of the following observations should be reported immediately to the physician? A. The stoma is slightly edematous. B. The stoma is dark red to purple. C. The stoma oozes a small amount of blood. D. The stoma does not expel stool.

B. The stoma is dark red to purple. A dark red to purple stoma indicates inadequate blood supply. Major color changes in a stoma, with the stoma becoming pale or dark, are a sign that the tissue is not receiving the blood supply that it should. Option A: Mild edema is normal in the early post-op period. A normal stoma in the days after surgery may be swollen and may also produce mucus. While the stoma itself should be moist, the skin around the stoma should be normal in appearance. Option C: Slight oozing of blood is normal in the early post-op period. The skin may be tender initially during the healing process and may feel irritated by normal cleaning. The skin immediately surrounding the stoma and stoma can be irritated by the cleaning process. A small amount of blood from the stoma itself is not unusual while it is healing. Option D: The colostomy would typically not begin functioning until 2-4 days after surgery. A stoma should be a beefy red or pink color. The tissue that makes a stoma is the lining of the intestine and should be moist and shiny. It is very similar in appearance to the inside of the mouth along the cheek.

The nurse would assess the client experiencing an acute episode of cholecystitis for pain that is located in the right A. Upper quadrant and radiates to the left scapula and shoulder. B. Upper quadrant and radiates to the right scapula and shoulder. C. Lower quadrant and radiates to the umbilicus. D. Lower quadrant and radiates to the back.

B. Upper quadrant and radiates to the right scapula and shoulder During an acute "gallbladder attack," the client may complain of severe right upper quadrant pain that radiates to the right scapula and shoulder. This is governed by the pattern on dermatomes in the body. Acute cholecystitis is inflammation of the gallbladder that occurs due to occlusion of the cystic duct or impaired emptying of the gallbladder. Often this impaired emptying is due to stones or biliary sludge. Option A: When cystic duct blockage is caused by a stone, it is called acute calculous cholecystitis. It is important to know, one can have pain due to temporary obstruction by gallstones, and that is called biliary colic. The diagnosis of biliary colic is upgraded to acute calculous cholecystitis if the pain does not resolve in six hours. If no stone is identified, it is called acute acalculous cholecystitis. Option C: Cases of chronic cholecystitis present with progressing right upper quadrant abdominal pain with bloating, food intolerances (especially greasy and spicy foods), increased gas, nausea, and vomiting. Pain in the mid-back or shoulder may also occur. This pain could be present for years until diagnosis. Option D: The pathophysiologic mechanism of acute cholecystitis is blockage of the cystic duct. Cholecystitis is a condition best treated with surgery; however, it can be treated conservatively if necessary. This condition can be associated with or without the presence of gallstones and can also be classified as acute or chronic.

The nurse instructs the nursing assistant on how to provide oral hygiene for a client who cannot perform this task for himself. Which of the following techniques should the nurse tell the assistant to incorporate into the client's daily care? A. Assess the oral cavity each time mouth care is given and record observations. B. Use a soft toothbrush to brush the client's teeth after each meal. C. Swab the client's tongue, gums, and lips with a soft foam applicator every 2 hours. D. Rinse the client's mouth with mouthwash several times a day.

B. Use a soft toothbrush to brush the client's teeth after each meal. A soft toothbrush should be used to brush the client's teeth after each meal and more often as needed. Mechanical cleaning is necessary to maintain oral health, simulate gingiva, and remove plaque. Gently brush teeth in circular motions at a 45-degree angle to the gum line. Brush for 1-2 minutes to remove plaque build-up. Option A: Assessing the oral cavity and recording observations is the responsibility of the nurse, not the nursing assistant. Inspect the patient's oral cavity (lips, gums, teeth, tongue, inside cheeks, and mucosa of floor and roof of the mouth) for any sign of dryness, edema, redness, bleeding, or other debris. Option C: Swabbing with a safe foam applicator does not provide enough friction to clean the mouth. In a comatose patient, use an Asepto syringe or irrigating bulb without a needle to rinse the oral cavity. Swab or use oral suction to remove pooled secretions Option D: Mouthwash can be a drying irritant and is not recommended for frequent use. Use 0.05% Cetylpyridinium Chloride (antifungal and antibacterial) containing mouthwash to reduce the risk of infection and to remove plaque.

When planning home care for a client with hepatitis A, which preventive measure should be emphasized to protect the client's family? A. Keeping the client in complete isolation B. Using good sanitation with dishes and shared bathrooms C. Avoiding contact with blood-soiled clothing or dressing D. Forbidding the sharing of needles or syringes

B. Using good sanitation with dishes and shared bathrooms. Hepatitis A is transmitted through the fecal-oral route or from contaminated water or food. Measures to protect the family include good handwashing, personal hygiene and sanitation, and the use of standard precautions. According to the WHO, the most effective way to prevent HAV infection is to improve sanitation, food safety, and immunization practices. Option A: Complete isolation is not required. No specific treatment is needed for most patients with acute, uncomplicated HAV infection beyond supportive care. Complete recovery from symptoms may take several weeks to months. Option C: HAV is most commonly transmitted through the oral-fecal route via exposure to contaminated food, water, or close physical contact with an infectious person. According to the World Health Organization (WHO), infection rates in developed countries are low. Option D: Avoiding the sharing of needles or syringes are precautions needed to prevent transmission of hepatitis B. Globally, the rates of HAV have decreased due to improvements in public healthcare policies, sanitation, and education, but infection rates of other hepatitis viruses appear to be increasing.

The nurse is caring for a male client with a diagnosis of chronic gastritis. The nurse monitors the client knowing that this client is at risk for which vitamin deficiency? A. Vitamin A B. Vitamin B12 C. Vitamin C D. Vitamin E

B. Vitamin B12 Chronic gastritis causes deterioration and atrophy of the lining of the stomach, leading to the loss of the function of the parietal cells. The source of the intrinsic factor is lost, which results in the inability to absorb vitamin B12. This leads to the development of pernicious anemia. Option A: Autoimmune gastritis (pernicious anemia) is the most common cause of severe vitamin B12 deficiency due to food-cobalamin malabsorption in the elderly, nevertheless use of medications, as proton pump inhibitors, histamine H2 blockers, metformin, or cholestyramine can interfere with or reduce vitamin B12 absorption. Option C: The most common is vitamin B12 deficiency, which results in megaloblastic anemia and iron malabsorption, leading to iron deficiency anemia. However, in recent years the deficiency of several other vitamins and micronutrients, such as vitamin C, vitamin D, folic acid, and calcium, has been increasingly described in patients with chronic gastritis. Option D: The reduction of intrinsic factor levels results in vitamin B12 malabsorption. Indeed, the intrinsic factor has a key role in binding vitamin B12 in the duodenum and transporting it to the terminal ileum for absorption. More recently, iron deficiency and iron-deficiency anemia have been reported in the setting of CAAG, particularly in younger patients.

The nurse is caring for a client with chronic gastritis. The nurse monitors the client, knowing that this client is at risk for which of the following vitamin deficiencies? A. Vitamin A B. Vitamin B12 C. Vitamin C D. Vitamin E

B. Vitamin B12 Chronic gastritis causes deterioration and atrophy of the lining of the stomach, leading to the loss of the functioning parietal cells. The source of the intrinsic factor is lost, which results in the inability to absorb vitamin B12. This leads to the development of pernicious anemia. Acute gastritis will evolve to chronic, if not treated. Helicobacter pylori (H. pylori) is the most common cause of gastritis worldwide. However, 60 to 70% of H. pylori-negative subjects with functional dyspepsia or non-erosive gastroesophageal reflux were also found to have gastritis. Option A: Pernicious anemia can develop in AMAG with an overall prevalence of 0.1%. It can present with fatigue, dizziness, irritability, depression, insomnia, and mood swings. Occasionally, it can present with normal vitamin B12 levels. Most individuals are already treated for their iron, folate, or vitamin B12 deficiency without a confirmed underlying diagnosis. Option C: Chronic atrophic gastritis is often diagnosed late, as many individuals can be asymptomatic for years and diagnosed incidentally. It is essential to investigate those with risk factors, such as the new onset of upper gastrointestinal symptoms in those over 50 years of age, pernicious anemia, or with a family history. Option D: The incidental finding of anemia (iron deficiency or B12) should be investigated alongside medical treatment to avoid delay in diagnosis. The classical presentation would suggest individuals presenting with epigastric pain, nausea, and occasionally vomiting; however, this is rare. The most common presentation is anemia.

The RN had taught the pt with chronic pancreatitis about risk factor modification to reduce the incidence of recurrences. The RN determines that pt has understood the information if pt states it will be necessary to control which of the following. A. diabetes mellitus B. alcohol intake C. duodenal ulcer D. Crohn's disease

B. alcohol intake

A pt with cholelithiasis experiences discomfort after ingesting fatty foods because A. fatty foods are hard to digest B. bile flow into intestine is obstructed C. liver is manufacturing inadequate bile D. there is inadequate closure of ampulla of water

B. bile flow into intestine is obstructed

The nurse is reviewing the medication record of a client with acute gastritis. Which medication, if noted on the client's record, would the nurse ? A. digoxin (Lanoxin) B. indomethacin (Indocin) C. furosemide (Lasix) D. propranolol hydrochloride (Inderal)

B. indomethacin (Indocin) Indomethacin (Indocin) is an NSAID and can cause ulceration of the esophagus, stomach, duodenum, or small intestine. Indomethacin is contraindicated in a client with GI disorders. Nonsteroidal anti-inflammatory drugs (NSAIDs) such as indomethacin are capable of producing injury to gastrointestinal mucosa in experimental animals and humans, and their use is associated with a significant risk of hemorrhage, erosions, and perforation of both gastric and intestinal ulcers Option A: Digoxin is not contraindicated for acute gastritis. The molecular basis for the gastrointestinal toxicity of NSAIDs is widely believed to be their inhibitory activity against cyclooxygenase, which causes them to block the production of prostaglandins and their therapeutic actions. Option C: The Food and Drug Administration (FDA) has approved furosemide to treat conditions with volume overload and edema secondary to congestive heart failure exacerbation, liver failure, or renal failure, including nephrotic syndrome. It is not contraindicated in a client with gastritis. Option D: Propranolol can be used to ameliorate the sympathetic response in angina, tachyarrhythmias, prevention of acute ischemic attacks, migraine prophylaxis, and restless leg syndrome. Propranolol can be used in almost all cases if the desired result is to slow contractility and decrease a patient's heart rate.

Which pt will be discharged home first? A. obese older adult who has laparoscopic surgery B. middle aged thin adult who had laparoscopic C. middle aged thin adult who as traditional laparotomy D. older obese adult who has traditional lapaprotomy

B. middle aged thin adult who had laparoscopic

Which pt is at highest risk for developing gallstones? A. obese male with history of COPD B. obese female on hormone replacement therapy C. thin male with history of CABG D. thin female who has recently given birth

B. obese female on hormone replacement therapy

A pt is suspected of having a peptic ulcer. When obtaining a history from this pt, the RN should expect the reported pain to A. intensify when pt vomits B. occur 1-3 hours after meals C. increase when eating fatty foods D. begin in epigastrium and radiate across abdomen

B. occur 1-3 hours after meals

The pt with pancreatitis asks what is the reason for the NG tube? Which fact should form the basis of the RN's reply? A. prevent vomiting B. remove gastric contents so they do not enter intestine C. allow for monitoring of gastric pH D. protect gastric lining from pancreatic enzymes

B. remove gastric contents so they do not enter intestine

An enema is prescribed for a client with suspected appendicitis. Which of the following actions should the nurse take? A. Prepare 750 ml of irrigating solution warmed to 100° F. B. the physician about the order. C. Provide privacy and explain the procedure to the client. D. Assist the client to the left lateral Sims position.

B. the physician about the order. Enemas are contraindicated in an acute abdominal condition of unknown origin as well as after recent colon or rectal surgery or myocardial infarction. People with symptoms of appendicitis should not take laxatives or enemas to relieve constipation because these medicines could cause the appendix to burst. The other answers are correct only when enema administration is appropriate. Option A: Warming the enema solution to body temperature may be beneficial as heat stimulates the rectal mucosa. Dougherty and Lister (2004) recommend a solution temperature of 40.5-43.3°C for non-oil-based enemas. Cold solutions should be avoided as they may cause cramping. Option C: Obtain informed consent, identifying allergies and any contraindications. Provide the patient with reassurance. Assess patient privacy and dignity and take steps to maximize both. Option D: Position the patient on the left side, lying with the knees drawn to the abdomen. This eases the passage and flow of fluid into the rectum. Gravity and the anatomical structure of the sigmoid colon also suggest that this will aid enema distribution and retention.

A 26-yr-old woman is being evaluated for vomiting and abdominal pain. Which question from the nurse will be most useful in determining the cause of the patient's symptoms? A. "What type of foods do you eat?" B. "Is it possible that you are pregnant?" C. "Can you tell me more about the pain?" D. "What is your usual elimination pattern?"

C A complete description of the pain provides clues about the cause of the problem. Although the nurse should ask whether the patient is pregnant to determine whether the patient might have an ectopic pregnancy and before any radiology studies are done, this information is not the most useful in determining the cause of the pain. The usual diet and elimination patterns are less helpful in determining the reason for the patient's symptoms.

A patient has just been admitted to the emergency department with nausea and vomiting. Which information requires the most rapid intervention by the nurse? A. The patient has been vomiting for 4 days. B. The patient takes antacids 8 to 10 times a day. C. The patient is lethargic and difficult to arouse. D. The patient had a small intestinal resection 2 years ago.

C A lethargic patient is at risk for aspiration, and the nurse will need to position the patient to decrease aspiration risk. The other information is also important to collect, but it does not require as quick action as the risk for aspiration.

After change-of-shift report, which patient would the nurse assess first? A. A 42-yr-old patient who has acute gastritis and ongoing epigastric pain B. A 70-yr-old patient with a hiatal hernia who experiences frequent heartburn C. A 60-yr-old patient with nausea and vomiting who is lethargic with dry mucosa D. A 53-yr-old patient who has dumping syndrome after a recent partial gastrectomy

C A patient with nausea and vomiting who is lethargic with dry mucosa is at high risk for problems such as aspiration, dehydration, and fluid and electrolyte disturbances. The other patients will also need to be assessed, but the information about them indicates symptoms that are typical for their diagnoses and are not life threatening.

Which action would the nurse include in the plan of care for a patient who is being admitted with a C. difficile infection? A. Teach the patient about proper food storage. B. Order a diet without dairy products for the patient. C. Place the patient in a private room on contact isolation. D. Teach the patient about why antibiotics will not be used.

C Because C. difficile is highly contagious, the patient would be placed in a private room, and contact precautions would be used. There is no need to restrict dairy products for this type of diarrhea. Metronidazole (Flagyl) is frequently used to treat C. difficile infections. Improper food handling and storage do not cause C. difficile.

A patient with a stroke is unconscious and unresponsive to stimuli. After learning that the patient has a history of gastroesophageal reflux disease (GERD), which assessment would the nurse plan to make more frequently than is routine? A. Apical pulse B. Bowel sounds C. Breath sounds D. Abdominal girth

C Because GERD may cause aspiration, the unconscious patient is at risk for developing aspiration pneumonia. Bowel sounds, abdominal girth, and apical pulse will not be affected by the patient's stroke or GERD and do not require more frequent monitoring than the routine.

A patient in the emergency department has just been diagnosed with peritonitis from a ruptured diverticulum. Which prescribed intervention will the nurse implement first? A. Send the patient for a CT scan. B. Insert a urinary catheter to drainage. C. Infuse metronidazole (Flagyl) 500 mg IV. D. Place a nasogastric tube to intermittent low suction.

C Because peritonitis can be fatal if treatment is delayed, the initial action would be to start antibiotic therapy (after any ordered cultures are obtained). The other actions can be done after antibiotic therapy is initiated.

Which prescribed action will the nurse implement first for a patient who has vomited 1100 mL of blood? A. Give an IV H2 receptor antagonist. B. Draw blood for type and crossmatch. C. Administer 1 L of lactated Ringer's solution. D. Insert a nasogastric (NG) tube and connect to suction.

C Because the patient has vomited a large amount of blood, correction of hypovolemia and prevention of hypovolemic shock are the priorities. The other actions also are important to implement quickly but are not the highest priorities.

A patient had an abdominal-perineal resection for colon cancer. Which action is most important for the nurse to include in the plan of care for the day after surgery? A. Teach about a low-residue diet. B. Monitor output from the stoma. C. Assess the drainage and incision. D. Encourage acceptance of the stoma.

C Because the perineal wound is at high risk for infection, the initial care is focused on assessment and care of this wound. Teaching about diet is best done closer to discharge from the hospital. There will be very little drainage into the colostomy until peristalsis returns. The patient will be encouraged to assist with the colostomy, but this is not the highest priority in the immediate postoperative period.

A patient who has cirrhosis and esophageal varices is being treated with propranolol. Which finding is the best indicator to the nurse that the medication has been effective? A. The patient reports no chest pain. B. Blood pressure is 130/80 mm Hg. C. Stools test negative for occult blood. D. The apical pulse rate is 68 beats/min.

C Because the purpose of b-blocker therapy for patients with esophageal varices is to decrease the risk for bleeding from esophageal varices, the best indicator of the effectiveness for propranolol is the lack of blood in the stools. Although propranolol is used to treat hypertension, angina, and tachycardia, the purpose for use in this patient is to decrease the risk for bleeding from esophageal varices.

After a patient has had a hemorrhoidectomy at an outpatient surgical center, which instructions will the nurse include in discharge teaching? A. Maintain a low-residue diet until the area is healed. B. Avoid using any topical preparations on the surgical area. C. Take prescribed pain medications before you expect a bowel movement. D. Delay having a bowel movement for several days until you are well healed.

C Bowel movements may be very painful, and patients may avoid defecation unless pain medication is taken before the bowel movement. A high-residue diet will increase stool bulk and prevent constipation. Delay of bowel movements is likely to lead to constipation. Topical preparations that provide anesthesia or reduce internal sphincter spasms such as topical lidocaine, 2% diltiazem, and glyceryl trinitrate may be used. Warm Sitz baths are used to relieve pain and keep the surgical area clean.

Which patient statement indicates that the nurse's postoperative teaching after a gastroduodenostomy has been effective? A. "I will drink more liquids with my meals." B. "I should choose high carbohydrate foods." C. "Vitamin supplements may prevent anemia." D. "Persistent heartburn is expected after surgery."

C Cobalamin deficiency may occur after partial gastrectomy, and the patient may need to receive cobalamin via injections or nasal spray. Although peptic ulcer disease may recur, persistent heartburn is not expected after surgery, and the patient should call the health care provider if this occurs. Ingestion of liquids with meals is avoided to prevent dumping syndrome. Foods that have moderate fat and low carbohydrate should be chosen to prevent dumping syndrome.

A patient born in 1955 had hepatitis A infection 1 year ago. Which action would the nurse include in care when the patient is seen for a routine annual physical examination? A. Start the hepatitis B immunization series. B. Teach the patient about hepatitis A immune globulin. C. Ask whether the patient has been screened for hepatitis C. D. Test for anti-hepatitis-A virus immune globulin M (anti-HAV-IgM).

C Current CDC guidelines indicate that all patients who were born between 1945 and 1965 should be screened for hepatitis C because many persons who are positive have not been diagnosed. Although routine hepatitis B immunization is recommended for infants, children, and adolescents, vaccination for hepatitis B is recommended only for adults at risk for blood-borne infections. Because the patient has already had hepatitis A, immunization, and anti-HAV IgM levels will not be needed.

A patient returned from a laparoscopic Nissen fundoplication for hiatal hernia 4 hours ago. Which assessment finding is most important for the nurse to address immediately? A. The patient reports 7/10 (0 to 10 scale) abdominal pain. B. The patient is experiencing intermittent waves of nausea. C. The patient has no breath sounds in the left anterior chest. D. The patient has hypoactive bowel sounds in all four quadrants.

C Decreased breath sounds on one side may indicate a pneumothorax, which requires rapid diagnosis and treatment. The nausea and abdominal pain would be addressed, but they are not as high priority as the patient's respiratory status. Decreased bowel sounds are expected after surgery and require ongoing monitoring but no other action.

A 36-yr-old female patient is receiving treatment for chronic hepatitis B with pegylated interferon (Pegasys). Which finding is important to communicate to the health care provider to suggest a change in therapy? A. Nausea and anorexia B. Weight loss of 2 lb (1 kg) C. Report of severe depression D. Hemoglobin level of 10.4 g/dL

C Depression is a side effect of therapy. We must screen patients for depression and other mood disorders before starting treatment and monitor them frequently while on therapy. Anemia, weight loss, and nausea are common adverse effects of the prescribed regimen and may require actions such as patient teaching, but they would not require immediate cessation of the therapy.

Which activity in the care of a patient with a new colostomy could the nurse delegate to assistive personnel (AP)? A. Document the appearance of the stoma. B. Place a pouching system over the ostomy. C. Drain and measure the output from the ostomy. D. Check the skin around the stoma for breakdown.

C Draining and measuring the output from the ostomy is included in AP education and scope of practice. The other actions should be implemented by LPNs or RNs.

Which diet choice by the patient with an acute exacerbation of inflammatory bowel disease (IBD) indicates a need for more teaching? A. Scrambled eggs B. White toast and jam C. Oatmeal with cream D. Pancakes with syrup

C During acute exacerbations of IBD, the patient would avoid high-fiber foods such as whole grains. High-fat foods also may cause diarrhea in some patients. The other choices are low residue and would be appropriate for this patient.

The nurse is caring for a patient with pancreatic cancer. Which nursing action would be the highest priority? A. Offer psychologic support for depression. B. Offer high-calorie, high-protein dietary choices. C. Administer prescribed opioids to relieve pain as needed. D. Teach about the need to avoid scratching any pruritic areas.

C Effective pain management will be necessary in order for the patient to improve nutrition, be receptive to teaching, or manage anxiety or depression.

The nurse has completed teaching a patient with newly diagnosed eosinophilic esophagitis about the management of the disease. Which patient action indicates that the teaching has been effective? A. Patient orders nonfat milk for each meal. B. Patient uses the prescribed corticosteroid inhaler. C. Patient schedules an appointment for allergy testing. D. Patient takes ibuprofen (Advil) to control throat pain.

C Eosinophilic esophagitis is frequently associated with environmental allergens, so allergy testing is used to determine possible triggers. Corticosteroid therapy may be prescribed, but the medication will be swallowed, not inhaled. Milk is a frequent trigger for attacks. NSAIDs are not used for eosinophilic esophagitis.

Which response by the nurse best explains the purpose of propranolol for a patient who was admitted with bleeding esophageal varices? A. The medication will reduce the risk for aspiration. B. The medication will inhibit development of gastric ulcers. C. The medication will prevent irritation of the enlarged veins. D. The medication will decrease nausea and improve the appetite.

C Esophageal varices are dilated submucosal veins. Patients with varices who are at risk for bleeding often receive a nonselective -blocker (nadolol, propranolol) to reduce bleeding risk. -Blockers decrease high portal pressure, which decreases the risk for rupture. Propranolol does not decrease the risk for peptic ulcers, reduce nausea, or help prevent aspiration pneumonia.

Which patient choice for a snack 3 hours before bedtime indicates that the nurse's teaching about gastroesophageal reflux disease (GERD) has been effective? A. Chocolate pudding B. Glass of low-fat milk C. Cherry gelatin with fruit D. Peanut butter and jelly sandwich

C Gelatin and fruit are low fat and will not decrease lower esophageal sphincter (LES) pressure. Foods such as chocolate are avoided because they lower LES pressure. Milk products increase gastric acid secretion. High-fat foods such as peanut butter decrease both gastric emptying and LES pressure.

Which assessment finding would the nurse need to report most quickly to the health care provider regarding a patient who has acute pancreatitis? A. Nausea and vomiting B. Hypotonic bowel sounds C. Muscle twitching and finger numbness D. Upper abdominal tenderness and guarding

C Muscle twitching and finger numbness indicate hypocalcemia, which may lead to tetany unless calcium gluconate is administered. Although the other findings should also be reported to the health care provider, they do not indicate complications that require rapid action.

A patient had an incisional cholecystectomy 6 hours ago. Which action would the nurse identify as the highest priority for the patient to accomplish? A. Perform leg exercises hourly while awake. B. Ambulate the evening of the operative day. C. Turn, cough, and deep breathe every 2 hours. D. Choose preferred low-fat foods from the menu.

C Postoperative nursing care after a cholecystectomy focuses on prevention of respiratory complications because the surgical incision is high in the abdomen and impairs coughing and deep breathing. The other nursing actions are also important to implement but are not as high a priority as ensuring adequate ventilation.

In reviewing the medical record for a patient admitted with acute pancreatitis, the nurse sees that the patient has a positive Cullen's sign. Indicate the area in the accompanying figure where the nurse will assess for this change. A. 1 B. 2 C. 3 D. 4

C The area around the umbilicus would be indicated. Cullen's sign consists of ecchymosis around the umbilicus. Cullen's sign occurs because of seepage of bloody exudates from the inflamed pancreas and indicates severe acute pancreatitis.

A patient has peptic ulcer disease associated with Helicobacter pylori. Which medications will the nurse plan to teach the patient? A. Sucralfate (Carafate), nystatin, and bismuth (Pepto-Bismol) B. Metoclopramide (Reglan), bethanechol, and promethazine C. Amoxicillin (Amoxil), clarithromycin, and omeprazole (Prilosec) D. Famotidine (Pepcid), magnesium hydroxide (Mylanta), and pantoprazole

C The drugs used in triple drug therapy include a proton pump inhibitor such as omeprazole and the antibiotics amoxicillin and clarithromycin. The other combinations listed are not included in the protocol for H. pylori infection.

An older patient reports chronic constipation. When would the nurse suggest that the patient regularly attempt defecation? A. Right after awakening in the morning B. Before eating breakfast C. Immediately after the first daily meal D. Right before bedtime

C The gastrocolic reflex is most active after the first daily meal. Awakening, the anticipation of eating, and bedtime timing do not stimulate these reflexes.

A 76-yr-old patient with obstipation has a fecal impaction and is incontinent of liquid stool. Which action would the nurse take first? A. Administer bulk-forming laxatives. B. Assist the patient to sit on the toilet. C. Manually remove the hard stool. D. Increase the patient's oral fluid intake.

C The initial action with a fecal impaction is manual disimpaction. The other actions will be used to prevent future constipation and impactions.

A 19-yr-old woman is brought to the emergency department with a knife handle protruding from her abdomen. Which action would the nurse take during the initial assessment of the patient? A. Remove the knife and assess the wound. B. Determine the presence of Rovsing sign. C. Check for circulation and tissue perfusion. D. Insert a urinary catheter and assess for hematuria.

C The initial assessment is focused on determining whether the patient has hypovolemic shock. The knife should not be removed until the patient is in surgery, where bleeding can be controlled. Rovsing sign is assessed in the patient with suspected appendicitis. Assessment for bladder trauma is not part of the initial assessment.

Which action would the nurse in the emergency department take first for a patient who arrives vomiting blood? A. Insert a large-gauge IV catheter. B. Draw blood for coagulation studies. C. Check blood pressure and heart rate. D. Place the patient in the supine position.

C The nurse's first action would be to determine the patient's hemodynamic status by assessing vital signs. Drawing blood for coagulation studies and inserting an IV catheter are also appropriate. However, the vital signs may indicate the need for more urgent actions. Because aspiration is a concern for this patient, the nurse will need to assess the patient's vital signs and neurologic status before placing the patient in a supine position.

At his first postoperative checkup appointment after a gastrojejunostomy (Billroth II), a patient reports that dizziness, weakness, and palpitations occur about 20 minutes after each meal. Which action would the nurse teach the patient to take? A. Increase the amount of fluid with meals. B. Eat foods that are higher in carbohydrates. C. Lie down for about 30 minutes after eating. D. Drink sugared fluids or eat candy after meals.

C The patient is experiencing symptoms of dumping syndrome, which may be reduced by lying down for a short rest after eating. Increasing fluid intake and choosing high carbohydrate foods will increase the risk for dumping syndrome. Having a sweet drink or hard candy will correct the hypoglycemia that is associated with dumping syndrome but will not prevent dumping syndrome.

How would the nurse prepare a patient with ascites for paracentesis? A. Place the patient on NPO status. B. Assist the patient to lie flat in bed. C. Ask the patient to empty the bladder. D. Position the patient on the right side.

C The patient should empty the bladder to decrease the risk of bladder perforation during the procedure. The patient would be positioned in Fowler's position and would not be able to lie flat without compromising breathing. Because no sedation is required for paracentesis, the patient does not need to be NPO.

When taking the blood pressure (BP) on the right arm of a patient who has severe acute pancreatitis, the nurse notices carpal spasms of the patient's right hand. Which action would the nurse take next? A. Ask the patient about any arm pain. B. Retake the patient's blood pressure. C. Check the calcium level in the health record. D. Notify the health care provider immediately.

C The patient with acute pancreatitis is at risk for hypocalcemia, and the assessment data indicate a positive Trousseau's sign. The health care provider would be notified after the nurse checks the patient's calcium level. There is no indication that the patient needs to have the BP rechecked or that there is any arm pain.

A young adult patient is admitted to the hospital for evaluation of right lower quadrant abdominal pain with nausea and vomiting. Which action would the nurse take? A. Assist the patient to cough and deep breathe. B. Palpate the abdomen for rebound tenderness. C. Suggest the patient lie on the side, flexing the right leg. D. Encourage the patient to sip clear, noncarbonated liquids.

C The patient's clinical manifestations are consistent with appendicitis. Lying still with the right leg flexed is often the most comfortable position. Checking for rebound tenderness frequently is unnecessary and uncomfortable for the patient. The patient should be NPO in case immediate surgery is needed. The patient will need to know how to cough and deep breathe postoperatively, but coughing will increase pain at this time.

A 72-yr-old patient with dehydration caused by an exacerbation of ulcerative colitis is receiving 5% dextrose in normal saline at 125 mL/hour. Which assessment finding by the nurse is most important to report to the health care provider? A. Skin is dry with tenting and poor turgor. B. Patient has not voided for the last 2 hours. C. Crackles are heard halfway up the posterior chest. D. Patient has had 5 loose stools over the previous 6 hours.

C The presence of crackles in an older patient receiving IV fluids at a high rate suggests volume overload and a need to reduce the rate of the IV infusion. The other data will be reported but are consistent with the patient's age and diagnosis and do not require a change in the prescribed treatment.

A patient is admitted to the emergency department with severe abdominal pain and rebound tenderness. Vital signs include temperature 102F (38.3C), pulse 120 beats/min, respirations 32 breaths/min, and blood pressure (BP) 82/54 mm Hg. Which prescribed intervention would the nurse implement first? A. Administer IV ketorolac 15 mg for pain relief. B. Send a blood sample for a complete blood count (CBC). C. Infuse a liter of lactated Ringer's solution over 30 minutes. D. Send the patient for an abdominal computed tomography (CT) scan.

C The priority for this patient is to treat the patient's hypovolemic shock with fluid infusion. The other actions should be implemented after starting the fluid infusion.

Which finding is most important for the nurse to communicate to the health care provider about a patient who received a liver transplant 1 week ago? A. Dry palpebral and oral mucosa B. Crackles at bilateral lung bases C. Temperature 100.8F (38.2C) D. No bowel movement for 4 days

C The risk of infection is high in the first few months after liver transplant, and fever is frequently the only sign of infection. The other patient data indicate the need for further assessment or nursing actions and might be communicated to the health care provider, but they do not indicate a need for urgent action.

During change-of-shift report, the nurse learns about the following four patients. Which patient would the nurse assess first? A. A patient who has compensated cirrhosis and reports anorexia B. A patient with chronic pancreatitis who has gnawing abdominal pain C. A patient with cirrhosis and ascites who has a temperature of 102F (38.8C) D. A patient recovering from a laparoscopic cholecystectomy who has severe shoulder pain

C This patient's history and fever suggest possible spontaneous bacterial peritonitis, which would require rapid assessment and interventions such as antibiotic therapy. The clinical manifestations for the other patients are consistent with their diagnoses and do not indicate complications are occurring.

A woman who was recently diagnosed with esophageal cancer tells the nurse, "I do not feel ready to die yet." Which response would the nurse provide? A. "Y ou may have quite a few years still left to live." B. "Thinking about dying will only make you feel worse." C. "Having this new diagnosis must be very hard for you." D. "It is important that you be realistic about your prognosis."

C This response is open ended and will encourage the patient to further discuss feelings of anxiety or sadness about the diagnosis. Patients with esophageal cancer have a low survival rate, so the response "You may have quite a few years still left to live" is misleading. The response beginning, "Thinking about dying" indicates that the nurse is not open to discussing the patient's fears of dying. The response beginning, "It is important that you be realistic" discourages the patient from feeling hopeful, which is important to patients with any life-threatening diagnosis.

Which information about an 80-yr-old male patient at the senior center is of most concern to the nurse? A. Decreased appetite B. Occasional indigestion C. Unintended weight loss D. Difficulty chewing food

C Unintentional weight loss is not a normal finding and may indicate a problem such as cancer or depression. Poor appetite, difficulty in chewing, and indigestion are common in older patients. These will need to be addressed but are not of as much concern as the weight loss.

A patient vomiting blood-streaked fluid is admitted to the hospital with acute gastritis. What would the nurse ask the patient about to determine possible risk factors for gastritis? A. The amount of saturated fat in the diet B. A family history of gastric or colon cancer C. Use of nonsteroidal antiinflammatory drugs D. A history of a large recent weight gain or loss

C Use of an NSAID is associated with damage to the gastric mucosa, which can result in acute gastritis. Family history, recent weight gain or loss, and fatty foods are not risk factors for acute gastritis.

An adult with E. coli O157:H7 food poisoning is admitted to the hospital with bloody diarrhea and dehydration. Which prescribed action will the nurse question? A. Infuse lactated Ringer's solution at 250 mL/hr. B. Monitor blood urea nitrogen and creatinine daily. C. Administer loperamide (Imodium) after each stool. D. Provide a clear liquid diet and progress diet as tolerated.

C Use of antidiarrheal agents is avoided with this type of food poisoning because the drugs slow GI motility and can prolong infection. IV fluids, clear oral fluids, and monitoring renal function are appropriate for dehydration.

While interviewing a young adult patient, the nurse learns that the patient has a family history of familial adenomatous polyposis (FAP). Which area of patient knowledge would the nurse plan to assess? A. Preventing noninfectious hepatitis B. Treating inflammatory bowel disease C. Risk for developing colorectal cancer D. Using antacids and proton pump inhibitors

C FAP is a genetic condition that greatly increases the risk for colorectal cancer. Noninfectious hepatitis, use of medications that treat increased gastric pH, and inflammatory bowel disease are not related to FAP.

A patient with chronic hepatitis B infection declines to take any medication that requires an injection. Which medication, if prescribed, requires further discussion with the health care provider? A. Tenofovir (Viread) B. Adefovir (Hepsera) C. Lamivudine (Epivir) D. Pegylated a-interferon (Pegasys)

C Pegylated a-interferon is administered subcutaneously. Tenofovir (Viread), adefovir (Hepsera), and entecavir (Baraclude) are administered orally. The medications are all appropriate for a patient with chronic hepatitis B infection.

A patient calls the clinic reporting diarrhea for 24 hours. Which action would the nurse take first? A. Inform the patient that testing of blood and stools will be needed. B. Suggest that the patient drink clear liquid fluids with electrolytes. C. Ask the patient to describe the stools and any associated symptoms. D. Advise the patient to use over-the-counter antidiarrheal medication.

C The initial response by the nurse should be further assessment of the patient. The other responses may be appropriate, depending on what is learned in the assessment.

When obtaining a nursing history on a client with a suspected gastric ulcer, which signs and symptoms would the nurse expect to see? Select all that apply. A. Epigastric pain at night B. Relief of epigastric pain after eating C. Vomiting D. Weight loss

C and D. Vomiting and weight loss are common with gastric ulcers. Gastric ulcers are a break in the mucosa of the stomach lining that penetrates through the muscularis mucosa and extends more than 5 mm in diameter. When alterations occur to the defense mechanisms of the stomach, it can cause changes in the gastric mucosa which will eventually result in erosion and then ulceration. Option A: Clients with a gastric ulcer are most likely to complain of a burning epigastric pain that occurs about one hour after eating. They often describe this pain as a sharp or burning type of pain that typically doesn't radiate. The most common finding on the physical exam is epigastric tenderness. Option B: Eating frequently aggravates the pain. Clients with duodenal ulcers are more likely to complain about pain that occurs during the night and is frequently relieved by eating. The typical presentation of a patient with gastric ulcers is epigastric pain that is worse with eating. It often correlates with mild nausea and early satiety. Option C: These symptoms may continue for weeks or months before patients seek medical help. Patients may present with upper GI bleeding. The clinician should ask if they are having any black tarry stools, hematemesis, coffee-ground emesis, or bright red blood per rectum. Option D: Alarm symptoms should be recognized which would make the need for an EGD more urgent. Alarm symptoms include unintentional weight loss, bleeding, age over 50, nausea, and vomiting. If a gastric ulcer is present on EGD, biopsies of the mucosa surrounding the ulcer will be necessary to rule out gastritis, Helicobacter pylori infection, and malignancy.

Nurse Farrah is providing care for Kristoff who has jaundice. Which statement indicates that the nurse understands the rationale for instituting skin care measures for the client? A. "Jaundice is associated with pressure ulcer formation." B. "Jaundice impairs urea production, which produces pruritus." C. "Jaundice produces pruritus due to impaired bile acid excretion." D. "Jaundice leads to decreased tissue perfusion and subsequent breakdown."

C. "Jaundice produces pruritus due to impaired bile acid excretion." Jaundice is a symptom characterized by increased bilirubin concentration in the blood. Bile acid excretion is impaired, increasing the bile acids in the skin and causing pruritus. Patients with jaundice often nominate pruritus as their most troublesome symptom to control and the symptom that has the most negative influence on their quality of life. The presence of pruritus can cause severe sleep deprivation resulting in lassitude, fatigue, depression, and suicidal ideation Option A: Jaundice is not associated with pressure ulcer formation. However, edema and hypoalbuminemia are. Itching in patients with primary biliary cirrhosis may be severe in the early stages of the disease when bile salt concentrations are low but cease to be a significant symptom when liver failure and cholestasis is advanced. Option B: Consequently bile salts emerged as the primary causative agents in pruritus. This was supported by the observation that feeding bile salts to cholestatic patients worsened pruritus, intradermal injection of bile salts in healthy volunteers caused local itching, and administration of anion exchange resins to bind luminal bile salts decreased itching intensity. Option D: Jaundice itself does not lead to decreased tissue perfusion. Histamine is the principal mediator of allergic reactions and is released by mast cells and circulating basophils. Bile salts, particularly chenodeoxycholate and deoxycholate, stimulate the release of histamine from mast cells and plasma histamine concentrations are increased in pruritic patients.

You're caring for Carin who has just had ileostomy surgery. During the first 24 hours post-op, how much drainage can you expect from the ileostomy? A. 100 ml B. 500 ml C. 1500 ml D. 5000 ml

C. 1500 ml The large intestine absorbs large amounts of water so the initial output from the ileostomy may be as much as 1500 to 2000 ml/24 hours. Gradually, the small intestine absorbs more fluid, and the output decreases. The major function of the small intestine is to absorb the body's nutrients and water. Enzymes released into the small intestine break food into small particles so that vitally needed proteins, carbohydrates, fats, vitamins, and minerals can be absorbed. Option A: 100 ml is too low an amount for ileostomy drainage. There are occasions when the ileostomy does not function for short periods of time. This is normal. However, if the stoma is not active for 4 to 6 hours and is accompanied by cramps and/or nausea, the intestine could be obstructed. An obstruction (or blockage) may be partial; that is, some liquid may pass through. Option B: Loose stool can come from eating certain foods and is usually temporary. Raw fruits and vegetables, milk, fruit juice, prune juice, or contaminated drinking water are examples. Loose stool may also be caused by emotional stress. Some people with ileostomies may always have "watery discharge" and this is normal for them. 800-1,000 cc/day is normal for an ileostomy. Option D: A drainage of 5000 ml in the first 24 hours may dehydrate the patient. Dehydration is a common concern with symptoms of increased thirst, dry mouth, decreased urine output, and fatigue. Increase any type of fluids such as Pedialyte® which is high in potassium and sodium. Avoid high-sugar drinks which can cause osmotic diarrhea. Daily intake of fluids should be 8-10 (8 oz.) glasses.

Nurse Berlinda is assigned to a 41-year-old client who has a diagnosis of chronic pancreatitis. The nurse reviews the laboratory result, anticipating a laboratory report that indicates a serum amylase level of: A. 45 units/L B. 100 units/L C. 300 units/L D. 500 units/L

C. 300 units/L The normal serum amylase level is 25 to 151 units/L. With chronic cases of pancreatitis, the rise in serum amylase levels usually does not exceed three times the normal value. In acute pancreatitis, the value may exceed five times the normal value. Basic lab studies for chronic pancreatitis can include a CBC, BMP, LFTs, lipase, amylase, lipid panel, and a fecal-elastase-1 value. Lipase and amylase levels can be elevated, but they are usually normal secondary to significant pancreatic scarring and fibrosis. Of note, amylase and lipase values should not be considered diagnostic or prognostic. Option A: 45 units/L is within normal limits. Serum amylase and lipase levels may be slightly elevated in chronic pancreatitis; high levels are found only during acute attacks of pancreatitis. In the later stages of chronic pancreatitis, atrophy of the pancreatic parenchyma can result in normal serum enzyme levels because of significant fibrosis of the pancreas, resulting in decreased concentrations of these enzymes within the pancreas. Option B: 100 units/L is within normal limits. When pancreatic tissue damage (eg. pancreatitis) or pancreatic duct is blocked, serum amylase levels increased. In acute pancreatitis, lipase levels are often very high; 10.5 times the normal level can be increased Option D: 500 units/L is an extremely elevated level seen in acute pancreatitis. In acute pancreatitis, blood amylase increased. Sometimes up to 4-6 times the highest normal level rises.

To avoid fecal impaction, psyllium (Metamucil) should be administered with at least how many ounces of fluid? A. 4 B. 6 C. 8 D. 10

C. 8 Bulk-forming laxatives must be given with at least 8 ounces of liquid plus additional liquid each day to prevent intestinal obstruction. Bulk-forming laxatives retain fluid in the stool and increase stool weight and consistency. Psyllium, dietary fiber, carboxymethylcellulose, and methylcellulose are common examples. It is important to take ample amounts of water for bulk-forming agents to work. Lack of water, in turn, leads to bloating and can cause bowel obstruction. Option A: Most laxatives are safe when used appropriately and in patients without contraindications. Bulk-forming agents like lactulose can have adverse effects like bloating, nausea, vomiting, and diarrhea. With prokinetic agents, adverse effects like a headache, nausea, and diarrhea have been described. Option B: Stimulant laxatives are known to cause abdominal pain. Cisapride and tegaserod were withdrawn from the market after cardiovascular adverse effects, including prolonged QT interval that increases the risk for Torsades de Pointes. Mineral oil can cause aspiration and lipoid pneumonia. Option D: Osmotic agents like magnesium can cause metabolic disturbances, especially in the presence of renal involvement. Also, magnesium excretion depends on renal function, and its use requires caution in renal impairment. Osmotic agents result in volume load and should be used with caution in renal or cardiac dysfunction.

The mechanism of action of diphenoxylate (Lotomil) is: A. An increase in intestinal excretion of water. B. An increase in intestinal motility. C. A decrease in peristalsis in the intestinal wall. D. A decrease in the reabsorption of water in the bowel.

C. A decrease in peristalsis in the intestinal wall. Diphenoxylate acts on the smooth muscle of the intestinal tract to inhibit GI motility and excessive propulsion of the GI tract (peristalsis). Diphenoxylate is an active ingredient of commonly available antimotility agents. Its indication is for the treatment of diarrhea in adults and children 13 years or older, or as add-on therapy in the management of acute non-infectious diarrhea. Option A: By acting on the presynaptic opioid receptors, it blocks the release of acetylcholine in the synaptic cleft and hence inhibits the motility and secretory action of the enteric nervous system. This action leads to a decrease in segmental contractions and prolongation of gastrointestinal transit time. Diphenoxylate reduces the epithelial secretion of fluid and electrolytes and enhances active absorption by mild action on delta receptors. Option B: It does not have analgesic effects of morphine at standard doses, but at higher doses, it can lead to CNS effects, like euphoria. The drug can have a misuse potential if used for a prolonged time and classified as Schedule V drug under Food and Drug Administration. Option D: Diphenoxylate can precipitate GI complications, including sepsis and prolonged diarrhea, when administered in patients with infectious diarrhea. This effect is because of the prolongation of GI transit time and decreased GI motility, which leads to bacterial overgrowth and release of enterotoxins into the bloodstream, creating a septic shock-like picture.

Proton pump inhibitors cause: A. Gastric ulcer formation B. GERD C. Achlorhydria D. Diverticulosis

C. Achlorhydria Because the proton pump inhibitors stop the final step of acid secretion, they can block up to 90% of acid secretion, leading to achlorhydria (without acid). Achlorhydria or hypochlorhydria refers to conditions in which production of hydrochloric acid in the stomach is respectively absent or reduced. It is usually secondary to an underlying medical condition. Short-term standard-dose treatment with PPI has been shown to have low risk, but long-term use of PPI has been linked to hypochlorhydria. Option A: The most common causes of peptic ulcers are infection with the bacterium Helicobacter pylori (H. pylori) and long-term use of nonsteroidal anti-inflammatory drugs (NSAIDs) such as ibuprofen (Advil, Motrin IB, others) and naproxen sodium (Aleve). Stress and spicy foods do not cause peptic ulcers. Option B: GERD is caused by frequent acid reflux. When one swallows, a circular band of muscle around the bottom of the esophagus (lower esophageal sphincter) relaxes to allow food and liquid to flow into the stomach. Then the sphincter closes again. If the sphincter relaxes abnormally or weakens, stomach acid can flow back up into the esophagus. This constant backwash of acid irritates the lining of the esophagus, often causing it to become inflamed. Option D: Diverticulosis is thought to occur due to peristalsis abnormalities (e.g., intestinal spasms), intestinal dyskinesia, or high segmental intraluminal pressures. Although the exact cause is unknown, some environmental and lifestyle risk factors have been linked to this condition.

The RN would expect a history of LUQ pain worse after eating and not relieved by vomiting when assessing pt with which problem? A. cholecystitis B. splenomegaly C. acute pancreatitis D. hepatitis

C. Acute pancreatitis

A patient is admitted with lacerated liver as a result of blunt abdominal trauma. Which of the following nursing interventions would not be appropriate for this patient? A. Monitor for respiratory distress. B. Monitor for coagulation studies. C. Administer pain medications as ordered. D. Administer normal saline, crystalloids as ordered.

C. Administer pain medications as ordered. Pain medication may mask signs and symptoms of hemorrhage, further decrease blood pressure, and interfere with assessment of neurologic status and additional abdominal injury. With the potential for hemorrhage, nonsteroidal anti-inflammatory drugs (NSAIDs) probably should be avoided. Acetaminophen with or without small quantities of mild narcotic analgesics may be all that should be prescribed initially. Minimize use of analgesics in patients who are admitted for observation. Option A: The evaluation of any trauma patient begins with evaluating the airway, accessing the breathing, and managing the circulation. The diagnosis of intra-abdominal injury following blunt trauma depends primarily on the hemodynamic status of the patient. Once the airway is protected, it is mandatory to protect the cervical spine. Option B: Because the presentation is often not straightforward, the diagnosis can be difficult and often time-consuming. Besides pain, the patient may present with bleeding per rectum, unstable vital signs, and the presence of peritonitis. The physical exam may reveal marks from a lap belt, ecchymosis, abdominal distention, absent bowel sounds, and tenderness to palpation. Option D: After the primary survey is complete, patients who are hypotensive require aggressive fluid resuscitation. If hemodynamic instability persists, blood should be typed and crossed, but in the meantime, immediate transfusion with O negative blood can be done (O+ for males and women past childbearing years). All patients with blunt abdominal trauma who have signs of peritonitis, frank bleeding, or worsening of clinical signs require an immediate laparotomy.

Medical management of the client with diverticulitis should include which of the following treatments? A. Reduced fluid intake. B. Increased fiber in the diet. C. Administration of antibiotics. D. Exercises to increase intra-abdominal pressure.

C. Administration of antibiotics. Antibiotics are used to reduce inflammation. The most common regimen used in the United States consists of quinolones (ciprofloxacin) or sulfa drugs (trimethoprim/sulfamethoxazole) in combination with metronidazole (or clindamycin, if the patient is intolerant to metronidazole) or single-agent amoxicillin-clavulanate for 7 to 10 days. Option A: The client isn't typically isn't allowed anything orally until the acute episode subsides. Parenteral fluids are given until the client feels better. It is recommended that the client drink eight 8-ounce glasses of water per day. Bowel rest is preferred in patients requiring inpatient admission. Typically, defervescence and improvement in leukocytosis should be observed for two to four days of hospitalization, if not an alternative diagnosis or complications should be suspected. Option B: Gradually increase fiber in the diet to improve intestinal motility. The patient should start a high-fiber diet, drink ample water, maintain a healthy weight, and exercise. Fiber softens and adds bulk to stools, helping them pass more easily through the colon. It also reduces pressure in the digestive tract. Option D: During the acute phase, activities that increase intra-abdominal pressure should be avoided to decrease pain and the chance of intestinal obstruction. An attack of diverticulitis without complications may respond to antibiotics within a few days if treated early. To help the colon rest, the doctor may recommend bed rest and a liquid diet, along with a pain reliever. An acute attack with severe pain or severe infection may require a hospital stay.

A client with ulcerative colitis has an order to begin salicylate medication to reduce inflammation. The nurse instructs the client to take the medication: A. 30 minutes before meals B. On an empty stomach C. After meals D. On arising

C. After meals Salicylate compounds act by inhibiting prostaglandin synthesis and reducing inflammation. The nurse teaches the client to take the medication with a full glass of water and to increase fluid intake throughout the day. This medication needs to be taken after meals to reduce GI irritation. Option A: The medication should not be taken 30 minutes before meals. Aspirin absorption from the gastrointestinal (GI) tract depends on the formulation state. When consumed as a liquid preparation, it is rapidly absorbed as opposed to tablets. Its hydrolysis yields salicylic acid. Salicylic acid has a narrow therapeutic window. If maintained within that narrow range, it provides the appropriate anti-inflammatory effect. Option B: Aspirin, if taken on an empty stomach, may cause GI upset. Aspirins absorption is pH sensitive at the level of the small intestine. Absorption is higher through the small intestine than the stomach for the same pH range. At pH 3.5 or 6.5, aspirin's intestinal absorption is greater than the gastric absorption of the compound. The stomach does not absorb aspirin at pH 6.5. Option D: Taking aspirin upon arising may put the client at risk for GI upset. The most common side effect of aspirin is gastrointestinal upset ranging from gastritis to gastrointestinal bleed. Aspirin increases the risk of GI bleeding in patients who already suffer from peptic ulcer disease or gastritis. The risk of bleeding is still present even without these conditions if there is concomitant consumption of alcohol or if the patient is on warfarin.

When administering sucralfate (Carafate) to a patient with a nasogastric tube, it is important to: A. Crush the tablet into a fine powder before mixing with water. B. Administer with a bolus tube feeding. C. Allow the tablet to dissolve in water before administering. D. Administer with an antacid for maximum benefit.

C. Allow the tablet to dissolve in water before administering. It is important to give sucralfate on an empty stomach so that it may dissolve and form a protective barrier over the gastric mucosa. Sucralfate exhibits its action by forming a protective layer, increasing bicarbonate production, exhibiting anti-peptic effects, promoting tissue growth, regeneration, and repair. Option A: The tablet form will not dissolve in water when crushed; it must be left whole and allowed to dissolve. Crushing the medication so that it will not dissolve could lead to clogging of the nasogastric tube and decreased effectiveness of the drug. Option B: Sucralfate is a basic aluminum salt of sucrose octasulfate. When given orally, it disintegrates in the stomach in the presence of acid and binds to normal and damaged mucosa forming a protective layer. It releases aluminum and binds to positively charged compounds like proteins, peptides, glycoproteins, and glyco lipoproteins, forming an adhesive layer, thereby protecting the mucosa. Option D: It prevents hydrolysis by preventing the formation of the enzyme-substrate complex. It adsorbs to pepsin and decreases its concentration. By forming a polyanion gel, it acts as a physical barrier between luminal contents and mucosa. It increases prostaglandin-dependent and independent production of bicarbonate by stomach and duodenum.

A client is to take one daily dose of ranitidine (Zantac) at home to treat her peptic ulcer. The nurse knows that the client understands proper drug administration of ranitidine when she says that she will take the drug at which of the following times? A. Before meals B. With meals C. At bedtime D. When pain occurs

C. At bedtime Ranitidine blocks the secretion of hydrochloric acid. Clients who take only one daily dose of ranitidine are usually advised to take it at bedtime to inhibit the nocturnal secretion of acid. Clients who take the drug twice a day are advised to take it in the morning and at bedtime. Ranitidine's acid-lowering effect is more pronounced for basal and nocturnal acid secretion than it is for food-stimulated acid secretion. Option A: Ranitidine is available as tablets, capsules, or oral syrup. Ranitidine solution or the dissolved tablet may be mixed with select enteral tube feeding solutions. When dosed orally, ranitidine has a bioavailability of 50%, which is relatively unaffected by food. The peak levels occur 2 to 3 hours post-administration for oral administration and occur 15 minutes after intramuscular administration. Option B: Ranitidine is a competitive inhibitor of histamine H2-receptors. The reversible inhibition of the H2-receptors in gastric parietal cells results in a reduction in both gastric acid volume and concentration. Ranitidine is primarily excreted unchanged in the urine, with a half-life ranging from 2.5 to 3 hours, and because of the renal elimination, the half-life may increase to 4 to 5 hours in patients with kidney dysfunction. Option D: Clinicians urge patients to seek professional medical care and contraindicate the over-the-counter product when they have difficulty or pain when swallowing food or have blood in vomit or stool. Concurrent use is contraindicated with other acid reducers, kidney disease, or in patients younger than 12 years old.

The nurse is preparing a discharge teaching plan for the client who had an umbilical hernia repair. Which of the following would the nurse include in the plan? A. Restricting pain medication. B. Maintaining bedrest. C. Avoiding coughing. D. Irrigating the drain.

C. Avoiding coughing. Coughing is avoided to prevent disruption of the tissue integrity, which can occur because of the location of this surgical procedure. Splint the stomach by placing a pillow over the abdomen with firm pressure before coughing or movement to help reduce the pain. Option A: The client should take analgesics as needed and as prescribed to control pain. Most non-opioid analgesics are classified as non-steroidal anti-inflammatory drugs (NSAIDs). They are used to treat mild pain and inflammation or combined with narcotics. Narcotics or opioids are used for severe pain. Option B: Bedrest is not required following this surgical procedure. The client may slowly increase his activity. He should get up and walk every hour or so to prevent blood clot formation. After recovery, the client may return to work within 2 or 3 days. There should be no lifting anything above 10 lbs, climbing, or any strenuous activities for 4 to 6 weeks. Option D: A drain is not used in this surgical procedure, although the client may be instructed in simple dressing changes. Do not soak in a bathtub until the stitches or staples are removed. A small amount of drainage from the incision is normal.

Which of the following tests can be performed to diagnose a hiatal hernia? A. Colonoscopy B. Lower GI series C. Barium swallow D. Abdominal x-rays

C. Barium swallow A barium swallow with fluoroscopy shows the position of the stomach in relation to the diaphragm. A barium swallow involves drinking a special liquid, then taking X-rays to help see problems in the esophagus (such as swallowing disorders) and the stomach (such as ulcers and tumors). It also shows how big the hiatal hernia is and if there is twisting of the stomach as a result of the hernia. Option A: A colonoscopy shows disorders of the intestine. Colonoscopy is a diagnostic as well as a therapeutic procedure performed to evaluate the large intestine (i.e., colon, rectum, and anus) as well as the distal portion of the small intestine (terminal ileum). It is performed using a hand-held flexible tube-like device called the colonoscope, which has a high definition camera mounted at the tip of the scope, as well as accessory channels that allow insertion of equipment and fluids to cleanse the colonoscope lense and colonic mucosa Option B: A lower GI series shows disorders of the intestine. A lower GI series is a procedure in which a doctor uses X-rays and a chalky liquid called barium to view the large intestine. The barium will make the large intestine more visible on an x-ray. A lower GI series is also called a barium enema. Option D: Abdominal x-ray uses a very small dose of ionizing radiation to produce pictures of the inside of the abdominal cavity. It is used to evaluate the stomach, liver, intestines, and spleen and may be used to help diagnose unexplained pain, nausea, or vomiting.

The nurse has given instructions to the client with an ileostomy about foods to eat to thicken the stool. The nurse determines that the client needs further instructions if the client starts to eat which of the following foods to make the stools less watery? A. Pasta B. Boiled rice C. Bran D. Low-fat cheese

C. Bran Foods that help thicken the stool of the client with an ileostomy include pasta, boiled rice, and low-fat cheese. Bran is high in dietary fiber and thus will increase the output of watery stool by increasing propulsion through the bowel. Ileostomy output is liquid. The addition or elimination of various foods can help thicken or loosen this liquid drainage. Option A: Eat foods that thicken the stool such as rice, pasta, cheese, bananas, applesauce, smooth peanut butter, pretzels, yogurt, and marshmallows. Drink 2 or 3 glasses of fluid that will replace electrolytes like sports drinks, fruit or vegetable juice, and broth but limit these items. Too much sugar drinks can produce diarrhea. Option B: Slowly increase intake of high-fiber foods, such as whole grains, bread, and cereals. As a daily goal, women need 25 grams of fiber; men need 38 grams of fiber. For more fluid, fiber, and vitamin punch, eat at least five servings of fruits and vegetables per day. An easy way to do this is to fill half of the plate with fruit and/or vegetables at every meal Option D: Any bland, low-fat, low-fiber diet is likely to help alleviate diarrhea. By bland, low-fat, low-fiber, we're referring to foods such as white bread, peeled potatoes, peeled and cooked fruit, white pasta, and rice. Another way to approach this is to avoid spicy, fatty, fried, or raw food and any food that's difficult to chew.

A 30-year old client experiences weight loss, abdominal distention, crampy abdominal pain, and intermittent diarrhea after the birth of her 2nd child. Diagnostic tests reveal gluten-induced enteropathy. Which foods must she eliminate from her diet permanently? A. Milk and dairy products B. Protein-containing foods C. Cereal grains (except rice and corn) D. Carbohydrates

C. Cereal grains (except rice and corn) To manage gluten-induced enteropathy, the client must eliminate gluten, which means avoiding all cereal grains except for rice and corn. In initial disease management, clients eat a high-calorie, high-protein diet with mineral and vitamin supplements to help normalize nutritional status. Gluten is a group of proteins found in certain grains, such as wheat, rye, and barley. Option A: Most dairy products are naturally gluten-free. However, those that are flavored and contain additives should always be double-checked for gluten. Some common gluten-containing ingredients that may be added to dairy products include thickeners, malt, and modified food starch. Option B: Many foods contain protein, including animal and plant-based sources. Most are naturally gluten-free. However, gluten-containing ingredients, such as soy sauce, flour, and malt vinegar are often used as fillers or flavorings. They may be added to sauces, rubs, and marinades that are commonly paired with protein sources. Option D: Wheat, rye, and barley are the major foods that need to be avoided while following a gluten-free diet. Gluten is also commonly added to processed foods, such as canned and boxed items.

The nurse is caring for a client with poorly controlled GERD. The nurse is providing education regarding foods that can exacerbate the condition. Which of the following would be an appropriate food for this client to eliminate? A. Whole grain foods B. Purine containing foods like organ meats (liver, kidneys) C. Chocolate D. Gluten containing foods

C. Chocolate Chocolate contributes to GERD symptoms, because it decreases the tone of the esophageal sphincter which worsens the reflux. The other foods that have this effect include coffee, soda, tea, peppermint, and fried or fatty foods. Whole grain foods may help improve symptoms of GERD as they have high amounts of fiber. Purine containing foods like organ meats do not decrease esophageal sphincter tone. Gluten containing foods are not associated with worsening GERD symptoms by decreasing esophageal sphincter tone.

Mandy, an adolescent girl is admitted to an acute care facility with severe malnutrition. After a thorough examination, the physician diagnoses anorexia nervosa. When developing the plan of care for this client, the nurse is most likely to include which nursing diagnosis? A. Hopelessness B. Powerlessness C. Chronic low self-esteem D. Deficient knowledge

C. Chronic low self-esteem Young women with chronic low self-esteem — are at the highest risk for anorexia nervosa because they perceive being thin as a way to improve their self-confidence. Anorexia nervosa is an illness of starvation, brought on by severe disturbance of body image and a morbid fear of obesity. People with anorexia nervosa attempt to maintain a weight that's far below normal for their age and height. Option A: Hopelessness is an inappropriate nursing diagnosis because clients with anorexia nervosa seldom feel hopeless; instead, they use food to control their desire to be thin and hope that restricting food intake will achieve this goal. Feelings of personal ineffectiveness, low self-esteem, and perfectionism are often part of the problem. Option B: Major physical and psychological changes in adolescence can contribute to the development of eating disorders. Feelings of powerlessness and loss of control of feelings (in particular sexual sensations) lead to an unconscious desire to desexualize self. The patient often believes that these fears can be overcome by taking control of bodily appearance, development, and function. Option D: Anorexia nervosa doesn't result from a knowledge deficit, such as one regarding good nutrition. The patient sees herself as weak-willed, even though part of a person may feel a sense of power and control (dieting, weight loss). The patient feels helpless to change and requires assistance to problem-solve methods of control in life situations.

Nurse Joy is preparing to administer medication through a nasogastric tube that is connected to suction. To administer the medication, the nurse would: A. Position the client supine to assist in medication absorption B. Aspirate the nasogastric tube after medication administration to maintain patency C. Clamp the nasogastric tube for 30 minutes following administration of the medication D. Change the suction setting to low intermittent suction for 30 minutes after medication administration

C. Clamp the nasogastric tube for 30 minutes following administration of the medication. If a client has a nasogastric tube connected to suction, the nurse should wait up to 30 minutes before reconnecting the tube to the suction apparatus to allow adequate time for medication absorption. Flush 10 ml of water between medications. This step prevents interactions between medications. After the last medication has been given, flush the tube with 30 ml of water. Flushing prevents blocking of the tube. Option A: The client should not be placed in the supine position because of the risk of aspiration. Help the patient to a high sitting position unless contraindicated. This position reduces the risk of aspiration during swallowing. Pour medication and water solution into the 60 ml syringe, release pinch, and allow fluid to drain slowly by gravity into the gastric tube. Option B: Aspirating the nasogastric tube will remove the medication just administered. Patients with a gastric tube (nasogastric, nasointestinal, percutaneous endoscopic gastrostomy (PEG), or jejunostomy (J) tube) will often receive medication through this tube (Lynn, 2011). Liquid medications should always be used when possible because absorption is better and less likely to cause blockage of the tube. Option D: Low intermittent suction also will remove the medication just administered. Determine if medication should be given with or without food. If the medication is to be given on an empty stomach, the enteral feeding may need to be stopped from 30 minutes before until 30 minutes after the medication is given. Follow specific medication guidelines to ensure adequate absorption and distribution of the medication.

A patient has a severe exacerbation of ulcerative colitis. Long-term medications will probably include: A. Antacids B. Antibiotics C. Corticosteroids D. Histamine2-receptor blockers

C. Corticosteroids Medications to control inflammation such as corticosteroids are used for long-term treatment. First-line treatment is sulfasalazine and 5-aminosalicylates, given orally or rectally, which have a remission rate of about 50%. Glucocorticoids, orally or rectally, can be added for those who fail to achieve remission within two weeks. Except for glucocorticoids, all of these medications can be used in the maintenance of remission. Option A: Antacids are a group of drugs that have been on the market for many years. They were initially first-line defense against peptic ulcer disease; however, the discovery of proton pump inhibitors revolutionized the treatment of peptic ulcer disease. Currently, antacid use is restricted to the relief of mild intermittent gastroesophageal reflux disease (GERD) associated with heartburn. Option B: Antibiotic therapy is often initiated before an exact infectious disease diagnosis is made and microbiological results are available. Antibiotics used in this manner are referred to as empiric therapy. This approach attempts to cover all potential pathogens. When microbiology tests result and antibiotic susceptibilities are known, definitive antibiotic therapy can then be tailored to the specific infection etiology. Option D: H2 receptor blockers, or H2 receptor antagonists (H2RAs), are a class of gastric acid-suppressing agents frequently used in various gastric conditions. They are FDA-approved for short-term use in treating uncomplicated gastroesophageal reflux disease (GERD), gastric or duodenal ulcers, gastric hypersecretion, and mild to infrequent heartburn or indigestion.

A male client who is recovering from surgery has been advanced from a clear liquid diet to a full liquid diet. The client is looking forward to the diet change because he has been "bored" with the clear liquid diet. The nurse would offer which full liquid item to the client? A. Tea B. Gelatin C.Custard D.Popsicle

C. Custard Full liquid food items include items such as plain ice cream, sherbet, breakfast drinks, milk, pudding, and custard, soups that are strained, and strained vegetable juices. A clear liquid diet consists of foods that are relatively transparent. A patient prescribed a full liquid diet follows a specific diet type requiring all liquids and semi-liquids but no forms of solid intake. Option A: Tea is included in the clear liquid diet. Unlike a clear liquid diet, which includes only liquids and semi-liquids that are non-opaque, a full liquid diet is more inclusive, as it allows all types of liquids. Option B: A clear liquid diet is a specific dietary plan that only includes liquids that are fully transparent at room temperature. Some items that may be allowed include water, ice, fruit juices without pulp, sports drinks, carbonated drinks, gelatin, tea, coffee, clear broths, and clear ice pops. Option D: A popsicle is included in the clear liquid diet. The clear liquid diet assists in maintaining hydration, it provides electrolytes and calories, and offers some level of satiety when a full diet is not appropriate, but may struggle to provide adequate caloric needs if employed for more than five days.

In a client with Crohn's disease, which of the following symptoms should not be a direct result of antibiotic therapy? A. Decrease in bleeding. B. Decrease in temperature. C. Decrease in body weight. D. Decrease in the number of stools.

C. Decrease in body weight A decrease in body weight may occur during therapy due to inadequate dietary intake, but isn't related to antibiotic therapy. Effective antibiotic therapy will be noted by a decrease in temperature, number of stools, and bleeding. For people with Crohn's, antibiotics may help lower the amount and change the composition of bacteria in the intestines, which may relieve symptoms. Option A: Antibiotics also work to control infections. They may aid in healing abscesses and fistulas. Abscesses are small pockets of infection, and they can contain fluid, dead tissue, and bacteria. Fistulas are unusual connections between your intestines and other body parts, or between two loops of your intestines. Abscesses and fistulas occur when your bowels are inflamed or injured. Option B: Metronidazole is used alone or in combination with ciprofloxacin, metronidazole (Flagyl) is commonly used to treat complications such as abscesses and fistulas. It may also help reduce disease activity and prevent a recurrence. Option D: Rifaximin (Xifaxan) has been used for years to treat diarrhea. However, it has recently emerged as a promising treatment for Crohn's. Possible side effects may include skin rash or hives; bloody urine or diarrhea, and fever.

Five days after undergoing surgery, a client develops a small bowel obstruction. A Miller-Abbott tube is inserted for bowel decompression. Which nursing diagnosis takes priority? A. Imbalanced nutrition: Less than body requirements B. Acute pain C. Deficient fluid volume D. Excess fluid volume

C. Deficient fluid volume Fluid shifts to the site of the bowel obstruction, causing a fluid deficit in the intravascular spaces. Monitor I&O. Note number, character, and amount of stools; estimate insensible fluid losses (diaphoresis). Measure urine specific gravity; observe for oliguria. This provides information about overall fluid balance, renal function, and bowel disease control, as well as guidelines for fluid replacement. Option A: If the obstruction isn't resolved immediately, the client may experience an imbalanced nutritional status (less than body requirements); however, deficient fluid volume takes priority. Avoid or limit foods that might cause or exacerbate abdominal cramping, flatulence (milk products, foods high in fiber or fat, alcohol, caffeinated beverages, chocolate, peppermint, tomatoes, orange juice). Individual tolerance varies, depending on the stage of disease and area of bowel affected. Option B: The client may also experience pain, but that nursing diagnosis is also of lower priority than deficient fluid volume. Encourage the patient to assume a position of comfort (knees flexed). This reduces abdominal tension and promotes a sense of control. Option D: Note possible conditions or processes that may lead to deficits such as fluid loss, limited intake, fluid shifts, environmental factors to assess causative and precipitating factors. Fluid loss may be an effect of diarrhea or vomiting).

The student nurse is preparing a teaching care plan to help improve nutrition in a patient with achalasia. You include which of the following: A. Swallow foods while leaning forward. B. Omit fluids at mealtimes. C. Eat meals sitting upright. D. Avoid soft and semi soft foods.

C. Eat meals sitting upright. Eating in the upright position aids in emptying the esophagus. Doing the opposite of the other three also may be helpful. Achalasia is an esophageal smooth muscle motility disorder that occurs due to a failure of relaxation of the lower esophageal sphincter. This condition causes a functional obstruction at the gastroesophageal junction. Option A: The majority of patients with achalasia typically present with dysphagia, initially with solids than to liquids though 70-97% of patients will have dysphagia to both solids and liquids at presentation. Dysphagia and regurgitation are the most common presenting symptoms in achalasia. Option B: Instruct patient regarding eating small amounts of bland food followed by a small amount of water. Instruct to remain in an upright position at least 1-2 hours after meals, and to avoid eating within 2-4 hours of bedtime. Gravity helps control reflux and causes less irritation from reflux action into the esophagus. Option D: Patients need to understand necessary lifestyle changes following myotomy, such as the need to eat small food boluses in an upright position, which allows gravity to assist with food transit and never to lay flat but rather at 30 to 45 degrees due to increased risk for aspiration.

Which of the following factors can cause hepatitis A? A. Contact with infected blood. B. Blood transfusions with infected blood. C. Eating contaminated shellfish. D. Sexual contact with an infected person.

C. Eating contaminated shellfish. Hepatitis A can be caused by consuming contaminated water, milk, or food — especially shellfish from contaminated water. The most common mode of transmission of hepatitis A is via the fecal-oral route from contact with food, water, or objects contaminated by fecal matter from an infected individual. It is more commonly encountered in developing countries where due to poverty and lack of sanitation, there is a higher chance of fecal-oral spread. Option A: Hepatitis B is caused by blood and sexual contact with an infected person. Transfusion of blood and blood products, injection drug use with shared needles, needlesticks, or wounds caused by other instruments in healthcare workers and hemodialysis are all examples of parenteral and percutaneous exposures, but parenteral mode remains the dominant mode of transmission both globally and in the United States. Option B: Hepatitis C is usually caused by contact with infected blood, including receiving blood transfusions. Transmission can be parenteral, perinatal, and sexual, with the most common mode being the sharing of contaminated needles among IV drug users. Also, other high-risk groups include people who require frequent blood transfusions and organ transplantation of organs from infected donors. Option D: Hepatitis B and C can be caused by sexual contact with an infected person. Intravenous drug users, men who have sex with men, healthcare workers with exposure to infected body fluids, patients who require frequent and multiple blood transfusions, people who have multiple sexual partners, prisoners, partners of hepatitis B virus carriers, and persons born in endemic areas are all at high risk for hepatitis B virus infection.

Which of the following tests can be useful as a diagnostic and therapeutic tool in the biliary system? A. Ultrasonography B. MRI C. Endoscopic retrograde cholangiopancreatography (ERCP) D. Computed tomography scan (CT scan)

C. Endoscopic retrograde cholangiopancreatography (ERCP) ERCP permits direct visualization of the pancreatic and common bile ducts. Its therapeutic value is in retrieving gallstones from the distal and common bile ducts and dilating strictures. Endoscopic retrograde cholangiopancreatography (ERCP) is a combined endoscopic and fluoroscopic procedure in which an endoscope is advanced into the second part of the duodenum, thus allowing other tools to be passed into the biliary and pancreatic ducts via the major duodenal papilla. Option A: Ultrasonography aids in the diagnosis of cholecystitis, gallstones, pancreatitis, and metastatic disease. It also identifies edema, inflammation, and fatty or fibrotic infiltrates or calcifications. A procedure that uses high-energy sound waves to look at tissues and organs inside the body. The sound waves make echoes that form pictures of the tissues and organs on a computer screen (sonogram). Ultrasonography may be used to help diagnose diseases, such as cancer. Option B: MRI detects hepatic neoplasms, cysts, abscesses, and hematomas. Magnetic resonance imaging ( MRI ) uses a large magnet and radio waves to look at organs and structures inside the body. Health care professionals use MRI scans to diagnose a variety of conditions, from torn ligaments to tumors. MRIs are very useful for examining the brain and spinal cord. Option D: A CT Scan can be done without a contrast medium. It can detect tumors, cysts, pseudocysts, abscesses, hematomas, and obstructions of the liver, biliary tract and pancreas. The CT scan is essentially an X-ray study, where a series of rays are rotated around a specified body part, and computer-generated cross-sectional images are produced. The advantage of these tomographic images compared to conventional X-rays is that they contain detailed information of a specified area in cross-section, eliminating the superimposition of images, which provides a tremendous advantage over plain films.

Which of the following symptoms is common with a hiatal hernia? A. Left arm pain B. Lower back pain C. Esophageal reflux D. Abdominal cramping

C. Esophageal reflux Esophageal reflux is a common symptom of a hiatal hernia. This seems to be associated with chronic exposure of the lower esophageal sphincter to the lower pressure of the thorax, making it less effective. In a hiatal hernia, the stomach pushes through that opening and into the chest and compromises the lower esophageal sphincter (LES). This laxity of the LES can allow gastric content and acid to back up into the esophagus and is the leading cause of gastroesophageal reflux disease (GERD). Option A: There is no left arm pain in hiatal hernia. The typical presentation leading to an evaluation for a hiatal hernia is gastroesophageal reflux disease (GERD). Patients typically complain of heartburn and sometimes regurgitation. While heartburn is the most common complaint, some patients will present with extra-esophageal symptoms such as a chronic cough or asthma. Option B: Lower back pain is not associated with hiatal hernia. The presentation of regurgitation or extra-esophageal symptoms typically is a sign of disease progression. However, not all patients with regurgitation have GERD, and it is important to note if the regurgitated food is digested or undigested. Option D: Abdominal cramping is not a symptom of hiatal hernia. Physical examination in patients with a hiatal hernia and GERD rarely helps confirm the diagnosis. The presence of abnormal supraclavicular lymph nodes in patients with heartburn and dysphasia may suggest esophageal or gastric cancer and is an important part of the evaluation.

Polyethylene glycol-electrolyte solution (GoLYTELY) is prescribed for the female client scheduled for a colonoscopy. The client begins to experience diarrhea following the administration of the solution. What action by the nurse is appropriate? A. Start an IV infusion B. Administer an enema C. Cancel the diagnostic test D. Explain that diarrhea is expected

C. Explain that diarrhea is expected. The solution GoLYTELY is a bowel evacuant used to prepare a client for a colonoscopy by cleansing the bowel. The solution is expected to cause mild diarrhea and will clear the bowel in 4 to 5 hours. Polyethylene glycol electrolyte (PEG) is essential for a wide range of bowel preparation, with advantages such as high security, reliable effect, no dehydration, and electrolyte disturbance. Option A: Starting an IV is unnecessary. Surveys, such as those conducted by Seo et al., have shown that colon cleanliness was the highest at time intervals of 3-5 h after a one-time oral administration of 4 L of PEG solution, whereas colon cleanliness was significantly decreased at time intervals of <3 or >7 h. Option B: Administering an enema would be inappropriate. Bacteria in intestinal feces account for 20%-30% of the solid weight of feces. This also accords with earlier observations that the PEG solution can only effectively remove solid residues in feces and has no significant effect on colonic bacteria and flora. Option C: Cancelling the test would be inappropriate. PEG solution combined with lactulose improves the quality of bowel preparation in patients with long interval P-C, which allows patients to have no restriction on the time of colonoscopy, and benefits more patients who need a colonoscopy.

A client with gastric cancer may exhibit which of the following symptoms? A. Abdominal cramping B. Constant hunger C. Feeling of fullness D. Weight gain

C. Feeling of fullness The client with gastric cancer may report a feeling of fullness in the stomach, but not enough to cause him to seek medical attention. The most common physical examination finding is a palpable abdominal mass indicating advanced disease. The patient may also present with signs of metastatic lymphatic spread distribution, including Virchow's node (left supraclavicular adenopathy), Sister Mary Joseph node (periumbilical nodule), and Irish node (left axillary node). Option A: Abdominal cramping isn't associated with gastric cancer. Results from several studies have demonstrated an approximately sixfold increase in H. pylori infection in patients with gastric cancer, particularly adenocarcinoma of the distal stomach, including both intestinal and diffuse types. As mentioned above, H. pylori trigger inflammation, resulting in stomach atrophy and subsequent metaplasia culminating in carcinoma. Option B: Anorexia (not increased hunger) is a common symptom of gastric cancer. In the United States, most patients have symptoms of an advanced stage at the time of presentation. The most common presenting symptoms for gastric cancers are non-specific weight loss, persistent abdominal pain, dysphagia, hematemesis, anorexia, nausea, early satiety, and dyspepsia. Option D: Patients presenting with locally advanced or metastatic disease usually present with significant abdominal pain, potential ascites, weight loss, fatigue, and have visceral metastasis on scans, and can have a gastric-outlet obstruction.

Sucralfate (Carafate) achieves a therapeutic effect by: A. Neutralizing gastric acid. B. Enhancing gastric absorption. C. Forming a protective barrier around gastric mucosa. D. Inhibiting gastric acid secretion.

C. Forming a protective barrier around gastric mucosa. Sucralfate has a local effect only on the gastric mucosa. It forms a paste-like substance in the stomach, which adheres to the gastric lining, protecting against adverse effects related to gastric acid. It also stimulates healing of any ulcerated areas of the gastric mucosa. Option A: The antacids reduce the acid reaching the duodenum by neutralizing the acid present in the stomach. The salts' mechanism of neutralization of acid varies, and each salt has a different mechanism with the ultimate goal of acid neutralization. Option B: Drugs that affect gastric emptying (eg, parasympatholytic drugs) affect the absorption rate of other drugs. Food may enhance the extent of absorption for poorly soluble drugs (eg, griseofulvin), reduce it for drugs degraded in the stomach (eg, penicillin G), or have little or no effect. Option D: H2RAs decrease gastric acid secretion by reversibly binding to histamine H2 receptors located on gastric parietal cells, thereby inhibiting the binding and action of the endogenous ligand histamine. H2 blockers thus function as competitive antagonists.

The nurse is caring for a client who is taking esomeprazole. The nurse knows that this medication is used to treat which of the following? A. Candidiasis B. Gastroparesis C. Gastroesophageal reflux disorder (GERD) D. Irritable bowel syndrome (IBS)

C. Gastroesophageal reflux disorder (GERD) Esomeprazole (Nexium) Is a proton pump inhibitor used to treat gastroesophageal reflux disorder (GERD) and heartburn. Candidiasis Is a yeast infection and an antifungal such as fluconazole would be used to treat. gastroparesis may be treated with gut motility stimulator such as metoclopramide. IBS is treated primarily through diet modification Esomeprazole is not used for this condition.

Which of the following diagnostic tests may be performed to determine if a client has gastric cancer? A. Barium enema B. Colonoscopy C. Gastroscopy D. Serum chemistry levels

C. Gastroscopy A gastroscopy will allow direct visualization of the tumor. Patients presenting with any symptoms suspicious for gastric cancer should undergo an upper endoscopy over barium study (except for limited plastic presenting as leather-flask appearance). Although upper endoscopy is more invasive and costly, it offers tissue diagnosis by direct biopsy of esophageal, gastric, or duodenal lesions. Option A: A barium enema is a radiographic (X-ray) examination of the lower gastrointestinal (GI) tract. The large intestine, including the rectum, is made visible on X-ray film by filling the colon with a liquid suspension called barium sulfate (barium). Barium highlights certain areas in the body to create a clearer picture. Option B: A colonoscopy would help diagnose colon cancer. Synchronous or metachronous colorectal cancer is reportedly detected in approximately 1% of patients with gastric cancer. Therefore, screening colonoscopy before surgical interventions for the stomach is now well established. Option D: Serum chemistry levels don't contribute data useful to the assessment of gastric cancer. Staging pre-preoperative evaluations include chest and abdominal imaging to rule out metastasis and to determine surgical resectability. Abdominopelvic computerized tomography is performed early to rule out gross metastatic disease but does not accurately assess T, N, and small peritoneal metastases with an overall accuracy of 42% to 82%.

A nurse is planning medication administration for a client who has all of the following oral medications due at 0900: Calcium carbonate, Codeine, Levetiracetam, Metoclopramide. What is the most appropriate action by the nurse? A. Give Metoclopramide 1 hour before the others B. Give Levetiracetam 30 minutes after the others C. Give Calcium carbonate 1 hour after the others D. Give Codeine 30 minutes before the others

C. Give Calcium carbonate 1 hour after the others Calcium carbonate is an antacid, which should always be given one hour after other oral medications, otherwise it may impair absorption of those medications. In this case, it would be appropriate to give the other medications at 0830 and the calcium carbonate at 0930. Metoclopramide Is a prokinetic agent. It will improve GI motility, but will not impair absorption of other medications. It is safe to administer concurrently with other medications Levetiracetam is an antileptic medication. It is not necessary to separate administration for using this drug. Codeine is an opioid analgesic. There is no need to separate administration of this oral medication from the others. It would be wise, however, to ensure the client is alert enough after taking codeine to be able to take any other medications.

A client with advanced cirrhosis has been diagnosed with hepatic encephalopathy. The nurse expects to assess for: A. Malaise B. Stomatitis C. Hand tremors D. Weight loss

C. Hand tremors Hepatic encephalopathy results from the accumulation of neurotoxins in the blood, therefore the nurse wants to assess for signs of neurological involvement. Flapping of the hands (asterixis), changes in mentation, agitation, and confusion are common. During the intermediate stages of HE, a characteristic jerking movement of the limbs is often observed (e.g., asterixis) when the patient attempts to hold arms outstretched with hands bent upward at the wrist. Option A: Malaise is not related to neurological involvement. Other physical signs may include hyperreflexia, a positive Babinski's sign, or Parkinsonian symptoms (e.g., rigidity or tremors). Symptoms typically include confusion, personality changes, disorientation, and a depressed level of consciousness. The earliest stage is often characterized by an inverted sleep-wake pattern wherein patients are found to be sleeping during the day and awake throughout the night. Option B: Stomatitis is not related to neurological involvement. Throughout the intermediate stages, patients tend to experience worsening levels of confusion, lethargy, and personality changes. In the advanced stages, hepatic encephalopathy may eventually lead to coma (e.g., hepatic coma or coma hepaticum) and ultimately to death. Option D: These clients typically have ascites and edema so experience weight gain. In order to make a diagnosis of HE, there must be confirmed the presence of liver disease (e.g., abnormal liver function tests, ultrasound or liver biopsy demonstrating liver disease) or a portosystemic shunt, and exclusion of other potential etiologies (e.g., intracranial lesions, masses, hemorrhage or stroke; seizure activity; post-seizure encephalopathy; intracranial infections; or toxic encephalopathy from other causes).

Which of the following factors is believed to be linked to Crohn's disease? A. Constipation B. Diet C. Hereditary D. Lack of exercise

C. Hereditary Although the definite cause of Crohn's disease is unknown, it's thought to be associated with infectious, immune, or psychological factors. Because it has a higher incidence in siblings, it may have a genetic cause. More than a hundred genes associated with IBD have been identified. In Crohn's disease particularly, there appears to be a genetic association with phenotypes. Specifically, NOD2/CARD15 mutations were found to be associated with a phenotype of Crohn's disease which was associated in those diagnosed at a younger age Option A: Constipation does not cause Crohn's disease. Although the exact etiology of inflammatory bowel disease (IBD) is not known, there is substantial evidence to suggest that the disease is resulting from an inappropriate immune response in the bowel to situations from environmental factors such as drugs, toxins, infections or intestinal microbes in a genetically susceptible host. Option B: Diet is not a cause of Crohn's disease. In the future, genotyping could potentially provide prognostic information on the severity of the disease. Furthermore, it could predict which patients should be considered for surgical management vs. medical management based on a more detailed understanding of genetic analysis. Option D: Lack of exercise does not cause Crohn's disease. This condition results from a combination of genetic, environmental, and lifestyle factors, many of which are unknown. Many of the major genes related to Crohn's disease, including NOD2, ATG16L1, IL23R, and IRGM, are involved in immune system function.

Eleanor, a 62 y.o. woman with diverticulosis is your patient. Which interventions would you expect to include in her care? A. Low-fiber diet and fluid restrictions. B. Total parenteral nutrition and bed rest. C. High-fiber diet and administration of psyllium. D. Administration of analgesics and antacids.

C. High-fiber diet and administration of psyllium. She needs a high-fiber diet and a psyllium (bulk laxative) to promote normal soft stools. Diverticulosis is a clinical condition in which multiple sac-like protrusions (diverticula) develop along the gastrointestinal tract. Though diverticula may form at weak points in the walls of either the small or large intestines, the majority occur in the large intestine (most commonly the sigmoid colon). Option A: Most clinicians recommend increasing fiber content in the diet, which can help increase the bulk of the stools. Fiber also helps to control the recurrence of symptoms. Patients with diverticular disease are also advised to avoid seeds, corn, and nuts because of the belief that these foods could cause diverticulitis. Option B: Greater intestinal bulk reduces the number of spasms, and as a result, decreases intestinal pressures. Studies found no positive or negative association between diverticular disease and consumption of nuts, grains, potassium, ?-carotene, vitamin C, and magnesium. Option D: Treatment is typically aimed at reducing intestinal spasms, which is potentially achievable by increasing fiber and fluids in the diet. Most bleeding associated with diverticulosis is self-limiting and does not require intervention. In some cases, however, endoscopic, radiologic, or surgical intervention may be required to stop persistent bleeding ((e.g., injection, coagulation (cautery, argon plasma coagulator), or mechanical devices (clips, bands, loops)).

Kevin has a history of peptic ulcer disease and vomits coffee-ground emesis. What does this indicate? A. He has fresh, active upper GI bleeding. B. He needs immediate saline gastric lavage. C. His gastric bleeding occurred 2 hours earlier. D. He needs a transfusion of packed RBCs.

C. His gastric bleeding occurred 2 hours earlier. Coffee-ground emesis occurs when there is upper GI bleeding that has undergone gastric digestion. For blood to appear as coffee-ground emesis, it would have to be digested for approximately 2 hours. The term "coffee-grounds" describes gastric aspirate or vomitus that contains dark specks of old blood. UGIB is classified as any blood loss from a gastrointestinal source above the ligament of Treitz. Option A: The clinical presentation can vary but should be well-characterized. Hematemesis is the overt bleeding with vomiting of fresh blood or clots. Patients may also present with syncope or orthostatic hypotension if bleeding is severe enough to cause hemodynamic instability. Option B: Patients must have a minimum of two large-bore peripheral access catheters (at least 18-gauge). Intravenous fluids should be administered to maintain adequate blood pressure and hemodynamic stability. If patients are not able to protect their airways or have ongoing severe hematemesis, elective endotracheal intubation is advised. Option D: Blood transfusions should be given to target a hematocrit above 20%, with a hematocrit above 30% targeted in high-risk patients, such as the elderly and patients with coronary artery disease. There is no evidence that higher targets for hematocrit goals should be sought as that higher targets can even be deleterious.

A client's ulcerative colitis symptoms have been present for longer than 1 week. The nurse recognizes that the client should be assessed carefully for signs of which of the following complications? A. Heart failure B. DVT C. Hypokalemia D. Hypocalcemia

C. Hypokalemia Excessive diarrhea causes significant depletion of the body's stores of sodium and potassium as well as fluid. The client should be closely monitored for hypokalemia and hyponatremia. Intestinal inflammatory processes reduce the absorption of sodium, chloride and calcium, while they increase potassium secretion. In addition, mild to severe metabolic alkalosis may occur in IBD patients, mainly depending on the severity of the disease and the part of the gastrointestinal tract being affected. Ulcerative colitis may place the client at risk for heart failure, DVT, or hypocalcemia, but this can rarely occur. Option A: High levels of cytokines, C-reactive protein (CRP), and homocysteine in IBD patients may lead to endothelial dysfunction, an early sign of atherosclerosis. IBD patients, in general, do not show the typical risk factors for cardiovascular disease but changes in lipid profiles similar to the ones seen in cardiovascular events have been reported recently. Option B: Patients with inflammatory bowel disease are at increased risk of thromboembolic complications which may affect patients' morbidity and mortality. Many factors have been investigated as determinants of the pro-thrombotic tendency such as acquired risk factors or genetic and immune abnormalities, but a unique cause has not been found. Option D: An inability to absorb enough calcium is one reason why people with IBD may experience calcium deficiency. Malabsorption occurs mainly in people with Crohn's disease where the inflammation is located in the small intestine. Calcium is absorbed in the small intestine, so people who have ulcerative colitis, which mainly has inflammation in the large intestine, may have better calcium absorption.

A client who has ulcerative colitis has persistent diarrhea. He is thin and has lost 12 pounds since the exacerbation of his ulcerative colitis. The nurse should anticipate that the physician will order which of the following treatment approaches to help the client meet his nutritional needs? A. Initiate continuous enteral feedings. B. Encourage a high protein, high-calorie diet. C. Implement total parenteral nutrition. D. Provide six small meals a day.

C. Implement total parenteral nutrition. Food will be withheld from the client with severe symptoms of ulcerative colitis to rest the bowel. To maintain the client's nutritional status, the client will be started on TPN. Dietary measures depend on the patient's condition (if disease is mild, the patient may do well on a low-residue, low-fat diet high in protein and calories with lactose restriction). In moderate disease, elemental enteral products may be given to provide nutrition without overstimulating the bowel. Patient with toxic colitis is NPO and placed on parenteral nutrition. Option A: Enteral feedings do not allow the bowel to rest. Recommend rest before meals. This quiets peristalsis and increases available energy for eating. Encourage bed rest and limited activity during the acute phase of illness. Decreasing metabolic needs aids in preventing caloric depletion and conserves energy. Option B: A high-calorie, high-protein diet will worsen the client's symptoms. Avoid or limit foods that might cause or exacerbate abdominal cramping, flatulence (milk products, foods high in fiber or fat, alcohol, caffeinated beverages, chocolate, peppermint, tomatoes, orange juice). Individual tolerance varies, depending on the stage of disease and area of bowel affected. Option D: Dividing the diet into 6 small meals does not allow the bowel to rest. Keep patient NPO as indicated. Resting the bowel decreases peristalsis and diarrhea, limiting malabsorption and loss of nutrients.

The nurse is reviewing the medication record of a female client with acute gastritis. Which medication, if noted on the client's record, would the nurse question? A. Digoxin (Lanoxin) B. Furosemide (Lasix) C. Indomethacin (Indocin) D. Propranolol hydrochloride (Inderal)

C. Indomethacin (Indocin) Indomethacin (Indocin) is a nonsteroidal anti-inflammatory drug and can cause ulceration of the esophagus, stomach, or small intestine. Indomethacin is contraindicated in a client with gastrointestinal disorders. Nonsteroidal anti-inflammatory drugs (NSAIDs) such as indomethacin are capable of producing injury to gastrointestinal mucosa in experimental animals and humans, and their use is associated with a significant risk of hemorrhage, erosions, and perforation of both gastric and intestinal ulcers. Option A: Digoxin is a cardiac medication. The molecular basis for the gastrointestinal toxicity of NSAIDs is widely believed to be their inhibitory activity against cyclooxygenase, which causes them to block the production of prostaglandins and their therapeutic actions. Option B: Furosemide (Lasix) is a loop diuretic. Furosemide is not contraindicated in clients with gastric disorders. Suppression of prostaglandin synthesis in NSAIDs is associated with reduction of gastric mucosal blood flow, disturbance of microcirculation, decrease in mucus secretion, lipid peroxidation, and neutrophil activation, which are involved in the pathogenesis of gastrointestinal mucosal disorders Option D: Propranolol (Inderal) is a ?-adrenergic blocker. While the presence of acid in the lumen of the stomach may not be a primary factor in the pathogenesis of NSAID-induced gastropathy, it can make an important contribution to the severity of these lesions by impairing the restitution process, interfering with hemostasis, and inactivating several growth factors that are important in mucosal defense and repair.

Which of the following definitions best describes gastritis? A. Erosion of the gastric mucosa. B. Inflammation of a diverticulum. C. Inflammation of the gastric mucosa. D. Reflux of stomach acid into the esophagus.

C. Inflammation of the gastric mucosa. Gastritis is an inflammation of the gastric mucosa that may be acute (often resulting from exposure to local irritants) or chronic (associated with autoimmune infections or atrophic disorders of the stomach). The current classification of gastritis centers on time course (acute versus chronic), histological features, anatomic distribution, and underlying pathological mechanisms. Acute gastritis will evolve to chronic, if not treated. Option A: Erosion of the mucosa results in ulceration. With peptic ulcers, there is usually a defect in the mucosa that extends to the muscularis mucosa. Once the protective superficial mucosal layer is damaged, the inner layers are susceptible to acidity. Further, the ability of the mucosal cells to secrete bicarbonate is compromised. Option B: Inflammation of a diverticulum is called diverticulitis. Acute diverticulitis is inflammation due to micro-perforation of a diverticulum. The diverticulum is a sac-like protrusion of the colon wall. Diverticulitis can present in about 10% to 25% of patients with diverticulosis. Diverticulitis can be simple or uncomplicated and complicated. Option D: Reflux of stomach acid is known as gastroesophageal disease. Gastroesophageal reflux disease (GERD) is a chronic gastrointestinal disorder characterized by the regurgitation of gastric contents into the esophagus. It is one of the most commonly diagnosed digestive disorders in the US with a prevalence of 20%, resulting in a significant economic burden in direct and indirect costs and adversely affects the quality of life.

When a client has peptic ulcer disease, the nurse would expect a priority intervention to be: A. Assisting in inserting a Miller-Abbott tube. B. Assisting in inserting an arterial pressure line. C. Inserting a nasogastric tube. D. Inserting an I.V.

C. Inserting a nasogastric tube. An NG tube insertion is the most appropriate intervention because it will determine the presence of active GI bleeding. Monitor the client's fluid intake and urine output. Assess for the signs of hematemesis or melena. The client with a bleeding ulcer may vomit bright red blood or coffee grounds emesis. Melena occurs when there is bleeding in the upper GI tract. Option A: A Miller-Abbott tube is a weighted, mercury-filled balloon tube used to resolve bowel obstructions. The modifications of lifestyle behaviors such as alcohol use, coffee, and other caffeinated beverages, and the overuse of aspirin or other nonsteroidal anti-inflammatory drugs is necessary to prevent recurrent ulcer development and prevent complications during the healing phase. Option B: There is no evidence of shock or fluid overload in the client; therefore, an arterial line is not appropriate at this time. Monitor the client's vital signs, and observe BP and HR for signs of orthostatic changes. Option D: An IV is optional. Administer IV fluids, volume expanders, and blood products as ordered. Isotonic fluids, volume expanders, and blood products can restore or expand intravascular volume.

The nurse is caring for a female client following a Billroth II procedure. Which postoperative order should the nurse question and verify? A. Leg exercises B. Early ambulation C. Irrigating the nasogastric tube D. Coughing and deep-breathing exercises

C. Irrigating the nasogastric tube. In a Billroth II procedure, the proximal remnant of the stomach is anastomosed to the proximal jejunum. Patency of the nasogastric tube is critical for preventing the retention of gastric secretions. The nurse should never irrigate or reposition the gastric tube after gastric surgery unless specifically ordered by the physician. In this situation, the nurse should clarify the order. Option A: Leg exercises are allowed to prevent thrombosis. Discuss the importance of eating small, frequent meals slowly and in a relaxed atmosphere; resting after meals; avoiding extremely hot or cold food; restricting high-fiber foods, caffeine, milk products, and alcohol, excess sugars and salt; and taking fluids between meals, rather than with food. Option B: Early ambulation is recommended after the procedure. Also, discuss the reasons for and importance of cessation of smoking. Smoking stimulates gastric acid production and may cause vasoconstriction, compromising mucous membranes and increasing the risk of gastric irritation and ulceration. Option D: The client may do coughing and deep breathing exercises. Discuss and identify stressful situations and how to avoid them. Investigate job-related issues. This can alter gastric motility, interfering with optimal digestion. The patient may require vocational counseling if a change in employment is indicated.

A client is scheduled for a gastric emptying study. Which of the following best describes a gastric emptying study? A. It involves drinking barium and watching the stomach empty B. It involves a contrast medium taken orally to assess stomach emptying C. It is a radionuclide study that scans the stomach emptying D. It involves a small camera at the end of a flexible tube

C. It is a radionuclide study that scans the stomach emptying This type of test is performed when a client experiences vomiting, abdominal pain or gastroparesis. It is a radionuclide study in which the individual consumes a liquid or a solid mill that contains a radioactive isotope for visualization. The test involves timing how long the meal takes to get through the stomach. A barium swallow is done to determine the cause of painful swallowing, unexplained weight loss, or other abnormalities in the upper GI tract Contrast medium uses iodine dye which interacts with X-rays but is not radioactive A small camera at the end of a flexible tube is used to check the structures inside the GI tract. It is used for procedures such as endoscopy or a colonoscopy

The nurse is providing discharge instructions to a male client following gastrectomy and instructs the client to take which measure to assist in preventing dumping syndrome? A. Ambulate following a meal B. Eat high carbohydrate foods C. Limit the fluid taken with meal D. Sit in a high-Fowler's position during meals

C. Limit the fluid taken with meals. Dumping syndrome is a term that refers to a constellation of vasomotor symptoms that occurs after eating, especially following a Billroth II procedure. Early manifestations usually occur within 30 minutes of eating and include vertigo, tachycardia, syncope, sweating, pallor, palpitations, and the desire to lie down. The nurse should instruct the client to decrease the amount of fluid taken at meals. Option A: The nurse should instruct the client to lie down for 30 minutes after eating to delay gastric emptying, and to take antispasmodics as prescribed. Identify symptoms that may indicate dumping syndrome, (weakness, profuse perspiration, epigastric fullness, nausea and vomiting, abdominal cramping, faintness, flushing, explosive diarrhea, and palpitations occurring within 15 min to 1 hr after eating). Option B: The nurse should instruct the client to avoid high-carbohydrate foods, including fluids such as fruit nectars. Review dietary needs and regimen (low-carbohydrate, low-fat, high-protein) and the importance of maintaining vitamin supplementation. This may prevent deficiencies, enhance healing, and promote cooperation with therapy. A low-fat diet may be required to reduce the risk of alkaline reflux gastritis. Option D: The nurse should instruct the client to assume a low Fowler's position during meals. Avoid placing the patient in a supine position, have the patient sit upright after meals. Supine position after meals can increase regurgitation of acid. Instruct the patient to chew food thoroughly and eat slowly. Well-masticated food is easier to swallow. Food should be cut into small pieces.

The nurse is performing an abdominal assessment and inspects the skin of the abdomen. The nurse performs which assessment technique next? A. Palpates the abdomen for size B. Palpates the liver at the right rib margin C. Listens to bowel sounds in all four quadrants D. Percusses the right lower abdominal quadrant

C. Listens to bowel sounds in all four quadrants The appropriate sequence for abdominal examination is inspection, auscultation, percussion, and palpation. Auscultation is performed after inspection to ensure that the motility of the bowel and bowel sounds are not altered by percussion or palpation. Therefore, after inspecting the skin on the abdomen, the nurse should listen for bowel sounds. Option A: The ideal position for abdominal examination is to sit or kneel on the right side of the patient with the hand and forearm in the same horizontal plane as the patient's abdomen. There are three stages of palpation that include superficial or light palpation, deep palpation, and organ palpation and should be performed in the same order. Option B: The examiner should begin with superficial or light palpation from the area furthest from the point of maximal pain and move systematically through the nine regions of the abdomen. If no pain is present, any starting point can be chosen. Several sources mention that the abdomen should first gently be examined with the fingertips. Option D: A proper technique of percussion is necessary to gain maximum information regarding abdominal pathology. While percussing, it is important to appreciate tympany over air-filled structures such as the stomach and dullness to percussion which may be present due to an underlying mass or organomegaly (for example, hepatomegaly or splenomegaly).

Janice is waiting for discharge instructions after her herniorrhaphy. Which of the following instructions do you include? A. Eat a low-fiber diet. B. Resume heavy lifting in 2 weeks. C. Lose weight, if obese. D. Resume sexual activity once discomfort is gone.

C. Lose weight, if obese. Because obesity weakens the abdominal muscles, advise weight loss for the patient who has had a hernia repair. Every surgery has risks and complications but people with a high BMI have much higher risks for complications after hernia repair surgery, like a higher risk for the hernia to recur after surgical repair. This may require a second surgery. Option A: Eat lots of whole grains, fruits, green leafy vegetables. Avoid dairy products or foods that cause constipation. Avoid foods that cause constipation such as dairy products, red meat, processed foods such as pizza, frozen dinners, pasta, sugar products such as cakes, pies, pastries, doughnuts, and caffeine and caffeine drinks. Option B: The dressing is removed on postoperative day 5, and stitches are removed on postoperative day 7. Patients should be advised to avoid strenuous activities for a few weeks. Typically, light work can be resumed after 1 week, heavier jobs after 6 weeks. Option D: Postoperative sexual function is an important factor to consider because the operation is performed in the inguinal region, in close proximity to the testicular structures and nerves which are important for sexual function. Furthermore, modern techniques of hernia repair are based on the implantation of a mesh to reinforce the inguinal floor. The first results of a prospective study of the Department of General Surgery Charité, Campus Mitte, showed no significant influence of the hernia repair with mesh on sexual function for at least 3 months postoperatively.

Marie, a 51-year-old woman, is diagnosed with cholecystitis. Which diet, when selected by the client, indicates that the nurse's teaching has been successful? A. 4-6 small meals of low-carbohydrate foods daily B. High-fat, high-carbohydrate meals C. Low-fat, high-carbohydrate meals D. High-fat, low protein meals

C. Low-fat, high-carbohydrate meals Option C: For the client with cholecystitis, fat intake should be reduced. The calories from fat should be substituted with carbohydrates. Option A: Reducing carbohydrate intake would be contraindicated. Options B & D: Any diet high in fat may lead to another attack of cholecystitis.

The nurse would an order for which type of antacid in patients with chronic renal failure? A. Aluminum-containing antacids B. Calcium-containing antacids C. Magnesium-containing antacids D. All of the above.

C. Magnesium-containing antacids Magnesium-containing antacids can cause hypermagnesemia in patients with chronic renal failure. Antacids that contain magnesium have a laxative effect that may cause diarrhea, and in patients with renal failure, they may cause increased magnesium levels in the blood, because of the reduced ability of the kidneys to eliminate magnesium from the body in the urine. Option A: Aluminum-containing antacids may be used as a phosphate binder in patients with chronic renal failure. With the recognition that long-term ingestion of large doses of phosphate-binding antacids containing aluminum can result in aluminum retention and toxicity, the use of phosphate-binding agents free of this element, such as calcium carbonate, has been recommended for patients with chronic renal failure. Option B: Calcium-containing antacids are also appropriate because these patients may be hypocalcemic. Calcium salts neutralize gastric acidity resulting in increased gastric and duodenal bulb pH; they additionally inhibit the proteolytic activity of pepsin if the pH is greater than 4 and increase lower esophageal sphincter tone. The calcium released from calcium carbonate is known to increase peristalsis in the esophagus, pushing the acid into the stomach and providing relief from symptoms of heartburn. Option D: The average therapeutic dose of antacid is 10 to 15 mL (1 tablespoon or one package content) of liquid or 1 to 2 tablets 3 to 4 times a day. Periodic monitoring of calcium and phosphorus plasma concentrations is a suggested practice in patients on chronic therapy.

Pierre, who is diagnosed with acute pancreatitis, is under the care of Nurse Bryan. Which intervention should the nurse include in the care plan for the client? A. Administration of vasopressin and insertion of a balloon tamponade B. Preparation for a paracentesis and administration of diuretics C. Maintenance of nothing-by-mouth status and insertion of nasogastric (NG) tube with low intermittent suction D. Dietary plan of a low-fat diet and increased fluid intake to 2,000 ml/day

C. Maintenance of nothing-by-mouth status and insertion of nasogastric (NG) tube with low intermittent suction With acute pancreatitis, the client is kept on nothing-by-mouth status to inhibit pancreatic stimulation and secretion of pancreatic enzymes. NG intubation with low intermittent suction is used to relieve nausea and vomiting, decrease painful abdominal distention, and remove hydrochloric acid. Prolonged bowel rest by nothing per os (NPO) to minimize pancreatic secretion was an important part of the therapy for any patient with acute pancreatitis. Option A: Vasopressin would be appropriate for a client diagnosed with bleeding esophageal varices. The most common cause of late death in acute necrotizing pancreatitis is represented by organ failure through infected pancreatic necrosis (IPN). Therefore there might be a theoretical benefit from antibiotic prophylaxis. Option B: Paracentesis and diuretics would be appropriate for a client diagnosed with portal hypertension and ascites. Fluid therapy in acute pancreatitis can be seen as double edge sword with risk of necrosis through tissue hypoperfusion by using low fluid quantities and liquid sequestration and increased morbidity with too high volumes Option D: A low-fat diet and increased fluid intake would further aggravate pancreatitis. The concept of nutritional support in AP has gradually moved towards enteral feeding, due to large evidence proving safety and efficiency. Timing and mode of nutritional support in acute pancreatitis should be based on risk prediction of severity.

Which of the following associated disorders may the client with Crohn's disease exhibit? A. Ankylosing spondylitis B. Colon cancer C. Malabsorption D. Lactase deficiency

C. Malabsorption Because of the transmural nature of Crohn's disease lesions, malabsorption may occur with Crohn's disease. A number of factors related to Crohn's disease may contribute to malabsorption, like inflammation. Persistent, long-term inflammation of the small intestine in people with small bowel Crohn's disease often leads to damage of the intestinal lining. This can interfere with the organ's ability to absorb nutrients properly. Option A: Ankylosing spondylitis is more commonly associated with ulcerative colitis. Between 5 and 10% of cases of ankylosing spondylitis (AS) are associated with inflammatory bowel disease (IBD), ulcerative colitis. A much larger percentage of AS patients have subclinical gut inflammation manifested either by endoscopic findings or by histology. Option B: In general, having ulcerative colitis can increase the risk for colorectal cancer. Recent research shows that colorectal cancer rates are dropping among people with inflammatory bowel diseases such as ulcerative colitis. People with ulcerative colitis should receive regular screenings for colorectal cancer. Option D: Lactase deficiency is caused by a congenital defect in which an enzyme isn't present. Lactose intolerance is usually the result of the body not producing enough lactase. Lactase is an enzyme (a protein that causes a chemical reaction to occur) normally produced in the small intestine that's used to digest lactose. If the client has a lactase deficiency, it means his body does not produce enough lactase.

The client with a duodenal ulcer may exhibit which of the following findings during an assessment? A. Hematemesis B. Malnourishment C. Melena D. Pain with eating

C. Melena The client with a duodenal ulcer may have bleeding at the ulcer site, which shows up as melena (black tarry stool). Duodenal ulcers occur when there is a disruption to the surface of the mucosa of the duodenum. These ulcers are part of peptic ulcer disease, which involves the stomach and first part of the duodenum. The other findings are consistent with a gastric ulcer. Option A: Patients who initially present with ulcer-related complications may present with symptoms suggestive of upper GI bleed, including melena, hematemesis, elevated BUN, and anemia of varying degrees in severity with associated fatigue. Patients who present with more alarming symptoms such as anemia, melena, or hematemesis, which may represent perforation or bleeding, will likely require more invasive forms of evaluation. Option B: The presentation of patients with symptoms consistent with dyspepsia or peptic ulcer disease, and most specifically, duodenal ulcers, can vary highly depending on the degree of disease progression and time when a patient seeks treatment. Other common signs and symptoms include epigastric abdominal pain, bloating, nausea and vomiting, and weight gain due to improved symptoms post meals. Option D: Overall, dyspepsia is the most common symptom for patients who do experience symptoms. The location of the disease can also be differentiated based on symptoms. The pain associated with duodenal ulcers improves after meals, while the pain associated with gastric ulcers generally intensifies after meals.]

After a subtotal gastrectomy, care of the client's nasogastric tube and drainage system should include which of the following nursing interventions? A. Irrigate the tube with 30 ml of sterile water every hour if needed. B. Reposition the tube if it is not draining well. C. Monitor the client for N/V, and abdominal distention. D. Turn the machine to high suction if the drainage is sluggish on low suction.

C. Monitor the client for N/V, and abdominal distention. Nausea, vomiting, or abdominal distention indicated that gas and secretions are accumulating within the gastric pouch due to impaired peristalsis or edema at the operative site and may indicate that the drainage system is not working properly. Assess the comfort of the client. Check for presence of nausea and vomiting, feeling of fullness, or pain. May indicate operation of NG suction or blockage in the tube. Option A: Saline solution is used to irrigate nasogastric tubes. Hypotonic solutions such as water increase electrolyte loss. In addition, a physician's order is needed to irrigate the NG tube, because this procedure could disrupt the suture line. Irrigations are recorded as intake. Drainage from the NG tube is measured as output every 8 hours. If drainage is copious, more frequent emptying of the collection container will be necessary. Documentation provides an accurate record of the client's response to NG drainage. Option B: After gastric surgery, only the surgeon repositions the NG tube because of the danger of rupturing or dislodging the suture line. Always verify if the NG tube placed is in the stomach by aspirating a small amount of stomach contents. An X-ray study is the best way to verify placement. Option D: The amount of suction varies with the type of tube used and is ordered by the physician. High suction may create too much tension on the gastric suture line. Inspect suction apparatus. Check that setting is correct for the type of suction (continuous or intermittent), range of suction (low, medium, high), and that movement of drainage through the tubing is present.

Your patient has a GI tract that is functioning, but has the inability to swallow foods. Which is the preferred method of feeding for your patient? A. TPN B. PPN C. NG feeding D. Oral liquid supplements

C. NG feeding Because the GI tract is functioning, feeding methods involve the enteral route which bypasses the mouth but allows for a major portion of the GI tract to be used. Nasogastric tubes can be placed to administer medications or nutrition in patients who have a functional gastrointestinal tract but are unable to tolerate oral intake. This is most common in patients who have suffered a stroke or other malady, which has left them unable to swallow effectively. Option A: Parenteral nutrition is the intravenous administration of nutrition outside of the gastrointestinal tract. Total parenteral nutrition (TPN) is when the IV administered nutrition is the only source of nutrition the patient is receiving. Total parenteral nutrition is indicated when there is an inadequate gastrointestinal function and contraindications to enteral nutrition. Option B: Enteral diet intake is preferred over parenteral as it is inexpensive and associated with fewer complications such as infection and blood clots but requires a functional GI system. Option D: Nasogastric tubes may be placed for nutritional support while waiting to see how much function the patient will recover, and if the patient does not recover their swallowing ability or will otherwise require long term nutritional support, then a more permanent feeding tube should be placed such as gastrostomy or jejunostomy feeding tube.

The nurse is caring for a female client with active upper GI bleeding. What is the appropriate diet for this client during the first 24 hours after admission? A. Regular diet B. Skim milk C. Nothing by mouth D. Clear liquids

C. Nothing by mouth Shock and bleeding must be controlled before oral intake, so the client should receive nothing by mouth. When the bleeding is controlled, the diet is gradually increased, starting with ice chips and then clear liquids. In patients hospitalized for acute upper gastrointestinal bleeding due to an ulcer with high risk of rebleeding or with variceal bleeding, it is recommended to wait at least 48 h after endoscopic therapy before initiating oral or enteral feeding. Option A: A regular diet is . Proton pump inhibitors (PPIs) are beneficial for both ulcer and non-ulcer diseases as they reduce the risk of re-bleeding by clot stabilization. Endoscopy should only be performed after hemodynamic stability has been achieved and should not be delayed by more than 24 hours. Option B: Skim milk shouldn't be given because it increases gastric acid production, which could prolong bleeding. Further research found that ingesting milk increases the production of stomach acid, which can worsen gastritis symptoms. Any relief gastritis sufferers experience after drinking a glass of milk is likely to be temporary; within a half-hour, symptoms are usually worse, not better. Option D: A liquid diet is the first diet offered after bleeding and shock are controlled. They can be fed with clear liquids soon after endoscopy. Clear liquids provide the advantage that if the patient starts to bleed again, sedation and anesthesia can be given within two hours after the last ingestion

Brenda, a 36 y.o. patient is on your floor with acute pancreatitis. Treatment for her includes: A. Continuous peritoneal lavage. B. Regular diet with increased fat. C. Nutritional support with TPN. D. Insertion of a T tube to drain the pancreas.

C. Nutritional support with TPN. With acute pancreatitis, you need to rest the GI tract by TPN as nutritional support. In cases of severe pancreatitis or where peroral intake is not tolerated, nasojejunal feeding is superior to parenteral nutrition as it helps to minimize bacterial translocation by maintaining the intestinal barrier. Option A: The foundation of management for acute pancreatitis remains early aggressive fluid resuscitation. Lactated Ringer's solution is the recommended fluid with an initial bolus of 15 to 20 mL/kg and following rates of 3 mL/kg per hour (usually approximately 250 to 500 mL per hour) for the first 24 hours if no other contraindications are present. Option B: Common practice is to keep nothing by mouth until abdominal pain, nausea, vomiting, appetite, and ileus improve. Early feeding in mild pancreatitis is safe and does not exacerbate symptoms. Soft, low residue, low-fat diet is recommended for initial feeding and advanced to regular consistency as tolerated. Option D: Further management depends upon the etiology of pancreatitis. In gallstone pancreatitis, early cholecystectomy is strongly recommended. Early ERCP (within 24 hours of presentation) is of benefit in cases of concurrent cholangitis and obvious biliary obstruction.

Which of the following symptoms best describes Murphy's sign? A. Periumbilical ecchymosis exists. B. On deep palpation and release, pain elicited. C. On deep inspiration, pain is elicited and breathing stops. D. Abdominal muscles are tightened in anticipation of palpation.

C. On deep inspiration, pain is elicited and breathing stops. Murphy's sign is elicited when the client reacts to pain and stops breathing. It's a common finding in clients with cholecystitis. Murphy's sign is elicited in patients with acute cholecystitis by asking the patient to take in and hold a deep breath while palpating the right subcostal area. If pain occurs on inspiration, when the inflamed gallbladder comes into contact with the examiner's hand, Murphy's sign is positive. Option A: Periumbilical ecchymosis, Cullen's sign, is present in peritonitis. Cullen sign manifests as superficial edema with bruising in the subcutaneous fatty tissue around the periumbilical region. Originally described in association with ectopic pregnancy. Option B: Pain on deep palpation and release is rebound tenderness. Blumberg's sign (also referred to as rebound tenderness or the Shyotkin-Blumberg sign) is a clinical sign in which there is pain upon removal of pressure rather than the application of pressure to the abdomen. (The latter is referred to simply as abdominal tenderness.) It is indicative of peritonitis. Option D: Tightening up abdominal muscles in anticipation of palpation is guarding. Investigations of relationships of specific pain behaviors with pain intensity and fear of movement are rare. Guarding, defined as "behavior that is aimed at preventing or alleviating pain" and which includes stiffness, hesitation, and bracing, has been shown to predict work loss over 3 months in injured workers.

The hospitalized client with GERD is complaining of chest discomfort that feels like heartburn following a meal. After administering an ordered antacid, the nurse encourages the client to lie in which of the following positions? A. Supine with the head of the bed flat. B. On the stomach with the head flat. C. On the left side with the head of the bed elevated 30 degrees. D. On the right side with the head of the bed elevated 30 degrees.

C. On the left side with the head of the bed elevated 30 degrees. The discomfort of reflux is aggravated by positions that compress the abdomen and the stomach. These include lying flat on the back or on the stomach after a meal or lying on the right side. The left side-lying position with the head of the bed elevated is most likely to give relief to the client. Heartburn is the most common feature of GERD. This becomes more severe with vigorous exercise, bending, or lying down. Option A: Avoid placing the patient in supine position, have the patient sit upright after meals. Supine position after meals can increase regurgitation of acid. Instruct the patient to chew food thoroughly and eat slowly. Well-masticated food is easier to swallow. Food should be cut into small pieces. Option B: Elevate HOB while in bed to prevent aspiration by preventing the gastric acid to flow back into the esophagus. Instruct the patient to avoid highly seasoned food, acidic juices, alcoholic drinks, bedtime snacks, and foods high in fat. These can reduce the lower esophageal sphincter pressure. Option D: Instruct the patient to avoid bending over, coughing, straining at defecations, and other activities that increase reflux. This promotes comfort by the decrease in intra-abdominal pressure, which reduces the reflux of gastric contents.

During the assessment of a client's mouth, the nurse notes the absence of saliva. The client is also complaining of pain near the area of the ear. The client has been NPO for several days because of the insertion of a NG tube. Based on these findings, the nurse suspects that the client is developing which of the following mouth conditions? A. Stomatitis B. Oral candidiasis C. Parotitis D. Gingivitis

C. Parotitis The lack of saliva, pain near the area of the ear, and the prolonged NPO status of the client should lead the nurse to suspect the development of parotitis, or inflammation of the parotid gland. Parotitis usually develops in cases of dehydration combined with poor oral hygiene or when clients have been NPO for an extended period. Preventative measures include the use of sugarless hard candy or gum to stimulate saliva production, adequate hydration, and frequent mouth care. Option A: Stomatitis (inflammation of the mouth) produces excessive salivation and a sore mouth. Stomatitis may involve swelling and redness of the oral mucosa or discrete, painful ulcers (single or multiple). Less commonly, whitish lesions form, and, rarely, the mouth appears normal despite significant symptoms. Symptoms hinder eating, sometimes leading to dehydration and malnutrition. Secondary infection occasionally occurs, especially in immunocompromised patients. Option B: Oral candidiasis or thrush is an infection of the oral cavity by Candida albicans. Oral candidiasis is generally obtained secondary to immune suppression, whether a patient's oral cavity has decreased immune function or if it is systemic. This immunosuppression is dose-dependent. Option D: Gingivitis is an inflammatory condition of the gingival tissue, most commonly caused by bacterial infection. Unlike periodontitis, there is no attachment loss and therefore no migration of the junctional epithelium. The condition is restricted to the soft-tissue area of the gingival epithelium and connective tissue.

Which of the following conditions is most likely to directly cause peritonitis? A. Cholelithiasis B. Gastritis C. Perforated ulcer D. Incarcerated hernia

C. Perforated ulcer The most common cause of peritonitis is a perforated ulcer, which can pour contaminates into the peritoneal cavity, causing inflammation and infection within the cavity. Peritonitis is most often caused by the introduction of infection into the otherwise sterile peritoneal environment through organ perforation, but it may also result from other irritants, such as foreign bodies, bile from a perforated gallbladder or a lacerated liver, or gastric acid from a perforated ulcer. The other conditions don't by themselves cause peritonitis. Option A: However, if cholelithiasis leads to rupture of the gallbladder, peritonitis may develop. Biliary peritonitis is caused by perforation of the gallbladder, bile duct, or upper gastrointestinal tract. Nontraumatic perforation of the bile duct is a disease in which the extrahepatic duct or intrahepatic duct is perforated spontaneously without traumatic or iatrogenic injury. Option B: If gastritis leads to erosion of the stomach wall, peritonitis may develop. If an ulcer goes through (perforates) the stomach wall, stomach contents may spill into the abdominal cavity, resulting in inflammation and usually an infection of the lining of the abdominal cavity (peritonitis) and sudden worsening of pain. Some complications of gastritis are slow to develop. Option D: If an incarcerated hernia leads to rupture of the intestines, peritonitis may develop. Hernias that are incarcerated may be painful or may cause obstruction if the urinary bladder, ovary, or intestines are involved. Hernias that have strangulated are generally painful and may cause nausea, vomiting, peritoneal signs, peritonitis, sepsis, or even cardiovascular collapse.

The most important pathophysiological factor contributing to the formation of esophageal varices is: A. Decreased prothrombin formation. B. Decreased albumin formation by the liver. C. Portal hypertension. D. Increased central venous pressure.

C. Portal hypertension As the liver cells become fatty and degenerate, they are no longer able to accommodate a large amount of blood necessary for homeostasis. The pressure in the liver increases and causes increased pressure in the venous system. As the portal pressure increases, fluid exudes into the abdominal cavity. This is called ascites. Option A: Esophageal varices are dilated submucosal distal esophageal veins connecting the portal and systemic circulations. They form due to portal hypertension, which commonly is a result of cirrhosis, resistance to portal blood flow, and increased portal venous blood inflow. Variceal rupture is the most common fatal complication of cirrhosis. The severity of liver disease correlates with the presence of varices and the risk of bleeding. Option B: Increased resistance to portal flow at the level of hepatic sinusoids is caused by intrahepatic vasoconstriction due to decreased nitric oxide production, and increased release of endothelin-1 (ET-1), angiotensinogen, and eicosanoids; sinusoidal remodeling disrupting blood flow' and increased portal flow is caused by hyperdynamic circulation due to splanchnic arterial vasodilation through mediators such as nitric oxide, prostacyclin, and TNF. Option D: Portal hypertension causes portocaval anastomosis to develop to decompress portal circulation. Normal portal pressure is between 5-10 mmHg but in the presence of portal obstruction, the pressure may be as high as 15-20 mmHg. Since the portal venous system has no valves, resistance at any level between the splanchnic vessels and the right side of the heart results in retrograde flow and elevated pressure.

Your patient with peritonitis is NPO and complaining of thirst. What is your priority? A. Increase the I.V. infusion rate. B. Use diversion activities. C. Provide frequent mouth care. D. Give ice chips every 15 minutes.

C. Provide frequent mouth care. Frequent mouth care helps relieve dry mouth. Maintain NPO with nasogastric or intestinal aspiration. This reduces hyperactivity of bowel and diarrhea losses. Observe skin or mucous membrane dryness, turgor. Note peripheral and sacral edema. Hypovolemia, fluid shifts, and nutritional deficits contribute to poor skin turgor, taut edematous tissues. Option A: Administer plasma or blood, fluids, electrolytes, diuretics as indicated. Replenishes circulating volume and electrolyte balance. Colloids (plasma, blood) help move water back into the intravascular compartment by increasing the osmotic pressure gradient. Diuretics may be used to assist in the excretion of toxins and to enhance renal function. Option B: Change position frequently, provide frequent skincare, and maintain dry or wrinkle-free bedding. Edematous tissue with compromised circulation is prone to breakdown. Option D: Eliminate noxious sights and smells from the environment. Limit intake of ice chips. This reduces gastric stimulation and vomiting response. Excessive use of ice chips during gastric aspiration can increase gastric washout of electrolytes.

Which of the following treatments is used for rectal cancer but not for colon cancer? A. Chemotherapy B. Colonoscopy C. Radiation D. Surgical resection

C. Radiation A client with rectal cancer can expect to have radiation therapy in addition to chemotherapy and surgical resection of the tumor. Radiation therapy isn't usually indicated in colon cancer. Neoadjuvant combined CRT is recommended for locally advanced resectable colon cancer (cT3-4N0-2M0). Adjuvant therapy is strongly suggested for all pathological T3 and/or N positive tumors. Option A: Palliative systemic chemotherapy is offered to non-surgical candidates with unresectable locally advanced disease or high metastatic burden to improve quality of life and prolongs life expectancy. Rectal cancer has strong recommendations for neoadjuvant therapy for stage II (T3 or T4 node-negative) and III (node-positive), although the best regimen has not been established. Option B: A colonoscopy is performed to diagnose the disease. The initial evaluation may involve barium enema or computed tomography (CT) colonography, but endoscopy is ultimately required for tissue biopsy. Flexible sigmoidoscopy is no replacement for a complete diagnostic colonoscopy; still, it is a screening modality that reduces CRC mortality. Option D: Endoscopic resection (ER) is reserved for selected candidates with favorable-risk and early-stage (cT1N0M0) found in a completely excised rectal polyp. Upfront Rca surgical resection is appropriate for lesions that do not invade the muscularis propria and negative lymphatic nodes (cT2N0M0) on appropriate surgical candidates.

The nurse is caring for a hospitalized female client with a diagnosis of ulcerative colitis. Which finding, if noted on assessment of the client, would the nurse report to the physician? A. Hypotension B. Bloody diarrhea C. Rebound tenderness D. A hemoglobin level of 12 mg/dL

C. Rebound tenderness Rebound tenderness may indicate peritonitis. During the physical exam, pertinent findings include fever and abdominal tenderness to palpation which usually is diffuse with wall rigidity in more septic presentations. Signs of peritonitis must be reported to the physician. It is important to conduct a thorough exam as certain thoracic or pelvic pathologies can mimic peritoneal irritation (empyema causing diaphragmatic irritation and cystitis/pyelonephritis causing peritoneum adjacent pain). Option A: Because of the blood loss, the client may be hypotensive. In ulcerative colitis, bleeding can arise from the lining of the rectum or large intestine, and this blood can be visible in the stool. The bleeding generally comes from the ulcers that have formed in the lining of the large intestine or rectum. Option B: Bloody diarrhea is expected to occur in ulcerative colitis. Ulcerative colitis's main symptom is bloody diarrhea, with or without mucus. Associated symptoms also include urgency or tenesmus, abdominal pain, malaise, weight loss, and fever, depending on the extent and severity of the disease. Option D: Because of the blood loss, the client may be hypotensive and the hemoglobin level may be lower than normal. Signs of peritonitis must be reported to the physician. It may also be necessary to treat the loss of blood that has happened. If the client developed anemia from blood loss, he may need to supplement with iron, folic acid, or vitamin B12, depending on what your health provider says. In serious cases of blood loss, a blood transfusion might be required.

A client with rectal cancer may exhibit which of the following symptoms? A. Abdominal fullness B. Gastric fullness C. Rectal bleeding D. Right upper quadrant pain

C. Rectal bleeding Rectal bleeding is a common symptom of rectal cancer. Rectal cancer may be missed because other conditions such as hemorrhoids can cause rectal bleeding. Symptoms according to tumor location on the clinical presentation of rectosigmoid are more frequently associated with a change in bowel habits (diminish stool caliber), bright red blood per rectum (hematochezia), pain (tenesmus), leakage diarrhea (mucus discharge), and constipation (obstruction). Option A: Abdominal fullness may occur with colon cancer. Physical examination should explore signs of ascites, hepatomegaly, and lymphadenopathy and must extend to a digital rectal exam for fixed mass. A thorough family history is of great relevance in identifying familial clusters and inherited patterns that would change the surveillance and therapy of a high-risk patient. Option B: Gastric fullness may occur with gastric cancer. The most common physical examination finding is a palpable abdominal mass indicating advanced disease. The patient may also present with signs of metastatic lymphatic spread distribution, including Virchow's node (left supraclavicular adenopathy), Sister Mary Joseph node (periumbilical nodule), and Irish node (left axillary node). Option D: Right upper quadrant pain may occur with liver cancer. Most patients are initially asymptomatic from hepatocellular carcinoma but often present with related symptoms due to chronic liver disease. Patients may complain of upper abdominal discomfort and distention, weight loss, fever, poor appetite, early satiety, diarrhea, and other symptoms.

A 53 y.o. patient has undergone a partial gastrectomy for adenocarcinoma of the stomach. An NG tube is in place and is connected to low continuous suction. During the immediate postoperative period, you expect the gastric secretions to be which color? A. Brown B. Clear C. Red D. Yellow

C. Red Normally, drainage is bloody for the first 24 hours after a partial gastrectomy; then it changes to brown-tinged and then to yellow or clear. Drainage will be bloody for the first 12 hours, and then should clear and turn greenish. Continued or recurrent bleeding suggests complications. A decline in output may reflect the return of GI function. Option A: This tube will be set to suction and will drain out brownish-colored stomach acid. When it runs from brown to light green to clear, this is an indication that things are moving through the stomach and feedings may be possible. Option B: Gastric aspirate is usually cloudy and green, tan or off-white, or brown. Intestinal aspirate is generally clear and yellow to bile-colored. Pleural fluid is pale yellow and serous; tracheobronchial secretions are usually tan or off-white mucus. Option D: Normal color of gastric drainage is light yellow to green in color due to the presence of bile. Bloody drainage may be expected after gastric surgery but must be monitored closely. The presence of coffee-ground type drainage may indicate bleeding.

A male client with a peptic ulcer is scheduled for a vagotomy and the client asks the nurse about the purpose of this procedure. Which response by the nurse best describes the purpose of a vagotomy? A. Halts stress reactions B. Heals the gastric mucosa C. Reduces the stimulus to acid secretions D. Decreases food absorption in the stomach

C. Reduces the stimulus to acid secretions. A vagotomy, or cutting of the vagus nerve, is done to eliminate parasympathetic stimulation of gastric secretion. A vagotomy is a type of surgery that removes all or part of the vagus nerve. This nerve runs from the bottom of the brain, through the neck, and along the esophagus, stomach, and intestines in the gastrointestinal (GI) tract. Option A: Vagotomy was once commonly performed to treat and prevent PUD; however, with the availability of excellent acid secretion control with H2-receptor antagonists, proton pump inhibitors, and anti-Helicobacter pylori medications, the need for surgical management of this condition has greatly decreased. Option B: The relevant physiology revolves around the mechanisms relating to stomach acid secretion. Intraluminal gastric acid is released by the parietal cells, mainly located in the body of the stomach. Parietal cells are stimulated via 3 mechanisms: gastrin, acetylcholine, and histamine. All 3 mechanisms activate the hydrogen-potassium ATPase-releasing hydrogen ion into the stomach lumen. Option D: The indications for vagotomy are few with the advancements of medical therapy. Generally, acid-reducing operations are reserved for complicated ulcer disease in a stable patient who has failed maximum medical therapy. The type of surgery performed depends on the type of ulcer (duodenal versus gastric), the complication of PUD (bleeding, perforation, obstruction, intractability), and the location of the ulcer (types I to V gastric ulcers as described by the Modified Johnson Classification system).

The client being treated for esophageal varices has a Sengstaken-Blakemore tube inserted to control the bleeding. The most important assessment is for the nurse to: A. Check that the hemostat is on the bedside. B. Monitor IV fluids for the shift. C. Regularly assess respiratory status. D. Check that the balloon is deflated on a regular basis.

C. Regularly assess respiratory status. The respiratory system can become occluded if the balloon slips and moves up the esophagus, putting pressure on the trachea. This would result in respiratory distress and should be assessed frequently. Scissors should be kept at the bedside to cut the tube if distress occurs. This is a safety intervention. Option A: Variceal band ligation is preferred to sclerotherapy for bleeding varices and for non-bleeding medium-to-large varices to decrease bleeding risk. Ligation has lower rates of rebleeding, fewer complications, more rapid cessation of bleeding, and a higher rate of variceal eradication. Option B: Treat coagulopathy as necessary. Fresh frozen plasma may increase blood volume and increase rebleeding risk. IV octreotide to lower portal venous pressure as adjuvant to endoscopic management. An IV bolus of 50 micrograms followed by a drip of 50 micrograms/hr. Option D: If endoscopic treatment fails, consider self-expanding esophageal metal stents or peroral placement of Sengstaken-Blakemore-type tubes for up to 24 hours to stabilize the patient for TIPS. As many as two-thirds of patients with variceal bleeding develop an infection, most commonly spontaneous bacterial peritonitis, UTI, or pneumonia. Antibiotic prophylaxis with oral norfloxacin 400 mg or IV ceftriaxone, 1 g q24h for up to a week, is indicated.

Britney, a 20 y.o. student is admitted with acute pancreatitis. Which laboratory findings do you expect to be abnormal for this patient? A. Serum creatinine and BUN B. Alanine aminotransferase (ALT) and aspartate aminotransferase (AST) C. Serum amylase and lipase D. Cardiac enzymes

C. Serum amylase and lipase Pancreatitis involves the activation of pancreatic enzymes, such as amylase and lipase. These levels are elevated in a patient with acute pancreatitis. The diagnosis of acute pancreatitis has been defined by the Revised Atlanta Classification and requires at least 2 of 3 criteria to be met: 1) a lipase or amylase level that is three times the upper limit of normal 2) abdominal pain that is consistent with pancreatitis 3) abdominal imaging consistent with acute pancreatitis. Option A: Early changes in BUN level may reflect several important physiologic processes in acute pancreatitis. In addition to intravascular volume depletion, a rise in BUN level may be secondary to impairment in renal function or potentially concurrent upper gastrointestinal hemorrhage. Renal failure is a relatively common form of organ dysfunction among patients with acute pancreatitis Option B: Jaundice with increased ALT suggests gallstone etiology requiring ERCP. ALT or AST levels more than three times the upper limit of normal indicates gallstones as the cause of acute pancreatitis. However, the absence of elevated transaminases does not rule out gallstones. ALT has high specificity, but low sensitivity for gallstone pancreatitis. Option D: Acute pancreatitis can be associated with electrical changes mimicking acute coronary syndrome with normal coronary arteries. The association of acute pancreatitis with ST-segment elevation and elevated cardiac enzymes has been reported in few observations. The pathophysiological mechanisms of this association remain poorly understood.

Your goal is to minimize David's risk of complications after a herniorrhaphy. You instruct the patient to: A. Avoid the use of pain medication. B. Cough and deep breath Q2H. C. Splint the incision if he can't avoid sneezing or coughing. D. Apply heat to scrotal swelling.

C. Splint the incision if he can't avoid sneezing or coughing. Teach the patient to avoid activities that increase intra-abdominal pressure such as coughing, sneezing, or straining with a bowel movement. Patients should be advised to avoid strenuous activities for a few weeks. Typically, light work can be resumed after 1 week, heavier jobs after 6 weeks. Option A: Postoperative chronic pain is more frequent than was previously understood and has become one of the most important primary endpoints in hernia surgery. In published reports, the incidence of post herniorrhaphy pain has ranged from 0% to more than 30%. Chronic inguinodynia is defined as pain persisting more than 3 months post herniorrhaphy, after the process of wound healing is complete. Option B: Recurrence in Lichtenstein hernioplasty may be due to inaccurate execution of the technique (inadequate size or improper fixation of the mesh) or to an overlooked hernia at the primary operation. To avoid the latter, the patient should be asked to cough, and the region should be carefully examined for an indirect hernia, a direct hernia, a femoral hernia, or a combined hernia. Option D: After the procedure, the patient is asked to rest for a few hours. He or she may be discharged later the same day on a day-care basis. Early mobilization is the key to rapid convalescence. Patients can safely ambulate on the evening of the operation. If general or regional anesthesia is used, the patient may be hospitalized for a few days.

Which of the following medications is most effective for treating the pain associated with irritable bowel disease? A. Acetaminophen B. Opiates C. Steroids D. Stool softeners

C. Steroids The pain with irritable bowel disease is caused by inflammation, which steroids can reduce. If a patient has concerns about abdominal pain, bloating, cramping, and changes in bowel habits, a visit to a primary care physician is advised. If IBS is diagnosed, a gastroenterology consultation will be needed as they can guide management and treatment. Option A: Acetaminophen has little effect on the pain. Prednisone is a steroid with anti-inflammatory effects. It is used to treat inflammation in ulcerative colitis and Crohn's disease. However, steroids do not prevent symptoms from returning and have many side effects. Sometimes, it is necessary to use steroids to treat IBD, especially during "flares." Option B: Opiate narcotics won't treat its underlying cause. In inflammatory bowel disease (IBD), corticosteroids are the mainstay of "rescue" therapy for patients who are experiencing a disease flare. Corticosteroids often lead to rapid resolution of IBD symptoms, which aligns with patient preferences for treatment. However, this may lead to inappropriate use of these medications for maintenance when other treatment options may be better in the longer term. Option D: Stool softeners aren't necessary. For constipation, fiber supplements and laxatives can be helpful whereas, in those with diarrhea, medications such as loperamide or probiotics can be helpful. Escalation of therapy to corticosteroid-sparing maintenance therapy such as immunomodulators or biological agents can improve disease outcomes and avoid the complications of prolonged steroid use.

The nurse is performing colostomy irrigation on a client. During the irrigation, a client begins to complain of abdominal cramps. Which of the following is the most appropriate nursing action? A. Notify the physician. B. Increase the height of the irrigation. C. Stop the irrigation temporarily. D. Medicate with dilaudid and resume the irrigation.

C. Stop the irrigation temporarily. If cramping occurs during colostomy irrigation, the irrigation flow is stopped temporarily and the client is allowed to rest. Cramping may occur from an infusion that is too rapid or is causing too much pressure. Have the colostomy patient sit on or near the toilet for about 15 to 20 minutes so the initial colostomy returns can drain into the toilet. (If the patient is on bed rest, allow the colostomy to drain into the bedpan.) Option A: The physician does not need to be notified. Unless contraindicated or otherwise ordered by the physician, it is best to establish a routine of daily irrigation in accordance with the patient's former bowel habits. Option B: Increasing the height of the irrigation will cause further discomfort. Hold the enema can approximately 12 inches above the bed and allow the solution to flow in slowly to avoid painful cramps usually caused by too rapid flow. Option C: Medicating the client for pain is not the most appropriate action. If cramping occurs, slow down the flow rate and ask the patient to deep breathe until cramps subside. Cramping during irrigation may indicate that the flow is too fast or the water is too cold.

The nurse is monitoring a client for the early signs of dumping syndrome. Which symptom indicates this occurrence? A. Abdominal cramping and pain B. Bradycardia and indigestion C. Sweating and pallor D. Double vision and chest pain

C. Sweating and pallor Early manifestations of dumping syndrome occur 5 to 30 minutes after eating. Symptoms include vertigo, tachycardia, syncope, sweating, pallor, palpitations, and the desire to lie down. In early dumping, the symptoms usually occur within 10 to 30 minutes after a meal. The rapid transit of hyperosmolar chyme from the stomach into the duodenum causes fluid to shift from the vasculature to the intestinal lumen, leading to increased volume in the small bowel. Option A: Late dumping, also known as postprandial hyperinsulinemic hypoglycemia, usually occurs 1 to 3 hours after a high-carbohydrate meal. There is an association with hypoglycemia, but the exact mechanism is unknown. It is proposed that the rapid absorption of carbohydrates exaggerates the glucose-mediated insulin response. Option B: There may be GI or vasomotor symptoms. GI symptoms include nausea, vomiting, diarrhea, or belching. Vasomotor symptoms include shock, syncope, near-syncope, palpitations, dizziness, desire to lie down, or diaphoresis. Option D: GI hormones such as enteroglucagon, pancreatic polypeptide, peptide YY, vasoactive intestinal polypeptide, glucagon-like peptide, and neurotensin have been evident with higher values after meals. Hormonal imbalances may cause delayed motility, decreased gastric and intestinal secretions, which delay the digestion and transit of food that arrives at the small bowel.

A nurse is preparing to remove a nasogastric tube from a female client. The nurse should instruct the client to do which of the following just before the nurse removes the tube? A. Exhale B. Inhale and exhale quickly C. Take and hold a deep breath D. Perform a Valsalva maneuver

C. Take and hold a deep breath. When the nurse removes a nasogastric tube, the client is instructed to take and hold a deep breath. This will close the epiglottis. This allows for easy withdrawal through the esophagus into the nose. The nurse removes the tube with one smooth, continuous pull. Option A: The patient should take a deep breath, and hold it, not exhale. An NG tube should be removed if it is no longer required. The process of removal is usually very quick. Prior to removing an NG tube, verify physician orders. If the NG tube was ordered to remove gastric content, the physician's order may state to "trial" clamping the tube for a number of hours to see if the patient tolerates its removal. During the trial, the patient should not experience any nausea, vomiting, or abdominal distension. Option B: Inhaling and exhaling quickly could make the removal uncomfortable. Instruct the patient to take a deep breath and hold it. This prevents aspiration; holding the breath closes the glottis. Option D: Performing the Valsalva maneuver is inappropriate. Kink the NG tube near the nares and gently pull out the tube in a swift, steady motion, wrapping it in hand as it is being pulled out. Dispose of the tube in a garbage bag.

In a client with diarrhea, which outcome indicates that fluid resuscitation is successful? A. The client passes formed stools at regular intervals. B. The client reports a decrease in stool frequency and liquidity. C. The client exhibits firm skin turgor. D. The client no longer experiences perianal burning.

C. The client exhibits firm skin turgor. A client with diarrhea has a nursing diagnosis of Deficient fluid volume related to excessive fluid loss in the stool. Expected outcomes include firm skin turgor, moist mucous membranes, and urine output of at least 30 ml/hr. Evaluate dehydration by observing skin turgor over the sternum and inspecting for longitudinal furrows of the tongue. Option A: The client also has a nursing diagnosis of diarrhea, with expected outcomes of passage of formed stools at regular intervals and a decrease in stool frequency and liquidity. Give antidiarrheal drugs as ordered. Record number and consistency of stools per day; if desired, use a fecal incontinence collector for accurate measurement of output. Option B: Have patient keep a diary that includes the following: time of day defecation occurs; usual stimulus for defecation; consistency, amount, and frequency of stool; type of, amount of, and time food consumed; fluid intake; history of bowel habits and laxative use; diet; exercise patterns; obstetrical/gynecological, medical, and surgical histories; medications; alterations in perianal sensations; and present bowel regimen. Option D: The client is at risk for impaired skin integrity related to irritation from diarrhea; expected outcomes for this diagnosis include absence of erythema in perianal skin and mucous membranes and the absence of perianal tenderness or burning. Mild cleansing of the perianal skin after each bowel movement will prevent excoriation. Barrier creams can be used to protect the skin.

A client with suspected gastric cancer undergoes an endoscopy of the stomach. Which of the following assessments made after the procedure would indicate the development of a potential complication? A. The client complains of a sore throat. B. The client displays signs of sedation. C. The client experiences a sudden increase in temperature. D. The client demonstrates a lack of appetite.

C. The client experiences a sudden increase in temperature. The most likely complication of an endoscopic procedure is perforation. A sudden temperature spike within 1 to 2 hours after the procedure is indicative of perforation and should be reported immediately to the physician. This most commonly occurs when additional procedures are carried out at the same time. The infections are normally minor and treatable with a course of antibiotics. Option A: A sore throat is to be anticipated after an endoscopy. Risks of endoscopy may include persistent pain in the area of the endoscopy or a numb throat for a few hours due to the use of a local anesthetic. Option B: Clients are given sedatives during the procedure, so it is expected that they will display signs of sedation after the procedure is completed. Risks of endoscopy may include over-sedation, although sedation is not always necessary. Option D: A lack of appetite could be the result of many factors, including the disease process. There may be some soreness. With this type of endoscopy, there may be bloating and soreness, but these usually resolve quickly.

A client has been taking aluminum hydroxide 30 mL six times per day at home to treat his peptic ulcer. He tells the nurse that he has been unable to have a bowel movement for 3 days. Based on this information, the nurse would determine which of the following is the most likely cause of the client's constipation? A. The client has not been including enough fiber in his diet. B. The client needs to increase his daily exercise. C. The client is experiencing a side effect of the aluminum hydroxide. D. The client has developed a gastrointestinal obstruction.

C. The client is experiencing a side effect of the aluminum hydroxide. It is most likely that the client is experiencing a side effect of the antacid. Antacids with aluminum salt products, such as aluminum hydroxide, form insoluble salts in the body. These precipitate and accumulate in the intestines, causing constipation. Disturbances of gut motility occur frequently under a high-dose antacid regimen. Typical symptoms are diarrhea and constipation. They are due to the cations of the antacids. Aluminum causes constipation, magnesium induces diarrhea, and calcium has no definite motor effect. Option A: Increasing dietary fiber intake may be a beneficial lifestyle change for the client but is not likely to relieve constipation caused by the aluminum hydroxide. Fiber adds bulk to the stool and makes defecation easier because it passes through the intestine essentially unchanged. Option B: Urge the patient for some physical activity and exercise. Consider isometric abdominal and gluteal exercises. Movement promotes peristalsis. Abdominal exercises strengthen abdominal muscles that facilitate defecation. Option D: Constipation, in isolation from other symptoms, is not a sign of bowel obstruction. Encourage the patient to take in fluid 2000 to 3000 mL/day, if not contraindicated medically. Sufficient fluid is needed to keep the fecal mass soft. But take note of some patients or older patients having cardiovascular limitations requiring less fluid intake.

Mucosal barrier fortifiers are used in peptic ulcer disease management for which of the following indications? A. To inhibit mucus production. B. To neutralize acid production. C. To stimulate mucus production. D. To stimulate hydrogen ion diffusion back into the mucosa.

C. To stimulate mucus production. The mucosal barrier fortifiers stimulate mucus production and prevent hydrogen ion diffusion back into the mucosa, resulting in accelerated ulcer healing. Sucralfate, a polymer of sucrose with aluminum hydroxide, forms a protective coating on the mucosal lining, particularly in ulcerated areas. In the presence of acid, it becomes a gel that adheres to epithelial cells and ulcer craters. Option A: Misoprostol is a prostaglandin analog that increases the release of bicarbonate and mucin (a component of mucus) and reduces acid secretion by binding to prostaglandin receptors on parietal cells. Because NSAIDs (nonsteroidal anti-inflammatory drugs) inhibit prostaglandin formation, a synthetic prostaglandin such as misoprostol is sometimes given to reduce NSAID-induced damage. Option B: Antacids neutralize acid production. The antacids reduce the acid reaching the duodenum by neutralizing the acid present in the stomach. The salts' mechanism of neutralization of acid varies, and each salt has a different mechanism with the ultimate goal of acid neutralization. Option D: The mucosal barrier is the name given to the barrier in the stomach that resists the back-diffusion of hydrogen ions. The barrier is a layer of thick mucus secreted together with an alkaline fluid. Since the mucus is a gel, it entraps the alkaline fluid so that the stomach is coated.

Dark, tarry stools indicate bleeding in which location of the GI tract? A. Upper colon B. Lower colon C. Upper GI tract D. Small intestine

C. Upper GI tract Melena is the passage of dark, tarry stools that contain a large amount of digested blood. It occurs with bleeding from the upper GI tract. The clinical presentation can vary but should be well-characterized. Hematemesis is the overt bleeding with vomiting of fresh blood or clots. Melena refers to dark and tarry-appearing stools with a distinctive smell. The term "coffee-grounds" describes gastric aspirate or vomitus that contains dark specks of old blood. Option A: UGIB is described as blood loss from a gastrointestinal source above the ligament of Treitz. It can manifest as hematemesis, which can be bright red emesis or coffee-ground emesis, hematochezia, or melena. Patients may also present with symptoms secondary to blood loss, such as syncopal episodes, fatigue, and weakness. Option B: Gastrointestinal bleeding can fall into two broad categories: upper and lower sources of bleeding. The anatomic landmark that separates upper and lower bleeds is the ligament of Treitz, also known as the suspensory ligament of the duodenum. This peritoneal structure suspends the duodenojejunal flexure from the retroperitoneum. Option D: Bleeding that originates above the ligament of Treitz usually presents either as hematemesis or melena whereas bleeding that originates below most commonly presents as hematochezia.

Which of the following interventions should be included in the medical management of Crohn's disease? A. Increasing oral intake of fiber. B. Administering laxatives. C. Using long-term steroid therapy. D. Increasing physical activity.

C. Using long-term steroid therapy Management of Crohn's disease may include long-term steroid therapy to reduce the inflammation associated with the deeper layers of the bowel wall. Most people with Crohn's disease need to take steroids (such as prednisolone) from time to time. Steroids can relieve symptoms by reducing inflammation in the digestive system - they usually start to work in a few days or weeks. are usually taken as tablets once a day - sometimes they're given as injections. Option A: Other management focuses on bowel rest (not increasing oral intake). Provide colon rest by omitting or decreasing the stimulus of foods and fluids. Gradual resumption of liquids may prevent cramping and recurrence of diarrhea; however, cold fluids can increase intestinal motility. Option B: Reducing diarrhea with medications, not giving laxatives, is the management of Crohn's disease. Observe and record stool frequency, characteristics, amount, and precipitating factors. Helps differentiate the individual disease and assess the severity of an episode. Option D: The pain associated with Crohn's disease may require bed rest, not an increase in physical activity. Promote bedrest, provide a bedside commode. Rest decreases intestinal motility and reduces the metabolic rate when infection or hemorrhage is a complication. Urge to defecate may occur without warning and be uncontrollable, increasing the risk of incontinence or falls if facilities are not close at hand.

The nurse would anticipate using which medication if sclerotherapy has not been used? A. Neomycin B. Propranolol C. Vasopressin D. Cimetidine

C. Vasopressin Vasopressin is the drug of choice when sclerotherapy is contraindicated. Vasoactive drugs stop bleeding in most patients, and emergency sclerotherapy may carry risks to the patient and is more demanding on the healthcare system. Sclerotherapy did not appear to be superior to vasoactive drugs in terms of control of bleeding, the number of transfusions, 42?day rebleeding and mortality, or rebleeding and mortality before other elective treatments. Option A: Neomycin is used in preventing encephalopathy when blood is broken down. Neomycin belongs to a group of antibiotics known as aminoglycosides. Like others in the aminoglycoside family, neomycin works by inhibiting bacterial protein synthesis leading to its bactericidal effect. This group of medications is particularly effective in killing gram-negative organisms allowing for good coverage of enteric organisms. Option B: Propranolol may or may not be used to decrease cardiac output and hepatic venous pressure. Propranolol can be used to ameliorate the sympathetic response in angina, tachyarrhythmias, prevention of acute ischemic attacks, migraine prophylaxis, and restless leg syndrome. Propranolol can be used in almost all cases if the desired result is to slow contractility and decrease a patient's heart rate. Option D: Cimetidine is a drug with the indication of peptic ulcer disease, gastroesophageal reflux disease, and for dermatological conditions including warts, urticaria, mastocytosis, and erythropoietic protoporphyria. This medication is an H2 receptor antagonist.

A female client with viral hepatitis A is being treated in an acute care facility. Because the client requires enteric precautions, the nurse should: A. Place the client in a private room. B. Wear a mask when handling the client's bedpan. C. Wash the hands after touching the client. D. Wear a gown when providing personal care for the client.

C. Wash the hands after touching the client. To maintain enteric precautions, the nurse must wash the hands after touching the client or potentially contaminated articles and before caring for another client. Enteric precautions are taken to prevent infections that are transmitted primarily by direct or indirect contact with fecal material. They're indicated for patients with known or suspected infectious diarrhea or gastroenteritis. Option A: A private room is warranted only if the client has poor hygiene — for instance, if the client is unlikely to wash the hands after touching infective material or is likely to share contaminated articles with other clients. Option B: For enteric precautions, the nurse need not wear a mask. While taking care of the client, the nurse may wear gloves and a gown if she might have contact with body fluids (stool, urine, saliva). A sign on the door to the client's room reminds staff to wear a gown and gloves when inside the room. Staff will wash their hands before entering and leaving the client's room. Option D: For enteric precautions, the nurse must wear a gown only if soiling from fecal matter is likely. Wash hands before entering and before leaving the client's room. Make sure to use soap and water when leaving the room. Be sure other visitors do this too.

A pt had an ERCP to remove a gallstone. Which intervention should RN implement? A. assess for rectal bleeding B. increase fluid intake C. assess gag reflex D. keep in supine position

C. assess gag reflex

The RN is analyzing the physical assessment findings of spider angiomas, palmar erythema, peripheral edema, ascites and change in mental status of the pt. These findings are consistent with which of the following disorders? A. cholelithiasis B. hepatitis C. cirrhosis D. pancreatitis

C. cirrhosis

The pt is admitted to the hospital with viral hepatitis, complaining of "no appetite" and "losing taste for food". To provide adequate nutrition, the RN would instruct the pt to A. eat a good supper when anorexia is not as severe B. eat less often, preferably only 3 large meals daily C. increase intake of fluids including juices D. select foods high in fat

C. increase intake of fluids including juices

A pt suspected of having cirrhosis asks what test will confirm the diagnosis. What is the RN's most appropriate reply? A. measures of liver enzymes : ALT & AST B. CT scan C. liver biopsy D. PT : prothrombin time

C. liver biopsy

An adult female is diagnosed with cholecystitis and is being treated conservatively at home. The RN visits her at home and begins teaching diet for healthy weight loss and reducing pain episodes. What diet should the RN recommend? A. very low calorie diet to promote faster weight loss B. higher fat diet to stimulate gallbladder to empty C. low carb, low fat, higher protein diet D. low protein diet with adequate carb and no fat at all

C. low carb, low fat, higher protein diet

Your patient Maria takes NSAIDS for her degenerative joint disease, and has developed peptic ulcer disease. Which drug is useful in preventing NSAID-induced peptic ulcer disease? A. calcium carbonate (Tums) B. famotidine (Pepcid) C. misoprostol (Cytotec) D. sucralfate (Carafate)

C. misoprostol (Cytotec) Misoprostol restores prostaglandins that protect the stomach from NSAIDS, which diminish the prostaglandins. Currently, misoprostol is FDA-approved only for the prevention and treatment of NSAID-induced gastric ulcers in patients taking NSAIDs and at high risk for ulceration. It has an indication (but not FDA approved) in the short-term treatment of active duodenal or gastric ulcers with other etiologies. Option A: Calcium carbonate is an inorganic salt primarily used in the management and treatment of low calcium conditions, GERD, CKD, and a variety of other indicated conditions. It is classified as a calcium supplement, antacid, and as a phosphate binder. Option B: Famotidine is a competitive histamine H-receptor antagonist (H2RA) that binds to the H-receptors located on the basolateral membrane of the parietal cell in the stomach, effectively blocking histamine actions. Its pharmacologic activity results in the inhibition of gastric secretion by suppressing acid concentration and volume of gastric secretion. Option D: Sucralfate is a medication used to treat duodenal ulcers, epithelial wounds, chemotherapy-induced mucositis, radiation proctitis, ulcers in Behcet disease, and burn wounds. Sucralfate exhibits its action by forming a protective layer, increasing bicarbonate production, exhibiting anti-peptic effects, promoting tissue growth, regeneration, and repair.]

A patient unable to tolerate oral medications may be prescribed which of the following proton pump inhibitors to be administered intravenously? A. lansoprazole (Prevacid) B. omeprazole (Prilosec) C. pantoprazole (Protonix) D. esomeprazole (Nexium)

C. pantoprazole (Protonix) Pantoprazole is the only proton pump inhibitor that is available for intravenous administration. For erosive esophagitis associated with gastroesophageal reflux disease, pantoprazole administration can be oral or intravenous. For Zollinger-Ellison syndrome, pantoprazole administration can also be oral or intravenous. Intravenously, the recommendation is to administer 80 mg of pantoprazole every 12 hours. The other medications in this category may only be administered orally. Option A: As proton pumps recycle periodically in the stomach, it may take a few days for PPIs to achieve a full effect - and of note, their duration of action is slower than some other medications that affect acid production, such as histamine-receptor blockers. These medications are best administered before food intake as proton pumps become activated during meals, and administration of PPIs prior to food intake will enhance the drug's efficacy. Option B: Omeprazole should be ingested 30 to 60 minutes before meals. It may be taken with antacids. When taken twice daily, the first dose should be before breakfast and the second dose before dinner. The capsule and tablet should be swallowed whole, not crushed or chewed. However, it is permissible to open the capsule and mix the contents with one tablespoon of applesauce, soft enough to be swallowed without chewing. Option D: Most practitioners recommend that the patient take the PPI first thing in the morning when taken once daily. If twice-daily dosing is employed, then a second dose is usually added approximately 30 minutes before dinner. For some select patients with nighttime predominance of symptoms, the timing of once-daily administration may change to 30 minutes pre-dinner.

In planning preoperative teaching for a patient undergoing a Roux-en-Y gastric bypass as treatment for morbid obesity, the nurse places the highest priority on A. demonstrating range-of-motion exercises to the legs. B. teaching the patient about the postoperative presence of a NG tube connected to suction. C. teaching the patient proper coughing and deep-breathing techniques and methods of turning and positioning. D. discussing the necessary postoperative modifications in lifestyle.

C. teaching the patient proper coughing and deep-breathing techniques and methods of turning and positioning.

Which assessment finding is of most concern for a patient with acute pancreatitis? A. Absent bowel sounds B. Abdominal tenderness C. Left upper quadrant pain D. Palpable abdominal mass

D A palpable abdominal mass may indicate the presence of a pancreatic abscess, which will require rapid surgical drainage to prevent sepsis. Absent bowel sounds, abdominal tenderness, and left upper quadrant pain are common in acute pancreatitis and do not require rapid action to prevent further complications.

A 40-yr-old male patient has had a herniorrhaphy to repair an incarcerated inguinal hernia. Which patient teaching will the nurse provide before discharge? A. Soak in Sitz baths several times each day. B. Cough 5 times each hour for the next 48 hours. C. Avoid using acetaminophen (Tylenol) for pain. D. Apply a scrotal support and ice to reduce swelling.

D A scrotal support and ice are used to reduce edema and pain. Coughing will increase pressure on the incision. Sitz baths will not relieve pain and would not be of use after this surgery. Acetaminophen can be used for postoperative pain.

The nurse is providing preoperative teaching for a patient scheduled for an abdominal-perineal resection. Which information will the nurse include? A. The patient will need to be on bedrest for three days after surgery. B. An ileal-anal reservoir will be surgically created in 8 to 12 weeks. C. The patient will have a temporary colostomy for 6-12 months. D. The site for the stoma will be marked on the abdomen before surgery.

D A wound, ostomy, continence nurse (WOCN) would select the site where the ostomy will be positioned and mark the abdomen preoperatively. The site would be within the rectus muscle, on a flat surface, and in a place that the patient is able to see. A permanent colostomy is created with this surgery and no further surgery would be planned. The patient will be encouraged to walk the day after surgery.

Which finding is likely in the nurse's assessment of a patient who has a large bowel obstruction? A. Referred back pain B. Metabolic alkalosis C. Projectile vomiting D. Abdominal distention

D Abdominal distention is seen in lower intestinal obstruction. Referred back pain is not a common clinical manifestation of intestinal obstruction. Metabolic alkalosis is common in high intestinal obstruction because of the loss of HCl acid from vomiting. Projectile vomiting is associated with higher intestinal obstruction.

Which breakfast choice indicates a patient's good understanding of information about a diet for celiac disease? A. Wheat toast with butter B. Oatmeal with nonfat milk C. Bagel with low-fat cream cheese D. Corn tortilla with scrambled eggs

D Avoidance of gluten-containing foods is the only treatment for celiac disease. Corn does not contain gluten, but oatmeal and wheat do.

Which topic would the nurse anticipate teaching to a patient who has a new report of heartburn? A. Radionuclide tests B. Barium swallow exam C. Endoscopy procedures D. Proton pump inhibitors

D Because diagnostic testing for heartburn that is probably caused by gastroesophageal reflux disease (GERD) is expensive and uncomfortable, proton pump inhibitors are frequently used for a short period as the first step in the diagnosis of GERD. The other tests may be used but are not usually the first step in diagnosis.

A young adult contracts hepatitis from contaminated food. Which result would the nurse expect serologic testing to reveal during the acute (icteric) phase of the patient's illness? A. Antibody to hepatitis D (anti-HDV) B. Hepatitis B surface antigen (HBsAg) C. Anti-hepatitis A virus immunoglobulin G (anti-HAV IgG) D. Anti-hepatitis A virus immunoglobulin M (anti-HAV IgM)

D Hepatitis A is transmitted through the oral-fecal route, and antibody to HAV IgM appears during the acute phase of hepatitis A. The patient would not have antigen for hepatitis B or antibody for hepatitis D. Anti-HAV IgG would indicate past infection and lifelong immunity.

A patient who takes a nonsteroidal antiinflammatory drug (NSAID) daily for the management of severe rheumatoid arthritis has recently developed melena. What would the nurse anticipate teaching the patient? A. Substitution of acetaminophen (Tylenol) for the NSAID B. Use of enteric-coated NSAIDs to reduce gastric irritation C. Reasons for using corticosteroids to treat the rheumatoid arthritis D. Misoprostol (Cytotec) to protect the gastrointestinal (GI) mucosa

D Misoprostol, a prostaglandin analog, reduces acid secretion and the incidence of upper GI bleeding associated with NSAID use. Enteric coating of NSAIDs does not reduce the risk for GI bleeding. Corticosteroids increase the risk for ulcer development and will not be substituted for NSAIDs for this patient. Acetaminophen will not be effective in treating rheumatoid arthritis.

A patient with acute pancreatitis is NPO and has a nasogastric (NG) tube to suction. Which information obtained by the nurse indicates that these therapies have been effective? A. Bowel sounds are present. B. Grey Turner sign resolves. C. Electrolyte levels are normal. D. Abdominal pain is decreased.

D NG suction and NPO status will decrease the release of pancreatic enzymes into the pancreas and decrease pain. Although bowel sounds may be hypotonic with acute pancreatitis, the presence of bowel sounds does not indicate that treatment with NG suction and NPO status has been effective. Electrolyte levels may be abnormal with NG suction and must be replaced by appropriate IV infusion. Although Grey Turner sign will eventually resolve, it would not be appropriate to wait for this to occur to determine whether treatment was effective.

The health care provider prescribes antacids and sucralfate (Carafate) for treatment of a patient's peptic ulcer. Which medication schedule would the nurse teach the patient? A. Sucralfate at bedtime and antacids before each meal B. Sucralfate and antacids together 0 minutes before meals C. Antacids 30 minutes before each dose of sucralfate is taken D. Antacids after meals and sucralfate 30 minutes before meals

D Sucralfate is most effective when the pH is low and should not be given with or soon after antacids. Antacids are most effective when taken after eating. Administration of sucralfate 30 minutes before eating and antacids just after eating will ensure that both drugs can be most effective. The other regimens will decrease the effectiveness of the medications.

Which finding indicates to the nurse that a patient's transjugular intrahepatic portosystemic shunt (TIPS) placed 3 months ago has been effective? A. Increased serum albumin level B. Decreased indirect bilirubin level C. Improved alertness and orientation D. Fewer episodes of bleeding varices

D TIPS is used to lower pressure in the portal venous system and decrease the risk of bleeding from esophageal varices. Indirect bilirubin level and serum albumin levels are not affected by shunting procedures. TIPS will increase the risk for hepatic encephalopathy.

A patient has cirrhosis and 4+ pitting edema. Which focused data would the nurse assess? A. Hemoglobin B. Temperature C. Activity level D. Albumin level

D The low oncotic pressure caused by hypoalbuminemia is a major pathophysiologic factor in the development of edema. The other parameters are not directly associated with the patient's edema.

Which assessment would the nurse perform first for a patient who just vomited bright red blood? A. Measuring the quantity of emesis B. Palpating the abdomen for distention C. Auscultating the chest for breath sounds D. Taking the blood pressure (BP) and pulse

D The nurse is concerned about blood loss and possible hypovolemic shock in a patient with acute gastrointestinal bleeding. BP and pulse are the best indicators of these complications. The other information is important to obtain, but BP and pulse rate are the best indicators for assessing intravascular volume.

A patient is diagnosed with stomach cancer after an unintended 20-lb weight loss. Which action would the nurse include in the plan of care? A. Refer the patient for hospice services. B. Infuse IV fluids through a central line. C. Teach the patient about antiemetic therapy. D. Offer supplemental feedings between meals.

D The patient data indicate a poor nutritional state and improvement in nutrition will be helpful in improving the response to therapies such as surgery, chemotherapy, or radiation. Nausea and vomiting are not common clinical manifestations of stomach cancer. There is no indication that the patient requires hospice or IV fluid infusions.

Which condition would the nurse anticipate when caring for a patient with a history of a total gastrectomy? A. Constipation B. Dehydration C. Elevated total serum cholesterol D. Cobalamin (vitamin B12) deficiency

D The patient with a total gastrectomy does not secrete intrinsic factor, which is needed for cobalamin (vitamin B12) absorption. Because the stomach absorbs only small amounts of water and nutrients, the patient is not at higher risk for dehydration, elevated cholesterol, or constipation.

A nurse is considering which patient to admit to the same room as a patient who is hospitalized with acute rejection 3 weeks after a liver transplant. Which patient would be the best choice? A. Patient who is receiving chemotherapy for liver cancer B. Patient who is receiving treatment for acute hepatitis C C. Patient who has a wound infection after cholecystectomy D. Patient who requires pain management for chronic pancreatitis

D The patient with chronic pancreatitis does not present an infection risk to the immunosuppressed patient who had a liver transplant. The other patients either are at risk for infection or currently have an infection, which will place the immunosuppressed patient at risk for infection.

After several days of antibiotic therapy for pneumonia, an older hospitalized patient develops watery diarrhea. Which action would the nurse take first? A. Notify the health care provider. B. Obtain a stool specimen for analysis. C. Teach the patient about hand washing. D. Place the patient on contact precautions.

D The patient's history and new onset diarrhea suggest a C. difficile infection, which requires implementation of contact precautions to prevent spread of the infection to other patients. The other actions are also appropriate but can be accomplished after contact precautions are implemented.

A patient with cirrhosis has ascites and 4+ edema of the feet and legs. Which nursing action will be included in the plan of care? A. Restrict daily dietary protein intake. B. Reposition the patient every 4 hours. C. Perform passive range of motion twice daily. D. Place the patient on a pressure-relief mattress.

D The pressure-relieving mattress will decrease the risk for skin breakdown for this patient. Adequate dietary protein intake is necessary in patients with ascites to improve oncotic pressure. Repositioning the patient every 4 hours will not be adequate to maintain skin integrity. Passive range of motion will not take the pressure off areas such as the sacrum that are vulnerable to breakdown.

A patient is transferred from the recovery room to a surgical unit after a transverse colostomy. The nurse observes the stoma to be deep pink with edema and a small amount of sanguineous drainage. Which action would the nurse take? A. Place ice packs around the stoma. B. Notify the surgeon about the stoma. C. Monitor the stoma every 30 minutes. D. Document stoma assessment findings.

D The stoma appearance indicates good circulation to the stoma. There is no indication that surgical intervention is needed or that frequent stoma monitoring is required. Swelling of the stoma is normal for 2 to 3 weeks after surgery. An ice pack is not needed.

Which statement by a patient with jaundice indicates a need for teaching? A. "I used cough syrup several times a day last week." B. "I take a baby aspirin every day to prevent strokes." C. "I take an antacid for indigestion several times a week" D. "I use acetaminophen (Tylenol) every 4 hours for pain."

D Chronic use of high doses of acetaminophen can be hepatotoxic and may have caused the patient's jaundice. The other patient statements require further assessment by the nurse but do not indicate a need for patient education.

When teaching an elderly client how to prevent constipation, which of the following instructions should the nurse include? A. "Drink 6 glasses of fluid each day." B. "Avoid grain products and nuts." C. "Add at least 4 grams of bran to your cereal each morning." D. "Be sure to get regular exercise."

D. "Be sure to get regular exercise." Exercise helps prevent constipation. Urge the patient for some physical activity and exercise. Consider isometric abdominal and gluteal exercises. Movement promotes peristalsis. Abdominal exercises strengthen abdominal muscles that facilitate defecation. Option A: Fluids and dietary fiber promote normal bowel function. The client should drink eight to ten glasses of fluid each day. Encourage the patient to take in fluid 2000 to 3000 mL/day, if not contraindicated medically. Sufficient fluid is needed to keep the fecal mass soft. But take note of some patients or older patients having cardiovascular limitations requiring less fluid intake. Option B: Grain products should be a part of the client's diet. A balanced diet that comprises adequate fiber, fresh fruits, vegetables, and grains is important. Twenty grams of fiber per day is suggested. Option C: Although adding bran to cereal helps prevent constipation by increasing dietary fiber, the client should start with a small amount and gradually increase the amount as tolerated to a maximum of 2 grams a day. Assist the patient to take at least 20 g of dietary fiber (e.g., raw fruits, fresh vegetables, whole grains) per day.

A client is taking an antacid for treatment of a peptic ulcer. Which of the following statements bestindicates that the client understands how to correctly take the antacid? A. "I should take my antacid before I take my other medications." B. "I need to decrease my intake of fluids so that I don't dilute the effects of my antacid." C. "My antacid will be most effective if I take it whenever I experience stomach pains." D. "It is best for me to take my antacid 1 to 3 hours after meals."

D. "It is best for me to take my antacid 1 to 3 hours after meals." Antacids are most effective if taken 1 to 3 hours after meals and at bedtime. When an antacid is taken on an empty stomach, the duration of the drug's action is greatly decreased. Taking antacids 1 to 3 hours after a meal lengthens the duration of action, thus increasing the therapeutic action of the drug. Option A: Antacids should be administered about 2 hours after other medications to decrease the chance of drug interactions. Antacids are available as over-the-counter medications, meaning that patients do not require a prescription to obtain them. This situation results in the improper use of these medications with little to no relief of symptoms. Option B: It is not necessary to decrease fluid intake when taking antacids. The dose for antacids depends upon the age of the patient, the purpose of administration (neutralization of acid or off-label use), and the presence of other comorbidities like renal or hepatic impairment. Option C: Antacids are a combination of various compounds with various salts of calcium, magnesium, and aluminum as the active ingredients. The antacids act by neutralizing the acid in the stomach and by inhibiting pepsin, which is a proteolytic enzyme. Each of these cationic salts has a characteristic pharmacological property that determines its clinical use.

You're discharging Nathaniel with hepatitis B. Which statement suggests understanding by the patient? A. "Now I can never get hepatitis again." B. "I can safely give blood after 3 months." C. "I'll never have a problem with my liver again, even if I drink alcohol." D. "My family knows that if I get tired and start vomiting, I may be getting sick again."

D. "My family knows that if I get tired and start vomiting, I may be getting sick again." Hepatitis B can recur. Patients infected with HBV could be asymptomatic initially and, depending on the particular genotype, might not be symptomatic throughout the infected state. In these particular cases, careful history taking is important to establish a diagnosis. However, when symptomatic from acute HBV infection, patients can present with serum sickness-like syndrome manifested as fever, skin rash, arthralgia, and arthritis. This syndrome usually subsides with the onset of jaundice. Patients may also have fatigue, abdominal pain, nausea, and anorexia. Option A: Unlike hepatitis A and hepatitis E, in which there is no chronic state, HBV infection has the potential for the development of a chronic state. Chronic hepatitis B predisposes a patient to the development of portal hypertension, cirrhosis, and its complications or hepatocellular carcinoma (HCC). Option B: Patients who have had hepatitis are permanently barred from donating blood. HBsAg is transmitted via blood contact or body secretions, and the risk of acquiring hepatitis B is considerably higher in individuals with close contact with HBsAg-positive patients. Option C: Alcohol is metabolized by the liver and should be avoided by those who have or had hepatitis B. As such, patients with HBV infection should be monitored closely, and a referral to a specialist is highly recommended. Fulminant liver failure from HBV infection requires an emergent liver transplant evaluation at a liver transplant center.

A patient with Crohn's disease is admitted after 4 days of diarrhea. Which of the following urine-specific gravity values do you expect to find in this patient? A. 005 B. 011 C. 020 D. 030

D. 030 The normal range of specific gravity of urine is 010 to 025; a value of 030 may be seen with dehydration. Specific gravity measures the kidney's ability to concentrate or dilute urine in relation to plasma. Because urine is a solution of minerals, salts, and compounds dissolved in water, the specific gravity is greater than 000. The more concentrated the urine, the higher the urine-specific gravity. Option A: In diabetes insipidus, there is an absence or decrease of antidiuretic hormone. Without antidiuretic hormone, the kidneys produce an excessive amount of urine, often up to 15 to 20 liters per day with low specific gravity. Option B: The first and most common reason for an increase in urine-specific gravity is dehydration. The second reason for a high specific gravity is an increased secretion of antidiuretic hormone (ADH). ADH causes increased tubular water re-absorption and decreased urine volume. Trauma, stress reactions, surgery, and many drugs cause an increase in ADH secretion. Option C: Glomerulonephritis and pyelonephritis cause a decreased urine volume and low specific gravity. In these diseases, damage to the kidney's tubules affects the ability of the kidney to reabsorb water. As a result, the urine remains dilute.

A nurse is preparing to care for a female client with esophageal varices who just had a Sengstaken-Blakemore tube inserted. The nurse gathers supplies, knowing that which of the following items must be kept at the bedside at all times? A. An obturator B. Kelly clamp C. An irrigation set D. A pair of scissors

D. A pair of scissors When the client has a Sengstaken-Blakemore tube, a pair of scissors must be kept at the client's bedside at all times. The client needs to be observed for sudden respiratory distress, which occurs if the gastric balloon ruptures and the entire tube moves upward. If this occurs, the nurse immediately cuts all balloon lumens and removes the tube. Sengstaken-Blakemore tube placement is indicated for unstable patients with uncontrolled hemorrhage. Sengstaken-Blakemore tube placements can temporarily control the hemorrhage. Option A: An obturator is kept at the bedside of a client with a tracheostomy. This is a piece of rigid plastic, silicone, or metal that fits inside the outer cannula when a tracheostomy tube is being inserted. It helps guide the tracheostomy tube into place, causing less damage to the tissues. Option B: A Kelly clamp is kept at the bedside of a client with a tracheostomy. Clinicians must be prepared in case of emergency as the medical condition of a patient with tracheostomy and/or mechanical ventilation may change quickly. Emergency equipment is necessary at the bedside as well as during the transportation. Option C: An irrigation set may be kept at the bedside, but it is not the priority item. Airway protection remains the foremost focus. If the patient is requiring a Sengstaken-Blakemore tube placement, they have likely already been intubated for airway protection, but if not, endotracheal intubation should be performed prior to placement. Following intubation, the patient should be placed in the supine position with the head of the bed elevated to 45 degrees.

The nurse is monitoring a female client with a diagnosis of peptic ulcer. Which assessment findings would most likely indicate perforation of the ulcer? A. Bradycardia B. Numbness in the legs C. Nausea and vomiting D. A rigid, board-like abdomen

D. A rigid, board-like abdomen Perforation of an ulcer is a surgical emergency and is characterized by sudden, sharp, intolerable severe pain beginning in the mid epigastric area and spreading over the abdomen, which becomes rigid and board-like. Perforated peptic ulcer (PPU) is a serious complication of PUD and patients with PPU often present with an acute abdomen that carries a high risk for morbidity and mortality. The lifetime prevalence of perforation in patients with PUD is about 5%. PPU carries mortality ranging from 1.3% to 20%. Option A: Tachycardia may occur as hypovolemic shock develops. The classic triad of sudden onset of abdominal pain, tachycardia, and abdominal rigidity is the hallmark of perforated peptic ulcers. Early diagnosis, prompt resuscitation, and urgent surgical intervention are essential to improve outcomes. Option B: Numbness in the legs is not an associated finding. Symptoms of PUD include abdominal pain, upper abdominal discomfort, bloatedness, and feeling of fullness. When PUD worsens and eventually perforates, gastric juice and gas enter the peritoneal cavity leading to chemical peritonitis. Option C: Nausea and vomiting may occur. Sudden onset of abdominal pain or acute deterioration of the ongoing abdominal pain is typical of PPU. Typically the pain never completely subsides despite usual premedical remedies and forces the patient to seek medical attention.

Which of the following would be an expected nutritional outcome for a client who has undergone a subtotal gastrectomy for cancer? A. Regain weight loss within 1 month after surgery. B. Resume normal dietary intake of three meals per day. C. Control nausea and vomiting through regular use of antiemetics. D. Achieve optimal nutritional status through oral or parenteral feedings.

D. Achieve optimal nutritional status through oral or parenteral feedings. An appropriate expected outcome is for the client to achieve optimal nutritional status through the use of oral feedings or total parenteral nutrition (TPN). TPN may be used to supplement oral intake, or it may be used alone if the client cannot tolerate oral feedings. Maintain patency of NG tube. Notify the physician if the tube becomes dislodged. This provides rest for the GI tract during the acute postoperative phase until the return of normal function. Option A: The client would not be expected to regain lost weight within 1 month after surgery. Note admission weight and compare with subsequent readings. This provides information about the adequacy of dietary intake and determination of nutritional needs. Option B: The client would not be expected to tolerate a normal dietary intake of three meals per day. Monitor tolerance to fluid and food intake, noting abdominal distension, reports of increased pain, cramping, nausea, and vomiting. Complications of paralytic ileus, obstruction, delayed gastric emptying, and gastric dilation may occur, possibly requiring reinsertion of the NG tube. Option C: Nausea and vomiting would not be considered an expected outcome of gastric surgery, and regular use of antiemetics would not be anticipated. Progress diet as tolerated, advancing from clear liquid to bland diet with several small feedings. Usually, the NG tube is clamped for specified periods of time when peristalsis returns to determine tolerance. After the NG tube is removed, intake is advanced gradually to prevent gastric irritation and distension.

The nurse caring for a client with small bowel obstruction would plan to implement which nursing intervention first? A. Administering pain medication B. Obtaining a blood sample for laboratory studies C. Preparing to insert a nasogastric (NG) tube D. Administering I.V. fluids

D. Administering I.V. fluids. I.V. infusions containing normal saline solution and potassium should be given first to maintain fluid and electrolyte balance. Maintenance of bowel rest requires alternative fluid replacement to correct losses and anemia. Fluids containing sodium may be restricted in presence of regional enteritis. Option A: Pain medication often is withheld until the obstruction is diagnosed because analgesics can decrease intestinal motility. Provide comfort measures (back rub, reposition) and diversional activities. Promotes relaxation, refocuses attention, and may enhance coping abilities. Option B: A blood sample is then obtained for laboratory studies to aid in the diagnosis of bowel obstruction and guide treatment. Blood studies usually include a complete blood count, serum electrolyte levels, and blood urea nitrogen level. Option C: For the client's comfort and to assist in bowel decompression, the nurse should prepare to insert an NG tube next. Resume or advance diet as indicated (clear liquids progressing to bland, low residue; then high-protein, high-calorie, caffeine-free, non-spicy, and low-fiber as indicated).

Nurse Hannah is teaching a group of middle-aged men about peptic ulcers. When discussing risk factors for peptic ulcers, the nurse should mention: A. A sedentary lifestyle and smoking. B. A history of hemorrhoids and smoking. C. Alcohol abuse and a history of acute renal failure. D. Alcohol abuse and smoking.

D. Alcohol abuse and smoking. Risk factors for peptic (gastric and duodenal) ulcers include alcohol abuse, smoking, and stress. Peptic ulcer disease (PUD) has various causes; however, Helicobacter pylori-associated PUD and NSAID-associated PUD account for the majority of the disease etiology. Option A: A sedentary lifestyle isn't a risk factor for peptic ulcers. PUD is a global problem with a lifetime risk of development ranging from 5% to 10%. Overall, there is a decrease in the incidence of PUD worldwide due to improved hygienic and sanitary conditions combined with effective treatment and judicious use of NSAIDs. Option B: A history of hemorrhoids isn't a risk factor for peptic ulcers. Gastric and duodenal ulcers can be differentiated from the timing of their symptoms in relation to meals. Nocturnal pain is common with duodenal ulcers. Those with gastric outlet obstruction commonly report a history of abdomen bloating and or fullness. Option C: Chronic renal failure, not acute renal failure, is associated with duodenal ulcers. The prognosis of PUD is excellent after the underlying cause is successfully treated. Recurrence of the ulcer may be prevented by maintaining good hygiene and avoiding alcohol, smoking, and NSAIDs.

The client with peptic ulcer disease is scheduled for a pyloroplasty. The client asks the nurse about the procedure. The nurse plans to respond knowing that a pyloroplasty involves: A. Cutting the vagus nerve. B. Removing the distal portion of the stomach. C. Removal of the ulcer and a large portion of the cells that produce hydrochloric acid. D. An incision and resuturing of the pylorus to relax the muscle and enlarge the opening from the stomach to the duodenum.

D. An incision and resuturing of the pylorus to relax the muscle and enlarge the opening from the stomach to the duodenum. Pyloroplasty is surgery to widen the opening in the lower part of the stomach (pylorus) so that stomach contents can empty into the small intestine (duodenum). The pylorus is a thick, muscular area. When it thickens, food cannot pass through. Option A: A vagotomy involves cutting the vagus nerve. A vagotomy is a type of surgery that removes all or part of the vagus nerve. This nerve runs from the bottom of the brain, through the neck, and along the esophagus, stomach, and intestines in the gastrointestinal (GI) tract. Option B: A subtotal gastrectomy involves removing the distal portion of the stomach. Gastrectomy is a surgery that's done to treat stomach cancer. During gastrectomy, the surgeon may remove part or all of the stomach. A subtotal gastrectomy includes removing the part of the stomach with cancer, nearby lymph nodes, and possibly parts of other organs near the tumor. Option C: A Billroth II procedure involves removal of the ulcer and a large portion of the tissue that produces hydrochloric acid. There are many variations on the procedure, but they generally involve resection of the diseased portion of the distal stomach and a side-to-side anastomosis of the residual stomach to jejunum through the transverse mesocolon. It can be performed with either an antecolic or a retrocolic anastomosis.

The client with GERD complains of a chronic cough. The nurse understands that in a client with GERD this symptom may be indicative of which of the following conditions? A. Development of laryngeal cancer. B. Irritation of the esophagus. C. Esophageal scar tissue formation. D. Aspiration of gastric contents.

D. Aspiration of gastric contents Clients with GERD can develop pulmonary symptoms such as coughing, wheezing, and dyspnea that are caused by the aspiration of gastric contents. It is frequently thought that GERD plays a big role in chronic cough; there are reports that 25% or more of chronic cough cases are associated with GERD. Option A: GERD does not predispose the client to the development of laryngeal cancer. The most intuitive theory is called the reflux theory, whereby reflux rises above the esophagus and upper esophageal sphincter, resulting in microaspiration as microdroplets land in the larynx or occasionally enter the bronchial tree, directly causing cough as a protective mechanism against reflux. Option B: Irritation of the esophagus can develop as a result of GERD. However, GERD is more likely to cause painful and difficult swallowing. In the reflex theory, because of the common embryologic origin of the respiratory tract and the digestive tract, a little bit of reflux in the esophagus can lead to an esophagobronchial reflex that causes cough. Option C: Esophageal scar tissue formation can develop as a result of GERD. GERD occurs in approximately 20% of Americans, and chronic cough is a very common problem, which patients with GERD are not immune to developing. Due to the baseline GERD rate of 20%, it is difficult to separate the presence of the disorder from the causative effect of the disorder.

Which of the following nursing interventions should be implemented to manage a client with appendicitis? A. Assessing pain. B. Encouraging oral intake of clear fluids. C. Providing discharge teaching. D. Assessing for symptoms of peritonitis.

D. Assessing for symptoms of peritonitis The focus of care is to assess for peritonitis, or inflammation of the peritoneal cavity. Peritonitis is most commonly caused by appendix rupture and invasion of bacteria, which could be lethal. Monitor vital signs. Note onset of fever, chills, diaphoresis, changes in mentation, reports of increasing abdominal pain. This can be suggestive of the presence of infection or developing sepsis, abscess, peritonitis. Option A: The client with appendicitis will have pain that should be controlled with analgesia. Assess pain, noting location, characteristics, severity (0-10 scale). Investigate and report changes in pain as appropriate. Keep the client at rest in semi-Fowler's position to lessen the pain. Gravity localizes inflammatory exudate into the lower abdomen or pelvis, relieving abdominal tension, which is accentuated by a supine position. Option B: The nurse should discourage oral intake in preparation for surgery. Aperients should also be avoided as induced peristalsis may cause perforation. If appendicitis has been diagnosed regular analgesia, usually an opioid depending on the pain severity, should be given to make the patient comfortable before treatment. Option C: Discharge teaching is important; however, in the acute phase, management should focus on minimizing preoperative complications and recognizing when such may be occurring.

Bismuth subsalicylate (Pepto-Bismol), as an absorbent, has which of the following mechanisms of action? A. Decreased GI motility. B. Decreased gastric secretions. C. Increased fluid absorption. D. Binding to diarrhea-causing bacteria for excretion.

D. Binding to diarrhea-causing bacteria for excretion. Absorbent antidiarrheal medications bind to diarrhea-causing bacteria to form a nonabsorbable complex, which is then excreted in the stool. Bismuth subsalicylate (BSS) exhibits many of its properties due to its formulation as an insoluble salt of salicylic acid and trivalent bismuth. The mechanism of action through which BSS works is complex. Option A: In the stomach, BSS hydrolyzes into two compounds, bismuth, and salicylic acid. The salicylate compound is almost completely absorbed into the bloodstream, while bismuth salt is minimally absorbed. Option B: The bismuth that remains in the gastrointestinal tract forms other bismuth salts. These bismuth salts contain bactericidal and antimicrobial activity, and prevent bacteria from binding and growing on the mucosal cells of the stomach. This is the mechanism by which BSS helps eradicate H. pylori. Option C: The prevention of bacterial binding to the mucosal cells provides many benefits, which include prevention of intestinal secretion, promotion of fluid absorption, reduction of inflammation, and promotion of the healing of any present ulcer in the stomach.

Surgical management of ulcerative colitis may be performed to treat which of the following complications? A. Gastritis B. Bowel herniation C. Bowel outpouching D. Bowel perforation

D. Bowel perforation Perforation, obstruction, hemorrhage, and toxic megacolon are common complications of ulcerative colitis that may require surgery. Perforation can also present in severe ulcerative colitis even in the absence of toxic megacolon. Most perforations occur in the left colon, commonly in the sigmoid colon. Perforations tend to occur more often during the first episodes of colitis. Option A: Gastritis isn't associated with irritable bowel diseases. The current classification of gastritis is based on time course (acute versus chronic), histological features, anatomic distribution, and underlying pathological mechanisms. Acute gastritis will evolve to chronic, if not treated. Helicobacter pylori (H. pylori) is the most common cause of gastritis worldwide. Option B: Inguinal hernias are considered to have both a congenital and acquired component. Most adult hernias are considered acquired. However, there is evidence to suggest genetics also play a role. Patients with a known family history of a hernia are at least 4 times more likely to have an inguinal hernia than patients with no known family history. Option C: Outpouching of the bowel is diverticulosis. Diverticulosis is a clinical condition in which multiple sac-like protrusions (diverticula) develop along the gastrointestinal tract. Though diverticula may form at weak points in the walls of either the small or large intestines, the majority occur in the large intestine (most commonly the sigmoid colon).

Sitty, a 66 y.o. patient underwent a colostomy for a ruptured diverticulum. She did well during the surgery and returned to your med-surg floor in stable condition. You assess her colostomy 2 days after surgery. Which finding do you report to the doctor? A. Blanched stoma B. Edematous stoma C. Reddish-pink stoma D. Brownish-black stoma

D. Brownish-black stoma A brownish-black color indicates lack of blood flow, and maybe necrosis. Necrosis occurs if the blood supply to the stoma is restricted. Initially, the stoma will become a darker red/purple and may even turn black, which is an indication that the blood supply is impaired. It may also feel cold and hard to touch. It is vital that you seek urgent medical attention. Option A: A blanched or pale stoma indicates possible decreased blood flow and should be assessed regularly. 2 days postoperatively, the stoma should be edematous and reddish-pink. Option B: It is normal for the stoma to be edematous postoperatively. Stoma edema begins immediately postoperatively. The stoma looks swollen within 4 to 6 hours, swelling progresses for the first 2 days, and by the fifth day subsides markedly. Option C: A healthy stoma is pinkish-red and moist. The stoma should stick out slightly from the skin. It is normal to see a little mucus. Spots of blood or a small amount of bleeding from the stoma is normal.

If a client had irritable bowel syndrome, which of the following diagnostic tests would determine if the diagnosis is Crohn's disease or ulcerative colitis? A. Abdominal computed tomography (CT) scan B. Abdominal x-ray C. Barium swallow D. Colonoscopy with biopsy

D. Colonoscopy with biopsy A colonoscopy with biopsy can be performed to determine the state of the colon's mucosal layers, presence of ulcerations, and level of cytologic development. Diagnosis of ulcerative colitis is made clinically with supportive findings on endoscopy, biopsy, and by negative stool examination for infectious causes. Multiple biopsies should be obtained to confirm the diagnosis. Option A: Computed tomography (CT) of the abdomen and pelvis is a diagnostic imaging test used to help detect diseases of the small bowel, colon, and other internal organs and is often used to determine the cause of unexplained pain. CT scanning is fast, painless, noninvasive and accurate. Option B: An abdominal x-ray or CT scan wouldn't provide the cytologic information necessary to diagnose which disease it is. Abdominal x-ray is a commonly performed diagnostic x-ray examination that produces images of the organs in the abdominal cavity including the stomach, liver, intestines, and spleen. Option C: A barium swallow doesn't involve the intestine. The barium swallow study, also known as a barium esophagogram or esophagram, is a contrast-enhanced radiographic study commonly used to assess structural characteristics of the entire esophagus. It may be used for the diagnosis of a wide range of pathologies including esophageal motility disorders, strictures, and perforations.

One of your patients is receiving digitalis orally and is also to receive an antacid at the same time. Your most appropriate action, based on the pharmacokinetics of antacids, is to: A. Delay the digitalis for 1 to 2 hours until the antacid is absorbed. B. Give the antacid at least 2 to 4 hours before administering the digitalis. C. Administer both medications as ordered and documented in the nurse's notes. D. Contact the physician regarding the drug interaction and request a change in the time of dosing of the drugs.

D. Contact the physician regarding the drug interaction and request a change in the time of dosing of the drugs. When antacids are taken with acidic drugs (for example, digoxin Digitek, phenytoin, chlorpromazine, Thorazine,isoniazid,), they cause the absorption of the acidic drugs to be decreased, which causes low blood concentrations of the drugs, which ultimately results in reduced effects of the drugs. Option A: Avoid taking antacid medicines for indigestion or heartburn within two hours of taking a digoxin dose, because they can stop the digoxin from being absorbed properly if taken too close together. Option B: Numerous pharmacological agents have been shown to produce clinically significant pharmacokinetic interactions with digoxin. Drugs that reduce digoxin absorption include the antacids aluminum hydroxide, magnesium hydroxide, and magnesium trisilicate, the antidiarrheals kaolin and pectin, the hypocholesterolemic agent cholestyramine and the chemotaxins cyclophosphamide, vincristine, and bleomycin. Option C: Alteration of the timing of the administration of digoxin from that of a second drug (e.g. as with liquid antacids, kaolin-pectin, or meals of high fiber content), or alteration of the dose of digoxin based on the anticipated change in kinetics (as with spironolactone, quinidine or medications altering gut function or motility), can improve the clinical effectiveness of digoxin therapy.

Donald is a 61 y.o. man with diverticulitis. Diverticulitis is characterized by: A. Periodic rectal hemorrhage. B. Hypertension and tachycardia. C. Vomiting and elevated temperature. D. Crampy and lower left quadrant pain and low-grade fever.

D. Crampy and lower left quadrant pain and low-grade fever. One sign of acute diverticulitis is crampy lower left quadrant pain. A low-grade fever is another common sign. Clinical manifestation of acute diverticulitis varies depending on the severity of the disease. Patients with uncomplicated diverticulitis typically present with left lower quadrant abdominal pain, reflecting that propensity of left-sided disease in Western nations. However, patients of Asian descent present with predominantly right-sided abdominal pain. Option A: On physical examination, tenderness to palpation over the area of inflammation is almost always present due to irritation of the peritoneum. A mass may be felt in approximately 20% of patients if an abscess is present. Bowel sounds are usually hypoactive but can be normoactive. Option B: Patients can present with peritoneal signs (rigidity, guarding, rebound tenderness) with bowel wall perforation. On the other hand, fever is almost always present, but hypotension and shock are uncommon. Option C: Change in bowel habits, either diarrhea (35%) or constipation (50%), can be associated with abdominal pain. Patients may also experience nausea and vomiting, possibly secondary to bowel obstruction. Fever is not uncommon in patients with abscesses and perforation.

A female client with hepatitis C develops liver failure and GI hemorrhage. The blood products that would most likely bring about hemostasis in the client are: A. Whole blood and albumin. B. Platelets and packed red blood cells. C. Fresh frozen plasma and whole blood. D. Cryoprecipitate and fresh frozen plasma.

D. Cryoprecipitate and fresh frozen plasma. The liver is vital in the synthesis of clotting factors, so when it's diseased or dysfunctional, as in hepatitis C, bleeding occurs. Treatment consists of administering blood products that aid clotting. These include fresh frozen plasma containing fibrinogen and cryoprecipitate, which have most of the clotting factors. Option A: Although administering whole blood, albumin, and packed cells will contribute to hemostasis, those products aren't specifically used to treat hemostasis. Whole blood is often divided into component parts for ease of storage and administration. These typically include Red Blood Cells (RBC), Platelets (thrombocytes), and Plasma. Option B: Platelets are helpful, but the best answer is cryoprecipitate and fresh frozen plasma. Platelets are typically given when patients have a low platelet count (thrombocytopenia) or have platelets that are dysfunctional, due to medications or other acquired or inherited lesions. Option C: The indications for whole blood and blood component transfusion consist of increasing hemoglobin and oxygenation of tissues, maintaining adequate blood volume to avoid ischemia and hypovolemic shock, and to reconstitute platelets, coagulation factors, and other plasma proteins to a functional status.

What assessment finding of a patient with acute pancreatitis would indicate a bluish discoloration around the umbilicus? A. Grey-Turner's sign B. Homan's sign C. Rovsing's sign D. Cullen's sign

D. Cullen's sign Cullen's sign is associated with pancreatitis when a hemorrhage is suspected. Cullen's sign is described as superficial edema with bruising in the subcutaneous fatty tissue around the periumbilical region. It is also known as periumbilical ecchymosis. It is most often recognized as a result of hemorrhagic pancreatitis. The sign can take 2-3 days before appearance and may be used as a clinical sign to help the diagnosis of acute pancreatitis. Option A: Grey-Turner's sign is ecchymosis in the flank area suggesting retroperitoneal bleed. Grey Turner's sign is an uncommon subcutaneous manifestation of intra-abdominal pathology that manifests as ecchymosis or discoloration of the flanks. Classically it correlates with severe acute necrotizing pancreatitis, often in association with Cullen's sign (periumbilical ecchymosis). Option B: Homan's sign is called pain elicited by the dorsiflexion of the foot and suggests deep vein thrombosis. Homan's sign test also called dorsiflexion sign test is a physical examination procedure that is used to test for deep vein thrombosis (DVT). A positive Homan's sign in the presence of other clinical signs may be a quick indicator of DVT. Clinical evaluation alone cannot be relied on for patient management, but when carefully performed, it remains useful in determining the need for additional testing (like D-dimer test, ultrasonography, multidetector helical computed axial tomography (CT), and pulmonary angiography). Option C: Rovsing's sign is associated with appendicitis when pain is felt with pressure at McBurney's point. Rovsing's sign is a clinical finding that is indicative of acute appendicitis (the inflammation and possible infection of the appendix). A positive Rovsing's sign is characterized by right lower abdominal pain upon palpation of the left side of the lower abdomen

Katrina is diagnosed with lactose intolerance. To avoid complications with lack of calcium in the diet, which food should be included in the diet? A. Fruit B. Whole grains C. Milk and cheese products D. Dark green, leafy vegetables

D. Dark green, leafy vegetables Dark green, leafy vegetables are rich in calcium. Vegetables high in calcium include rhubarb, spinach, broccoli, and certain greens like kale. For example, one cup of cooked spinach has about 250 mg of calcium. Other foods for a lactose intolerance diet include pinto beans and calcium-fortified orange juice. Option A: Keep in mind that lactose-free products made from milk should be avoided by those with a dairy allergy, as they may contain milk proteins like casein or whey. Many healthy foods can easily fit into a lactose-free diet, including fruits, vegetables, whole grains, nuts, seeds, and legumes. Option B: Most fruits, vegetables, grains, and meats are lactose-free. These foods may contain lactose if prepared with milk-based ingredients (e.g. cream sauces, cheese sauces, bread made with milk, breaded and battered meats, etc). Option C: Fermented cheeses have less lactose than other dairy products, and the patient may be able to tolerate them in small amounts. They are worth including in a lactose intolerance diet because they are good sources of calcium and protein.

The nurse is assessing a male client 24 hours following a cholecystectomy. The nurse noted that the T-tube has drained 750 mL of green-brown drainage since the surgery. Which nursing intervention is appropriate? A. Clamp the T-tube B. Irrigate the T-tube C. Notify the physician D. Document the findings

D. Document the findings. Following cholecystectomy, drainage from the T-tube is initially bloody and then turns to a greenish-brown color. The drainage is measured as output. The amount of expected drainage will range from 500 to 1000 mL/day. The nurse would document the output. The fluid may appear bloody for the first day or 2. The color will eventually be golden yellow or greenish, depending on exactly where the catheter is inside the body. Option A: The doctor may order the t-tube to be clamped at times so bile can drain to the duodenum so fats can be digested during meal times. If choledocholithiasis persists, the T-tube can be clamped to promote stone passage. If signs or symptoms of cholangitis occur, the tube can be unclamped and repeat imaging is obtained. Option B: The client will need to flush the catheter with normal saline twice a day. If the doctor instructed to flush with less than 10 mL, squirt the extra saline out before connecting the syringe. Push the plunger of the syringe to push 1/3 of the normal saline into the catheter, and then pause. Push in another 1/3 of the normal saline, and pause again. Push in the rest of the normal saline into the catheter. Never pull back on the plunger. Option C: Notify the physician if the drainage is more than 500 mL/day. Watch for extremely thick, bad-smelling drainage with a fever or extremely bloody like bright red blood that looks fresh. Assess how well the patient tolerated the t-tube being clamped. If the patient develops abdominal pain, nausea, vomiting, etc. unclamp it and notify the physician.

A 52-year-old man was referred to the clinic due to increased abdominal girth. He is diagnosed with ascites by the presence of a fluid thrill and shifting dullness on percussion. After administering diuretic therapy, which nursing action would be most effective in ensuring safe care? A. Measuring serum potassium for hyperkalemia B. Assessing the client for hypervolemia C. Measuring the client's weight weekly D. Documenting precise intake and output

D. Documenting precise intake and output. For the client with ascites receiving diuretic therapy, careful intake and output measurement are essential for safe diuretic therapy. Diuretics lead to fluid losses, which if not monitored closely and documented, could place the client at risk for serious fluid and electrolyte imbalances. The most common adverse effect for any diuretic is mild hypovolemia, which can lead to transient dehydration and increased thirst. When there is an over-treatment with a diuretic, this could lead to severe hypovolemia, causing hypotension, dizziness, and syncope. Option A: Hypokalemia, not hyperkalemia, commonly occurs with diuretic therapy. Hypokalemia and metabolic alkalosis can occur with both loop and thiazide diuretics but are more common with loop diuretics. Loop agents increase distal Na+ delivery at macula densa and cause volume depletion, both of which indirectly activate the RAAS pathway. Option B: Because urine output increases, a client should be assessed for hypovolemia, not hypervolemia. Acid-base disorders usually accompany the electrolyte derangement due to their close association with their reabsorption in the renal tubules. Metabolic disturbances can lead to derangement of glucose, uric acid, or lipid levels with certain diuretics and are individually discussed. Option C: Weights are also an accurate indicator of fluid balance. However, for this client, weights should be obtained daily, not weekly. Diuretic treatment calls for careful assessment of extracellular fluid volume, urine output, electrolyte levels in plasma and urine, body weight, acid-base status, serum glucose, and BP regularly with particular emphasis on patients with cardiovascular, hepatic, renal, or metabolic disorders and in the elderly individuals.

Which of the following tests can be used to diagnose ulcers? A. Abdominal x-ray B. Barium swallow C. Computed tomography (CT) scan D. Esophagogastroduodenoscopy (EGD)

D. Esophagogastroduodenoscopy (EGD) The EGD can visualize the entire upper GI tract as well as allow for tissue specimens and electrocautery if needed. Esophagogastroduodenoscopy (EGD) is a diagnostic endoscopic procedure that includes visualization of the oropharynx, esophagus, stomach, and proximal duodenum. It is one of the most common procedures that a gastroenterologist performs. Option A: Abdominal x-ray uses a very small dose of ionizing radiation to produce pictures of the inside of the abdominal cavity. It is used to evaluate the stomach, liver, intestines, and spleen and may be used to help diagnose unexplained pain, nausea, or vomiting. Option B: The barium swallow could locate a gastric ulcer. The barium swallow study, also known as a barium esophagogram or esophagram, is a contrast-enhanced radiographic study commonly used to assess structural characteristics of the entire esophagus. It may be used for the diagnosis of a wide range of pathologies including esophageal motility disorders, strictures, and perforations. Option C: A CT scan isn't useful in the diagnosis of an ulcer. The CT scan is essentially an X-ray study, where a series of rays are rotated around a specified body part, and computer-generated cross-sectional images are produced. The advantage of these tomographic images compared to conventional X-rays is that they contain detailed information of a specified area in cross-section, eliminating the superimposition of images, which provides a tremendous advantage over plain films.

Which of the following diagnostic tests should be performed annually over age 50 to screen for colon cancer? A. Abdominal CT scan B. Abdominal x-ray C. Colonoscopy D. Fecal occult blood test

D. Fecal occult blood test Surface blood vessels of polyps and cancers are fragile and often bleed with the passage of stools. A fecal occult blood test is used to find blood in the feces, or stool, which can be a sign of polyps or cancer. A positive test, meaning that blood is found in the feces, can be from causes other than a colon polyp or cancer, including bleeding in the stomach or upper GI tract and even eating rare meat or other foods. There are 2 types of tests: guaiac (FOBT) and immunochemical (FIT). Polyps and cancers do not bleed continually, so FOBT must be done on several stool samples each year and should be repeated every year. Even then, this screening test provides a fairly small reduction in deaths from colorectal cancer, around 30% if done yearly and 18% if done every other year. Option A: Abdominal CT scan can help establish tumor size and metastasis. Ct colonography, sometimes called virtual colonoscopy, is a screening method being studied in some centers. It requires interpretation by a skilled radiologist to provide the best results. A radiologist is a doctor who specializes in obtaining and interpreting medical images. CT colonography may be an alternative for people who cannot have a standard colonoscopy due to the risk of anesthesia, which is medication to block the awareness of pain, or if a person has a blockage in the colon that prevents a full examination. Option B: Abdominal x-ray is a commonly performed diagnostic x-ray examination that produces images of the organs in the abdominal cavity including the stomach, liver, intestines, and spleen. When an abdominal x-ray is performed to provide pictures of the kidneys, ureters, and bladder, it's called a KUB x-ray. Option C: A colonoscopy can help locate a tumor as well as polyps, which can be removed before they become malignant. A colonoscopy allows the doctor to look inside the entire rectum and colon while a patient is sedated. A flexible, lighted tube called a colonoscope is inserted into the rectum and the entire colon to look for polyps or cancer. During this procedure, a doctor can remove polyps or other tissue for examination. The removal of polyps can also prevent colorectal cancer.

Your patient has a retractable gastric peptic ulcer and has had a gastric vagotomy. Which factor increases as a result of vagotomy? A. Peristalsis B. Gastric acidity C. Gastric motility D. Gastric pH

D. Gastric pH If the vagus nerve is cut as it enters the stomach, gastric acid secretion is decreased, but intestinal motility is also decreased and gastric emptying is delayed. Because gastric acids are decreased, gastric pH increases. The postoperative complications of truncal vagotomy are well documented. Resection of the vagal nerve trunks above the celiac and hepatic branches (differentiates TV versus SV) leads to parasympathetic denervation of the pylorus, liver, biliary tree, pancreas, and small and large intestines. Option A: The stomach loses the vagally mediated receptive relaxation. This leads to an increased intragastric pressure causing an increased emptying of liquids. The pylorus does not relax effectively, and a decrease in solid food emptying is seen. Option B: Gastrin is secreted by the G cells that are mainly located in the stomach antrum and pylorus. Acetylcholine is released in response to parasympathetic stimulation, which travels in the fibers of the vagus nerves. Histamine is released by the enterochromaffin-like cells. Option C: Gastric phase begins once food enters the stomach. It is stimulated by proximal stomach distension (vagally mediated). It is absent in patients with a vagotomy. The gastric phase is stimulated by amino acids and peptides, leading to G cell activation. It accounts for 60% of total acid production.

Your patient, post-op drainage of a pelvic abscess secondary to diverticulitis, begins to cough violently after drinking water. His wound has ruptured and a small segment of the bowel is protruding. What's your priority? A. Ask the patient what happened, call the doctor, and cover the area with a water-soaked bed sheet. B. Obtain vital signs, call the doctor, and obtain emergency orders. C. Have a CAN hold the wound together while you obtain vital signs, call the doctor and flex the patient's knees. D. Have the doctor called while you remain with the patient, flex the patient's knees, and cover the wound with sterile towels soaked in sterile saline solution.

D. Have the doctor called while you remain with the patient, flex the patient's knees, and cover the wound with sterile towels soaked in sterile saline solution. Call for help but stay with the patient. Tell the person who responds to notify the surgeon immediately. Lower the bed until it is flat or no steeper than 20 degrees. Have the patient bend his knees to reduce abdominal muscle tension. Soak sterile towels with sterile saline or use pre moistened sterile dressings. Option A: Using a water-soaked bed sheet to cover the area may lead to severe infection. Preventive measures to avoid wound dehiscence and wound evisceration include client coaching and teaching the client how to splint their incisional area when coughing, sneezing, vomiting and when doing planned, routine coughing and deep breathing exercises postoperatively. Option B: In some situations, a surgical incision may open during the recovery process, allowing organs to be visible or even spill out of the body. This is obviously a serious medical condition that should be treated as soon as possible. Vital signs may be taken after the surgeon has arrived or the patient's wound has been securely covered. Option C: It is not advisable to touch an evisceration. Covering the area with sterile, saline-soaked dressings could prevent an infection. Dehiscence and evisceration can be a life-threatening emergency; do not leave the client immediately call for help and, using a clean, sterile towel or sterile saline dampened dressing, cover the wound. Under no circumstance should reinserting the organs be attempted. Maintain light pressure on the wound and monitor the client for shock until help arrives.

A patient who underwent abdominal surgery now has a gaping incision due to delayed wound healing. Which method is correct when you irrigate a gaping abdominal incision with sterile normal saline solution, using a piston syringe? A. Rapidly instill a stream of irrigating solution into the wound. B. Apply a wet-to-dry dressing to the wound after the irrigation. C. Moisten the area around the wound with normal saline solution after the irrigation. D. Irrigate continuously until the solution becomes clear or all of the solution is used.

D. Irrigate continuously until the solution becomes clear or all of the solution is used. To wash away tissue debris and drainage effectively, irrigate the wound until the solution becomes clear of all the solution is used. Irrigation helps the wound to heal properly from the inside out; it helps prevent surface healing over an abscess pocket or infected tract. Continue to irrigate the wound until you have administered the prescribed amount of solution or until the solution returned is clear. Note the amount of solution administered. Remove and discard the catheter and syringe in the appropriate container. Option A: Gently instill a slow, steady stream of irrigating solution into the wound until the syringe empties. Make sure the solution flows from the clean tissue to the dirty area of the wound to prevent contamination of clean tissue by exudate. Be sure the solution reaches all areas of the wound. Option B: Keep the patient positioned to allow further wound drainage into the basin. Cleanse the area around the wound to help prevent skin breakdown and infection. Gently pack the wound, if ordered, and/or apply dressing. Option C: Observe for wound size including length, width, and depth; drainage characteristics including type, amount, color, and odor; wound bed tissue type/color including necrotic, slough, eschar, granulating, clean, non-granulating, epithelial; and symptoms of infection including redness, swelling, pain, discharge or increased temperature.

When counseling a client in ways to prevent cholecystitis, which of the following guidelines is mostimportant? A. Eat a low-protein diet. B. Eat a low-fat, low-cholesterol diet. C. Limit exercise to 10 minutes/day. D. Keep weight proportionate to height.

D. Keep weight proportionate to height. Obesity is a known cause of gallstones, and maintaining a recommended weight will help protect against gallstones. Excessive dietary intake of cholesterol is associated with the development of gallstones in many people. Being overweight makes one more likely to develop gallstones. To achieve a healthy weight, reduce calories and increase physical activity. Maintain a healthy weight by continuing to eat well and exercise. Option A: Dietary protein isn't implicated in cholecystitis. Diets high in fat and low in fiber may increase the risk of gallstones. To lower the risk, choose a diet high in fruits, vegetables, and whole grains. Option B: Liquid protein and low-calorie diets (with rapid weight loss of more than 5 lb (2.3kg)per week) are implicated as the cause of some cases of cholecystitis. Rapid weight loss can increase the risk of gallstones. If the client needs to lose weight, aim to lose 1 or 2 pounds (0.5 to about 1 kilogram) a week. Option C: Regular exercise (30 minutes/three times a week) may help reduce weight and improve fat metabolism. Reducing stress may reduce bile production, which may also indirectly decrease the chances of developing cholecystitis.

Anna is 45 y.o. and has a bleeding ulcer. Despite multiple blood transfusions, her HGB is 7.5g/dl and HCT is 27%. Her doctor determines that surgical intervention is necessary and she undergoes partial gastrectomy. Postoperative nursing care includes: A. Giving pain medication Q6H. B. Flushing the NG tube with sterile water. C. Positioning her in high Fowler's position. D. Keeping her NPO until the return of peristalsis.

D. Keeping her NPO until the return of peristalsis. After surgery, she remains NPO until peristaltic activity returns. This decreases the risk for abdominal distention and obstruction. Caution the patient to limit the intake of ice chips. Excessive intake of ice produces nausea and can wash out electrolytes via the NG tube. Option A: Monitor tolerance to fluid and food intake, noting abdominal distension, reports of increased pain, cramping, nausea, and vomiting. Complications of paralytic ileus, obstruction, delayed gastric emptying, and gastric dilation may occur, possibly requiring reinsertion of the NG tube. Option B: Maintain patency of NG tube. Notify the physician if the tube becomes dislodged. Provides rest for GI tract during the acute postoperative phase until the return of normal function. The physician or surgeon may need to reposition the tube endoscopically to prevent injury to the operative area. Option C: Auscultate for resumption of bowel sounds and note passage of flatus. Peristalsis can be expected to return about the third postoperative day, signaling readiness to resume oral intake.

Jordin is a client with jaundice who is experiencing pruritus. Which nursing intervention would be included in the care plan for the client? A. Administering vitamin K subcutaneously B. Applying pressure when giving I.M. injections C. Decreasing the client's dietary protein intake D. Keeping the client's fingernails short and smooth

D. Keeping the client's fingernails short and smooth The client with pruritus experiences itching, which may lead to skin breakdown and possibly infection from scratching. Keeping his fingernails short and smooth helps prevent skin breakdown and infection from scratching. Encourage the patient to adopt skin care routines to decrease skin irritation. One of the first steps in the management of pruritus is promoting healthy skin and healing of skin lesions. Option A: Administering vitamin K subcutaneously is important if the client develops bleeding problems. Instruct the client to bathe or shower using lukewarm water and mild soap or nonsoap cleansers. Long bathing or showering in hot water causes drying of the skin and can aggravate itching through vasodilation. Option B: Applying pressure when giving I.M. injections is important if the client develops bleeding problems. Instruct the client to allow the skin to air dry or gently pat the skin dry after bathing. Avoid rubbing or brisk drying. Rubbing the skin with a towel can irritate the skin and exacerbate the itch-scratch cycle. Option C: Decreasing the client's dietary intake is appropriate if the client's ammonia levels are increased. The patient who scratches the skin to relieve intense itching may cause open skin lesions with an increased risk for infection. Characteristic patterns associated with scratching include reddened papules that run together and become confluent, widespread erythema, and scaling or lichenification.

While palpating a female client's right upper quadrant (RUQ), the nurse would expect to find which of the following structures? A. Sigmoid colon B. Appendix C. Spleen D. Liver

D. Liver The RUQ contains the liver, gallbladder, duodenum, head of the pancreas, hepatic flexure of the colon, portions of the ascending and transverse colon, and a portion of the right kidney. Begin palpation over the right lower quadrant, near the anterior iliac spine. Palpate for the liver with one or two hands palm down moving upward 2-3 cm at a time towards the lower costal margin. Option A: The sigmoid colon is located in the left lower quadrant. The 40cm long sigmoid colon is located in the left lower quadrant of the abdomen, extending from the left iliac fossa to the level of the S3 vertebra. Option B: The appendix is located in the right lower quadrant. The appendix sits at the junction of the small intestine and large intestine. It's a thin tube about four inches long. Normally, the appendix sits in the lower right abdomen. Option C: The spleen is located in the left upper quadrant. The spleen is a fist-sized organ in the upper left side of the abdomen, next to the stomach and behind the left ribs. It's an important part of the immune system, but one can survive without it.

A client presents to the emergency room, reporting that he has been vomiting every 30 to 40 minutes for the past 8 hours. Frequent vomiting puts him at risk for which of the following? A. Metabolic acidosis with hyperkalemia B. Metabolic acidosis with hypokalemia C. Metabolic alkalosis with hyperkalemia D. Metabolic alkalosis with hypokalemia

D. Metabolic alkalosis with hypokalemia Gastric acid contains large amounts of potassium, chloride, and hydrogen ions. Excessive loss of these substances, such as from vomiting, can lead to metabolic alkalosis and hypokalemia. Vomiting or nasogastric (NG) suction generates metabolic alkalosis by the loss of gastric secretions, which are rich in hydrochloric acid (HCl). Whenever a hydrogen ion is excreted, a bicarbonate ion is gained in the extracellular space. Option A: Hyperkalemia can be the direct cause of metabolic acidosis from its effects on multiple components of renal ammonia metabolism. The first major finding in these studies is that hyperkalemia itself causes reversible metabolic acidosis by inhibiting ammonia excretion. Option B: The most common cause for hypokalemia and metabolic acidosis is GI loss (eg, diarrhea, laxative use). Other less common etiologies include renal loss of potassium secondary to RTA or salt-wasting nephropathy. The urine pH, the urine AG, and the urinary K+ concentration can distinguish these conditions. Option C: The first clue to metabolic alkalosis is often an elevated bicarbonate concentration that is observed when serum electrolyte measurements are obtained. Remember that an elevated serum bicarbonate concentration may also be observed as a compensatory response to primary respiratory acidosis. However, a bicarbonate concentration greater than 35 mEq/L is almost always caused by metabolic alkalosis.

Which of the following tasks should be included in the immediate postoperative management of a client who has undergone gastric resection? A. Monitoring gastric pH to detect complications. B. Assessing for bowel sounds. C. Providing nutritional support. D. Monitoring for symptoms of hemorrhage.

D. Monitoring for symptoms of hemorrhage. The client should be monitored closely for signs and symptoms of hemorrhage, such as bright red blood in the nasogastric tube suction, tachycardia, or a drop in blood pressure. Identify signs and symptoms requiring medical evaluation such as persistent nausea and vomiting or abdominal fullness; weight loss; diarrhea; foul-smelling fatty or tarry stools; bloody or coffee-ground vomitus or presence of bile, fever. Instruct the patient to report changes in pain characteristics. Option A: Gastric pH may be monitored to evaluate the need for histamine-2 receptor antagonists. Caution the patient to read labels and avoid products containing ASA, ibuprofen. This can cause gastric irritation and bleeding. Review medication purpose, dosage, and schedule, and possible side effects. Option B: Bowel sounds may not return for up to 72 hours postoperatively. Auscultate for resumption of bowel sounds and note passage of flatus. Peristalsis can be expected to return about the third postoperative day, signaling readiness to resume oral intake. Option C: Nutritional needs should be addressed soon after surgery. Monitor tolerance to fluid and food intake, noting abdominal distension, reports of increased pain, cramping, nausea, and vomiting. Avoid milk and high-carbohydrate foods in the diet because this may trigger dumping syndrome.

The nurse is reviewing the physician's orders written for a client admitted with acute pancreatitis. Which physician order would the nurse if noted on the client's chart? A. NPO status B. Insert a nasogastric tube C. An anticholinergic medication D. Morphine for pain

D. Morphine for pain Meperidine (Demerol) rather than morphine is the medication of choice because morphine can cause spasms in the sphincter of Oddi. Meperidine is usually effective in relieving pain and may be preferred over morphine, which can have a side effect of biliary-pancreatic spasms. Paravertebral block has been used to achieve prolonged pain control. Option A: Maintain NPO status and gastric suctioning in acute phase; prevents stimulation and release of pancreatic enzymes (secretin), released when chyme and HCl enter the duodenum. Administer hyperalimentation and lipids, if indicated. IV administration of calories, lipids, and amino acids should be instituted before nutrition and nitrogen depletion are advanced. Option B: Oral feedings given too early in the course of illness may exacerbate symptoms. Loss of pancreatic function and reduced insulin production may require the initiation of a diabetic diet. Option C: Provide medium-chain triglycerides (MCTs) (MCT, Portagen). MCTs are elements of enteral feedings (NG or J-tube) that provide supplemental calories and nutrients that do not require pancreatic enzymes for digestion and absorption.

Which clinical manifestation would the nurse expect a client diagnosed with acute cholecystitis to exhibit? A. Jaundice, dark urine, and steatorrhea B. Acute right lower quadrant (RLQ) pain, diarrhea, and dehydration C. Ecchymosis petechiae, and coffee-ground emesis D. Nausea, vomiting, and anorexia

D. Nausea, vomiting, and anorexia Acute cholecystitis is an acute inflammation of the gallbladder commonly manifested by the following: anorexia, nausea, and vomiting; biliary colic; tenderness and rigidity the right upper quadrant (RUQ) elicited on palpation (e.g., Murphy's sign); fever; fat intolerance; and signs and symptoms of jaundice. Option A: Jaundice, dark urine, and steatorrhea are clinical manifestations of the icteric phase of hepatitis. Patients in this phase present with dark-colored urine and pale-colored stool. Some patients develop jaundice and right upper quadrant pain with liver enlargement. Option B: Cases of chronic cholecystitis present with progressing right upper quadrant abdominal pain with bloating, food intolerances (especially greasy and spicy foods), increased gas, nausea, and vomiting. Pain in the mid back or shoulder may also occur. Option C: Ecchymosis, petechiae, and coffee-ground emesis are clinical manifestations of esophageal bleeding. The coffee-ground appearance indicates old bleeding. The clinical presentation can vary but should be well-characterized. Hematemesis is the overt bleeding with vomiting of fresh blood or clots. Melena refers to dark and tarry-appearing stools with a distinctive smell. The term "coffee-grounds" describes gastric aspirate or vomitus that contains dark specks of old blood.

Which of the following substances is most likely to cause gastritis? A. Milk B. Bicarbonate of soda, or baking soda C. Enteric-coated aspirin D. Nonsteroidal anti-inflammatory drugs

D. Nonsteroidal anti-inflammatory drugs NSAIDs are a common cause of gastritis because they inhibit prostaglandin synthesis. When NSAIDs irritate the gastric mucosa, they weaken the resistance to acid, causing gastritis, ulcers, bleeding, or perforation. The damage ranges from superficial injury to single or multiple ulcers, some of which may bleed. Suppression of prostaglandin synthesis can occur systemically with both oral and parenteral NSAID therapy. The antiplatelet activity of some NSAIDs in low doses may cause bleeding from preexisting ulcers Option A: Milk, once thought to help gastritis, has little effect on the stomach mucosa. Studies on certain ingredients found in high concentrations in milk, such as calcium and amino acids, have suggested that milk may be harmful to ulcer patients because of its potential to directly stimulate acid secretion. Therapeutic endeavors have therefore aimed at reducing gastric acid secretion or neutralizing its effect. Option B: Bicarbonate of soda, or baking soda, may be used to neutralize stomach acid, but it should be used cautiously because it may lead to metabolic acidosis. The Canadian Society of Intestinal Research reminds people that baking soda is a temporary solution to acid reflux. They advise people that sodium bicarbonate is available from a pharmacy in the form of tablets and effervescent powder. Option C: ASA with enteric coating shouldn't contribute significantly to gastritis because the coating limits the aspirin's effect on the gastric mucosa. Enteric-coated aspirin is designed to resist dissolving and being absorbed in the stomach. As such, enteric-coated aspirin passes into the small intestine, where it's absorbed into the bloodstream. The purported goal is to prevent stomach ulcers and bleeding that can sometimes occur with aspirin use.

Which rationale supports explaining the placement of an esophageal tamponade tube in a client who is hemorrhaging? A. Allowing the client to help insert the tube B. Beginning teaching for home care C. Maintaining the client's level of anxiety and alertness D. Obtaining cooperation and reducing fear

D. Obtaining cooperation and reducing fear An esophageal tamponade tube would be inserted in critical situations. Typically, the client is fearful and highly anxious. The nurse, therefore, explains the placement to help obtain the client's cooperation and reduce his fear. Option A: The client would not be helping to insert the tube. Option B: This type of tube is used only short-term and is not indicated for home use. The tube is large and uncomfortable. Option C: A client's anxiety should be decreased, not maintained, and depending on the degree of hemorrhage, the client may not be alert.

The pain of a duodenal ulcer can be distinguished from that of a gastric ulcer by which of the following characteristics? A. Early satiety B. Pain on eating C. Dull upper epigastric pain D. Pain on an empty stomach]

D. Pain on an empty stomach Pain on an empty stomach is relieved by taking foods or antacids. The location of the disease can also be differentiated based on symptoms. The pain associated with duodenal ulcers improves after meals, while the pain associated with gastric ulcers generally intensifies after meals. The other symptoms are those of a gastric ulcer. Option A: Early satiety is seen in gastric ulcers. The mechanism by which H. pylori predisposes individuals is unclear. However, the thinking is that H. pylori colonization and persistent inflammation lead to the weakening of the mucosal surface layer causing it to be vulnerable to exposure to gastric acid. Option B: The typical presentation of a patient with gastric ulcers is epigastric pain that is worse with eating. It often correlates with mild nausea and early satiety. They often describe this pain as a sharp or burning type of pain that typically doesn't radiate. Option C: The most common finding on the physical exam is epigastric tenderness. These symptoms may continue for weeks or months before patients seek medical help. Patients may present with upper GI bleeding.

Peritonitis can occur as a complication of: A. Septicemia B. Multiple organ failure C. Hypovolemic shock D. Peptic ulcer disease

D. Peptic ulcer disease Perforation is a life-threatening complication of peptic ulcer disease and can result in peritonitis. Since the peritoneum completely covers the stomach, perforation of the wall creates a communication between the gastric lumen and the peritoneal cavity. If the perforation occurs acutely, there is no time for an inflammatory reaction to wall off the perforation, and the gastric content is free to enter the general peritoneal cavity, causing chemical peritonitis. Option A: Septic shock is a serious illness and despite all the advances in medicine, it still carries high mortality which can exceed 40%. Mortality does depend on many factors including the type of organism, antibiotic sensitivity, number of organs affected, and patient age. The more factors that match SIRS, the higher the mortality. Option B: The high mortality of patients with multiple organ failure provided a focus for the problems that ultimately led to death for many patients in the intensive care unit. The frequency of infection, sepsis, or inflammation in producing multiple organ failure led to clinical trials of so-called magic bullets for the treatment of patients with sepsis. Option C: Patients with volume depletion may complain of thirst, muscle cramps, and/or orthostatic hypotension. Severe hypovolemic shock can result in mesenteric and coronary ischemia that can cause abdominal or chest pain. Agitation, lethargy, or confusion may result from brain malperfusion.

During the initial assessment of a patient post-endoscopy, the nurse notes absent bowel sounds, tachycardia, and abdominal distention. The nurse would anticipate: A. Ischemic bowel B. Peritonitis C. Hypovolemic shock D. Perforated bowel

D. Perforated bowel Invasive diagnostic testing can cause perforated bowel. Perforation is widely recognized as one of the most serious complications of endoscopy of the lower gastrointestinal tract. The risk of perforation ranges from 0.027% to 0.088% for flexible sigmoidoscopy, from 0.016% to 0.2% for diagnostic colonoscopy, and up to 5% for therapeutic endoscopy. Option A: Ischemic bowel is usually not related. As the volume of both diagnostic and therapeutic endoscopic procedures increases, the absolute number of perforations will undoubtedly increase even with a relatively constant perforation rate. Option B: Peritonitis can be a complication after initial perforation. The risk factors contributing to perforation are well established in the literature. They include patient-related factors such as advanced age, female sex, diverticular disease, previous abdominal surgery, and colonic stricture in addition to therapeutic procedures such as endoscopic resection and dilation. Option C: Hypovolemic shock can occur if peritonitis is allowed to continue. If the peritoneum is weakened or injured, inflammation and infection can spread through the peritoneal cavity. Peristaltic action decreases, leading to bowel obstruction. Large amounts of fluid from the intravascular space move into the peritoneal cavity, causing hypovolemia and hemoconcentration.

The client who has undergone the creation of a colostomy has a nursing diagnosis of Disturbed body image. The nurse would evaluate that the client is making the most significant progress toward identified goals if the client: A. Watch the nurse empty the colostomy bag. B. Look at the ostomy site. C. Read the ostomy product literature. D. Practice cutting the ostomy appliance.

D. Practice cutting the ostomy appliance. The client is expected to have a body image disturbance after a colostomy. The client progresses through normal grieving stages to adjust to this change. The client demonstrates the greatest deal of acceptance when the client participates in the actual colostomy care. Each of the options represents an interest in colostomy care but is a passive activity. The correct option shows the client is participating in self-care. Option A: Provide opportunities for patient/SO to view and touch stoma, using the moment to point out positive signs of healing, normal appearance, and so forth. Remind the patient that it will take time to adjust, both physically and emotionally. Option B: Although integration of stoma into body image can take months or even years, looking at the stoma and hearing comments (made in a normal, matter-of-fact manner) can help the patient with this acceptance. Touching stoma reassures patient/SO that it is not fragile and that slight movements of stoma actually reflect normal peristalsis. Option C: Provide an opportunity for the patient to deal with ostomy through participation in self-care. Independence in self-care helps improve self-confidence and acceptance of the situation. Plan/schedule care activities with the patient. Promotes a sense of control and gives a message that a patient can handle the situation, enhancing self-concept.

Which phase of hepatitis would the nurse incur strict precautionary measures at? A. Icteric B. Non-icteric C. Post-icteric D. Pre-icteric

D. Pre-icteric Pre-icteric is the infective phase and precautionary measures should be strictly enforced. However, most patients are not always diagnosed during this phase. Nonspecific symptoms occur; they include profound anorexia, malaise, nausea and vomiting, a newly developed distaste for cigarettes (in smokers), and often fever or right upper quadrant abdominal pain. Urticaria and arthralgias occasionally occur, especially in HBV infection. Option A: During the icteric phase, precautionary measures should already be in place. After 3 to 10 days, the urine darkens, followed by jaundice. Systemic symptoms often regress, and patients feel better despite worsening jaundice. The liver is usually enlarged and tender, but the edge of the liver remains soft and smooth. Mild splenomegaly occurs in 15 to 20% of patients. Jaundice usually peaks within 1 to 2 weeks. Option B: There is no non-icteric phase. Some manifestations of acute hepatitis are virus-specific, but in general, acute infection tends to develop in predictable phases. Acute viral hepatitis is a common, worldwide disease that has different causes; each type shares clinical, biochemical, and morphologic features. The term acute viral hepatitis often refers to infection of the liver by one of the hepatitis viruses. Option C: During the post-icteric phase, precautionary measures should already be in place. During this 2- to 4-week period, jaundice fades. Appetite usually returns after the first week of symptoms. Acute viral hepatitis usually resolves spontaneously 4 to 8 weeks after symptom onset.

After an abdominal resection for colon cancer, Madeline returns to her room with a Jackson-Pratt drain in place. The purpose of the drain is to: A. Irrigate the incision with a saline solution. B. Prevent bacterial infection of the incision. C. Measure the amount of fluid lost after surgery. D. Prevent accumulation of drainage in the wound.

D. Prevent accumulation of drainage in the wound. A Jackson-Pratt drain promotes wound healing by allowing fluid to escape from the wound. JP drains are often placed in wounds during surgery to prevent the collection of fluid underneath the incision site. This is a closed, air-tight drainage system which operates by self-suction. The drain(s) promote healing by keeping excess pressure off the incision and decreasing the risk of infection. Option A: JP drains do not irrigate the incision with saline solution. The drain is sutured (stitched) in place at the skin at the site of insertion to promote stability. Clots in the tubing are expected as long as they do not interfere with the drainage collection. The drain(s) is left in place until the drainage is approximately 30 cc's or less (or 30 ml's, or 1 ounce) per drain for each of 2 consecutive days. Option B: It does not prevent bacterial infection. After surgery, there is continued oozing and shedding of cells and bodily fluids at the surgical site. The Jackson Pratt drain removes fluid and this removal of fluid speeds healing. Option C: The drain will automatically suction fluid out when the bulb is compressed. The bulb has to be compressed very well and the drain tab has to be closed in order for the suction to work. When the bulb can maintain its compressed shape, it is a sign that suction is in effect.

The nurse is assessing for stoma prolapse in a client with a colostomy. The nurse would observe which of the following if stoma prolapse occurred? A. Sunken and hidden stoma B. Dark- and bluish-colored stoma C. Narrowed and flattened stoma D. Protruding stoma

D. Protruding stoma A prolapsed stoma is one which the bowel protruded through the stoma. Prolapse is a complication associated more with colostomies than with ileostomies, and is more frequent in those with loop colostomies, particularly loop stomas located in the transverse colon, than with end stomas. (End stomas result from a complete cut through the intestine with the end pulled through the abdominal wall, while loop stomas result when an intestinal loop is pulled through the abdominal wall and an incision made into part of the loop.) Option A: A stoma retraction is characterized by sinking of the stoma. Stoma retraction is caused by excessive tension on the bowel or stoma placed at a poorly selected site. Stoma retraction can lead to skin irritation and improper fixation of the stoma appliance. In mildly symptomatic cases, a convex faceplate and a tight belt may be used to control leakage around the appliance. Option B: Ischemia of the stoma would be associated with a dusky or bluish color. Signs of ischemia usually arise within 24 hours. The stoma first appears edematous with bluish discoloration and then progresses to necrosis. A common cause of ischemia is an inadequate arterial blood supply secondary to damage to or an inappropriately divided vascular arcade supplying the left colon. Option C: A stoma with a narrowed opening at the level of the skin or fascia is said to be stenosed. Stoma stenosis is the narrowing or constriction of the stoma or its lumen. This condition may occur at the skin or fascial level of the stoma. Causes include hyperplasia, adhesions, sepsis, radiation of the intestine before stoma surgery, local inflammation, hyperkeratosis, and surgical technique.

During the procedure, the client begins to cough and has difficulty breathing. Which of the following is the appropriate nursing action? A. Quickly insert the tube B. Notify the physician immediately C. Remove the tube and reinsert when the respiratory distress subsides D. Pull back on the tube and wait until the respiratory distress subsides

D. Pull back on the tube and wait until the respiratory distress subsides During the insertion of a nasogastric tube, if the client experiences difficulty breathing or any respiratory distress, withdraw the tube slightly, stop the tubing advancement, and wait until the distress subsides. The most common indication for placement of a nasogastric tube is to decompress the stomach in the setting of distal obstruction. Less commonly, nasogastric tubes can be placed to administer medications or nutrition in patients who have a functional gastrointestinal tract but are unable to tolerate oral intake. Option A: Quickly inserting the tube is not an appropriate action because, in this situation, it may be likely that the tube has entered the bronchus. A common error when placing the tube is to direct the tube in an upward direction as it enters the nares; this will cause the tube to push against the top of the sinus cavity and cause increased discomfort. Option B: Notifying the physician immediately is unnecessary. If there is a great deal of difficulty in passing the tube, a helpful maneuver is to withdraw the tube and attempt again after a short break in the contralateral nares as the tube may have become coiled in the oropharynx or nasal sinus. Option C: Removing the tube is unnecessary. The tip should instead be directed parallel to the floor, directly toward the back of the patient's throat. At this time, the patient can be given a cup of water with a straw in it to sip from to help ease the passage of the tube. The tube should be advanced with firm, constant pressure while the patient is sipping.

The nurse is caring for a hospitalized client with a diagnosis of ulcerative colitis. Which finding, if noted on assessment of the client, would the nurse report to the physician? A. Bloody diarrhea B. Hypotension C. A hemoglobin of 12 mg/dL D. Rebound tenderness

D. Rebound tenderness Rebound tenderness may indicate peritonitis. During the physical exam, pertinent findings include fever and abdominal tenderness to palpation which usually is diffuse with wall rigidity in more septic presentations. Signs of peritonitis must be reported to the physician. It is important to conduct a thorough exam as certain thoracic or pelvic pathologies can mimic peritoneal irritation (empyema causing diaphragmatic irritation and cystitis/pyelonephritis causing peritoneum adjacent pain). Option A: Bloody diarrhea is expected to occur in ulcerative colitis. Ulcerative colitis's main symptom is bloody diarrhea, with or without mucus. Associated symptoms also include urgency or tenesmus, abdominal pain, malaise, weight loss, and fever, depending on the extent and severity of the disease. Option B: Because of the blood loss, the client may be hypotensive. In ulcerative colitis, bleeding can arise from the lining of the rectum or large intestine, and this blood can be visible in the stool. The bleeding generally comes from the ulcers that have formed in the lining of the large intestine or rectum. Option C: Because of the blood loss, the hemoglobin level may be lower than normal. It may also be necessary to treat the loss of blood that has happened. If the client developed anemia from blood loss, he may need to supplement with iron, folic acid, or vitamin B12, depending on what your health provider says. In serious cases of blood loss, a blood transfusion might be required.

A client with a peptic ulcer is scheduled for a vagotomy. The client asks the nurse about the purpose of this procedure. The nurse tells the client that the procedure: A. Decreases food absorption in the stomach. B. Heals the gastric mucosa. C. Halts stress reactions. D. Reduces the stimulus to acid secretions.

D. Reduces the stimulus to acid secretions. A vagotomy, or cutting the vagus nerve, is done to eliminate parasympathetic stimulation of gastric secretion. A vagotomy is a type of surgery that removes all or part of the vagus nerve. This nerve runs from the bottom of the brain, through the neck, and along the esophagus, stomach, and intestines in the gastrointestinal (GI) tract. Option A: The indications for vagotomy are few with the advancements of medical therapy. Generally, acid-reducing operations are reserved for complicated ulcer disease in a stable patient who has failed maximum medical therapy. The type of surgery performed depends on the type of ulcer (duodenal versus gastric), the complication of PUD (bleeding, perforation, obstruction, intractability), and the location of the ulcer (types I to V gastric ulcers as described by the Modified Johnson Classification system). Option B: The relevant physiology revolves around the mechanisms relating to stomach acid secretion. Intraluminal gastric acid is released by the parietal cells, mainly located in the body of the stomach. Parietal cells are stimulated via 3 mechanisms: gastrin, acetylcholine, and histamine. All 3 mechanisms activate the hydrogen-potassium ATPase-releasing hydrogen ion into the stomach lumen. Option C: Vagotomy was once commonly performed to treat and prevent PUD; however, with the availability of excellent acid secretion control with H2-receptor antagonists, proton pump inhibitors, and anti-Helicobacter pylori medications, the need for surgical management of this condition has greatly decreased.

Which of the following therapies is not included in the medical management of a client with peritonitis? A. Broad-spectrum antibiotics B. Electrolyte replacement C. I.V. fluids D. Regular diet

D. Regular diet The client with peritonitis usually isn't allowed anything orally until the source of peritonitis is confirmed and treated. In bacterial peritonitis, the type of antibiotic therapy administered depends on the infecting organism. Keep the patient N.P.O., administer supplemental parenteral fluids and electrolytes as prescribed, and administer medication to manage pain as ordered. Insert a nasogastric (NG) tube to decompress the bowel. Option A: The client also requires broad-spectrum antibiotics to combat the infection. The primary focus of management is the identification and targeted treatment of offending agent(s) via antibiotics and/or surgical intervention. The non-operative measure includes broad-spectrum antibiotic administration with appropriate stewardship, tailoring the regimen to accomplish increased efficacy targeting the identified microorganisms. Option B: I.V. fluids are given to maintain hydration and hemodynamic stability and to replace electrolytes. Enteral nutrition is preferred to intravenous as soon as possible. The use of low-dose steroids for 7 days is indicated if the state of shock persists despite adequate resuscitation, or if there is poor response to vasopressors or adrenal insufficiency. Option C: Crystalloids are used, and a patient may easily require three to six liters in the first hours, depending on the state of the patient and the illness. Blood glucose is also closely monitored, aiming to maintain values around 220 mg/dL.

Rob is a 46 y.o. admitted to the hospital with a suspected diagnosis of Hepatitis B. He's jaundiced and reports weakness. Which intervention will you include in his care? A. Regular exercise. B. A low-protein diet. C. Allow the patient to select his meals. D. Rest period after small, frequent meals.

D. Rest period after small, frequent meals. Rest periods and small frequent meals are indicated during the acute phase of hepatitis B. Monitor dietary intake and caloric count. Suggest several small feedings and offer the "largest" meal at breakfast. Large meals are difficult to manage when a patient is anorexic. Anorexia may also worsen during the day, making intake of food difficult later in the day. Option A: Institute bed red or chair rest during the toxic state. Provide a quiet environment; limit visitors as needed. This promotes rest and relaxation. Available energy is used for healing. Activity and an upright position are believed to decrease hepatic blood flow, which prevents optimal circulation to the liver cells. Option B: If tolerated, a normal or increased protein intake helps with liver regeneration. Protein restriction may be indicated in severe disease (fulminant hepatitis) because the accumulation of the end products of protein metabolism can potentiate hepatic encephalopathy. Option C: Consult with a dietitian, nutritional support team to provide diet according to patient's needs, with fat and protein intake as tolerated. Useful in formulating dietary programs to meet individual needs. Fat metabolism varies according to bile production and excretion and may necessitate the restriction of fat intake if diarrhea develops.

A 29 y.o. patient has an acute episode of ulcerative colitis. What diagnostic test confirms this diagnosis? A. Barium Swallow B. Stool examination C. Gastric analysis D. Sigmoidoscopy

D. Sigmoidoscopy Sigmoidoscopy allows direct observation of the colon mucosa for changes, and if needed, biopsy. Colonoscopy or proctosigmoidoscopy might reveal loss of typical vascular pattern, granularity, friability, and ulceration which involve the distal rectum and proceed proximally in a symmetric, continuous, and circumferential pattern. The disease can range from disease isolated to the rectum and sigmoid colon (proctitis) to disease of the entire colon (pancolitis). Option A: The barium swallow study, also known as a barium esophagogram or esophagram, is a contrast-enhanced radiographic study commonly used to assess structural characteristics of the entire esophagus. It may be used for the diagnosis of a wide range of pathologies including esophageal motility disorders, strictures, and perforations. Option B: Diagnosis of ulcerative colitis is made clinically with supportive findings on endoscopy, biopsy, and by negative stool examination for infectious causes. Because colonic infection can produce clinical findings indistinguishable from idiopathic ulcerative colitis, microbiologic studies for bacterial infection and parasitic infestation should be included in the initial evaluation. Option C: Gastric acid analysis is rarely done in current practice. When conducted, samples of stomach contents obtained via nasogastric tube are used to measure gastric acid output in a basal and stimulated state. This information may be useful in a patient who develops a recurrent ulcer after surgical vagotomy for peptic ulcer disease. In this case, a positive acid response to stimulation (sham feeding) indicates an incomplete vagotomy.

Which of the following terms best describes the pain associated with appendicitis? A. Aching B. Fleeting C. Intermittent D. Steady

D. Steady The pain begins in the epigastrium or periumbilical region, then shifts to the right lower quadrant and becomes steady. The pain may be moderate to severe. Classically, appendicitis presents as an initial generalized or periumbilical abdominal pain that then localizes to the right lower quadrant. Initially, as the visceral afferent nerve fibers at T8 through T10 are stimulated, and this leads to vague centralized pain. Option A: Pain upon passive extension of the right leg with the patient in the left lateral decubitus position is known as psoas sign. This maneuver stretches the psoas major muscle, which can be irritated by an inflamed retrocecal appendix. Patients often flex the hip to shorten the psoas major muscle and relieve pain. Option B: As the appendix becomes more swollen and inflamed, it will irritate the lining of the abdominal wall, known as the peritoneum. This causes localized, sharp pain in the right lower part of the abdomen. Option C: As the appendix becomes more inflamed, and the adjacent parietal peritoneum is irritated, the pain becomes more localized to the right lower quadrant. The pain tends to be more constant and severe than the dull, aching pain that occurs when symptoms start.

Which of the following symptoms may be exhibited by a client with Crohn's disease? A. Bloody diarrhea B. Narrow stools C. N/V D. Steatorrhea

D. Steatorrhea Steatorrhea from malabsorption can occur with Crohn's disease. The inflammatory process caused by Crohn's disease, especially if it involves large tracts of intestine, alters the absorption mechanisms of different substances due to the lack of reabsorption of the bile salts, which normally stimulate the reabsorption of food fats. Option A: Ulcerative colitis's main symptom is bloody diarrhea, with or without mucus. Associated symptoms also include urgency or tenesmus, abdominal pain, malaise, weight loss, and fever, depending on the extent and severity of the disease. Option B: Narrow stools are associated with diverticular disease. Patients with complications of their diverticulitis may have more chronic or long-term symptoms. Thin stools or constipation may indicate the formation of a stricture. Dark, cloudy urine or passing air with the urine may indicate the formation of a fistula to the bladder. Option C: N/V are symptoms of ulcerative colitis. Ulcerative colitis itself is not an emergency situation, but serious symptoms may occur. Seek immediate medical care (call 911) for serious symptoms, such as severe abdominal pain, inability to pass gas or stool, and vomiting or vomiting blood.

Your patient recently had abdominal surgery and tells you that he feels a popping sensation in his incision during a coughing spell, followed by severe pain. You anticipate an evisceration. Which supplies should you take to his room? A. A suture kit. B. Sterile water and a suture kit. C. Sterile water and sterile dressings. D. Sterile saline solution and sterile dressings.

D. Sterile saline solution and sterile dressings. Saline solution is isotonic, or close to body fluids in content, and is used along with sterile dressings to cover an eviscerated wound and keep it moist. If the wound shows signs of evisceration, the wound can be covered with a sterile saline dressing until the herniating organs can be reduced back into the abdomen. Option A: A suture kit would be needed later during closure of the wound. Dehiscence is a partial or total separation of previously approximated wound edges, due to a failure of proper wound healing. This scenario typically occurs 5 to 8 days following surgery when healing is still in the early stages. The causes of dehiscence are similar to the causes of poor wound healing and include ischemia, infection, increased abdominal pressure, diabetes, malnutrition, smoking, and obesity Option B: When dehiscence is identified, it is crucial to determine the extent of wound failure. Superficial dehiscence can be managed with abdominal binders to reduce strain on the wound and prevent further dehiscence. The existing wound failure may be allowed to heal by secondary intention or can close secondarily. Option C: Deep dehiscence of the underlying fascia is a surgical emergency and must be closed in the operating room, as this can lead to evisceration. Secondary closure is preferred to healing by secondary intention in patients with superficial dehiscence. Patients who underwent secondary closure experience a faster healing time and fewer postoperative visits than the healing by secondary intention group.

Leigh Ann is receiving pancrelipase (Viokase) for chronic pancreatitis. Which observation best indicates the treatment is effective? A. There is no skin breakdown. B. Her appetite improves. C. She lost more than 10 lbs. D. Stools are less fatty and decrease in frequency.

D. Stools are less fatty and decreased in frequency. Pancrelipase provides the exocrine pancreatic enzyme necessary for proper protein, fat, and carb digestion. With increased fat digestion and absorption, stools become less frequent and normal in appearance. These agents serve as exogenous versions of digestive hormones and enzymes required for normal digestion and are ingested with meals to improve digestion, absorption, and abdominal pain frequently seen in chronic pancreatitis and exocrine pancreatic insufficiency. Option A: The most common adverse effects are headache, oral irritation, abdominal pain, lymphadenopathy, nasal congestion, and neck pain. Notably, a beta-hemolytic streptococcal infection has been seen with the use of pancrelipase. Rare (less than 1%), but severe side effects include cancer recurrence, anaphylaxis, duodenitis, and distal intestinal obstruction syndrome. Option B: Patients should take this medication with meals; a general rule of thumb is to take half a dose of pancrelipase with snacks. Most manufacturers recommend beginning with a dose of 500 units of lipase/kg with each meal. Most manufacturers recommend not to exceed 10,000 units of lipase/kg per day. Option C: Warnings include fibrosing colonopathy (especially when doses exceed 2500 lipase units/kg of body weight per meal or greater than 10,000 lipase units/kg of body weight per day) and hyperuricemia as porcine-derived pancreatic enzyme products contain purines that may increase blood uric acid levels.

Which area of the alimentary canal is the most common location for Crohn's disease? A. Ascending colon B. Descending colon C. Sigmoid colon D. Terminal ileum

D. Terminal ileum Studies have shown that the terminal ileum is the most common site for recurrence in clients with Crohn's disease. The terminal ileum is involved in 50-70% of children. Although any area of the GI system may be affected in patients with Crohn disease, the most common site of the chronic inflammatory process is the ileocecal region, followed by the colon (about 20%), the small intestine alone (about 30%), the stomach (rarely), and the mouth. The esophagus is very rarely involved. The other areas may be involved but aren't as common. Option A: More than half of these patients also have inflammation in various segments of the colon, usually the ascending colon. Gastric inflammation, duodenal inflammation, or both may be observed in as many as 30-40% of children with Crohn's disease. The primary pancreatic manifestation is pancreatitis. Option B: About 45% of cases of Crohn disease occur in the ileum and colon, 20% solely in the colon, 33% in the small bowel, and 5% in the gastroduodenal region and perianal region alone (fistula, abscess, anal ulcer or stricture, or fissure). Nearly 20-23% of patients with large bowel or small bowel disease have perianal complications, which may precede the development of intestinal symptoms and manifest as simple skin tags, anal fissures, perianal fistulae, or abscesses. Option C: Bowel obstruction is caused initially by significant edema of the mucosa and associated spasm of the bowel. Obstruction is intermittent and can often be reversed by means of conservative measures and anti-inflammatory agents. With further disease progression, the obstruction becomes chronic because of fibrotic scarring, luminal narrowing, and stricture formation.

Crohn's disease can be described as a chronic relapsing disease. Which of the following areas in the GI system may be involved with this disease? A. The entire length of the large colon. B. Only the sigmoid area. C. The entire large colon through the layers of mucosa and submucosa. D. The small intestine and colon; affecting the entire thickness of the bowel.

D. The small intestine and colon; affecting the entire thickness of the bowel Crohn's disease can involve any segment of the small intestine, the colon, or both, affecting the entire thickness of the bowel. In Crohn's disease, the inflammation extends through the entire thickness of the bowel wall from the mucosa to the serosa. The disease runs a relapsing and remitting course. Option A: Ulcerative colitis is an idiopathic inflammatory condition of the colon which results in diffuse friability and superficial erosions on the colonic wall and associated bleeding. It is the most common form of inflammatory bowel disease worldwide. Option B: Diverticula can form while straining during a bowel movement, such as with constipation. They are most common in the lower portion of the large intestine (called the sigmoid colon). Diverticulosis is very common and occurs in 10% of people over age 40 and in 50% of people over age 60. Option C: Characteristically, it involves inflammation restricted to the mucosa and submucosa of the colon. Typically, the disease starts in the rectum and extends proximally in a continuous manner. In the United States, the disease accounts for a quarter-million clinician visits annually.

You're doing preoperative teaching with Gertrude who has ulcerative colitis who needs surgery to create an ileoanal reservoir. Which information do you include? A. A reservoir is created that exits through the abdominal wall. B. A second surgery is required 12 months after the first surgery. C. A permanent ileostomy is created. D. The surgery occurs in two stages.

D. The surgery occurs in two stages. An ileoanal reservoir is created in two stages. The two surgeries are about 2 to 3 months apart. First, diseased intestines are removed and a temporary loop ileostomy is created. Second, the loop ileostomy is closed and stool goes to the reservoir and out through the anus. The ileoanal reservoir procedure is a surgical treatment option for chronic ulcerative colitis, colon cancer, and familial polyposis patients who need to have their large intestine (colon) removed. An ileoanal reservoir (or pouch) is an internal pouch formed of the small intestine. Option A: The first surgery removes the entire large bowel and the lining of the rectum, but leaves the rectal muscle intact. A reservoir or "pouch" is made out of the small intestine and then is connected to the anus. In the initial weeks after surgery, waste material coming through the ileostomy is liquid but then begins to thicken. A good diet with increased fluid intake is needed to keep well hydrated and nourished. Option B: Each patient considering this surgery is carefully evaluated to determine if this procedure is appropriate for them. This procedure is performed in one, two or three stages, but is most often done in two stages, usually 2-3 months apart. Option C: Next, a temporary ileostomy is made. An ileostomy is a surgically created opening between the small bowel and the skin of the abdomen through which stool and gas are passed. This temporary ileostomy diverts the stool; protecting the reservoir (pouch)while it heals.

A client is admitted to the hospital after vomiting bright red blood and is diagnosed with a bleeding duodenal ulcer. The client develops a sudden, sharp pain in the mid-epigastric area along with a rigid, board-like abdomen. These clinical manifestations most likely indicate which of the following? A. An intestinal obstruction has developed. B. Additional ulcers have developed. C. The esophagus has become inflamed. D. The ulcer has perforated.

D. The ulcer has perforated. The body reacts to perforation of an ulcer by immobilizing the area as much as possible. This results in boardlike muscle rigidity, usually with extreme pain. Perforation is a medical emergency requiring immediate surgical intervention because peritonitis develops quickly after perforation. These patients may also demonstrate signs and symptoms of septic shock, such as tachycardia, hypotension, and anuria. Option A: An intestinal obstruction would not cause mid-epigastric pain. Patients with perforated peptic ulcer disease usually present with a sudden onset of severe, sharp abdominal pain. Most patients describe generalized pain; a few present with severe epigastric pain. As even slight movement can tremendously worsen their pain, these patients assume a fetal position. Option B: The development of additional ulcers would not cause a rigid, boardlike abdomen. Abdominal examination in a perforated ulcer usually discloses generalized tenderness, rebound tenderness, guarding, and rigidity. Option C: Esophageal inflammation would not cause a rigid, boardlike abdomen. The degree of peritoneal findings is strongly influenced by a number of factors, including the size of the perforation, amount of bacterial and gastric contents contaminating the abdominal cavity, time between perforation and presentation, and spontaneous sealing of perforation.

Which of the following associated disorders may a client with ulcerative colitis exhibit? A. Gallstones B. Hydronephrosis C. Nephrolithiasis D. Toxic megacolon

D. Toxic megacolon Toxic megacolon is extreme dilation of a segment of the diseased colon caused by paralysis of the colon, resulting in complete obstruction. This disorder is associated with both Crohn's disease and ulcerative colitis. When the muscularis propria is involved it may lead to damage to the nerves resulting in dilatation, aperistalsis, and ischemia (toxic megacolon). Today, toxic megacolon is the most common cause of death in ulcerative colitis. At least 5% of patients develop colon cancer and this risk increases with the duration of the disease. The other disorders are more commonly associated with Crohn's disease. Option A: The prevalence of gallstone disease in patients with Crohn's disease is about two-fold higher than that in the general population. One hypothesis for the increased prevalence of gallstone disease in patients with Crohn's disease is that the bile acid malabsorption in patients with diseased or resected ileum may lead to cholesterol supersaturated bile. Option B: Ureteric stenosis with resulting hydronephrosis has been described as a serious complication of inflammatory bowel diseases, especially of Crohn's disease (CD), necessitating urological intervention. This complication predominantly affects the right renal system in CD and seems to be a mechanical obstruction caused by inflammatory penetration from the affected distal ileum into the retroperitoneum, irrespective of urolithiasis. Option C: In about 30% of the patients with Crohn's disease can be found extra-intestinals symptoms. Nephrolithiasis is one of them and the appearance of kidney stones, mainly of oxalate of calcium, is more common in these patients than in the general population.

Anthony, a 60 y.o. patient, has just undergone a bowel resection with a colostomy. During the first 24 hours, which of the following observations about the stoma should you report to the doctor? A. Pink color B. Light edema C. Small amount of oozing D. Trickles of bright red blood

D. Trickles of bright red blood After the creation of a colostomy, expect to see a stoma that is pink, slightly edematous, with some oozing. Bright red blood, regardless of amount, indicates bleeding and should be reported to the doctor. A colostomy is an operation that creates an opening for the colon, or large intestine, through the abdomen. A colostomy may be temporary or permanent. It is usually done after bowel surgery or injury. Most permanent colostomies are "end colostomies," while many temporary colostomies bring the side of the colon up to an opening in the abdomen. Option A: A normal stoma is moist and pink or red-colored. At first, the colostomy may appear dark red and swollen, with bruises. Within a few weeks, the color will lighten and bruises should disappear. Option B: Don't be alarmed by the size of the stoma for the first few weeks after surgery. The abdomen and the stoma will both be quite swollen. This is all quite normal after a stoma operation and nothing to be worried about. The stoma will go down in size quite considerably over the next 2-3 months. Option C: The stoma may also ooze blood for the first few days. It can take a few days for the stoma to start producing any output. The bowel will have gone into a shock-like state following surgery. Aid this process by being mobile and eating small regular amounts of food.

Colon cancer is most closely associated with which of the following conditions? A. Appendicitis B. Hemorrhoids C. Hiatal hernia D. Ulcerative colitis

D. Ulcerative colitis Chronic ulcerative colitis, granulomas, and familial polyposis seem to increase a person's chance of developing colon cancer. Having ulcerative colitis can increase the risk for colorectal cancer. Recent research shows that colorectal cancer rates are dropping among people with inflammatory bowel diseases such as ulcerative colitis. People with ulcerative colitis should receive regular screenings for colorectal cancer. The other conditions listed have no known effect on colon cancer risk. Option A: Appendicitis can be secondary to cecal pathology (polyp or cancer). Increasing age is a risk factor for malignancy coexisting with appendicitis. There is an increased coexistence of cancer post-appendicectomy in patients aged 50-54 years. Patients aged 55 years or over who have undergone appendectomy should be offered colonoscopy to exclude coexistent cecal pathology. Option B: Hemorrhoids don't cause or increase the risk for colon or rectal cancer. However, the two conditions may be mistaken for one another because they can produce similar symptoms, such as rectal bleeding, itching, and pain. Option C: Abdominal wall hernias may develop not because of cancer itself but due to its consequences, for example, increased intra-abdominal pressure secondary to obstructive colon cancer or a large pelvic tumor may cause a herniation.

Which of the following mechanisms can facilitate the development of diverticulosis into diverticulitis? A. Treating constipation with chronic laxative use, leading to dependence on laxatives. B. Chronic constipation causing an obstruction, reducing forward flow of intestinal contents. C. Herniation of the intestinal mucosa, rupturing the wall of the intestine. D. Undigested food blocking the diverticulum, predisposing the area to bacterial invasion.

D. Undigested food blocking the diverticulum, predisposing the area to bacterial invasion. Undigested food can block the diverticulum, decreasing blood supply to the area and predisposing the area to invasion of bacteria. Diverticulitis typically results from micro- or macroscopic perforation of a diverticulum, which may or may not result from obstruction (e.g., by a fecalith). Increased intraluminal pressures or inspissated (thickened and condensed) food matter, with resultant inflammation and focal necrosis, ultimately result in diverticular perforation. Option A: Chronic laxative use is a common problem in elderly clients, but it doesn't cause diverticulitis. Medications associated with an increased risk of diverticular bleeding or diverticulitis include nonsteroidal anti-inflammatory drugs, opiates, and steroids. Option B: Chronic constipation can cause an obstruction—not diverticulitis. The risk of diverticulitis and bleeding is significantly higher in patients with obesity or larger waist circumference. Smokers have been noted to have an increased incidence of diverticular abscess formation or perforation. Option C: Herniation of the intestinal mucosa causes an intestinal perforation. Inflammatory diseases of the bowel such as Crohn's disease and ulcerative colitis can also lead to perforation, especially Crohn's, which is characterized by full-thickness inflammation of the bowel wall.

A client being treated for chronic cholecystitis should be given which of the following instructions? A. Increase rest. B. Avoid antacids. C. Increase protein in diet. D. Use anticholinergics as prescribed.

D. Use anticholinergics as prescribed. Conservative therapy for chronic cholecystitis includes weight reduction by increasing physical activity, a low-fat diet, antacid use to treat dyspepsia, and anticholinergic use to relax smooth muscles and reduce ductal tone and spasm, thereby reducing pain. Anticholinergics such as atropine and propantheline (Pro-Banth?-ne) relieve reflex spasms and smooth muscle contraction and assists with pain management. Option A: Promote bedrest, allowing the patient to assume a position of comfort. Bedrest in low-Fowler's position reduces intra-abdominal pressure; however, the patient will naturally assume the least painful position. Encourage use of relaxation techniques. Provide diversional activities. Option B: Administer bile salts: Bilron, Zanchol, dehydrocholic acid (Decholin), as indicated. Promotes digestion and absorption of fats, fat-soluble vitamins, cholesterol. Useful in chronic cholecystitis. Option C: Begin a low-fat liquid diet after the NG tube is removed. Limiting fat content reduces stimulation of the gallbladder and pain associated with incomplete fat digestion and is helpful in preventing recurrence. Advance diet as tolerated, usually low-fat, high-fiber. Restrict gas-producing foods (onions, cabbage, popcorn) and foods or fluids high in fats (butter, fried foods, nuts).

During preparation for bowel surgery, a male client receives an antibiotic to reduce intestinal bacteria. Antibiotic therapy may interfere with the synthesis of which vitamin and may lead to hypoprothrombinemia? A. Vitamin A B. Vitamin D C. Vitamin E D. Vitamin K

D. Vitamin K Intestinal bacteria synthesize such nutritional substances as vitamin K, thiamine, riboflavin, vitamin B12, folic acid, biotin, and nicotinic acid. Therefore, antibiotic therapy may interfere with the synthesis of these substances, including vitamin K. Antibiotics, especially those known as cephalosporins, reduce the absorption of vitamin K in the body. Using them for more than 10 days may lower levels of vitamin K because these drugs kill not only harmful bacteria but also the bacteria that make vitamin K. Option A: Vitamin A is a general term that encompasses various fat-soluble substances such as retinol, retinyl palmitate, and beta-carotene. In the liver, retinol is esterified to retinyl esters and stored in the stellate cells. In the tissues, both retinol and beta-carotene are oxidized to retinal and retinoic acid, which are essential for vision and gene regulation, respectively. These active metabolites bind nuclear receptors of the RAR family to control gene expression. Option B: Vitamin D is labeled as the "sunshine vitamin," as it is produced in the skin on sun exposure. Vitamin D is a hormone obtained through dietary consumption and skin production. Ultraviolet B (UVB) radiation, wavelength (290 to 315 nm) converts 7-dehydrocholesterol in the skin to previtamin D. This previtamin D undergoes heat isomerization and is converted to vitamin D. Vitamin D from the skin and diet is metabolized in the liver to 25-hydroxyvitamin D (25 OH D), and 25-hydroxyvitamin D is useful in assessing vitamin D status. Option C: Intestinal bacteria don't synthesize vitamin E. Vitamin E or tocopherol is a fat-soluble vitamin functioning as an antioxidant, protecting the cell membrane. As with all fat-soluble vitamins, transport and absorption require intact fat digestion mechanisms. Fat metabolism involves lipases, both lingual and gastric, bile salts, pancreatic enzymes, and intestinal absorption.

Which of the following conditions can cause a hiatal hernia? A. Increased intrathoracic pressure. B. Weakness of the esophageal muscle. C. Increased esophageal muscle pressure. D. Weakness of the diaphragmatic muscle.

D. Weakness of the diaphragmatic muscle. A hiatal hernia is caused by weakness of the diaphragmatic muscle and increased intra-abdominal—not intrathoracic—pressure. This weakness allows the stomach to slide into the esophagus. The esophageal supports weaken, but esophageal muscle weakness or increased esophageal muscle pressure isn't a factor in hiatal hernia. A hiatal hernia is a medical condition in which the upper part of the stomach or other internal organ bulges through an opening in the diaphragm. Option A: The diaphragm is a muscular structure that assists in respiration and has a small opening, a hiatus, through which the esophagus passes prior to connecting to the stomach. This is called the gastroesophageal junction (GEJ). In a hiatal hernia, the stomach pushes through that opening and into the chest and compromises the lower esophageal sphincter (LES). Option B: Hiatal hernias may be congenital or acquired. There is an increased prevalence in older people. It is believed that muscle weakness with loss of flexibility and elasticity with age predisposes to the development of a hiatal hernia. This may cause the upper part of the stomach to not return to its natural position below the diaphragm during swallowing. Option C: Other predisposing factors have been identified, such as elevated intra abdominal pressure. This typically is a result of obesity, pregnancy, chronic constipation, and chronic obstructive pulmonary disease (COPD). Trauma, age, previous surgeries, and genetics can also play a role in the development of a hiatal hernia.

Which of the following laboratory results would be expected in a client with peritonitis? A. Partial thromboplastin time above 100 seconds B. Hemoglobin level below 10 mg/dL C. Potassium level above 5.5 mEq/L D. White blood cell count above 15,000

D. White blood cell count above 15,000 Because of infection, the client's WBC count will be elevated. Peritoneal fluid (in patients with ascites or who are on PD) should be sent for cell count and differential, gram stain, and culture. In cirrhosis, a polymorphonuclear (PMN) count >250 cells/milliliter (mL) is diagnostic of SBP. In PD, a WBC count >100 cells/mL suggests peritonitis. Option A: A PT time longer than 100 seconds may suggest disseminated intravascular coagulation, a serious complication of septic shock. Diagnosis of DIC involves a combination of laboratory tests and clinical evaluation. Laboratory findings suggestive of DIC include a low platelet count, elevated D-dimer concentration, decreased fibrinogen concentration, and prolongation of clotting times such as prothrombin time (PT). Option B: A hemoglobin level below 10 mg/dl may occur from hemorrhage. Hemorrhage in trauma patients is associated with an early decrease in Hgb level. Hgb < or =10 gm/dL in the first 30 minutes of patient arrival will correctly identify presence or absence of significant bleeding in almost 9 of 10 trauma patients. Option C: A potassium level above 5.5 mEq/L may indicate renal failure. When kidneys fail they can no longer remove excess potassium, so the level builds up in the body. High potassium in the blood is called hyperkalemia, which may occur in people with advanced stages of chronic kidney disease (CKD). Some of the effects of high potassium are nausea, weakness, numbness, and slow pulse.

The nurse is discharging a client who is newly diagnosed with GERD. which of the following medication prescriptions indicate the presence of this condition? A. Oxycodone B. Olanzapine C. Oxytocin D. Omeprazole

E. Omeprazole This medication is a proton pump inhibitor used to treat GERD and ulcers. Oxycodone is an opioid pain medication. Olanzapine is a mood stabilizing antipsychotic medication used for schizophrenia as well as depression and anorexia. Oxytocin is a labor inducing medication that contracts the smooth muscles of the uterus.

The nurse knows that which of the following most accurately describes the rationale for administering famotidine to a client with gastritis? A. Decrease inflammation in the stomach B. Reduce nausea and vomiting C. Alleviate client pain D. Decrease acid reflux

A. Decrease Inflammation in the stomach Famotidine is an H2 receptor blocker that decreases inflammation caused by gastritis by halting the histamine response to the inflamed area

Which topic would the nurse plan to teach to a patient with Crohn's disease who has megaloblastic anemia? A. Iron dextran infusions B. Oral ferrous sulfate tablets C. Routine blood transfusions D. Cobalamin (B12) supplements

D Crohn's disease frequently affects the ileum, where absorption of cobalamin occurs. Cobalamin must be administered regularly by nasal spray or IM to correct the anemia. Iron deficiency does not cause megaloblastic anemia. The patient may need occasional transfusions but not regularly scheduled transfusions.

Which information will the nurse provide for a patient with achalasia? A. A liquid diet will be necessary. B. Avoid drinking fluids with meals. C. Lying down after meals is recommended. D. Treatment may include endoscopic procedures.

D Endoscopic and laparoscopic procedures are the most effective therapy for improving symptoms caused by achalasia. Keeping the head elevated after eating will improve esophageal emptying. A semisoft diet is recommended to improve esophageal emptying. Patients are advised to drink fluids with meals.

An 80-yr-old patient who is hospitalized with peptic ulcer disease develops new-onset auditory hallucinations. Which prescribed medication will the nurse discuss with the health care provider before administration? A. Sucralfate (Carafate) B. Aluminum hydroxide C. Omeprazole (Prilosec) D. Metoclopramide (Reglan)

D Metoclopramide can cause central nervous system side effects ranging from anxiety to hallucinations. Hallucinations are not a side effect of proton pump inhibitors, mucosal protectants, or antacids.

Which question from the nurse would help determine if a patient's abdominal pain might indicate irritable bowel syndrome (IBS)? A. "Have you been passing a lot of gas?" B. "What foods affect your bowel patterns?" C. "Do you have any abdominal distention?" D. "How long have you had abdominal pain?"

D One criterion for the diagnosis of irritable bowel syndrome is the presence of abdominal discomfort or pain for at least 3 months. Abdominal distention, flatulence, and food intolerance are associated with IBS but are not diagnostic criteria.

Which action will be included in the care for a patient who has recently been diagnosed with asymptomatic nonalcoholic fatty liver disease (NAFLD)? A. Teach symptoms of variceal bleeding. B. Draw blood for hepatitis serology testing. C. Discuss the need to increase caloric intake. D. Review the patient's current medication list.

D Some medications can increase the risk for NAFLD, and they should be discontinued. NAFLD is not associated with hepatitis, weight loss is usually indicated, and variceal bleeding would not be expected in a patient with asymptomatic NAFLD.

A serum potassium level of 3.2 mEq/L (3.2 mmol/L) is reported for a patient with cirrhosis who has scheduled doses of spironolactone (Aldactone) and furosemide (Lasix) due. Which action would the nurse take? A. Withhold both drugs. B. Administer both drugs. C. Administer the furosemide. D. Administer the spironolactone.

D Spironolactone is a potassium-sparing diuretic and will help increase the patient's potassium level. The furosemide will further decrease the patient's potassium level and should be held until the nurse talks with the health care provider.

How would the nurse explain esomeprazole (Nexium) to a patient who has recurring heartburn? A. "It reduces gastroesophageal reflux by increasing the rate of gastric emptying." B. "It neutralizes stomach acid and provides relief of symptoms in a few minutes." C. "It coats and protects the lining of the stomach and esophagus from gastric acid." D. "It treats gastroesophageal reflux disease by decreasing stomach acid production."

D The proton pump inhibitors decrease the rate of gastric acid secretion. Promotility drugs such as metoclopramide (Reglan) increase the rate of gastric emptying. Cryoprotective medications such as sucralfate (Carafate) protect the stomach. Antacids neutralize stomach acid and work rapidly.

A nurse is educating a client about their new diagnosis of pyloric stenosis. The client indicates understanding of this diagnosis when they point to which location as the site of pyloric stenosis? (Refer to image) A. D B. C C. A D. B

D The pyloric valve is a muscle surrounding the opening between the stomach and the duodenum. Pyloric stenosis is the narrowing (stenosis) of the opening from the stomach to the part of the small intestine known as the duodenum, due to enlargement (hypertrophy) of the muscles surrounding this opening which spasms when the stomach empties. C is the body of the stomach A is the fundus of the stomach B is the esophagogastric junction

Which of the following dietary measures would be useful in preventing esophageal reflux? A. Eating small, frequent meals. B. Increasing fluid intake. C. Avoiding air swallowing with meals. D. Adding a bedtime snack to the dietary plan.

A. Eating small, frequent meals. Esophageal reflux worsens when the stomach is overdistended with food. Therefore, an important measure is to eat small, frequent meals. Encourage small frequent meals of high calories and high protein foods. Small and frequent meals are easier to digest. Instruct the patient to eat slowly and masticate foods well to help prevent reflux. Option B: Fluid intake should be decreased during meals to reduce abdominal distention. Avoid placing the patient in supine position, have the patient sit upright after meals. Supine position after meals can increase regurgitation of acid. Option C: Avoiding air swallowing does not prevent esophageal reflux. Assess for pulmonary symptoms resulting from reflux of gastric content. These include subsequent aspiration, chronic pulmonary disease, or nocturnal wheezing, bronchitis, asthma, morning hoarseness, and cough. Option D: Food intake in the evening should be strictly limited to reduce the incidence of nighttime reflux, so bedtime snacks are not recommended. Instruct the patient to avoid highly seasoned food, acidic juices, alcoholic drinks, bedtime snacks, and foods high in fat. These can reduce the lower esophageal sphincter pressure.

The nurse is performing an admission assessment on a client diagnosed with gastroesophageal reflux disease (GERD). Which signs and symptoms would indicate GERD? A. Pyrosis, water brash, and flatulence B. Weight loss, dysarthria, and diarrhea C. Decreased abdominal fat, proteinuria, and constipation D. Mid-epigastric pain, positive H. pylori test, and melena

A. Pyrosis, water brash, and flatulence

Which of the following measures should the nurse focus on for the client with esophageal varices? A. Recognizing hemorrhage. B. Controlling blood pressure. C. Encouraging nutritional intake. D. Teaching the client about varices.

A. Recognizing hemorrhage. Recognizing the rupture of esophageal varices, or hemorrhage is the focus of nursing care because the client could succumb to this quickly. A patient with bleeding esophageal varices is to be considered in critical condition. Nursing management is aimed at assisting the physician in controlling bleeding and preventing shock and death. Option B: Controlling blood pressure is also important because it helps reduce the risk of variceal rupture. As portal pressure increases, blood backs up into the spleen and bypasses the liver, returning to the right atrium via collateral circulation. The result is splenomegaly, ascites, and varicosities of the collateral veins (esophageal and gastric varices). Option C: It is also important to teach the client what foods he should avoid such as spicy foods. Additional teaching includes abstaining from alcohol, eating a healthy diet, and adhering to short-term antibiotic therapy to prevent infection. Because rebleeding is common. Option D: It is also important to teach the client what varices are. Assess for ecchymosis, epistaxis, petechiae, and bleeding gums. Monitor level of consciousness, vital signs, and urinary output to evaluate fluid balance. Use small-gauge needles, and apply pressure or cold for bleeding.

A young adult has been admitted to the emergency department with nausea and vomiting. Which action could the RN delegate to assistive personnel (AP)? A. Auscultate the bowel sounds. B. Assess for signs of dehydration. C. Assist the patient with oral care. D. Ask more questions about the nausea.

C Oral care is included in AP education and scope of practice. The other actions are all assessments that require more education and a higher scope of nursing practice.

Four hours after a bowel resection, a 74-yr-old male patient with a nasogastric tube to suction reports nausea and abdominal distention. Which action would the nurse take first? A. Auscultate for hypotonic bowel sounds. B. Notify the patient's health care provider. C. Check for tube placement and reposition it. D. Remove the tube and replace it with a new one.

C Repositioning the tube will frequently facilitate drainage. Because this is a common occurrence, it is not appropriate to notify the health care provider unless other interventions do not resolve the problem. Information about the presence or absence of bowel sounds will not be helpful in improving drainage. Removing the tube and replacing it are unnecessarily traumatic to the patient, so that would only be done if the tube was completely occluded.

A patient admitted with an abrupt onset of jaundice and nausea has abnormal liver function studies, but serologic testing is negative for viral causes of hepatitis. Which question would the nurse ask? A. "Have you taken corticosteroids?" B. "Do you have a history of IV drug use?" C. "Do you use any over-the-counter drugs?" D. "Have you recently traveled to another country?"

C The patient's symptoms, lack of antibodies for hepatitis, and the abrupt onset of symptoms suggest toxic hepatitis, which can be caused by commonly used over-the-counter drugs such as acetaminophen (Tylenol). Travel to a foreign country and a history of IV drug use are risk factors for viral hepatitis.

After change-of-shift report, which patient would the nurse assess first? A. A 42-yr-old patient who has acute gastritis and ongoing epigastric pain B. A 70-yr-old patient with a hiatal hernia who experiences frequent heartburn C. A 60-yr-old patient with nausea and vomiting who is lethargic with dry mucosa D. A 53-yr-old patient who has dumping syndrome after a recent partial gastrectomy

C A patient with nausea and vomiting who is lethargic with dry mucosa is at high risk for problems such as aspiration, dehydration, and fluid and electrolyte disturbances. The other patients will also need to be assessed, but the information about them indicates symptoms that are typical for their diagnoses and are not life threatening.

Which finding in the mouth of a patient who uses smokeless tobacco is suggestive of oral cancer? A. Bleeding during tooth brushing B. Painful blisters at the lip border C. Red patches on the buccal mucosa D. Curdlike plaques on the posterior tongue

C A red, velvety patch suggests erythroplasia, which has a high incidence (>50%) of progression to squamous cell carcinoma. The other lesions are suggestive of acute processes (e.g., gingivitis, oral candidiasis, herpes simplex).

A patient has arrived in the recovery area after an upper endoscopy. Which information collected by the nurse is most important to communicate to the health care provider? A. The patient is very drowsy. B. The patient reports a sore throat. C. The patient's temperature is 101.4F. D. The patient's pulse rate is 100 beats/min.

C A temperature elevation may indicate that an acute perforation has occurred. The other assessment data are normal immediately after the procedure.

Which finding by the nurse during abdominal auscultation indicates a need for a focused abdominal assessment? A. Loud gurgles B. High-pitched gurgles C. Absent bowel sounds D. Intermittent sounds

C Absent bowel sounds are abnormal and require further assessment by the nurse. Normal sounds are relatively high pitched intermittent gurgling.

Jerod is experiencing an acute episode of ulcerative colitis. Which is the priority for this patient? A. Replace lost fluid and sodium. B. Monitor for increased serum glucose level from steroid therapy. C. Restrict the dietary intake of foods high in potassium. D. Note any change in the color and consistency of stools.

A. Replace lost fluid and sodium.

The client has been admitted with a diagnosis of acute pancreatitis. The nurse would assess this client for pain that is: A. Severe and unrelenting, located in the epigastric area and radiating to the back. B. Severe and unrelenting, located in the left lower quadrant and radiating to the groin. C. Burning and aching, located in the epigastric area and radiating to the umbilicus. D. Burning and aching, located in the left lower quadrant and radiating to the hip.

A. Severe and unrelenting, located in the epigastric area and radiating to the back. The pain associated with acute pancreatitis is often severe and unrelenting, is located in the epigastric region, and radiates to the back. Acute pancreatitis is common and is the leading cause of hospitalization amongst gastrointestinal disorders in the United States. The severity of the disease varies widely, from mild disease needing conservative treatment to severe and complicated disease with high morbidity and mortality. Option B: The patient will commonly describe moderate to severe abdominal pain located in the epigastrium with nausea and anorexia. The nature of the pain can vary, often depending on whether the etiology is a biliary obstruction or a metabolic/toxicologic cause. Biliary etiology is more often described as a sharper pain, which radiates through to the back with more of acute onset. Option C: Metabolic and toxicologic causes, such as alcohol, often have a more indolent onset with more dull and generalized pain. A thorough history regarding alcohol use and medications should be gathered, keeping in mind that over five years of heavy alcohol use is often needed to induce alcohol-related pancreatitis. Option D: Family history should be reviewed, particularly when more common etiologies appear less likely, as there are rare genetically related cases of familial pancreatitis. A physical exam is often significant for elevated temperature, tachycardia, and in severe cases, hypotension. The abdominal exam will typically reveal epigastric tenderness with possible guarding and rigidity and decreased bowel sounds.

A pt experiencing severe jaundice due to cholethiaisis. Which action by CNA warrants intervention by RN? A. assisting pt to take hot soapy shower B. CNA applies emollient to pts legs C. puts mittens on hands of pt D. pats the skin dry with clean towel

A. assisting pt to take hot soapy shower

When caring for pt with pancreatitis, which question should be asked to check for the onset of hypocalcemia? A. do you have nubmness or tingling in your fingers or around your lips B. do you have any muscle cramps in your feet or legs? C. do you have any difficulty swallowing D. do you feel weak or dizzy when you stand up

A. do you have numbness or tingling in your fingers or around your lips

A RN is doing preop teaching for pt who will have a cholecystectomy. The RN teaches that special care must be taken to prevent what post op complication A. hypostatic pneumonia B. thrombus C. hemorrhage D. paralytic ileus

A. hypostatic pneumonia

Peptic ulcer disease may be caused by which of the following? A. Helicobacter pylori B. Clostridium difficile C. Candida albicans D. Staphylococcus aureus

A. Helicobacter pylori Helicobacter pylori is considered to be the major cause of ulcer formation. Peptic ulcer disease (PUD) has various causes; however, Helicobacter pylori-associated PUD and NSAID-associated PUD account for the majority of the disease etiology. H. pylorus is a gram-negative bacillus that is found within the gastric epithelial cells. This bacterium is responsible for 90% of duodenal ulcers and 70% to 90% of gastric ulcers. Other choices are not related to ulcer formation. Option B: Clostridium difficile is a gram-positive bacterium that is the cause most implicated in antibiotic-associated diarrhea. The emergence of a newer hypervirulent strain North American pulsed-field gel electrophoresis type 1 (NAP1) has been attributed to the increase in incidence and severity of C. difficile infections (CDI) over the last decade. Option C: Candida albicans is the most prevalent cause of fungal infections in people. Its species name, Albicans, comes from the Latin word for "white." The yeast appears white when cultured on a plate. And in the case of certain infections, like thrush, it can create white patches. Option D: Staphylococcus aureus is the most dangerous of all of the many common staphylococcal bacteria. These Gram-positive, sphere-shaped (coccal) bacteria often cause skin infections but can cause pneumonia, heart valve infections, and bone infections. Pain control with peptic ulcer disease includes all of the following except: A. Promoting physical and emotional rest. B. Identifying stressful situations. C. Eating meals when desired. D. Administering medications that decrease gastric acidity.] C. Eating meals when desired. Meals should be regularly spaced in a relaxed environment. Instruct the client that meals should be eaten at regularly spaced intervals in a relaxed setting. An irregular schedule of meals may interfere with the regular administration of medications. Option A: Encourage the use of nonpharmacological pain relief measures such as distraction, guided imagery. Massage, or music therapy. Non-pharmacological relaxation techniques will decrease the production of gastric acid, which in turn will reduce pain. Option B: Acknowledge awareness of the client's anxiety. Acknowledgement of the client's feelings validates the feelings and communicates the acceptance of those feelings. Open communication enables the client to develop a trusting relationship that aids in reducing anxiety and stress. Option D: Antacids buffer gastric acid and prevent the formation of peptin. This mechanism of action promotes the healing of the ulcer. Antibiotics treat the Helicobacter pylori infection and promote healing of the ulcer. As the ulcer heals, the client experiences less pain. H2 receptor antagonists block the secretion of gastric acid.

The nurse is caring for a patient with a biliary obstruction. Which condition would the nurse expect? A. Melena B. Steatorrhea C. Decreased serum cholesterol level D. Increased serum indirect bilirubin level

B A common bile duct obstruction will reduce the absorption of fat in the small intestine, leading to fatty stools. Gastrointestinal bleeding is not caused by common bile duct obstruction. Serum cholesterol levels are increased with biliary obstruction. Direct bilirubin level is increased with biliary obstruction.

A 33-yr-old male patient with a gunshot wound to the abdomen undergoes surgery, and a colostomy is formed as shown in the accompanying figure. Which information will the nurse include in patient teaching? A. Stool will be expelled from both stomas. B. This type of colostomy is usually temporary. C. Soft, formed stool can be expected as drainage. D. Irrigations can regulate drainage from the stomas.

B A loop, or double-barrel stoma, is usually temporary. Stool will be expelled from the proximal stoma only. The stool from the transverse colon will be liquid and regulation through irrigations will not be possible.

Which care activity for a patient with a paralytic ileus is appropriate for the registered nurse (RN) to delegate to assistive personnel (AP)? A. Auscultating for bowel sounds B. Brushing the teeth and tongue C. Assessing the nares for irritation D. Irrigating the nasogastric (NG) tube

B AP education and scope of practice include patient hygiene such as oral care. The other actions require education and scope of practice appropriate to the RN.

Which action would the nurse take after assisting with a needle biopsy of the liver at a patient's bedside? A. Elevate the head of the bed to facilitate breathing. B. Place the patient on the right side with the bed flat. C. Check the patient's postbiopsy coagulation studies. D. Position a sandbag over the liver to provide pressure.

B After a biopsy, the patient lies on the right side with the bed flat to splint the biopsy site. Coagulation studies are checked before the biopsy. A sandbag does not exert adequate pressure to splint the site.

Which risk factor would the nurse specifically ask about when a patient is being admitted with acute pancreatitis? A. Diabetes B. Alcohol use C. High-protein diet D. Cigarette smoking

B Alcohol use is one of the most common risk factors for pancreatitis in the United States. Cigarette smoking, diabetes, and high-protein diets are not risk factors.

The nurse is admitting a patient with new-onset steatorrhea. Which question is most important for the nurse to ask? A. "How much milk do you drink?" B. "Have you had a recent weight loss?" C. "Whattimeofdaydoyourbowelsmove?" D. "Do you eat meat or other animal products?"

B Although all the questions provide useful information, it is most important to determine if the patient has an imbalance in nutrition because of the steatorrhea.

A patient reports gas pains and abdominal distention 2 days after a small bowel resection. Which action would the nurse take? A. Administer morphine sulfate. B. Encourage the patient to ambulate. C. Offer the prescribed promethazine. D. Instill a mineral oil retention enema.

B Ambulation will improve peristalsis and help the patient eliminate flatus and reduce gas pain. A mineral oil retention enema is helpful for constipation with hard stool. A return-flow enema might be used to relieve persistent gas pains. Morphine will further reduce peristalsis. Promethazine is used as an antiemetic rather than to decrease gas pains or distention.

The nurse is assessing a patient who had a total gastrectomy 8 hours ago. Which information is most important to report to the health care provider? A. Hemoglobin (Hgb) 10.8 g/dL B. Temperature 102.1F (38.9C) C. Absent bowel sounds in all quadrants D. Scant nasogastric (NG) tube drainage

B An elevation in temperature may indicate leakage at the anastomosis, which may require return to surgery or keeping the patient NPO. The other findings are expected in the immediate postoperative period for patients who have this surgery and do not require any urgent action.

A patient being admitted with an acute exacerbation of ulcerative colitis reports crampy abdominal pain and passing 15 bloody stools a day. Which intervention would the nurse include in the patient's plan of care? A. Administer oral metoclopramide. B. Instruct the patient not to eat or drink. C. Administer cobalamin (vitamin B12) injections. D. Teach the patient about total colectomy surgery.

B An initial therapy for an acute exacerbation of inflammatory bowel disease (IBD) is to rest the bowel by making the patient NPO. Metoclopramide increases peristalsis and will worsen symptoms. Cobalamin (vitamin B12) is absorbed in the ileum, which is not affected by ulcerative colitis. Although total colectomy is needed for some patients, there is no indication that this patient is a candidate during this acute phase.

Which information from a 70-yr-old patient during a health history indicates to the nurse that the patient should be screened for hepatitis C? A. The patient had a blood transfusion in 2005. B. The patient used IV drugs about 30 years ago. C. The patient frequently eats in fast-food restaurants. D. The patient traveled to a country with poor sanitation.

B Any patient with a history of IV drug use should be tested for hepatitis C. Blood transfusions given after 1992 (when an antibody test for hepatitis C became available) do not pose a risk for hepatitis C. Hepatitis C is not spread by the oral-fecal route and therefore is not caused by contaminated food or by traveling in underdeveloped countries.

The nurse is caring for a patient who has cirrhosis. Which data obtained by the nurse during the assessment will be of most concern? A. The patient reports right upper-quadrant pain with palpation. B. The patient's hands flap back and forth when the arms are extended. C. The patient has ascites and a 2-kg weight gain from the previous day. D. The patient's abdominal skin has multiple spider-shaped blood vessels.

B Asterixis indicates that the patient has hepatic encephalopathy, and hepatic coma may occur. The spider angiomas and right upper quadrant abdominal pain are not unusual for the patient with cirrhosis and do not require a change in treatment. The ascites and weight gain indicate the need for treatment but not as urgently as the changes in neurologic status.

Which screening test would the nurse plan to teach a 45-yr-old male about during an annual wellness exam? A. Endoscopy B. Colonoscopy C. Computerized tomography D. Carcinoembryonic antigen (CEA)

B At age 45 years, persons with an average risk for colorectal cancer (CRC) would begin screening for CRC. Colonoscopy is the gold standard for CRC screening. The other diagnostic tests are not recommended as part of a routine annual physical exam at age 45 years.

A patient has been admitted with hypotension and dehydration after 3 days of nausea and vomiting. Which prescribed action will the nurse implement first? A. Insert a nasogastric (NG) tube. B. Infuse normal saline at 250 mL/hr. C. Administer IV ondansetron (Zofran). D. Provide oral care with moistened swabs.

B Because the patient has severe dehydration, rehydration with IV fluids is the priority. The other orders would be accomplished after the IV fluids are initiated.

Which statement by a patient with chronic atrophic gastritis indicates that the nurse's teaching regarding cobalamin injections has been effective? A. "The cobalamin injections will prevent gastric inflammation." B. "The cobalamin injections will prevent me from becoming anemic." C. "These injections will increase the hydrochloric acid in my stomach." D. "These injections will decrease my risk for developing stomach cancer."

B Cobalamin supplementation prevents the development of pernicious anemia. Chronic gastritis may cause achlorhydria, but cobalamin does not correct this. The loss of intrinsic factor secretion with chronic gastritis is permanent, and the patient will need lifelong supplementation with cobalamin. The incidence of stomach cancer is higher in patients with chronic gastritis, but cobalamin does not reduce the risk for stomach cancer.

Which information will the nurse provide for a patient with newly diagnosed gastroesophageal reflux disease (GERD)? A. "Peppermint tea may reduce your symptoms." B. "Keep the head of your bed elevated on blocks." C. "Avoideatingbetweenmealstoreduceacidsecretion." D. "Vigorousexercisemayincreasetheincidenceofreflux."

B Elevating the head of the bed will reduce the incidence of reflux while the patient is sleeping. Peppermint will decrease lower esophageal sphincter (LES) pressure and increase the chance for reflux. Small, frequent meals are recommended to avoid abdominal distention. There is no need to make changes in physical activities because of GERD.

Which nursing action would be included in the postoperative plan of care for a patient after a laparoscopic esophagectomy? A. Reposition the NG tube if drainage stops. B. Elevate the head of the bed to at least 30 degrees. C. Start oral fluids when the patient has active bowel sounds. D. Notify the doctor for any bloody nasogastric (NG) drainage.

B Elevation of the head of the bed decreases the risk for reflux and aspiration of gastric secretions. The NG tube would not be repositioned without consulting with the health care provider. Bloody NG drainage is expected for the first 8 to 12 hours. A swallowing study is needed before oral fluids are started.

Which action would the nurse include in the plan of care for a patient who has been diagnosed with chronic hepatitis B? A. Advise limiting alcohol intake to 1 drink daily. B. Schedule for liver cancer screening every 6 months. C. Initiate administration of the hepatitis C vaccine series. D. Monitor anti-hepatitis B surface antigen (anti-HBs) levels.

B Patients with chronic hepatitis are at higher risk for development of liver cancer and should be screened for liver cancer every 6 to 12 months. Patients with chronic hepatitis are advised to completely avoid alcohol. There is no hepatitis C vaccine. Because anti-HBs is present whenever there has been a past hepatitis B infection or vaccination, there is no need to regularly monitor for this antibody.

When caring for a patient who has cirrhosis, which nursing action could the registered nurse (RN) delegate to assistive personnel (AP)? A. Assessing the patient for jaundice B. Providing oral hygiene after a meal C. Palpating the abdomen for distention D. Teaching the patient the prescribed diet

B Providing oral hygiene is within the scope of AP. Assessments and assisting patients to choose therapeutic diets are nursing actions that require higher level nursing education and scope of practice and could be delegated to licensed practical/vocational nurses (LPNs/VNs) or RNs.

The nurse is planning care for a patient with acute severe pancreatitis. Which outcome would the nurse identify as the highest priority? A. Achieving fluid and electrolyte balance B. Maintaining normal respiratory function C. Expressing satisfaction with pain control D. Developing no ongoing pancreatic disease

B Respiratory failure can occur as a complication of acute pancreatitis and maintenance of adequate respiratory function is the priority goal. The other outcomes would also be appropriate for the patient.

A patient with blunt abdominal trauma from a motor vehicle crash undergoes peritoneal lavage. If the lavage returns brown fecal drainage, which action will the nurse plan to take next? A. Auscultate the bowel sounds. B. Prepare the patient for surgery. C. Check the patient's oral temperature. D. Obtain information about the accident.

B Return of brown drainage and fecal material suggests perforation of the bowel and the need for immediate surgery. Auscultation of bowel sounds, checking the temperature, and obtaining information about the accident are appropriate actions, but the priority is to prepare to send the patient for emergency surgery.

A patient with a family history of stomach cancer asks the nurse about ways to decrease the risk for developing stomach cancer. What would the nurse teach the patient to avoid? A. Emotionally stressful situations B. Smoked foods such as ham and bacon C. Foods that cause distention or bloating D. Chronic use of H2 blocking medications

B Smoked foods such as bacon, ham, and smoked sausage increase the risk for stomach cancer. Stressful situations, abdominal distention, and use of H2 blockers are not associated with an increased incidence of stomach cancer.

Which assessment information will be most important for the nurse to report to the health care provider about a patient who has acute cholecystitis? A. The patient's urine is bright yellow. B. The patient's stools are tan colored. C. The patient reports chronic heartburn. D. The patient has increased pain after eating.

B Tan or gray stools indicate biliary obstruction, which requires rapid intervention to resolve. The other data are not unusual for a patient with this diagnosis, and would be reported but do not require urgent intervention.

Which topic is most important for the nurse to include in teaching for a 41-yr-old patient diagnosed with early alcoholic cirrhosis? A. Taking lactulose B. Avoiding all alcohol use C. Maintaining good nutrition D. Using vitamin B supplements

B The disease progression can be stopped or reversed by alcohol abstinence. The other interventions may be used when cirrhosis becomes more severe to decrease symptoms or complications, but the priority for this patient is to stop the progression of the disease.

Which information about dietary management would the nurse include when teaching a patient with peptic ulcer disease (PUD)? A. "You will need to remain on a bland diet." B. "Avoid foods that cause pain after you eat them." C. "High-protein foods are least likely to cause pain." D. "You should avoid eating raw fruits and vegetables."

B The best information is that each person should choose foods that are not associated with postprandial discomfort. Raw fruits and vegetables may irritate the gastric mucosa but chewing well seems to decrease this problem and some patients tolerate these healthy foods well. High-protein foods help neutralize acid, but they also stimulate hydrochloric (HCl) acid secretion and may increase discomfort for some patients. Bland diets may be recommended during an acute exacerbation of PUD, but there is little evidence to support their ongoing use.

Which result is most important for the nurse to monitor to detect possible complications in a patient with severe cirrhosis who has bleeding esophageal varices? A. Bilirubin levels B. Ammonia levels C. Potassium levels D. Prothrombin time

B The protein in the blood in the gastrointestinal tract will be absorbed and may result in an increase in the ammonia level because the liver cannot metabolize protein very well. The prothrombin time, bilirubin, and potassium levels should also be monitored, but they will not be affected by the bleeding episode.

The nurse will plan to teach the patient with newly diagnosed achalasia that A. drinking fluids with meals should be avoided. B. lying down and resting after meals is recommended. C. a liquid or blenderized diet will be necessary. D. endoscopic procedures may be used for treatment.

D. endoscopic procedures may be used for treatment.

In planning the care of pt following a cholecystectomy laporatomy, which comfort measure should be used to reduce the risk of a pulmonary infection? A. maintain bed rest B. encourage ROM exercises C. assist pt to cough and deep breath D. have pt splint the abdominal incision with pillow during respiratory hygiene

D. have pt splint the abdominal incision with pillow during respiratory hygiene

When assessing a pt with portal hypertension, the RN should be alert for indications of A. liver abscess B. intestinal obstruction C. perforation of duodenum D. hemorrhage from esophageal varices

D. hemorrhage from esophageal varices

2 days after her cholecystectomy, a pt has been experiencing N/V. The pt has a t-tube in place. For what electrolyte imbalance will the RN monitor? A. hypernatremia B. hyperkalemia C. hypervolemia D. hypokalemia

D. hypokalemia

The pt with hepatitis is scheduled for a liver biopsy. The RN implements which of the following to assess for the most common symptom of bile peritonitis following biopsy A. monitoring for bloody diarrhea B. assessing for rebound tenderness C. assessing for increased flatulence D. monitoring for abdominal pain

D. monitoring for abdominal pain

Which pt requires immediate nursing intervention? pt who A. complains of epigastric pain after eating B. complains of anorexia and periumbilical pain C. presents with ribbonlike stools D. presents with rigid. board like abdomen

D. presents with rigid. board like abdomen

An adult with a body mass index (BMI) of 22 kg/m2 is being admitted to the hospital for elective knee surgery. Which assessment finding would the nurse report to the health care provider? A. Tympany on percussion of the abdomen B. Liver edge 3 cm below the costal margin C. Bowel sounds of 20/min in each quadrant D. Aortic pulsations visible in the epigastric area

B Normally the lower border of the liver is not palpable below the ribs, so this finding suggests hepatomegaly. Visible aortic pulsations in the epigastrium, active bowel sounds, and abdominal tympany are within normal findings for an adult of normal weight.

Calculate the flow rate for the following: Order: 1000 ml D5W IV 4pm-2am

100mL/hr 1000 ml/10hours

Which patient statement would indicate to the nurse that teaching after a laparoscopic cholecystectomy was effective? A. "I can take a shower and walk around the house tomorrow." B. "I need to limit my activities and not return to work for 4 weeks." C. "I can expect yellowish drainage from the incision for a few days." D. "I will follow a low-fat diet for life because I do not have a gallbladder."

A After a laparoscopic cholecystectomy, patients are discharged the same (or next) day and have few restrictions on activities of daily living. Drainage from the incisions would be abnormal, and the patient should be instructed to call the health care provider if this occurs. A low-fat diet may be recommended for a few weeks after surgery but will not be a lifelong requirement.

A patient with a new ileostomy asks how much it will drain after the bowel has adapted in a few months. How many cups of drainage per day would the nurse tell the patient to expect? A. 2 B. 3 C. 4 D. 5

A After the proximal small bowel adapts to reabsorb more fluid, the average amount of ileostomy drainage is about 500 mL daily. One cup is about 240 mL.

The RN is aware that pts with cholelithiasis and obstructive jaundice would be likely to exhibit which of the following signs and symptoms. select all that apply A. pruritus B. yellow sclera C. pale stool D. straw-colored urine

A, B, C

For a client with hepatic cirrhosis who has altered clotting mechanisms, which intervention would be most important? A. Allowing complete independence of mobility B. Applying pressure to injection sites C. Administering antibiotics as prescribed D. Increasing nutritional intake

B. Applying pressure to injection sites. The client with cirrhosis who has altered clotting is at high risk for hemorrhage. Prolonged application of pressure to injection or bleeding sites is important. Instruct patient/SO of signs and symptoms that warrant notification of health care provider: increased abdominal girth; rapid weight loss/gain; increased peripheral edema; increased dyspnea, fever; blood in stool or urine; excess bleeding of any kind; jaundice. Option A: Complete independence may increase the client's potential for injury, because an unsupervised client may injure himself and bleed excessively. Instruct SO to notify health care providers of any confusion, untidiness, night wandering, tremors, or personality change. Changes (reflecting deterioration) may be more apparent to SO, although insidious changes may be noted by others with less frequent contact with the patient. Option C: Antibiotics are important to promote liver regeneration. However, they are not most important for a client at high risk for hemorrhage. Some drugs are hepatotoxic (especially narcotics, sedatives, and hypnotics). In addition, the damaged liver has a decreased ability to metabolize all drugs, potentiating cumulative effect and/or aggravation of bleeding tendencies. Option D: Encourage the patient to eat; explain reasons for the types of diet. Feed the patient if tiring easily, or have SO assisted the patient. Include the patient in meal planning to consider his/her preferences in food choices. Improved nutrition and diet are vital to recovery. The patient may eat better if the family is involved and preferred foods are included as much as possible.

Which nursing action will the nurse include in the plan of care for a patient admitted with an exacerbation of inflammatory bowel disease (IBD)? A. Restrict IV fluid intake. B. Monitor stools for blood. C. Ambulate six times daily. D. Increase dietary fiber intake.

B Because anemia or hemorrhage may occur with IBD, stools should be assessed for the presence of blood. The other actions would not be appropriate for the patient with IBD. Dietary fiber may increase gastrointestinal motility and exacerbate the diarrhea, severe fatigue is common with IBD exacerbations, and dehydration may occur.

The nurse is providing discharge instructions to a client following gastrectomy. Which measure will the nurse instruct the client to follow to assist in preventing dumping syndrome? A. Eat high-carbohydrate foods. B. Limit the fluids taken with meals. C. Ambulate following a meal. D. Sit in a high-Fowler's position during meals

. B. Limit the fluids taken with meals. The nurse should instruct the client to decrease the amount of fluid taken at meals and to take antispasmodics as prescribed. Discuss the importance of eating small, frequent meals slowly and in a relaxed atmosphere; resting after meals; avoiding extremely hot or cold food; restricting high-fiber foods, caffeine, milk products, and alcohol, excess sugars and salt; and taking fluids between meals, rather than with food. Option A: The nurse should instruct the client to avoid high carbohydrate foods including fluids such as fruit nectars. Review dietary needs and regimen (low-carbohydrate, low-fat, high-protein) and the importance of maintaining vitamin supplementation. This may prevent deficiencies, enhance healing, and promote cooperation with therapy. A low-fat diet may be required to reduce the risk of alkaline reflux gastritis. Option C: The nurse should instruct the client to lie down for 30 minutes after eating to delay gastric emptying. Identify symptoms that may indicate dumping syndrome, (weakness, profuse perspiration, epigastric fullness, nausea and vomiting, abdominal cramping, faintness, flushing, explosive diarrhea, and palpitations occurring within 15 min to 1 hr after eating). Option D: The nurse should instruct the client to assume a low Fowler's position during meals. Avoid placing the patient in a supine position, have the patient sit upright after meals. Supine position after meals can increase regurgitation of acid. Instruct the patient to chew food thoroughly and eat slowly. Well-masticated food is easier to swallow. Food should be cut into small pieces.

The student nurse is participating in a colorectal cancer screening program. Which patient has the fewest risk factors for colon cancer? A. Janice, a 45 y.o. with a 25-year history of ulcerative colitis. B. George, a 50 y.o. whose father died of colon cancer. C. Herman, a 60 y.o. who follows a low-fat, high-fiber diet. D. Sissy, a 72 y.o. with a history of breast cancer.

. C. Herman, a 60 y.o. who follows a low-fat, high-fiber diet. Large population studies with variable strength evidence have found CRC protective factors such as physical activity, diet (fruits and vegetables, fiber, resistant starch, fish), vitamin supplements (folate, folic acid, pyridoxine B6, calcium, vitamin D, magnesium), garlic and coffee, and drugs aspirin, non-steroidal anti-inflammatory drugs (NSAIDs), hormonal replacement therapy in postmenopausal, statins, bisphosphonate and angiotensin inhibitors Option A: Inflammatory bowel disease (IBD), mainly ulcerative colitis, has a well-known association with Cca, with an estimated incidence 0.5% per year between 10 and 20 years after the time of IBD diagnosis and 1% per year after that reaching a 30% risk probability by the fourth decade of patients with pancolitis. Option B: Personal or family history of CRC, adenomatous polyps, and polyps with villous or tubulovillous dysplasia indicate a high risk for synchronous and metachronous CRC primary cancer up to 3% to 5% at 5 years or even longer after resection requiring a closer screening interval. Option D: Colon cancer (Cca) could present as sporadic (70%), familial clustering (20%), and inherited syndromes (10%). Sporadic Cca average age diagnosis is older than 50 years and mostly linked to environmental factors, different from a minority of patients with a true inherited pattern that carries a higher risk at a younger age (younger than 50 years), and the remaining 20% are familial clustering in the absence of identifiable inherited syndrome.

Which topic would the nurse plan to teach the patient diagnosed with acute hepatitis B? A. Administering a-interferon B. Measures for improving appetite C. Side effects of nucleotide analogs D. Ways to increase activity and exercise

B Maintaining adequate nutritional intake is important for regeneration of hepatocytes. Interferon and antivirals may be used for chronic hepatitis B, but they are not prescribed for acute hepatitis B infection. Rest is recommended.

Which diagnostic test would the nurse anticipate for an older patient who is vomiting "coffee-ground" emesis? A. Endoscopy B. Angiography C. Barium studies D. Gastric analysis

A Endoscopy is the primary tool for visualization and diagnosis of upper gastrointestinal (GI) bleeding. Angiography is used only when endoscopy cannot be done because it is more invasive and has more possible complications. Barium studies are helpful in determining the presence of gastric lesions, but not whether the lesions are actively bleeding. Gastric analysis testing may help with determining the cause of gastric irritation, but it is not used for acute GI bleeding.

Which finding indicates to the nurse that lactulose is effective for an older adult who has advanced cirrhosis? A. The patient is alert and oriented. B. The patient denies nausea or anorexia. C. The patient's bilirubin level decreases. D. The patient has at least one stool daily.

A The purpose of lactulose in the patient with cirrhosis is to lower ammonia levels and prevent encephalopathy. Although lactulose may be used to treat constipation, that is not the purpose for this patient. Lactulose will not decrease nausea and vomiting or lower bilirubin levels.

Calculate the flow rate for the following: Order of 500 mL over 1 hour and 45 mins

286mL/hr 500mL/1.75 hr (1hr + (45/60))

A young woman with Crohn's disease develops a fever and symptoms of a urinary tract infection (UTI) with tan, fecal-smelling urine. Which information will the nurse add to a teaching plan about UTIs for this patient that goes beyond a general teaching plan for UTIs? A. Fistulas can form between the bowel and bladder. B. Bacteria in the perianal area can enter the urethra. C. Drink adequate fluids to maintain normal hydration. D. Empty the bladder before and after sexual intercourse.

A Fistulas between the bowel and bladder occur in Crohn's disease and can lead to UTI. Teaching for UTI prevention in general includes good hygiene, adequate fluid intake, and voiding before and after intercourse.

A patient in the outpatient clinic is diagnosed with acute hepatitis C (HCV) infection. Which action would the nurse take? A. Schedule the patient for HCV genotype testing. B. Administer the HCV vaccine and immune globulin. C. Teach the patient about direct-acting antiviral treatment. D. Explain that the infection will resolve over a few months.

A Genotyping of HCV has an important role in managing treatment and is done before drug therapy is initiated. Because most patients with acute HCV infection convert to the chronic state, the nurse should not teach the patient that the HCV will resolve in a few months. Immune globulin or vaccine is not available for HCV. Direct-acting antiviral drugs are used for chronic HCV infection.

A patient has been admitted with acute liver failure. Which assessment data are most important for the nurse to communicate to the health care provider? A. Asterixis and lethargy B. Jaundiced sclera and skin C. Elevated total bilirubin level D. Liver 3 cm below costal margin

A The patient's findings of asterixis and lethargy are consistent with grade 2 hepatic encephalopathy. Patients with acute liver failure can deteriorate rapidly from grade 1 or 2 to grade 3 or 4 hepatic encephalopathy and need early transfer to a transplant center. The other findings are typical of patients with hepatic failure and would be reported but would not indicate a need for an immediate change in the therapeutic plan.

A woman receiving chemotherapy for breast cancer develops a Candida albicans oral infection. Which intervention would the nurse anticipate? A. Nystatin tablets B. Antiviral agents C. Referral to a dentist D. Hydrogen peroxide rinses

A C. albicans infections are treated with an antifungal such as nystatin. Peroxide rinses would be painful. Oral saltwater rinses may be used but will not cure the infection. Antiviral agents are used for viral infections such as herpes simplex. Referral to a dentist is indicated for gingivitis but not for Candida infection.

The nurse is assessing a patient with abdominal pain. How will the nurse document ecchymosis around the area of umbilicus? A. Cullen's sign B. Rovsing sign C. McBurney's sign D. Grey-Turner's sign

A Cullen's sign is ecchymosis around the umbilicus. Rovsing sign occurs when palpation of the left lower quadrant causes pain in the right lower quadrant. Grey Turner's sign is bruising over the flanks. Deep tenderness at McBurney's point (halfway between the umbilicus and the right iliac crest), known as McBurney's sign, is a sign of acute appendicitis.

Which menu choice by the patient with diverticulosis is best for preventing diverticulitis? A. Navy bean soup and vegetable salad B. Whole grain pasta with tomato sauce C. Baked potato with low-fat sour cream D. Roast beef sandwich on whole wheat bread

A A diet high in fiber and low in fats and red meat is recommended to prevent diverticulitis. Although all the choices have some fiber, the bean soup and salad will be the highest in fiber and the lowest in fat.

Which action would the nurse plan when admitting a patient with acute diverticulitis plan for initial care? A. Administer IV fluids. B. Prepare for colonoscopy. C. Encourage a high-fiber diet. D. Give stool softeners and enemas.

A A patient with acute diverticulitis will be NPO and given parenteral fluids. A diet high in fiber and fluids will be implemented before discharge. Bulk-forming laxatives, rather than stool softeners, are usually given. These will be implemented later in the hospitalization. The patient with acute diverticulitis will not have enemas or a colonoscopy because of the risk for perforation and peritonitis.

Which patient would the nurse assess first after receiving change-of-shift report? A. A patient with esophageal varices who has a rapid heart rate B. A patient with a history of gastrointestinal bleeding who has melena C. A patient with nausea who has a dose of metoclopramide (Reglan) due D. A patient who is crying after receiving a diagnosis of esophageal cancer

A A patient with esophageal varices and a rapid heart rate indicate possible hemodynamic instability caused by GI bleeding. The other patients require interventions, but their findings do not indicate acutely life-threatening complications.

A patient is awaiting surgery for acute peritonitis. Which action will the nurse plan to include in the preoperative care? A. Position patient with the knees flexed. B. Avoid use of opioids or sedative drugs. C. Offer frequent small sips of clear liquids. D. Assist patient to breathe deeply and cough.

A There is less peritoneal irritation with the knees flexed, which will help decrease pain. Opioids and sedatives are typically given to control pain and anxiety. Preoperative patients with peritonitis are given IV fluids for hydration. Deep breathing and coughing will increase the patient's discomfort.

The RN is teaching a pt who has been diagnosed with advanced pancreatic cancer. The pt asks the RN why it was not diagnosed earlier. Which of the following statements by the RN would be correct. Select all that apply A. the onset of pancreatic cancer is usually insidious B. pancreatic cancer causes black tarry stools C. pancreatic cancer presents with slow weight loss and jaundice D. pain with pancreatic cancer is dull epigastric pain that may not be reported right away

A, C, D

Mr. Hasakusa is in end-stage liver failure. Which interventions should the nurse implement when addressing hepatic encephalopathy? Select all that apply. A. Assessing the client's neurologic status every 2 hours B. Monitoring the client's hemoglobin and hematocrit levels C. Evaluating the client's serum ammonia level D. Monitoring the client's handwriting daily E. Preparing to insert an esophageal tamponade tube F. Making sure the client's fingernails are short

A, C, & D Hepatic encephalopathy results from an increased ammonia level due to the liver's inability to convert ammonia to urea, which leads to neurologic dysfunction and possible brain damage. Hepatic encephalopathy (HE) is a reversible syndrome observed in patients with advanced liver dysfunction. The syndrome is characterized by a spectrum of neuropsychiatric abnormalities resulting from the accumulation of neurotoxic substances in the bloodstream (and ultimately in the brain). Option A: The nurse should monitor the client's neurologic status. Symptoms typically include confusion, personality changes, disorientation, and a depressed level of consciousness. The earliest stage is often characterized by an inverted sleep-wake pattern wherein patients are found to be sleeping during the day and awake throughout the night. Option B: Monitoring the client's hemoglobin and hematocrit levels address esophageal bleeding. A diagnosis of HE should involve a thorough evaluation of the patient's vital signs and airway followed by classification of the symptoms according to the West-Haven Criteria. Option C: The nurse should monitor the client's serum ammonia level. Elevated blood ammonia levels are often seen in patients with hepatic encephalopathy. It is more useful, however, to assess the clinical improvement or deterioration of a patient undergoing treatment rather than monitor serial arterial blood ammonia measurements. Option D: The nurse should monitor the client's handwriting. During the intermediate stages of HE, a characteristic jerking movement of the limbs is often observed (e.g., asterixis) when the patient attempts to hold arms outstretched with hands bent upward at the wrist. Option E: Insertion of an esophageal tamponade tube addresses esophageal bleeding. Treatment for HE involves proper identification and treatment of the underlying cause. Antibiotics (e.g., rifaximin) neomycin/paromomycin/metronidazole, or vancomycin) are often given empirically due to the frequency of infection as an underlying cause. Option F: Keeping fingernails short addresses jaundice. Protein restriction is only of use in patients with acute flare-ups and is not justified in chronic cases. These patients need nutrition as they have a high catabolic rate and severe wasting.

Annabelle is being discharged with a colostomy, and you're teaching her about colostomy care. Which statement correctly describes a healthy stoma? A. "At first, the stoma may bleed slightly when touched." B. "The stoma should appear dark and have a bluish hue." C. "A burning sensation under the stoma faceplate is normal." D. "The stoma should remain swollen away from the abdomen."

A. "At first, the stoma may bleed slightly when touched." For the first few days to a week, slight bleeding normally occurs when the stoma is touched because the surgical site is still new. She should report profuse bleeding immediately. A small amount of blood from the stoma itself is not unusual while it is healing. Option B: A stoma should be a beefy red or pink color. The tissue that makes a stoma is the lining of the intestine and should be moist and shiny. It is very similar in appearance to the inside of the mouth along your cheek. Option C: The skin may be tender initially during the healing process and may feel irritated by normal cleaning. The skin immediately surrounding the stoma and stoma can be irritated by the cleaning process. Option D: A normal stoma in the days after surgery may be swollen and may also produce mucus. While the stoma itself should be moist, the skin around the stoma should be normal in appearance.

You are developing a care plan for Sally, a 67 y.o. patient with hepatic encephalopathy. Which of the following do you include? A. Administering a lactulose enema as ordered. B. Encouraging a protein-rich diet. C. Administering sedatives, as necessary. D. Encouraging ambulation at least four times a day.

A. Administering a lactulose enema as ordered. You may administer the laxative lactulose to reduce ammonia levels in the colon. Elevated ammonia levels disrupt the balance of excitatory and inhibitory neurotransmitters, further exacerbating neurological and motor function decline (Felipo, 2013). Patients who have high ammonia levels can experience HE, but in chronic liver failure, a higher ammonia level does not predict a more severe degree of HE. Option B: Protein restriction is reserved for patients who are severely protein-intolerant or for very short periods for patients with GI bleeding until symptoms resolve (Amodio et al., 2013). Dairy and vegetable proteins are preferred but are usually much less palatable. A fiber-rich diet is recommended to encourage fecal ammonia excretion while avoiding diarrhea that could potentially induce HE in patients already taking lactulose. Option C: It is suggested to proceed with caution when prescribing and administering any opioids for pain management because of their high-risk effects on the patient with chronic liver disease like sedation, constipation, and confusion, which are precipitating factors to induce HE. Doses may need to be lower with longer intervals between these doses for patient safety. Option D: Asterixis (flapping tremor), muscle twitching, and hyperreflexia may be observed in patients with OHE. These can be accompanied by other neuromuscular impairments such as bradykinesia and hyperactive deep tendon reflexes. Bradykinesia means "slow movement" and can present as decreased facial expressions, increased stillness, or difficulty with performing repetitive tasks such as finger tapping.

Jason, a 22 y.o. accident victim, requires an NG tube for feeding. What should you immediately do after inserting an NG tube for liquid enteral feedings? A. Aspirate for gastric secretions with a syringe. B. Begin feeding slowly to prevent cramping. C. Get an X-ray of the tip of the tube within 24 hours. D. Clamp off the tube until the feedings begin.

A. Aspirate for gastric secretions with a syringe. Aspirating the stomach contents confirms correct placement. If feeding is planned through the tube, then it is imperative to confirm its location as placing feeds into the lungs can cause potentially fatal complications. The ideal location for an NG tube placed for suction is within the stomach because placement past the pylorus can cause damage to the duodenum. The ideal location for an NG feeding tube is postpyloric to decrease the risk of aspiration. Option B: If the tube is being placed for the administration of medications or nutrition, intragastric placement must be confirmed. Introducing medication or tube feeds to the lungs can cause major complications, including death. Even in intubated patients, the NG tube can still be accidentally placed into the airway. Option C: If an X-ray is ordered, it should be done immediately, not in 24 hours. Taking an abdominal x-ray is the best way to confirm the location of the tube, even if there is the aspiration of gastric contents as the tube may be placed past the pylorus where it will aspirate not just gastric secretions but also hepatobiliary secretions leading to persistently high output even when the patient's acute issue has resolved. Option D: Once the tube has been advanced to the estimated necessary length, correct location is often made obvious by aspirating out a large amount of gastric contents. Pushing 50 cc of air through the tube using a large syringe while auscultating the stomach with a stethoscope is a commonly described maneuver to determine the location of the tube, but it is of able efficacy.

Type A chronic gastritis can be distinguished from type B by its ability to: A. Cause atrophy of the parietal cells. B. Affect only the antrum of the stomach. C. Thin the lining of the stomach walls. D. Decrease gastric secretions.

A. Cause atrophy of the parietal cells. Type A causes changes in parietal cells. Type A is caused by the immune system destroying stomach cells. And it can increase the risk of vitamin deficiencies, anemia, and cancer. Chronic gastritis occurs when the stomach lining becomes inflamed. Bacteria, consuming too much alcohol, certain medications, chronic stress, or other immune system problems can lead to inflammation. Option B: Type B, the most common type, is caused by Helicobacter pylori bacteria, and can cause stomach ulcers, intestinal ulcers, and cancer. Because chronic gastritis occurs over a long period of time it gradually wears away at the stomach lining. And it can cause metaplasia or dysplasia. These are precancerous changes in the cells that can lead to cancer if untreated. Option C: Type C is caused by chemical irritants like NSAIDs, alcohol, or bile. And it can also cause stomach lining erosion and bleeding. When inflammation occurs, the stomach lining changes and loses some of its protective cells. It may also cause early satiety. This is where the stomach feels full after eating just a few bites of food. Option D: A stressful lifestyle or a traumatic experience can decrease the stomach's ability to protect itself. In addition, the risk increases if the patient has autoimmune diseases or certain illnesses like Crohn's disease.

A male client with cholelithiasis has a gallstone lodged in the common bile duct. When assessing this client, the nurse expects to note: A. Yellow sclera B. Light amber urine C. Circumoral pallor D. Black, tarry stools

A. Yellow sclera Yellow sclera may be the first sign of jaundice, which occurs when the common bile duct is obstructed. Jaundice can be a sign of a common bile duct obstruction from an entrapped gallstone. In the presence of jaundice and abdominal pain, often, a procedure is an indication to go and retrieve the stone to prevent further sequelae. Option B: Urine normally is light amber. Usually, patients with symptoms from gallstones present with right upper abdominal pain after eating greasy or spicy foods. There is often nausea and vomiting. Pain can also be present in the epigastric area that radiates to the right scapula or mid-back. Option C: Circumoral pallor doesn't occur in common bile duct obstruction; it is a sign of hypoxia, respectively. The classic physical exam finding is a positive Murphy's sign, where the pain is elicited on deep palpation to the right upper quadrant underneath the rib cage upon deep inspiration. Option D: Black, tarry stools don't occur in common bile duct obstruction; they are signs of GI bleeding. Progression of this condition is indicated by neurologic changes and hypotension (Reynold's pentad). Other sequelae are acute pancreatitis with symptoms of midepigastric pain and intractable vomiting.

The client with a new colostomy is concerned about the odor from the stool in the ostomy drainage bag. The nurse teaches the client to include which of the following foods in the diet to reduce odor? A. Yogurt B. Broccoli C. Cucumbers D. Eggs

A. Yogurt The client should be taught to include deodorizing foods in the diet, such as beet greens, parsley, buttermilk, and yogurt. Drinking buttermilk and/or eating yogurt or parsley can help to reduce odors from colostomy and ileostomy bags. In the case of urostomy patients, asparagus and fish will make the urine smell stronger. Spinach also reduces odor but is a gas-forming food as well. Broccoli, cucumbers, and eggs are gas-forming foods. Option B: High-fiber foods are difficult to digest. Although very good for people's overall health and well-being, these slow-digesting foods break down or ferment in the digestive tract. The fermentation process produces odorous gas. Option C: In some cases, foods higher in fiber have a distinctive odor. The natural odor from these foods can also cause flatulence to be smelly. High-fiber foods often also contain more sulfur than other types. This can cause the makeup of a person's fart to change to include more sulfur, which has a distinct odor and will cause the person to produce smellier gas. Option D: When trying eggs, start with a small amount (such as 1 egg). Eggs may cause a bad odor (smell) when the client opens the pouch. Eating bland foods will help avoid uncomfortable symptoms such as diarrhea (loose or watery bowel movements), bloating, and gas.

Which information will the nurse include when teaching adults to decrease the risk for cancers of the tongue and buccal mucosa? A. Use sunscreen even on cloudy days. B. Avoid cigarettes and smokeless tobacco. C. Complete antibiotic courses used to treat throat infections. D. Use antivirals to treat herpes simplex virus (HSV) infections.

B Tobacco use greatly increases the risk for oral cancer. Acute throat infections do not increase the risk for oral cancer, although chronic irritation of the oral mucosa does increase risk. Sun exposure does not increase the risk for cancers of the buccal mucosa, although it increases risk for cancer of the lip. Human papillomavirus (HPV) infection is associated with an increased risk, but HSV infection is not a risk factor for oral cancer.

A 22-yr-old female patient with an exacerbation of ulcerative colitis is having 15 to 20 stools daily and has excoriated perianal skin. Which patient behavior indicates that the nurse's teaching about skin integrity has been effective? A. The patient uses incontinence briefs to contain loose stools. B. The patient uses witch hazel compresses to soothe irritation. C. The patient asks for antidiarrheal medication after each stool. D. The patient cleans the perianal area with soap after each stool.

B Witch hazel compresses are suggested to reduce anal irritation and discomfort. Incontinence briefs may trap diarrhea and increase the incidence of skin breakdown. Antidiarrheal medications cannot be given 15 to 20 times a day. The perianal area should be washed with plain water or pH balanced cleanser after each stool.

Arthur has a family history of colon cancer and is scheduled to have a sigmoidoscopy. He is crying as he tells you, "I know that I have colon cancer, too." Which response is most therapeutic? A. "I know just how you feel." B. "You seem upset." C. "Oh, don't worry about it, everything will be just fine." D. "Why do you think you have cancer?"

B. "You seem upset." Making observations about what you see or hear is a useful therapeutic technique. This way, you acknowledge that you are interested in what the patient is saying and feeling. Observations about the appearance, demeanor, or behavior of patients can help draw attention to areas that might pose a problem for them. Option A: Giving one's own opinion, evaluating, moralizing, or implying one's values by using words such as "nice", "bad", "right", "wrong", "should" and "ought" is not appropriate. Advanced levels of emotional support include sitting with patients and "providing opportunities for them to feel accompanied in their struggles," directly answering s, making the patient feel special, and making supportive gestures such as, when appropriate, holding the patient's hand. Option C: Giving the patient false reassurance is inappropriate. False reassurance is something a nurse might give to a patient in an effort to comfort or encourage them, but in reality, is not based on fact either. Option D: Probing is inappropriate in this situation. Nontherapeutic communication also includes probing, or the continuous ing of the client about something, that may, in turn, discourage proper communication between the nurse and patient. You're caring for Beth who underwent a Billroth II procedure (surgical removal of the pylorus and duodenum) for treatment of a peptic ulcer. Which findings suggest that the patient is developing dumping syndrome, a complication associated with this procedure? A. Flushed, dry skin. B. Headache and bradycardia. C. Dizziness and sweating. D. Dyspnea and chest pain. C. Dizziness and sweating. After a Billroth II procedure, a large amount of hypertonic fluid enters the intestine. This causes extracellular fluid to move rapidly into the bowel, reducing circulating blood volume and producing vasomotor symptoms. Vasomotor symptoms produced by dumping syndrome include dizziness and sweating, tachycardia, syncope, pallor, and palpitations. Option A: Dumping syndrome occurs in patients who have had gastric surgery. Symptoms of early dumping occur within 10 to 30 minutes after a meal. Late dumping can present 1 to 3 hours after a high-carbohydrate meal. In early dumping, the symptoms usually occur within 10 to 30 minutes after a meal. The rapid transit of hyperosmolar chyme from the stomach into the duodenum causes fluid to shift from the vasculature to the intestinal lumen, leading to increased volume in the small bowel. Option B: There may be GI or vasomotor symptoms. GI symptoms include nausea, vomiting, diarrhea, or belching. Vasomotor symptoms include shock, syncope, near-syncope, palpitations, dizziness, desire to lie down, or diaphoresis. The suggested division of meals recommended is at least six times per day. Liquids should be withheld until 30 minutes after the meal. In addition, simple sugars and milk products should be avoided. Option D: The enteric nervous system plays a major role in the regulation of gastric emptying, involving several gastrointestinal (GI) hormones and extrinsic innervation. Late dumping, also known as postprandial hyperinsulinemic hypoglycemia, usually occurs 1 to 3 hours after a high-carbohydrate meal. There is an association with hypoglycemia, but the exact mechanism is unknown. It is proposed that the rapid absorption of carbohydrates exaggerates the glucose-mediated insulin response.

The RN is caring for an African American pt who has been diagnosed with acute viral hepatitis. RN assesses for jaundice by checking which specific area? A. flexor surfaces of extremeties B. hard palate of mouth C. nailbeds D. skin

B. Hard palate of mouth

Which elevated lab test result is most indicative of acute pancreatitis? A. blood glucose B. serum amylase C. serum bilirubin level D. WBC count

B. Serum amylase

50 yo pt is admitted to hospital with severe back and abdominal pain, nausea, and occasional vomiting, temp of 101. He reports drinking 6-8 beers a day. Diagnosis of acute pancreatitis is made. Based on data, which is primary nursing diagnosis a. disturbed self-concept r/t illness b. acute pain r/t inflammation of pancreas c. deficient fluid volum r/t vomiting d. imbalanced nurtrition less than body requirements r/t nausea

B. acute pain r/t inflammation of pancreas

Pt with chronic pancreatitis is taking pancreatic enzyme supplement pancrease. RN understands that teaching has been effective when pt states that he will take medicine A. 30 mins before meals B. 30 mins after meals C. with meals D. in between meals

C. with meals

A patient who has chronic constipation asks the nurse about the use of psyllium (Metamucil). Which information would the nurse provide? A. Fiber-containing laxatives may reduce the absorption of fat-soluble vitamins. B. Dietary sources of fiber should be eliminated to prevent excessive gas formation. C. Use of this type of laxative to prevent constipation does not cause adverse effects. D. Large amounts of fluid should be taken to prevent impaction or bowel obstruction.

D A high fluid intake is needed when patients are using bulk-forming laxatives to avoid worsening constipation. Although bulk-forming laxatives are generally safe, the nurse should emphasize the possibility of constipation or obstipation if inadequate fluid intake occurs. Although increased gas formation is likely to occur with increased dietary fiber, the patient should gradually increase dietary fiber and eventually may not need the psyllium. Fat-soluble vitamin absorption is blocked by stool softeners and lubricants, not by bulk-forming laxatives.

The nurse is monitoring a client with a diagnosis of peptic ulcer. Which assessment finding would most likely indicate perforation of the ulcer? A. Bradycardia B. Numbness in the legs C. Nausea and vomiting D. A rigid, board-like abdomen

D. A rigid, board-like abdomen

You promote hemodynamic stability in a patient with upper GI bleeding by: A. Encouraging oral fluid intake. B. Monitoring central venous pressure. C. Monitoring laboratory test results and vital signs. D. Giving blood, electrolyte, and fluid replacement.

D. Giving blood, electrolyte, and fluid replacement. To stabilize a patient with acute bleeding, NS or LR solution is given I.V. until BP rises and urine output returns to 30ml/hr. Two large-caliber (18-gauge or larger) peripheral I.V. catheters or a central venous catheter should be inserted. For patients who are hemodynamically unstable, two 16-gauge I.V. catheters should be inserted. Option A: Nurses should expect to administer isotonic fluids, such as normal saline or lactated Ringer's solution, as well as any necessary blood products. Coagulopathies should be corrected with fresh frozen plasma and vitamin K. If necessary, more rapid reversal of anticoagulation can be achieved via prothrombin complex concentrate infusions; this is the preferred approach for patients with serious or life-threatening bleeding. Platelets should be administered for patients with thrombocytopenia or a platelet count below 50,000/mcL (normal, 150,000 to 450,000/mcL). Option B: Closely monitor the patient's clinical status, including airway, vital signs, cardiac rhythm, urine output, and nasogastric output if a nasogastric tube is in place. Initial treatment goals are focused on airway maintenance and volume resuscitation. Endotracheal intubation should be considered for those with ongoing hematemesis or altered respiratory or mental status. Option C: After initial fluid resuscitation, patients may require a blood transfusion, depending on their signs and symptoms and overall clinical presentation. A hemoglobin level maintained above 7 g/dL (normal, 13.5 to 17.5 g/dL in men) is recommended, but transfusions may be necessary for patients who are clinically unstable despite their hemoglobin levels.

The RN understands that the ascites seen in cirrhosis results in part from A. escape of lymph into abdominal cavity directly from the inflamed liver sinusoid B. increased plasma colloid osmotic pressure due to excessive liver growth and metabolism C. decreased levels of ADH and aldosterone due to increasing metabolic liver activity D. compression of portal veins with resultant increased back pressure in portal venous system

D. compression of portal veins with resultant increased back pressure in portal venous system

Which of the following pts is more likely to develop pancreatitis? A. 59 yo male with history of occasional alcohol use B. pt with renal problems and hypocalcemia C. pt recovering from myocardial infarction with hypercholesterolemia D. pt with stone lodged in common bile duct

D. pt with stone lodged in common bile duct


Ensembles d'études connexes

Conceptual Physics Final Review Chapters 11-14

View Set

Chapter 4: Common Reproductive Issues

View Set

Chapter 17. Understanding Accounting and Financial Information

View Set

med surg exam 5 practice (ch59pancreas/guallbladder)

View Set

Motion graph-Speed/time multiple choice #2

View Set

Breach of Contract - Contracts - Cha. 24

View Set